You are on page 1of 332

CHEMISTRY

CBSE (PART-I)

S.No. Content Page

SYLLABUS I & II

01. SOLUTION (UNIT-2) 01 – 70

02. ELECTROCHEMISTRY (UNIT-3) 71 – 136

03. CHEMICAL KINETICS (UNIT-4) 137 – 205

04. d & f-BLOCK (UNIT-8) 206 – 261

05. COORDINATION COMPOUND (UNIT-9) 262 - 328

"This booklet is colour coded for you convenience.


Red : Important [prepare with writing practice]
Blue : Analyse the concept involved.
Green : Read once
All rights including trademark and copyrights and rights of translation etc. reserved and vested exclusively with
Allen Career Institute Private Limited. (Allen)
No part of this work may be copied, reproduced, adapted, abridged or translated, transcribed, transmitted,
stored or distributed in any form retrieval system, computer system, photographic or other system or
transmitted in any form or by any means whether electronic, magnetic, chemical or manual, mechanical, digital,
optical, photocopying, recording or otherwise, or stood in any retrieval system of any nature without the written
permission of the Allen Career Institute Private Limited. Any breach will entail legal action and prosecution
without further notice.
This work is sold/distributed by Allen Career Institute Private Limited subject to the condition and undertaking
given by the student that all proprietary rights (under the Trademark Act, 1999 and Copyright Act, 1957) of the
work shall be exclusively belong to Allen Career Institute Private Limited. Neither the Study Materials and/or
Test Series and/or the contents nor any part thereof i.e. work shall be reproduced, modify, re-publish, sub-
license, upload on website, broadcast, post, transmit, disseminate, distribute, sell in market, stored in a retrieval
system or transmitted in any form or by any means for reproducing or making multiple copies of it.
Any person who does any unauthorised act in relation to this work may be liable to criminal prosecution and
civil claims for damages. Any violation or infringement of the propriety rights of Allen shall be punishable
under Section- 29 & 52 of the Trademark Act, 1999 and under Section- 51, 58 & 63 of the Copyright Act, 1957 and
any other Act applicable in India. All disputes are subjected to the exclusive jurisdiction of courts, tribunals and
forums at Kota, Rajasthan only.
Note:- This publication is meant for educational and learning purposes. All reasonable care and diligence have
been taken while editing and printing this publication. Allen Career Institute Private Limited shall not hold any
responsibility for any error that may have inadvertently crept in. Allen Career Institute Private Limited is not responsible for
the consequences of any action taken on the basis of this publication.
CBSE
ALLEN
CHEMISTRY (Code No. 043)
CLASS XII (2023 - 24)
SYLLABUS
Time : 3 Hours 70 Marks
S.No. Title Marks
1. Solutions 7
2. Electrochemistry 9
3. Chemical Kinetics 7
4. d -and f -Block Elements 7
5. Coordination Compounds 7
6. Haloalkanes and Haloarenes 6
7. Alcohols, Phenols and Ethers 6
8. Aldehydes, Ketones and Carboxylic Acids 8
9. Amines 6
10. Biomolecules 7
Total 70

Unit II : Solutions
Types of solutions, expression of concentration of solutions of solids in liquids, solubility of
gases in liquids, solid solutions, Raoult's law, colligative properties - relative lowering of vapour
pressure, elevation of boiling point, depression of freezing point, osmotic pressure, determination
of molecular masses using colligative properties, abnormal molecular mass, Van't Hoff factor.
Unit III : Electrochemistry
Redox reactions, EMF of a cell, standard electrode potential, Nernst equation and its application
to chemical cells, Relation between Gibbs energy change and EMF of a cell, conductance in
electrolytic solutions, specific and molar conductivity, variations of conductivity with
concentration, Kohlrausch's Law, electrolysis and law of electrolysis (elementary idea), dry cell-
electrolytic cells and Galvanic cells, lead accumulator, fuel cells, corrosion.
Unit IV : Chemical Kinetics
Rate of a reaction (Average and instantaneous), factors affecting rate of reaction: concentration,
temperature, catalyst; order and molecularity of a reaction, rate law and specific rate constant,
integrated rate equations and half-life (only for zero and first order reactions), concept of collision
theory (elementary idea, no mathematical treatment), activation energy, Arrhenius equation.
Unit VIII : d and f Block Elements
General introduction, electronic configuration, occurrence and characteristics of transition metals,
general trends in properties of the first-row transition metals – metallic character, ionization
enthalpy, oxidation states, ionic radii, colour, catalytic property, magnetic properties, interstitial
compounds, alloy formation, preparation and properties of K2Cr2O7 and KMnO4.

E I
Chemistry
ALLEN
Lanthanoids –
Electronic configuration, oxidation states, chemical reactivity and lanthanoid contraction and its
consequences.
Actinoids - Electronic configuration, oxidation states and comparison with lanthanoids.
Unit IX : Coordination Compounds
Coordination compounds - Introduction, ligands, coordination number, colour, magnetic
properties and shapes, IUPAC nomenclature of mononuclear coordination compounds. Bonding,
Werner's theory, VBT, and CFT; structure and stereoisomerism, the importance of coordination
compounds (in qualitative analysis, extraction of metals and biological system).
Unit X : Haloalkanes and Haloarenes.
Haloalkanes : Nomenclature, nature of C–X bond, physical and chemical properties, optical
rotation mechanism of substitution reactions.
Haloarenes : Nature of C–X bond, substitution reactions (Directive influence of halogen in
monosubstituted compounds only). Uses and environmental effects of - dichloromethane,
trichloromethane, tetrachloromethane, iodoform, freons, DDT.
Unit XI : Alcohols, Phenols and Ethers
Alcohols : Nomenclature, methods of preparation, physical and chemical properties (of primary
alcohols only), identification of primary, secondary and tertiary alcohols, mechanism of
dehydration, uses with special reference to methanol and ethanol.
Phenols : Nomenclature, methods of preparation, physical and chemical properties, acidic nature
of phenol, electrophilic substitution reactions, uses of phenols.
Ethers : Nomenclature, methods of preparation, physical and chemical properties, uses.
Unit XII : Aldehydes, Ketones and Carboxylic Acids
Aldehydes and Ketones : Nomenclature, nature of carbonyl group, methods of preparation,
physical and chemical properties, mechanism of nucleophilic addition, reactivity of alpha
hydrogen in aldehydes, uses.
Carboxylic Acids : Nomenclature, acidic nature, methods of preparation, physical and chemical
properties; uses.
Unit XIII : Amines
Amines : Nomenclature, classification, structure, methods of preparation, physical and chemical
properties, uses, identification of primary, secondary and tertiary amines.
Diazonium salts : Preparation, chemical reactions and importance in synthetic organic chemistry.
Unit XIV : Biomolecules
Carbohydrates - Classification (aldoses and ketoses), monosaccharides (glucose and fructose),
D-L configuration oligosaccharides (sucrose, lactose, maltose), polysaccharides (starch, cellulose,
glycogen); Importance of carbohydrates.
Proteins - Elementary idea of - amino acids, peptide bond, polypeptides, proteins, structure of
proteins - primary, secondary, tertiary structure and quaternary structures (qualitative idea only),
denaturation of proteins; enzymes. Hormones - Elementary idea excluding structure.
Vitamins - Classification and functions.
Nucleic Acids : DNA and RNA.

II E
CBSE
ALLEN

UNIT-2 : SOLUTIONS
UNIT INDEX
• Theory 01-20
• Exercise-1_Intext Questions 21-25
• Exercise-2_NCERT Exercise Questions 26-44
• Exercise-3_Exemplar 45-49
• Exercise-4_MCQ, A/R, Case Based Questions 50-56
• Previous Years Questions 57-66
• Practice Test 67-70
THEORY
INTRODUCTION:
A solution is a homogeneous mixture of two or more than two components.
• Homogeneous means : The mixture has same chemically composition and same physical
properties throughout the solution.
• When the solution is composed of only two chemical substances, it is termed as binary solution,
similarly, it is called ternary and quaternary, if it is composed by three and four components
respectively.
• For binary solution : Solution = solute + solvent
SOLUTE : Generally the component present in lesser amount than other component in solution, is
called solute.
SOLVENT : Generally, the component present in greater amount than all other components, is called
the solvent.
TYPES OF SOLUTIONS :

Type of Solutions Solute Solvent Common Example

Gaseous Solutions Gas Gas Mixture of oxygen and nitrogen gases


Liquid Gas Chloroform mixed with nitrogen gas
Solid Gas Camphor (C10H16O) in nitrogen gas
node06\B0BC-BD\Kota\Board Material\Chemistry\Booklets\CBSE\Part-1

Liquid Solutions Gas Liquid Oxygen dissolved in water


Liquid Liquid Ethanol dissolved in water
Solid Liquid Glucose dissolved in water

Solid Solutions Gas Solid Solution of hydrogen in palladium


Liquid Solid Amalgam of mercury with sodium
Solid Solid Copper dissolved in gold

E 1
Chemistry
ALLEN
CONCENTRATION TERMS
(a) Molarity (M)
The number of moles of solute present in one litre solution is called its molarity(M).
Number of moles of solute n
Molarity  =
volume of solution (in litre) V L 
(b) Molality (m)
The number of moles of solute present in 1 kg of the solvent is called molality of the solution.
Number of moles of solute Number of moles of solute 1000
Molality of a solution = =
Mass of solvent  kg  Mass of solvent  g 
(c) Concentration in terms of percentage
(i) Percent By Mass (w/W) or Mass percentage (w/W) :
Mass of solute (in g) present in 100 g of solution (g) is called mass percent of the solute.
Where 'w' gram of solute is dissolved in W gram of solvent.
Mass of solute  g   100 w 100
Mass percent  
Mass of solution  g  wW
Mass percent is independent of temperature.
(ii) Percent By Volume (v/V) or Volume percentage (v/V) :
This method is used for solutions of liquid in a liquid. The volume of liquid (solute) in mL
present in 100 mL of solution in is called volume percent.
Volume of solute  100
Volume percent =
Volume of solution
w
(iii) Percent by strength /percentage mass by volume   or Mass by volume
V
w
percentage   : Mass of solute (in g) present in 100 mL solution is called mass by
V
volume.
w mass of solute  g 
%    100
 V  volume of solution  mL 
(d) Parts Per Million (ppm)
This method is used for expressing the concentration of very dilute solutions such as hardness of
water, air pollution etc.
Number of parts of the component
Parts per million =  106
Total number of parts of all components of the solution
node06\B0BC-BD\Kota\Board Material\Chemistry\Booklets\CBSE\Part-1

Mass of solute  g   106


ppm of substance (by mass) =
Mass of solution (g)
Volume of solute(mL) 106
ppm (by volume) =
Volume of solution(mL)
 w mass of solute  g 
ppm  by   106
 V  volume of solution  mL 
2 E
CBSE
ALLEN
(e) Mole Fraction
The ratio of the number of moles of one component to the total number of moles of all the
components present in the solution, is called the mole fraction of that component.
moles of solute  n  nB
Mole fraction of solute XB = 
moles of solute  n   moles of solvent  N  n A  n B

moles of solvent  N  nA
Mole fraction of solvent XA = 
moles of solute  n   moles of solvent  N  n A  n B

XA  XB  1

SOLUBILITY
Solubility of a substance is it’s maximum amount that can be dissolved in a specified amount of
solvent at specific temperature and pressure.
• Solubility is affected by nature of solute and solvent as well as by temperature and pressure.
(I) Solubility of Solid in Liquid
or
Solid in liquid type of solution

Solute : Solid Solvent : Liquid

** The solution in which solid solute dissolve in liquid solvent at specific temperature
and pressure to form saturated solutions, known as solubility of solid in liquid.
Saturated Solutions : The solution in which rate of dissolution is equal to the rate of
crystallisation at specific temperature is known as saturated solution.
Factor affecting the solubiity of a solid in liquid :
1. Nature of solute and solvent : (like dissolve like)
Polar solutes are soluble in polar solvent and non polar solutes are soluble in non polar solvent
due to similar intermolecular forces.
Ex. NaCl dissolved in water because both are polar in nature.
Naphthalene dissolved in benzene because both are non-polar in nature.
2. Effect of temperature :
When solid solutes are dissolved in solvent then following equilibrium exists.
node06\B0BC-BD\Kota\Board Material\Chemistry\Booklets\CBSE\Part-1


Dissolution
Solute + Solvent  
 Solution
Crystallization

(solid) (Liq.) (Solid in liq.)


Solubility of solids is affected by temperature according to Le-chatlier's principle.
* If dissolution is exothermic then solubility decreases with increase in temperature.
H < 0  Temperature Solubility 

E 3
Chemistry
ALLEN
Ex. Dissolution of CaCO3 is more effective at low temperature because the dissolution of CaCO3 is
an exothermic phenomenon. Hence, according to Le-chatelier’s principle the dissolution increase
at low temperature.
* If endothermic then solubility increases with increase in temperature.
H > 0  Temperature Solubility 
Ex. The dissolution of NaCl in aqueous solution is more at high temperature because the dissolution
of NaCl in aqueous solution is an endothermic phenomenon. So, on increasing the temperature
solubility increases.
Note : Solubility of solids is not affected by pressure significantly since solids are highly
incompressible.
(II) SOLUBILITY OF GASES IN LIQUID
The solution in which gaseous solute dissolve in liquid solvent at specific temperature to
form saturated solution, known as solubility of gas in a liquid solution.
Factor affecting the solubility of gas in liquid solution :
1. Nature of solute :
The polar gas like NH3, HCl and SO2 are highly soluble in water while non-polar gases like
CO2, O2, N2 etc. are less soluble in water.
2. Temperature : Gas in a liquid is an exothermic phenomenon. So, on increasing
temperature solubility decreases according to Le-chatelier’s principle.
Ex. Gases always tends to be less soluble in liquid as the temperature is raised.
Ans. Gas + Liquid  Solution + E
(Solute) (Solvent)
Dissolution of gas in liquid is an exothermic process, as the temperature is raised, the
equilibrium shifts in reverse direction, It results in decrease of solubility of gas in liquid.
Ex. Aquatic species are more comfortable in cold water than that of warm water because
dissolution of oxygen in water is more at low temperature due to exothermic phenomena.
(III) Effect of Pressure :
The solubility of gas in liquid increases with increase in pressure.
Note : Solubility of a gas in liquid can be explained by HENRY’S LAW.
HENRY'S LAW
According to Henry’s law : “At constant temperature the solubility of a gas in a liquid is
node06\B0BC-BD\Kota\Board Material\Chemistry\Booklets\CBSE\Part-1

directly proportional to the partial pressure of the gas present above the surface of liquid or
solution.”
* Amount of gas which can be dissolved in solvent can be expressed in terms of
concentration i.e., mole fraction. Therefore, Henry’s law may be explained as follows :

Mole fraction of gas in the solution is proportional to the partial pressure of the gas over the
solution.

4 E
CBSE
ALLEN
or
The partial pressure of the gas in vapour phase (P) is proportional to the mole fraction of
the gas (X) in the solution. (This is most common definition)
Pg Xg ........(1)
Pg = KH Xg ........(2) KH = Henry's Constant
ng
Pg  K H .
ng  n

 n g  n 

ng
Pg  K H .
n
Note: (i) Henry's Constant is not a universal constant. It depends on nature of gas and
temperature.
(ii) KH increases with increase in temperature therefore solubility of
gas decreases. 
(iii) According to Henry’s law : P g

Pg = KH Xg
(y = mx) Xg 

APPLICATIONS OF HENRY’S LAW


(1) To increase the solubility of CO2 in soda water and soft drinks, the bottles are sealed under
low temperature and high pressure.
(2) The people, suffer from ANOXIA at high altitude due to less partial pressure of oxygen
solubility of oxygen in Haemoglobin decreases.
(3) At hight altitude the partial pressure of oxygen is less than that of ground level. This leads
to low concentration of oxygen in the blood and tissues of people which are living at high
altitude.
Low blood oxygen causes climbers to become weak and unable to think clearly. Such
symptoms is known as a condition called 'ANOXIA'.
(4) Scuba divers intake oxygen dilute with He. Due to low solubility of He in the blood and to
avoid 'BENDS' as well as toxic effect of high concentration of nitrogen(N2).
Note :- When the scuba divers comes towards surface, their pressure gradually decreases. This
node06\B0BC-BD\Kota\Board Material\Chemistry\Booklets\CBSE\Part-1

release the dissolved gases and leads to the formation of bubbles of N2 in the blood. This block
capillaries and creates and medical condition which is known as 'BENDS', which is painful and
dangerous for life.
(III) SOLUBILITY OF LIQUID IN LIQUID (LIQUID-LIQUID SOLUTION) :
• The solution in which solute and solvent both are liquid
• The solubility of liquid-liquid solution depends upon vapour pressure.
E 5
Chemistry
ALLEN
VAPOUR PRESSURE
At a constant temperature, the pressure exerted by the vapours of a liquid on its surface when
they (liquid and its vapours) are in equilibrium, is known as vapour pressure.
Factors affecting the Vapour Pressure
(1) Nature of liquid–
1
VP 
IMFOA

(2) Temperature–
Vapour Pressure  Temperature
• Solubility of liquid-liquid solution can be explained by Raoult’s law.
Raoult’s law :
According to Raoult’s law “The partial vapour pressure of any component is proportional to its
mole fraction.”
 According to Raoult’s law
P1  X1 …..(1)
P1  P1o X1 …..(2)
P1 = Vapour pressure of component 1 
o
P = Vapour pressure of that component 1 at pure state
1
P1
X1 = Mole fraction of that component 1
RAOULT’S LAW FOR VOLATILE LIQUID SOLUTION X1 
(Binary liquid solution in which both component are volatile)
According to Raoult’s law “The total pressure of the solution for a binary component is equal to the
sum of partial vapour pressure of that component.”
Let us consider that in a binary solution there are two volatile components 1 and 2, are present which
vapour pressure p1, p2 and mole fraction X1, X2 respectively.
 According to Raoult’s law : The partial vapour pressure of any component in volatile solution is
proportional to its mole fraction.
node06\B0BC-BD\Kota\Board Material\Chemistry\Booklets\CBSE\Part-1

P1  X1

P1  P1o X1 ………(1)
y = mx
P 2  X2

P2  P2o X 2 ………(2)

6 E
CBSE
ALLEN
 PT = P1 + P 2

 PT  P1o X1  P2o X 2 ………(3)


X1 + X2 = 1
X1 = 1 – X2
 PT = P1o (1  X 2 )  P2o X 2
PT  P1o  P1o X 2  P2o X 2

PT  P1o  X 2 (P2o  P1o ) ……..(3)


y c x m

P20
Vapour pressure 

P10
P2o > P1o

X1=1 Mole fraction X1=0


X2=0 X2  X2=1

Note :
(1) Total vapour pressure of the solution varies linearly with the mole fraction of component.
(2) Component (1) is less volatile than component (2).
RAOULT’S LAW FOR NON-VOLATILE SOLUTE:
Solute : Glucose, sugar, Urea (Non-volatile)
Solvent : Water (volatile)
* When solute is non-volatile then only the solvent molecules are present in vapour phase and
contribute to vapour pressure.
According to Raoult’s law for a solution which contain a non-volatile solute – “If solute is non-
volatile then its partial vapour pressure is equivalent to zero, hence only solvent molecules are
contributes in total pressure.”
 PT = P1 + P2 ……..(1) P1 = P1o X1 vapour pressure of
node06\B0BC-BD\Kota\Board Material\Chemistry\Booklets\CBSE\Part-1

pure solvent
(Component 2 is non-volatile solute) P2 = P2o X 2  0

 PT = P1 + 0 PT
PT  P1o X1 ……..(2)
0 X1  1
y m x
Equation (2) is a mathematical expression of Raoult’s law for a solution which contains
non-volatile solute.
E 7
Chemistry
ALLEN
Note :
(i) The composition of vapour phase in equilibrium with the solution is determined by the
partial pressure of the components.
(ii) If Y1 andY2 are mole fraction of the component (1) & (2) in vapour phase.
P1 Po Y
P1 = Y1 PTotal  Y1 =  1 1
PTotal PT
Note : Raoult’s law becomes a special case of ‘Henry’s law’.
According to Henry’s law
Pg = KH Xg ……(1)
According to Raoult’s law
PA = PAo X A ……(2)
The partial pressure of the volatile component or gas is directly proportion to its mole fraction in
solution. Only the proportionality constant KH differ from PAo .
Thus Raoult’s law becomes a special case of Henry’s law which KH becomes equal to PAo .
IDEAL SOLUTIONS (mixture of two liquids A and B)
• A solution which obeys Raoult's law exactly at all concentrations and at all temperatures is
called an ideal solution.
• For ideal solutions; A-A interactions and B-B interactions = A-B interactions.
An ideal solution possesses the following characteristics :
• Volume change on mixing should be zero. Vmix = 0, i.e., (Vsolute + Vsolvent = Vsolution)
• Heat change on mixing should be zero. Hmix = 0 (Heat is neither absorbed nor evolved)
• There should be no chemical reaction between liquid A and liquid B.
• Ideal solution must obey Raoult's law at all concentrations.
PA  PA0 .X A , PB  PB0 .X B
or
P1  P10 .X1 , P2  P20 .X 2
• observed VP = calculated VP
• observed BP = calculated BP
• G < 0

P20
Vapour pressure 

P10
node06\B0BC-BD\Kota\Board Material\Chemistry\Booklets\CBSE\Part-1

P2o > P1o

X1=1 Mole fraction X1=0


X2=0 X2  X2=1

8 E
CBSE
ALLEN
Example
(i) Benzene and toluene (ii) CCl4 and SiCl4
(iii) n-hexane and n-heptane (iv) C2H5Br and C2H5Cl
(v) PhCl and PhBr (vi) n-butylchloride and n-butylbromide
NON-IDEAL SOLUTIONS
• Those solutions which do not obey Raoult's law are called non-ideal solutions.
• For non ideal solutions ; A–A interactions or B-B interactions  A-B interactions.
• For such solutions ; PA  PAo X A ; PB  PBo X B
• Non ideal solutions are formed when the components differ much in their structures and
polarities.
So  H mixing  0 and Vmixing  0
Deviation from Raoult’s law
• Non ideal solutions show either positive or negative deviations from Raoult's law.
• The vapour pressure of such solutions is either higher or lower than expected by Raoult’s
law.
i.e., PT = PAo  (PBo  PAo )X 2 or PT  (P2o  P1o )X 2  P1o
If it is higher then solution shows +ve deviation and if it is lower then it shows –ve
deviation from Raoult’s law.
(a) Non ideal solutions having positive deviation from Raoult's law.
• The solutions in which the vapour pressure comes out to be more then expected on the
basis of Raoult’s law. It is due to decrease in IMFOA between new component hence
vapour pressure of the solution increases.
Properties :
• In these solutions A-B interactions are less than A-A and B-B molecular interactions.
PA  PAo X A or P1  P1o X1

PB  PBo X B or P2  P2o X 2
• The total vapour pressure of the solution will be greater than the corresponding vapour
pressure expected in case of an ideal solution of same composition. i.e.
Ptotal  ( PAo X A  PBo X B ) or Ptotal  ( P1o X1  P2o X 2 )
•  Hmix > 0 ; endothermic dissolution ; heat is absorbed.
node06\B0BC-BD\Kota\Board Material\Chemistry\Booklets\CBSE\Part-1

• Vmix >0. ; volume increases after dissolution, i.e., (Vsolute + Vsolvent < Vsolution).
• 'A' and 'B' escape easily showing higher vapour pressure than the expected ideally.
(B.P.)th > (B.P.)exp ; (V.P.)cal. < (V.P.)obs.
• (S)mix = +ve , G = –ve

Entropy change in mixing is positive.

E 9
Chemistry
ALLEN
Vapour pressure of solution
PT > P1 + P2

P2o
P1o

Vapour Pressure

X1=1 Mole Fraction X1=0


X2=0 X2  X2=1
X1

Ex.1 Mixture of ethyl alcohol + cyclo hexane


On mixing the two components, cyclohexane comes between two alcohol molecules by
breaking its H-bond hence vapour pressure of solution increase and solution shows +ve
deviation from Raoult’s law.
H—O---H—O---H—O---
C2H5 C2H5 C2H5
C6H12 C6H12

Examples : (i) Ethanol and cyclohexane (v) CCl4 and Benzene (ix) Benzene and Acetone
(ii) Ethanol and Water (vi) CCl4 and Toluene (x) CS2 and Acetone
(iii) Ethanol and Acetone (vii) CCl4 and CHCl3 (xi) CS2 and Acetaldehyde
(iv) Methanol and H2O (viii) CCl4 and Methanol
(b) Non ideal solutions having negative deviation from Raoult's Law
• The solution in which the vapour pressure comes out to be less than expected by Raoult’s
law. It is due to increase in IMFOA between the new components hence vapour pressure of
the solution decreases.
Properties :
• In these solutions the AB interactions are stronger than the AA and BB molecular
interactions.
PA  PAo X A ; PB  PBo X B
or
node06\B0BC-BD\Kota\Board Material\Chemistry\Booklets\CBSE\Part-1

P1  P X1
1
o
; P2  P2o X 2
• Ptotal  (PAo X A  PBo X B ) or Ptotal  (P1o X1 P2o X 2 )
Total vapour pressure is less than sum of individual vapour pressure.
• Hmix < 0 ; exothermic dissolution heat is evolved.
• Vmix < 0 ; volume decreases during dissolution, i.e., (Vsolute + Vsolvent > Vsolution).

10 E
CBSE
ALLEN
• Escaping tendency of both components 'A' and 'B' is lowered showing lower vapour
pressure than expected ideally.

P2°
P1°

Vapour Pressure

X1=1 Mole Fraction X1=0


X2=0 X2  X2=1
 X1

Ex.1 Mixture of acetone + Chloroform


A mixture of chloroform and acetone forms a solution with negative deviation from
Raoult’s law. This is because chloroform molecule is able to form hydrogen bond with
acetone molecules. as shown. This decreases the escaping tendency of molecules for each
components and consequently the vapour pressure decreases. Resulting in negative
deviation from Raoult’s law.
H3C  .. Cl
C=O . . ---H — C Cl
H3C Cl
Mixture of Phenol + Aniline
On mixing the phenol in Aniline, it forms H-bonding hence IMFOA, between the two
components increase and vapour pressure decrease.
H H
H—O - - - H — N - - - H — O - - - H — N - - -

Ex.2 (i) CHCl3 and CH3COCH3 (vi) H2O and HCl


(ii) CHCl3 and C6H6 (vii) H2O and HNO3
(iii) CHCl3 and C2H5OC2H5 (viii) CH3COOH and CH3OH
(iv) CHCl3 and HNO3 (ix) CH3COOH and C5H5N
(v) CHCl3 and CH3COOH (x) CH3COCH3 and Aniline
AZEOTROPIC MIXTURES :
node06\B0BC-BD\Kota\Board Material\Chemistry\Booklets\CBSE\Part-1

Some liquids on mixing, form azeotropes which are binary mixtures having the same
composition in liquid and vapour phase and boil at a constant temperature, the liquid and vapour
have the same composition, and no further separation occurs.
Components form azeotrope can't be separated by fractional distillation but can be separated by
azeotropic distillation.
Solutions showing Positive deviation form minimum boiling azeotrope and solutions showing
negative deviation form maximum boiling azeotrope.
E 11
Chemistry
ALLEN
 There are two types of azeotropes :
(1) Minimum boiling azeotrope
The solutions which show a large positive deviation from Raoult’s law form minimum
boiling azeotrope at a specific composition. For example, ethanol-water mixture (obtained
by fermentation of sugars) on fractional distillation gives a solution containing
approximately 95.5% v/v ethanol.
(2) Maximum boiling azeotrope
The solutions that show large negative deviation from Raoult’s law form maximum boiling
azeotrope at a specific composition. Ex. nitric acid and water. This azeotrope has the
approximate composition, 68% nitric acid and 32% water by mass with a boiling point of
393.5 K.
COLLIGATIVE PROPERTIES OF DILUTE SOLUTIONS (COLLIGATIVE PROPERTY)
Those physical properties of a solution which depends upon the relative number of particles of
solute and do not depend on nature of solute particles are called colligative properties.
CP  Number of solute particles
 Number of molecules (in the solution of non-electrolyte)
 Number of ions (in the solution of electrolytes)
 Number of moles of solute
 Mole fraction of solute
The following four properties are colligative property :
(1) Relative lowering in vapour pressure of solvent
(2) Elevation in boiling point (Tb) of solvent
(3) Depression in freezing point (Tf) of solvent
(4) Osmotic pressure ( or P) of solution
(a) Relative Lowering in Vapour Pressure
• When a non-volatile solute is dissolved in a pure solvent, the vapour pressure of the
solvent is lowered i.e. the vapour pressure of a solution is always lower than that of
pure solvent, because the escaping tendency of solvent molecules decreases
• Due to lesser solvent molecules per unit surface area.
RELATION BETWEEN MOLAR MASS OF NON-VOLATILE SOLUTE AND
RELATIVE LOWERING IN VAPOUR PRESSURE.
If at a certain temperature P1° is the vapour pressure of pure solvent, and Ps is the
vapour pressure of solution then
Lowering of Vapour pressure = P1o  PS ……..(1)
node06\B0BC-BD\Kota\Board Material\Chemistry\Booklets\CBSE\Part-1

P1o  PS
 Relative lowering of vapour pressure = ……..(2)
P1o
According to Raoult's law : 
When a non-volatile and non-electrolyte solute is dissolved in a pure solvent then
relative lowering of vapour pressure of solvent is equal to the mole fraction of the
solute, dissolved in the solution.
12 E
CBSE
ALLEN
P1o –PS
 = X2
P1o Component (2) is solute
P1o –PS n2
o
=
P1 n1 + n 2 Component (1) is solvbent
For a very dilute solution n2 << n1
P1o  PS n 2

P1o n1

P1o  PS w 2 / M 2
 
P1o w1 / M1

• Relative lowering depends upon relative number of solute particles. Therefore it is


called colligative properties.
(2) Elevation in Boiling Point
• The boiling point of a liquid is that temperature at which its vapour pressure
becomes equal to the atmospheric pressure.
• When a non-volatile solute is dissolved in a pure solvent, its vapour pressure is
decreased and boiling point increases. The difference of boiling points of the solution
and pure solvent is called elevation in boiling point.(Tb)
RELATION IN BETWEEN BOILING POINT ELEVATION AND MOLAR MASS OF
NON-VOLATILE SOLUTE :
There are a vapour pressure – temperature curve shows for pure solvent and non-volatile solute
containing solution.
Boiling point of pure solvent = Tbo
Boiling point of solution = Tb
Elevation in boiling point  Tb  = Tb – Tbo …….(1)

Experiments have shows that for dilute solution elevation in boiling point (Tb) is directly proportional
to molal concentration of the solute in solution.
1.013 bar
Tb  m or 1 Atm
Tb  K b .m ………(2)
Vapour pressure
node06\B0BC-BD\Kota\Board Material\Chemistry\Booklets\CBSE\Part-1

Kb = molal elevation constant or Ebullioscopic constant


(Unit = K.kg mol–1)
Kb The Tb produced when one mole of a non-volatile
Tb0 Tb
solute is dissolved in 1 kg of solvent. Tb
If w2 gm of solute of molar mass M2 is dissolved in w1 gm of Temperature
solvent then

E 13
Chemistry
ALLEN
w 2 / M2
Molality (m) =
w1 /1000
1000  w 2
 m= …….(3)
M 2  w1
Put the value of ‘m’ in eq. (2)
1000 × w 2
Tb = K b × ……(4)
M 2 × w1

Eq. (4) is relation in between molar mass of non-volatile solute and boiling point elevation.
• Kb depends only on nature of solvent which can be explained by thermodynamic relation.

M w R  Tb0 
2

Kb =
1000H vap

Where Tb0 = Boiling point of pure solvent.


Mw = Molar mass of pure solvent.
 Hvap = Enthalpy of vapourisation per mole of solvent
R = Gas constant
For water, Kb = 0.52 K kg mol–1
(3) DEPRESSION IN FREEZING POINT
• The freezing point of a liquid is that temperature at which the liquid and its solid state exist in
equilibrium with each other.
• On addition of non-volatile solute, vapour pressure of the solution decreases, liquid solution is to
freeze more to equalize its vapour pressure to the vapour pressure of the solid. Decrease in
freezing point of solution due to addition of non-volatile solute is known as depression in
freezing point.
• It may be defined as the temperature at which the liquid and solid states of a substance have the
same vapour pressure.
Relation in between molar mass of non-volatile solute and
depression in freezing point :
There are a vapour pressure – temperature curve shows for pure solvent and
non-volatile solute containing solution.
Vapour pressure

Freezing point of pure solvent = Tfo


node06\B0BC-BD\Kota\Board Material\Chemistry\Booklets\CBSE\Part-1

Freezing point of solution = Tf


Depression in freezing point  Tf  Tfo  Tf …….(1) Tf
Tf Tf0
The difference in freezing point of pure solvent and solution is known as
depression in freezing point. Temperature (K)

Experiments have shows that for dilute solution Tf is directly proportional to molal concentration of
the solute in solution.

14 E
CBSE
ALLEN
Tf  m
Tf  K f .m ………(2)
Kf = molal depression constant or Cryoscopic constant.
(Unit = K.kg mol–1)
Kf The Tf produced when 1 mole of a non-volatile solute dissolved in 1kg of solvent.
If w2 gm of solute of molar mass M2 is dissolved in w1 gm of solvent then
w 2 / M2
Molality (m) =
w1 /1000
1000  w 2
 m= …….(3)
M 2  w1
Put the value of ‘m’ in eq. (2)
1000 × w 2
Tf = K f × ……(4)
M 2 × w1
Eq. (4) is relation in between molar mass of non-volatile solute and depression in freezing point.
• Kf depends only on nature of solvent which can be explained by thermodynamic relation

R  Tf0  M w
2

Kf =
1000 H f
Where T0f = Freezing point of pure solvent
Mw = Molar mass of pure solvent
Hf = Enthalpy of fusion per mole of solvent
For water, Kf = 1.86 K kg mol–1
(d) Osmosis and Osmotic Pressure
Osmosis : Osmosis is defined as the spontaneous net flow of solvent molecules through
semipermeable membrane from a solvent to a solution or from a dilute solution to a concentrated
solution.
node06\B0BC-BD\Kota\Board Material\Chemistry\Booklets\CBSE\Part-1

Raisin
, sugar solution, shrink
10% 20%
H.C. (L.C.) 10% (H.C.)
20%

• Endo osmosis • exo osmosis


• Swelling • shrink

E 15
Chemistry
ALLEN
Osmotic Pressure (p or )
• The external pressure which must be applied on the p

solution in order to stop the flow of solvent into the h  = hdg


solution through semipermeable membrane is known
as osmotic pressure Solvent
or
Solution
• Hydrostatic pressure developed in a vertical column
when solution and solvent are separated by SPM. Semipermeable
membrane
Osmotic pressure = hydrostatic pressure  Level of solution rises in the thistle
funnel due to osmosis of solvent
 = hdg

where h = increase in level in the tube of unit cross section


d = density of solution
g = acceleration due to gravity
Examples of osmosis and osmotic pressure:
• The climbing of water up a tall tree from the soil is also due to osmosis.
• Bursting of red-blood cells when placed in water, is due to the movement of water into the cell.
• A raw mango when placed in a concentrated salt solution shrinks due to the movement of water
from the mango to the salt solution due to osmosis.
• When blood cells are placed in water containing less than 0.9% salt (mass to volume), they swell
up.
• Preservation of meat by adding salt and fruits/fruit juices by adding sugar is due to the loss of
water by any bacterium due to osmosis.
Van't Hoff law for Dilute Solution
According to it Gas equation PV = nRT is also followed by dilute solution when pressure of gas is
replaced by osmotic pressure of solution.
PV = nRT
  = osmotic pressure of solution (atm)
V = nRT
V = volume of solution (L)
n = moles of solute
R = (S) gas constant / solution constant = 0.0821 L atm mol–1K–1; 0.083 L bar mol–1K–1
n
    RT   CRT
node06\B0BC-BD\Kota\Board Material\Chemistry\Booklets\CBSE\Part-1

V
w 2 RT
 
M2 V
At constant temperature   C   is a colligative property.

• On the basis of osmotic pressure solution can be classified by following ways :

16 E
CBSE
ALLEN
Conditions to determine the accurate osmotic pressure :
(i) The temperature of system should be constant i.e., room temperature (25°C)
(ii) The solution should be dilute
(iii) The solute particle do not undergoes associate or dissociate in side the solution.
(iv) The spm should be perfect.
ISOTONIC, HYPERTONIC AND HYPOTONIC SOLUTIONS :
(i) Isotonic solutions : Solutions having same osmotic pressure are called isotonic solution.

1  2 ; primary condition

C1RT = C2RT (at same temperature)

C1  C2 (secondary condition) ; means

n1 n 2
 ; such solutions are known as isotonic
V1 V2

(ii) Hypertonic solution : The solution having higher concentration than other.
(iii) Hypotonic solution : The solution having lower concentration than other.
REVERSE OSMOSIS
If external pressure greater than osmotic pressure is applied, the flow of solvent molecules can be
made to proceed from solution towards pure solvent, i.e., in reverse direction of the ordinary
osmosis. Reverse osmosis is used for the desalination of sea water for getting fresh drinking
water.
Piston
(P) Pressure > 

Fresh water Salt water


Water
outlet
SPM

• Phenomena on the basis of osmosis People taking a lot of salt or salty food experience water
retention in tissue cells and intercellular spaces because of osmosis. This resulting puffiness or
swelling is called edema.
ABNORMAL COLLIGATIVE PROPERTIES
node06\B0BC-BD\Kota\Board Material\Chemistry\Booklets\CBSE\Part-1

• It has been observed that difference in the observed and calculated molecular masses of
solute is due to association or dissociation of solute molecules in solution. It results in a
change in the number of particles in solution.
Molar masses that are either lower or higher than the calculated (expected) or normal value are
called abnormal molar mass.

E 17
Chemistry
ALLEN
• van't Hoff factor (i) is defined as the ratio of the experimental or observed value of the
colligative property to the calculated value of the colligative property.
number of particles after dissociation or association
• i
number of particles before dissociation or association
observed colligative properties
=
calculated colligative properties
calculated molecular mass

observed molecular mass
• If i = 1 Neither dissociation nor association ; Ex. glucose, urea, sugar
• if i > 1 Dissociation will occur ; Ex. NaCl, KCl, BaCl2
• if i < 1 Association will occur ; Ex. CH3COOH, PhCOOH in benzene
CASE I :
DISSOCIATION OF SOLUTE : Molecules of electrolytes undergo ionization or dissociation
in polar solvents to give two or more particles in solution. This dissociation results in an increase
in the total number of particles, and therefore the value of colligative properties of such solutions
will be higher. As the colligative properties are inversely related to molecular weight, so the
molecular weight of ionizable solute will be less than the theoretical value.
Number of solute particles in solution increses.
• observed / experimental C.P. > calculated C.P. (normal)
• observed /experimental molecular weight of solute < calculated (normal) moleclar weight
of solute 
1
     C.P. 
molecular weight of solute
• Calculation of 'i' : Let solute be AxBy (electrolyte)
AxBy  xAy+ + yBx– ; x + y = n (total number of ions)
Initially 1 mol 0 0
After dissociation (1–) x y
Total number of solute particles = 1– + x + y
= 1 – + (x+y)
= (1– + n)mol
node06\B0BC-BD\Kota\Board Material\Chemistry\Booklets\CBSE\Part-1

Observed colligative property is proportional to observed number of solute particles (1– + n)
number of particles after dissociation 1    n
i =
number of particles before dissociation 1
i 1
i  1  (n  1) ; 
n 1
where  is the degree of dissociation

18 E
CBSE
ALLEN
• For strong electrolytes
If  = 1 or 100% then i  n
Ex. NaCl  i =2;  = 100%

K2SO4  i = 3  = 100%

Ex. K4[Fe(CN)6]  i =5 for  = 100%


For  = 50% i = 1 + (n–1)
i =3
If K4[Fe(CN)6] 50% ionised in aqueous solution.
 = 50% = 0.5
i = 1 –  + n
  = 1 – 0.5 + 5 × 0.5
= 0.5 + 2.5 = 3
CASE II :
ASSOCIATION OF SOLUTE : The formation of a bigger molecule by the union of two, three
or more solute molecules is called association. As a result, the total number of particles in
solution becomes less than the number of molecules initially dissolved in the solution and hence
the colligative properties will have lower value. As the molar mass of solute is inversely
proportional to the colligative properties, so the molar mass of solute will be greater than
theoretical value.
Number of solute particles in solution decreases.
• Observed /experimental C.P. < calculated C.P.
• Observed / experimental molecular weight of solute > normal molecular weight of solute

1
   C.P.  ; i < 1 for association.
Molecular weight of solute
• Calculation of i
nA –––––– [A]n
Initially 1 0
node06\B0BC-BD\Kota\Board Material\Chemistry\Booklets\CBSE\Part-1


After association (1–)
n
 
Total number of solute particles = 1     mol
 n

 
Observed C.P.  observed number of solute particles  1    
 n

E 19
Chemistry
ALLEN

1  
number of particles after association n
Van't hoff factor (i) = ; i
number of particle before association 1

 n(i  1)
i  1   
n 1 n
  = degree of association

n = number of solute particles which are associated.


1
• If  = 100% or 1 or  is not specified then i 
n

Some Important facts :


• Intramolecular H-bonding lowers boiling point.
• Benzoic acid and acetic acid dimerises in benzene.
• When egg is kept in saturated salt solution after removing the hard shell using dil. HCl, egg will
shrink due to osmosis.
• CaCl2 acts as a non-volatile solute and results depression in freezing point. Thus, snow is
reduced (as it melts) and prevents blocking of roads in cold region.
• Ethylene glycol is mixed with radiator water to decrease the freezing point of water in cold
region.
• Camphor is used for molecular mass determination due to its volatile nature.
• To increase the solubility of CO2 in soft drinks, the bottle is sealed under low temperature and
high pressure.
• Helium-oxygen mixture is used by deep sea divers because of its low solubility in blood.

node06\B0BC-BD\Kota\Board Material\Chemistry\Booklets\CBSE\Part-1

20 E
CBSE
ALLEN
EXERCISE-1 INTEXT QUESTIONS

1. Calculate the mass percentage of benzene (C6H6) and carbon tetrachloride (CCl4) if 22g of
benzene is dissolved in 122 g of carbon tetrachloride.
Ans. Total mass = 22 + 122 = 144 g
22×100
Mass % of benzene = = 15.28%
144
122
Mass % of CCl4 = ×100 = 84.72%
144
2. Calculate the mole fraction of benzene in a solution containing 30% by mass in carbon
tetrachloride.
Ans. Mass of benzene = 30 g
Mass of carbon tetrachloride = 100 – 30 = 70g
Molar mass of benzene (C6H6) = 78 g/mol
Molar mass of carbon tetrachloride = 154 g/mol
w 30
Moles of benzene = =  0.385
m 78
w 70
Moles of carbon tetrachloride = = = 0.455
m 154
Total moles = 0.385 + 0.455 = 0.84
moles of benzene 0.385
Mole fraction of benzene = = = 0.458
total moles 0.84
Mole fraction of carbon tetrachloride = 1 – 0.458 = 0.542
3. Calculate the molarity of each of the following solutions :
(a) 30 g of Co(NO3)2 . 6H2O in 4.3 L of solution,
(b) 30 mL of 0.5 M H2SO4 diluted to 500 mL.
Ans. (a) Molar mass of Co(NO3)2.6H2O = 291 g/mol
w 30
Molarity = = = 0.024 M
m× V(lit) 291× 4.3
(b) M1V1 = M2V2,
M1 = 0.5, V1 = 30 mL, M2 = ?, V2 = 500 mL
0.5 × 30 = M2 × 500
node06\B0BC-BD\Kota\Board Material\Chemistry\Booklets\CBSE\Part-1

or M2 = 0.03 M
4. Calculate the mass of urea (NH2CONH2) required in making 2.5 kg of 0.25 molal aqueous
solution.
Ans. 0.25 molal aqueous solution means
Moles of urea = 0.25, Mass of water = 1 kg = 1000 g
0.25 mole urea = 0.25 × 60 = 15 gram

E 21
Chemistry
ALLEN
Total mass of solution = 1000 + 15 = 1015g = 1.015 kg
 1.015 kg solution contains urea = 15 g
15× 2.5
 2.5 kg solution contains urea = = 36.94 gram
1.015
5. Calculate (a) molality, (b) molarity, (c) mole fraction of KI if the density of 20% (mass/mass)
aqueous KI is 1.202 g/mL.
Ans. 20% mass/mass aqueous KI means
Mass of KI = 20 g, Mass of water = 80g,
Mass of solution = 100 g
(a) For molality :
Molar mass of KI = 39 + 127 = 166
w 20
Molality = = = 1.51 mol kg –1
m× mass of solvent in kg 166×80×10-3
(b) For Molarity :
mass 100
Volume of solution = = = 83.2 ml.
density 1.202
w 20
Molarity = = = 1.45 mol L–1
m× V(lit) 166×83.2×10-3
w 20
(c) Moles of KI = = = 0.12
m 166
w 80
Moles of water = = = 4.44
m 18
Total moles = 0.120 + 4.444 = 4.56
0.12
Mole fraction of KI = = 0.0263
4.56
6. H2S, a toxic gas with rotten egg smell, is used for the qualitative analysis. If the solubility of H2S
in water at STP is 0.195 m, calculate Henry's law constant.
Mass of solvent (water) = 1kg = 1000 g.
Ans. Solubility = 0.195 mole in one kg of water
Mass of solvent (water) = 1kg = 1000 g
w 1000
Moles of water = = = 55.55
m 18
Total moles = 0.195 + 55.55
node06\B0BC-BD\Kota\Board Material\Chemistry\Booklets\CBSE\Part-1

 
Mole fraction of H2S X H2S in solution =
0.195
0.195 + 55.55
= 0.0035

Pressure of H2S at STP = 0.987 bar.


Since partial pressure of the gas is given,
p H2S = KH × X H2S
or 0.987 = KH × 0.0035 or KH = 282 bar

22 E
CBSE
ALLEN
7. Henry's law constant for CO2 in water is 1.67 × 108 Pa at 298 K. Calculate the quantity of CO2 in
500 mL of soda water when packed under 2.5 atmospheric CO2 pressure at 298 K.
Ans. KH = 1.67 × 108 Pa,
PCO2 = 2.5 atm = 2.5 × 101325 Pa

PCO2 = KH × X CO2

or 2.5 × 101325 = 1.67 × 108 × X CO2

or X CO2 = 1.517 × 10–3

n CO2 n CO2
X CO2 = = = 1.517×103
n H2O  n CO2 n H2O

n CO2 is neglected in denominator in comparison to n H2O .

Water = 500 mL = 500 g,


500
n H2O   27.78 mol
18
n CO2
= 1.517×103
n H2O

n CO2
or = 1.517×103
27.78
or n CO2 = 42.14 × 10–3

Mass of CO2 = moles × molar mass


= 42.14 × 10–3 × 44 = 1.854 gram
8. The vapour pressure of pure liquids A and B are 450 and 700 mm Hg respectively at 350 K. Find
out the composition of the liquid mixture if total vapour pressure is 600 mm Hg. Also find the
composition of the vapour phase.
Ans. PA0 = 450 mm Hg, PB0 = 700 mm Hg, PTotal = 600 mm Hg
Solution contains two liquids,

Applying Raoult's law liquids : Ps = PAo X A + PBo X B


node06\B0BC-BD\Kota\Board Material\Chemistry\Booklets\CBSE\Part-1

or Ps = PAo X A + PBo (1 – X A ) [ XA + XB = 1]

or Ps = PBo + X A (PAo – PBo )

600 = 700 + XA (450 – 700)


100
or XA = = 0.40
250

E 23
Chemistry
ALLEN
 XB = 1 – XA = 1 – 0.40 = 0.60
PA = PA0 X A = 450 × 0.40 = 180 mm Hg
PB = PBo X B = 700 × 0.60 = 420 mm Hg
Total vapour pressure = 180 + 420 = 600 mm Hg
PA 180
Mole fraction of A in vapour phase = = = 0.30
Ptotal 600
PB 420
Mole fraction of B in vapour phase = = = 0.70
Ptotal 600
9. Vapour pressure of pure water at 298 K is 23.8 mm Hg. 50 gram of urea (NH2CONH2)
is dissolved in 850 g of water. Calculate the vapour pressure of water for this solution and its
relative lowering.
PAo – PS w × M
Ans. Solution contains non-volatile solid. Our aim is to calculate Ps and =
PS m× W
PAo = 23.8 mm Hg , w = 50 g, W = 850 g, Ps = ?, M = 18 g/mol for H2O, m = 60 g/mol for urea

PAo – PS w × M
=
PS m× W
23.8 – Ps 50×18 3
or = =  0.0176
Ps 60×850 170
or Ps = 23.4 mm Hg
PAo  PS 23.8 – 23.387
Again, relative lowering in V.P. = = = 0.017
PS 23.4
10. Boiling point of water at 750 mm Hg is 99.63ºC. How much sucrose is to be added to 500 g of
water such that it boils at 100ºC ? Kb = 0.52 K kg/mol.
Ans. Tb = 100 – 99.63 = 0.37 K
w = ?, W = 500 g, M = 342 g/mol for sucrose,
Kb = 0.52 K kg/mol
K b × w ×1000
Tb =
M×W
0.52× w ×1000
or 0.37 =
node06\B0BC-BD\Kota\Board Material\Chemistry\Booklets\CBSE\Part-1

342×500
or w = 121.67 g
11. Calculate the mass of ascorbic acid (vitamin C, C6H8O6) to be dissolved in 75g acetic acid to
lower its melting point by 1.5ºC. Kf = 3.9 K kg/mol.
K f × w ×1000
Ans. Tf =
M×W
Formula for lowering in freezing point or melting point is same.
24 E
CBSE
ALLEN
Tf = 1.5 K, w = ?, W = 75g,
M for ascorbic acid = 176 g/mol
3.9× w ×1000
1.5 = w = 5.08 g
176× 75
12. Calculate the osmotic pressure in pascals exerted by a solution prepared by dissolving 1.0g of
polymer of molar mass 185000 in 450 mL of water at 37ºC.
wRT
Ans. Since mass of solute is given, V =
M
 = ?, V = 0.450 litre, w = 1g, M = 185000,
R = 8.314 kPa LK–1 mol–1 8.314 × 103 Pa LK–1 mol–1
T = 37 + 273 = 310 K
1×8.314×103 ×310
 × 0.450 =
185000
or  = 30.96 Pa  31 Pa (approximately)
node06\B0BC-BD\Kota\Board Material\Chemistry\Booklets\CBSE\Part-1

E 25
Chemistry
ALLEN
EXERCISE-2 NCERT EXERCISE

1. Define the term solution. How many types of solutions are formed? Write briefly about each type
with an example.
Ans. Homogeneous mixtures of two or more than two components are known as solutions.
There are three types of solutions.
(i) Gaseous solution : The solution in which the solvent is a gas in called a gaseous solution.
In these solutions, the solute may be liquid, solid, or gas. For example, a mixture of oxygen
and nitrogen gas is a gaseous solution.
(ii) Liquid solution : The solution in which the solvent is a liquid is known as a liquid
solution. The solute in these solutions may be gas, liquid, or solid.
For example, a solution of ethanol in water is a liquid solution.
(iii) Solid solution : The solution in which the solvent is a solid known as a solid solution. The
solute may be gas, liquid or solid. For example, a solution of copper in gold is a solid
solution.
2. Give an example of solid solutions in which the solute is a gas.
Ans. A solution of hydrogen in palladium is a solid solution in which the solute is a gas.
3. Define the following terms :
(i) Mole fraction (ii) Molality
(iii) Molarity (iv) Mass percentage
Ans. (i) Mole fraction : The mole fraction of a component in a mixture is defined as the ratio of the
number of moles of the component to the total number of moles of all the components in
the mixture.
Number of moles of the component
i.e., Mole fraction of a component = .
Total number of moles of all components

Mole fraction is denoted by 'x'.


If in a binary solution, the number of moles of the solute and the solvent are nA and nB
nA
respectively, then the mole fraction of the solute in the solution is given by, x A  .
nA  nB
node06\B0BC-BD\Kota\Board Material\Chemistry\Booklets\CBSE\Part-1

nB
Similarly, the mole fraction of the solvent in the solution is given as : x B  .
nA  nB

(ii) Molality : Molality (m) is defined as the number of moles of the solute per kilogram of
the solvent. It is expressed as :
Moles of solute
Molality (m) =
Mass of solvent in kg
26 E
CBSE
ALLEN
(iii) Molarity : Molarity (M) is defined as the number of moles of the solute dissolved in one
Litre of solution. It is expressed as :
Moles of solute
Molarity (M) =
Volume of solvent in Litre
(iv) Mass percentage : The mass percentage of a component of a solution is defined as the
mass of the solute in grams present in 100g of the solution. It is expressed as :
Mass of component in solution
Mass % of a component =  100
Total mass of solution
4. Concentrated nitric acid used in laboratory work is 68% nitric acid by mass in aqueous
solution. What should be the molarity of such a sample of the acid if the density of the solution is
1.504 g mL–1 ?
Ans. Concentrated nitric acid used in laboratory work is 68% nitric acid by mass in an aqueous
solution. This means that 68 g of nitric acid is dissolved in 100 g of the solution.
Molar mass of nitric acid (HNO3) = 1 × 1 + 14 + 3 × 16 = 63 g mol–1
68
Then, number of moles of HNO3 = mol = 1.079 mol
63
Given, Density of solution = 1.504 g mL–1
100
 Volume of 100 g solution = mL = 66.49 mL = 66.49 × 10–3 L
1.504
1.079
Molarity of solution = = 16.23 M
66.49 103 L
5. A solution of glucose in water is labelled as 10% (w/w), what would be the molality and mole
fraction of each component in the solution? If the density of solution is 1.2 g mL–1, then what
shall be the molarity of the solution?
Ans. 10% w/w solution of glucose in water means that 10 g of glucose in present in 100g of the
solution i.e., 10 g of glucose is present in (100 – 10) g = 90 g of water.
Molar mass of glucose (C6H12O6) = 6 × 12 + 12 × 1 + 6 × 16 = 180 g mol–1
10
Then, number of moles of glucose = mol = 0.056 mol
180
0.056
 Molality of solution = = 0.62 m
0.09 kg
node06\B0BC-BD\Kota\Board Material\Chemistry\Booklets\CBSE\Part-1

90g
Number of moles of water = = 5 mol
18 gmol1

Mole fraction of glucose  x g  


0.056
 = 0.011
0.056  5
And, mole fraction of water xw = 1 – xg = 1 – 0.011 = 0.989

E 27
Chemistry
ALLEN
If the density of the solution is 1.2 g mL–1, then the volume of the 100 g solution can be given as :
100 g
V= 1
= 83.33 mL = 83.33 × 10–3 L
1.2 g mL
0.056 mol
 Molarity of the solution = = 0.67 M
83.33 103 L
6. How many mL of 0.1 M HCl are required to react completely with 1 g mixture of Na2CO3 and
NaHCO3 containing equimolar amounts of both ?
Ans. Let the amount of Na2CO3 in the mixture be x g.
Then, the amount of NaHCO3 in the mixture is (1 – x) g.
Molar mass of Na2CO3 = 2 × 23 + 1 × 12 + 3 × 16 = 106 g mol–1
x
 Number of moles Na2CO3 = mol
106
Molar mass of NaHCO3 = 1 × 23 + 1 × 1 × 12 + 3 × 16 = 84 g mol–1
1 x
 Number of moles NaHCO3 = mol
84
x 1 x
According to the question,   84x = 106 – 106x
106 84
 190x = 106  x = 0.5579.
0.5579
Therefore, number of moles of Na2CO3 = mol = 0.0053 mol
106
1  0.5579
And, number of moles of NaHCO3 = = 0.0053 mol
84
HCl reacts with Na2CO3 and NaHCO3 according to the following equation.
2HCl  Na 2 CO3 
 2NaCl  H 2 O  CO 2
2mol 1mol

HCl  NaHCO3 
 NaCl  H 2 O  CO 2
1mol 1mol

1 mol of Na2CO3 reacts with 2 mol of HCl.


Therefore, 0.0053 mol of Na2CO3 reacts with HCl = 2 × 0.0053 mol = 0.0106 mol.
Similarly, 1 mol of NaHCO3 reacts with 1 mol of HCl.
Therefore, 0.0053 mol of NaHCO3 reacts with 0.0053 mol of HCl.
node06\B0BC-BD\Kota\Board Material\Chemistry\Booklets\CBSE\Part-1

Total moles of HCl required = (0.0106 + 0.0053) mol = 0.0159 mol


0.1 mol of HCl is preset in 1000 mL of the solution.
1000× 0.0159
Therefore 0.0159 mol of HCl is present in mol = 159 mL of the solution
0.1
Hence,159 mL of 0.1 M of HCl is required to react completely with 1 g of mixture of Na2CO3
and NaHCO3 containing equimolar amounts of both.

28 E
CBSE
ALLEN
7. A solution is obtained by mixing 300 g of 25% solution and 400 g of 40% solution by mass.
Calculate the mass percentage of the resulting solution.
Ans. Total amount of solute present in the mixture is given by.
25 40
300× + 400× = 75 + 160 = 235 g
100 100
Total amount of solution = 300 + 400 = 700 g
Therefore, mass percentage (w/w) of the solute in the resulting solution
235
=  100% = 33.57%
700
And, mass percentage (w/w) of the solvent in the resulting solution,
= (100 – 33.57%) = 66.43%
8. An antifreeze solution is prepared from 222.6 g of enthylene glycol (C2H6O2) and 200 g of water.
Calculate the molality of the solution. If the density of the solution is 1.072 g mL–1, then what
shall be the molarity of the solution?
Ans. Molar mass of ethylene glycol [C2H6O2] = 2 × 12 + 6 × 1 + 2 × 16 = 62 gmol–1
Mass of solvent (H2O) = 200 g = 0.200 kg
222.6g
Number of moles of enthylene glycol = = 3.59 mol
62 gmol1

3.59 mol
Therefore, molality of the solution = = 17.95 m
0.200 kg

Total mass of the solution = (222.6 + 200) g = 422.6 g


Given,
Density of the solution = 1.072 g mL–1
422.6 g
 Volume of the solution = 1
= 394.22 mL = 0.3942 × 10–3 L
1.072 g mL

3.59 mol
 Molarity of the solution = = 9.10 M
0.39422  103 L
9. A sample of drinking water was found to be severely contaminated with chloroform (CHCl3)
node06\B0BC-BD\Kota\Board Material\Chemistry\Booklets\CBSE\Part-1

supposed to be a carcinogen. The level of contamination was 15 ppm (by mass) :


(i) express this in percent by mass
(ii) determine the molality of chloroform in the water sample.
Ans. (i) 15 ppm (by mass) means 15 parts per million (106) of the solution.
15
Therefore, percent by mass = 100% =1.5 ×10–3%
106

E 29
Chemistry
ALLEN
(ii) Molar mass of chloroform (CHCl3) = 1 × 12 + 1 × 1 + 3 × 35.5 = 119.5 g mol–1
Now, according to the question,
15 g of chloroform is present in 106 g of the solution.
i.e., 15 g chloroform is present in (106 – 15)  106 g of water.
15
mol
 Molality of the solution = 119.5 = 1.25 × 10–4 m
106 ×10-3 kg
10. What role does the molecular interaction play in a solution of alcohol and water?
Ans. In pure alcohol and water, the molecules are held tightly by a strong hydrogen bonding. The
interaction between the molecules of alcohol and water is weaker than alcohol-alcohol and water-
water interactions. As a result, when alcohol and water are mixed, the intermolecular interactions
become weaker and the molecules can easily escape. This increase the vapour pressure of the
solution, which in turn lowers the boiling point of the resulting solution.
11. Why do gases always tend to be less soluble in liquids as the temperature is raised ?
Ans. Solubility of gases in liquids decreases with an increase in temperature. This is because
dissolution of gases in liquids is an exothermic process.
Gas + Liquid  Solution + Heat
Therefore, when the temperature is increased, heat is supplied and the equilibrium shifts
backwards, thereby decreasing the solubility of gases.
12. State Henry's law and mention some important applications?
Ans. Henry's law states that partial pressure of a gas in the vapour phase is proportional to the
mole fraction of the gas in the solution. If p is the partial pressure of the gas in the vapour
phase and x is the mole fraction of the gas, then Henry's law can be expressed as :
Pg = KHXg
where, KH is Henry's law constant
Some important applications of Henry's law are mentioned below.
(i) Bottles are sealed under high pressure to increase the solubility of CO2 in soft drinks and
soda water.
(ii) Henry's law states that the solubility of gases increases with an increase in pressure.
Therefore, when a scuba diver deep into the sea, the increased sea pressure causes the
nitrogen present in air to dissolve in his blood in great amounts. As a result, when he comes
back to the surface, the solubility of nitrogen again decreases and the dissolved gas is
released, leading to the formation of nitrogen bubbles in the blood. This results in the
blockage of capillaries and leads to a medical condition known as 'bends'..
node06\B0BC-BD\Kota\Board Material\Chemistry\Booklets\CBSE\Part-1

Hence, the oxygen tanks used by scuba divers are filled with air and diluted with helium to
avoid bends.
(iii) The concentration of oxygen is low in the blood and tissue of people living at high altitudes
such as climbers. This is because at high altitudes, partial pressure of oxygen is less than
that at ground level. Low-blood oxygen cause climbers to become weak and disables them
from thinking clearly. These are symptoms of anoxia.

30 E
CBSE
ALLEN
13. The partial pressure of ethane over a solution containing 6.56 × 10–3 g of ethane is 1 bar. If the
solution contains 5.00 × 10–2 g of ethane, then what shall be the partial pressure of the gas?
Ans. Molar mass of ethane (C2H6) = 2 × 12 + 6 × 1 = 30 g mol–1
6.56 103
 Number of moles present in 6.56 × 10–3 g of ethane = = 0.218 × 10–3 mol
30
= 2.18 × 10–4 mol
Let the number of moles of the solvent be x.
According to Henry's law, p = KHx
2.18  104 2.18  104
 1bar = K H .  1 bar = K H (Since x > > 2.18 × 10–4)
2.18 104  x x
x
 KH  bar
2.18 104
5.00  102
Number of moles present in 5.00 × 10–2 g of ethane = mol = 1.67 × 10–3 mol
30
According to Henry's law, Pg = KHXg
x 1.67  103
= 
2.18 104 1.67 103   x
x 1.67  103
= 4
 (Since, x >> 1.67 × 10–3)
2.18  10 x
= 7.64 bar
Hence, partial pressure of the gas shall be 7.64 bar.
14. What is meant by positive and negative deviations from Raoult's law and how is the sign of
mixH related to positive and negative deviations from Raoult's law?
Ans. According to Raoult's law, the partial vapour pressure of each volatile component in any solution is
directly proportional to its mole fraction. The solutions which obey Raoult's law over the entire range
of concentration are known as ideal solutions. The solutions that do not obey Raoult's law (non - ideal
solutions) have vapour pressure either higher or lower than that predicted by Raoult's law. If the
vapour pressure is higher, then the solution is said to exhibit positive deviation, and if it is lower, then
the solution is said to exhibit negative deviation from Raoult's law.

Vapour pressure of solution


PT > P1 + P2

P2o
P1o
Vapour Pressure
node06\B0BC-BD\Kota\Board Material\Chemistry\Booklets\CBSE\Part-1

X1=1 Mole Fraction X1=0


X2=0 X2  X2=1
X1

Vapour pressure of a two - component solution showing positive deviation from Raoult's
law.
E 31
Chemistry
ALLEN

P2°
P1°

Vapour Pressure
X1=1 Mole Fraction X1=0
X2=0 X2  X2=1
 X1

Vapour pressure of a two - component solution showing negative deviation from Raoult's law
In case of an ideal solution, the enthalpy of the mixing of the pure components for forming the
solutions is zero.
sol H = 0
In the case of solutions showing positive deviations, absorption of heat takes place.
sol H = Positive
In the case of solutions showing negative deviations, evolution of heat takes place.
sol H = Negative
15. An aqueous solution of 2% non-volatile solute exerts a pressure of 1.004 bar at the normal
boiling point of the solvent. What is the molar mass of the solute?
Ans. Here, Vapour pressure of the solution at normal boiling point ( P1 ) = 1.004 bar

Vapour pressure of pure water at normal boiling point ( P10 ) = 1.013 bar

Mass of solute, (w2) = 2g


Mass of solvent (water), (w1) = 98 g
Molar mass of solvent (water), (M1) = 18 g mol–1
According to Raoult's law,
P10  PS w 2  M1

PS M 2  w1
node06\B0BC-BD\Kota\Board Material\Chemistry\Booklets\CBSE\Part-1

1.013  1.004 2  18
 
1.004 M 2  98

1.004  2  18 36.144
 M2    40.98  41 g mol–1
0.009  98 0.882
Hence, the molar mass of the solute is 41.35 g mol–1.

32 E
CBSE
ALLEN
16. Heptane and octane form an ideal solution. At 373 K, the vapour pressure of the two liquid
components are 105.2 kPa and 46.8 kPa respectively. What will be the vapour pressure of a
mixture of 26.0 g of heptane and 35 g of octane ?
Ans. Vapour pressure of heptane ( P10 ) = 105.2 kPa

Vapour pressure of octane ( P20 ) = 46.8 kPa


We know that,
Molar mas of heptane (C7H16) = 7 × 12 + 16 × 1 = 100 g mol–1
26
 Number of moles of heptane = = 0.26 mol
100
Molar mass of octane (C8H18) = 8 × 12 + 18 × 1 = 114 g mol–1
35
 Number of moles of octane = mol = 0.31 mol
114
0.26
Mole fraction of heptane, X1  = 0.456
0.26  0.31
And, mole fraction of octane, X2 = 1 – 0.456 = 0.544
Now, partial pressure of heptane, P1 = P10 X1 = 105.2 × 0.456 = 47.97 kPa

Partial pressure of octane, P2 = P20 X 2 = 46.8 × 0.544 = 25.46 kPa


Hence, vapour pressure of solution, Ptotal  P1  P2 = 47.97 + 25.46 = 73.43 kPa
17. The vapour pressure of water is 12.3 kPa at 300 K. Calculate vapour pressure of 1 molal solution
of a non-volatile solute in it.
Ans. 1 molal solution means 1 mol of the solute is present in 1000 g of the solvent (water).
Molar mass of water = 18 g mol–1
1000
 Number of moles present in 1000 g of water = = 55.56 mol
18
1
Therefore, mole fraction of the solute in the solution is X 2   0.0176
1  55.56
It is given that,
Vapour pressure of water, P10 = 12.3 kPa
Applying the relation,
node06\B0BC-BD\Kota\Board Material\Chemistry\Booklets\CBSE\Part-1

12.3  PS
 0.0176 12.3 = 1.0176 PS
PS
12.3
PS =
1.0176
PS = 12.08 kPa (approximately)
Hence, the vapour pressure of the solution is 12.08 kPa.

E 33
Chemistry
ALLEN
18. Calculate the mass of a non-volatile solute (molar mass 40 g mol–1) which should be dissolved in
114 g octane to reduce its vapour pressure to 80%.
Ans. Molar mass of solute, M2 = 40 g mol–1
Mass of octane, w1 = 114 g
Molar mass of octane, (C8H18), M1 = 8 × 12 + 18 × 1 = 114 g mol–1
Applying the relation,
p10  ps w 2  M1 100  80 w 2 114
  
ps M 2  w1 80 40  114
20 w 2
  w2 = 10 g
80 40
Hence, the required mass of the solute is 10 g.
19. A solution containing 30 g of non-volatile solute exactly in 90 g of water has a vapour pressure
of 2.8 kPa at 298 K. Further, 18 g of water is then added to the solution and the new vapour
pressure becomes 2.9 kPa at 298 K. Calculate :
(i) molar mass of the solute
(ii) vapour pressure of water at 298 K.
Ans. (i) Let, the molar mass of the solute be M g mol–1
90 g
Now, the no. of moles of solvent (water), n1   5 mol
18g mol1
30 g 30
And, the no. of moles of solute, n 2  1
 mol
M mol M
PS = 2.8 kPa
PA0  PS n 2
Applying the relation : 
PS n1
30
P10  2.8 M
  .........(i)
2.8 5
After the addition of 18 g of water :
90  18g
n1   6mol
18
P1 = 2.9 kPa
P10  PS n 2
Again, applying the relation : 
node06\B0BC-BD\Kota\Board Material\Chemistry\Booklets\CBSE\Part-1

PS n1
30
P  2.9 M
0
 1
 .........(ii)
2.9 6
 (i) by (ii), we have :
Dividing equation
P1o  2.8 2.9 30 6M
 o  
2.8 P1  2.9 5M 30
34 E
CBSE
ALLEN
 14.5  P1o  2.8   16.8  P1o  2.9 

 8.12 = 2.3 PAo


 Vapour pressure of water ( P1o ) = 3.53 kPa
Put value of P1o in eq. (i)
3.53  2.8 30
 
2.8 5M
30  2.8 84
Molar mass of solute (M)    23u
0.73  5 3.65
20. A 5% solution (by mass) of cane sugar in water has freezing point of 271 K. Calculate the
freezing point of 5% glucose in water if freezing point of pure water is 273.15 K.
Ans. Here, Tf = (273.15 – 271) K = 2.15 K
Molar mass of sugar (C12H22O11) = 12 × 12 + 22 × 1 + 11 × 16 = 342 g mol–1
5% solution (by mass) of cane sugar in water means 5 g of cane sugar is present in
(100 – 5) g = 95g of water.
5
Now, number of moles of cane sugar = mol = 0.0146 mol
342
0.0146 mol
Therefore, molality of the solution, m  = 0.1537 mol kg–1
0.095 kg
Applying the relation,
Tf = Kf × m
Tf 2.15 K
 Kf  = = 13.99 K kg mol–1
m 0.1537 mol kg 1
Molar mass of glucose (C6H12O6) = 6 × 12 + 12 × 1 + 6 × 16 = 180 g mol–1.
5% glucose in water means 5g of glucose is present in (100 – 5) = 95 g of water.
5
 Number of moles of glucose = mol = 0.0278 mol
180
0.0278 mol
Therefore, molality of the solution, m  = 0.2926 mol kg–1
0.095 kg
Applying the relation,
Tf = Kf × m
= 13.99 K kg mol–1 × 0.2926 mol kg–1
node06\B0BC-BD\Kota\Board Material\Chemistry\Booklets\CBSE\Part-1

= 4.09 K (approximately)
Hence, the freezing point of 5% glucose solution is (273.15 – 4.09) K = 269.06 K.
21. Two element A and B form compounds having formula AB2 and AB4. When dissolved in 20 g of
benzene (C6H6), 1 g of AB2 lowers the freezing point by 2.3 K whereas 1.0 g of AB4 lowers it by
1.3K. The molar depression constant for benzene is 5.1 K kg mol–1. Calculate atomic masses of
A and B.

E 35
Chemistry
ALLEN
1000  w 2  k f
Ans. We know that M 2 
Tf  w1

1000  1 5.1
Then, M AB2  = 110.86 g mol–1
2.3  20
1000  1 5.1
M AB4  = 196.15 g mol–1
1.3  20
Now, we have the molar masses of AB2 and AB4 as 110.87 g mol–1 and 196.15 g mol–1
respectively.
Let the atomic masses of A and B are x and y respectively.
Now, we can write :
x + 2y = 110.87 ...........(i)
x + 4y = 196.15 ...........(ii)
Subtracting equation (i) from (ii), we have
2y = 85.28
 y = 42.64
Putting the value of 'y' in equation (1), we have
x + 2 × 42.64 = 110.87
x = 25.59 u
Hence, the atomic masses of A and B are 25.59 u and 42.64 u respectively.
22. At 300 K, 36g of glucose present in a litre of its solution has an osmotic pressure of 4.98 bar. If
the osmotic pressure of the solution is 1.52 bars at the same temperature, what would be its
concentration?
Ans. Here, T = 300 K;
 = 1.52 bar;
R = 0.083 bar L K–1 mol–1
Applying the relation,
 = CRT
 1.52
 C = = 0.061 mol
RT 0.083bar L.K 1mol1  300 K
node06\B0BC-BD\Kota\Board Material\Chemistry\Booklets\CBSE\Part-1

Since the volume of the solution is 1 L, the concentration of the solution would be 0.061 M.
23. Suggest the most important type of intermolecular attractive interaction in the following pairs.
(i) n - hexane and n-octane (ii) I2 and CCl4
(iii) NaClO4 and water (H2O) (iv) methanol and acetone
(v) acetonitrile (CH3CN) and acetone (C3H6O).

36 E
CBSE
ALLEN
Ans. (i) Van der Wall's forces of attraction.
(ii) Van der Wall's forces of attraction.
(iii) Ion-dipole interaction.
(iv) Dipole - dipole interaction.
(v) Dipole - dipole interaction.
24. Based on solute- solvent interactions, arrange the following in order of increasing solubility in
n-octane and explain. Cyclohexane, KCl,CH3OH, CH3CN.
Ans. n-octane is non-polar solvent. Therefore, the solubility of a non-polar solute is more than that of a
polar solute in the n-octane.
The order of increasing polartiy is :
Cyclohexane < CH3CN < CH3OH < KCl
Therefore, the order of increasing solubility is :
KCl < CH3OH < CH3CN < Cyclohexane.
25. Amongst the following compounds, identify which are insoluble, partially soluble and highly
soluble in water ?
(i) phenol (ii) toluene (iii) formic acid
(iv) ethlene glycol (v) chloroform (vi) pentanol.
Ans. (i) Phenol (C6H5OH) has the polar group –OH and non-polar group –C6H5. Thus, phenol is
partially soluble in water.
(ii) Toluene (C6H5–CH3) has no polar group. Thus, toluene is insoluble in water.
(iii) Formic acid (HCOOH) has the polar group –OH and can from H-bond with water. Thus,
formic acid is highly soluble in water.
OH
(iv) Ethylene glycol HO
has polar –OH group and can form H–bond. Thus, it is highly

soluble in water.
(v) Chloroform is insoluble in water.
(vi) Pentanol (C5H11OH) has polar –OH group, but it also contains a very bulky
non-polar –C5H11 group. Thus, pentanol is partially soluble in water.
26. If the density of some lake water is 1.25 g mL–1 and contains 92 g of Na+ ions per kg of water,
calculate the molality of Na+ ions in the lake.
92 g
Ans. Number of moles present in 92 g of Na+ ions = = 4 mol
23g mol1
4 mol
node06\B0BC-BD\Kota\Board Material\Chemistry\Booklets\CBSE\Part-1

Therefore, molality of Na+ ions in the lake = =5m


1kg
27. If the solubility product of CuS is 6 × 10–16, calculate the maximum molarity of CuS in aqueous
solution.
Ans. Solubility product of CuS, Ksp = 6 × 10–16
Let’s be the solubility of CuS is mol L–1.
 Cu 2  S2
CuS 
S S

E 37
Chemistry
ALLEN
Now, Ksp = [Cu2+] [S2+]
= s × s = s2
Then, we have, Ksp = s2 = 6 × 10–16
  s  6 1016 = 2.45 × 10–8 mol L–1
Hence, the maximum molarity of CuS in an aqueous solution is 2.45 × 10–8 mol L–1.
28. Calculate the mass percentage of aspirin (C9H8O4) in acetonitrile (CH3CN) when 6.5 g of C9H8O4
is dissolved in 450 g of CH3CN.
Ans. 6.5 g of aspirin (C9H8O4) is dissolved in 450 g of acetonitrile (CH3CN)
Then, total mass of the solution = (6.5 + 450) g = 456.5 g
6.5
Therefore, mass percentage of C9H8O4 =  100% = 1.424%
456.5
29. Nalorphene (C19H21NO3), similar to morphine, is used to combat withdrawal symptoms in
narcotic users. Dose of nalorphene generally given is 1.5 mg. Calculate the mass of 1.5 × 10–3 m
aqueous solution required for the above dose.
Ans. The molar mass of nalorphene (C19H21NO3) is given as :
19 × 12 + 21 × 1 + 1 × 14 + 3 × 16 = 311 g mol–1
In 1.5 × 10–3 m aqueous solutions of nalorphene,
1 kg (1000 g) of water contains 1.5 × 10–3 mol = 1.5 × 10–3 × 311 g = 0.4665 g nalorphene
Therefore, total mass of the solution = (1000 + 0.4665) g = 1000.4665 g
This implies that the mass of the solution containing 0.4665 g of nalorphene is 1000.4665 g.
Therefore, mass of the solution containing 1.5 mg of nalorphene is :
1000.4665 1.5 103
g = 3.2 g
0.4665
Hence, the mass of aqueous solution required is 3.2 g
Note : There is slight variation in this answer and the one given in the NCERT textbook.
30. Calculate the amount of benzoic acid (C6H5COOH) required for preparing 250 mL. of 0.15 M
solution in methanol.
Ans. 0.15 M solution of benzoic acid in methanol means, 1000 mL of solution contains 0.15 mol of
benzoic acid.
0.15  250
Therefore, 250 mL of solution contains = mol of benzoic acid
1000
= 0.0375 mol of benzoic acid
Molar mass of benzoic acid (C6H5COOH) = 7 × 12 + 6 × 1 + 2 × 16 = 122 g mol–1
node06\B0BC-BD\Kota\Board Material\Chemistry\Booklets\CBSE\Part-1

Hence, required benzoic acid = 0.0375 mol × 122 g mol–1 = 4.575 g


31. The depression in freezing point of water observed for the same amount of acetic acid,
trichloroacetic acid and trifluoroacetic acid increases in the order given above. Explain briefly.
H Cl F
Ans. H——C—COOH Cl—C—COOH F—C—COOH
H Cl F
Acetic acid Trichloroacetic acid Trifluoroacetic acid

38 E
CBSE
ALLEN
Among H, Cl, and F, H is least elecronegative while F is most electronegative. Then, F can
withdraw electrons towards itself more than Cl and H.
Thus, trifluoroacetic acid can easily lose H+ ions i.e. trifluoroacetic acid ionizes to the largest
extent. Now, the more ions produced, the greater is the depression of the freezing point. Hence,
the depression in the freezing point increases in the order.
Acetic acid < trichloroacetic acid < trifluoroacetic acid
32. Calculate the depression in the freezing point of water when 10 g, CH3CH2CHClCOOH is added
to 250 g of water, Ka = 1.4 × 10–3, Kf = 1.86 K kg mol–1.
Ans. Molar mass of CH3CH2CHClCOOH = 15 + 14 + 13 + 35.5 + 12 + 16 + 16 + 1 = 122.5 g mol–1
10 g
No. of moles present in 10 g CH3CH2CHClCOOH = = 0.0816 mol
122.5g mol-1
It is given that 10 g CH3CH2CHClCOOH is added to 250 g of water.
0.0186
Molality of the solution, =  1000 = 0.3265 mol kg–1
250
Let a be the degree of dissociation of CH3CH2CHClCOOH.
CH3CH2CHClCOOH undergoes dissociation according to the following equation :
CH3CH2CHClCOOH 
 CH3CH2CHClCOO– + H+
Initial conc. C mol L–1 0 0
At equilibrium C(1 – ) C C
C.C C 2
 Ka  =
C 1    1   
Since a is very small with respect to 1, 1 –  1
Now,  Ka = C2
Ka 1.4  103
    = 0.0655 (Ka = 1.4 × 10–3)
C 0.3264
Again, CH3CH2CHClCOOH 
 CH3CH2CHClCOO– + H+
Initial moles 1 0 0
At equilibrium 1–  
Total mass of equilibrium = 1 –  +  +  = 1 + 
1 
  i  = 1 +  = 1 + 0.0655 = 1.0655
1
Hence, the depression in the freezing point of water is given as :
Tf = i.Kfm = 1.0655 × 1.86 kg mol–1 × 0.364 mol kg–1 = 0.65 K
node06\B0BC-BD\Kota\Board Material\Chemistry\Booklets\CBSE\Part-1

33. 19.5 g of CH2FCOOH is dissolved in 500 g of water. The depression in the freezing point
of water observed is 1.0°C. Calculate the van't Hoff factor and dissociation constant of
fluoroacetic acid.
Ans. It is given that :
w1 = 500 g, w2 = 19.5 g
Kf = 1.86 K kg mol–1, Tf = 1 K

E 39
Chemistry
ALLEN
K f  w 2 1000
We know that : M2 
Tf  w1
1.86 K kg mol1 19.5g 1000g kg 1
= = 72.54 mol–1
500 g 1K
Therefore, observed molar mass of CH2FCOOH, (M2)obs = 72.54 mol
The calculated molar mass of CH2FCOOH is :
(M2)cal = 14 +19 + 12 + 16 + 16 + 1 = 78 g mol–1
(M 2 )cal 78g mol1
Therefore, van't Hoff factor, i   = 1.0753
(M 2 )obs 72.54 g mol1
Let a be the degree of dissociation of CH2FCOOH
CH2FCOOH 
 CH2FCOO– + H+
Initial conc. C mol L–1 0 0
At equilibrium C (1 – ) C C Total = C (1 + )
C 1   
 i
C
 i=1+
  = i – 1 = 1.0753 – 1 = 0.0753 = 7.53%
Now, the value of Ka is given as :
[CH 2 FCOO  ][H  ] C.C C 2
Ka  = =
[CH 2 FCOOH] C 1    1  
Taking the volume of the solution as 500 mL.
We have the concentration : 19.5 M
weight = 19.5 g
19.5
Concentration = 78 ×1000 = 0.5 M
500
C 2
Therefore, K a 
1 
0.5   0.0753
2
0.5  0.00567
= = = 0.00307 (approximately)
1  0.0753 0.9247
= 3.07 × 10–3
node06\B0BC-BD\Kota\Board Material\Chemistry\Booklets\CBSE\Part-1

34. Vapour pressure of water at 293 K is 17.535 mm Hg. Calculate the vapour pressure of water at
293 K when 25 g of glucose is dissolved in 450 g of water.
Ans. Vapour pressure of water, P10 = 17.535 mm of Hg
Mass of glucose, w2 = 25 g
Mass of water, w1 = 450 g

40 E
CBSE
ALLEN
We know that,
Molar mass of glucose (C6H12O6), M2 = 6 × 12 + 12 × 1 + 6 × 16 = 180 g mol–1
Molar mass of water, M1 = 18 g mol–1
25
Then, number of moles of glucose, n 2  = 0.139 mol
180 g mol1
450 g
And, number of moles of water, n1  = 25 mol
18g mol1
We know that,
 n1   25  17.535  25
P1  P1o     P1  17.535  
 n1  n 2   25  0.139  25.139

 17.535 – P1 = 0.097  P1 = 17.44 mm of Hg


Hence, the vapour pressure of water is 17.44 mm of Hg.
35. Henry's law constant for the molality of methane in benzene at 298 K is 4.27 × 105 mm Hg.
Calculate the solubility of methane in benzene at 298 K 760 mm Hg.
Ans. Here, P = 760 mm Hg ; kH = 4.27 × 105 mm Hg
According to Henry's law,
p 760 mm Hg
Pg = KHXg  x  =
KH 4.27 105 mm Hg
= 177.99 × 10–5 = 178 × 10–5 (approximately)
Hence, the mole fraction of methane in benzene is 178 × 10–5.
36. 100g of liquid A (molar mass 140 g mol–1) was dissolved in 1000 g of liquid B (molar mass
180 g mol–1). The vapour pressure of pure liquid B was found to be 500 torr. Calculate the
vapour pressure of pure liquid A and its vapour pressure in the solution if the total vapour
pressure of the solution is 475 Torr.
100
Ans. Number of moles of liquids A, n A  mol = 0.714 mol
140
1000
Number of moles of liquids B, n B  mol = 5.556 mol
180
Then, mole fraction of A,
nA 0.714
xA  = = 0.114
nA  nB 0.714  5.556
node06\B0BC-BD\Kota\Board Material\Chemistry\Booklets\CBSE\Part-1

And, mole fraction of B, xB = 1 – 0.114 = 0.886


Vapour pressure of pure liquid B, p0B = 500 torr
Therefore, vapour pressure of liquid B in the solution,
p B  p0B x B = 500 × 0.886 = 443 torr
Total vapour pressure of the solution, ptotal = 475 torr
E 41
Chemistry
ALLEN
 Vapour pressure of liquid A in the solution,
p A  p total  p B = 475 – 443 = 32 torr
pA 32
Now, p A  p 0A x A  p0A  = = 280.7
xA 0.114
Hence, the vapour pressure of pure liquid A is 280.7 torr.
37. Vapour pressure of pure acetone and chloroform at 328 K are 741.8 mm Hg and 632.8 mm Hg
respectively. Assuming that they form ideal solution over the entire range of composition, plot
ptotal' pchloroform' and pacetone as a function of xacetone. The experimental data observed for different
compositions of mixture is :
100 × xacetone 0 11.8 23.4 36.0 50.8 58.2 64.5 72.1
pacetone/mm Hg 0 54.9 110.1 202.4 322.7 405.9 454.1 521.1
pchloroform/mm Hg 632.8 548.1 469.4 359.7 257.7 193.6 161.2 120.7

Ans. From the question, we have the following data

100 × xacetone 0 11.8 23.4 36.0 50.8 58.2 64.5 72.1


pacetone/mm Hg 0 54.9 110.1 202.4 322.7 405.9 454.1 521.1
pchloroform/mm Hg 632.8 548.1 469.4 359.7 257.7 193.6 161.2 120.7
Ptotal/(mm Hg) 632.8 603.0 579.5 562.1 580.4 615.3 615.3 614.8

800
Vapour pressure (mm Hg)

700
Ptotal
600
500 Pch
loro
form
400 a

300 ane
200 P acet
100
0 10 20 30 40 50 60 70
100 × x acetone

It can be observed from the graph that the plot for the ptotal of the solution curves downwards.
Therefore, the solution shows negative deviation from the ideal behaviour.
node06\B0BC-BD\Kota\Board Material\Chemistry\Booklets\CBSE\Part-1

38. Benzene and toluene form ideal solution over the entire range of composition. The vapour
pressure of pure benzene and toluene at 300 K are 50.71 mm Hg and 32.06 mm Hg respectively.
Calculate the mole fraction of benzene in vapour phase if 80 g of benzene is mixed with 100 g of
toluene.
Ans. Molar mass of benzene (C6H6) = 6 × 12 + 6 × 1 = 78 g mol–1
Molar mass of toluene = 7 × 12 + 8 × 1 = 92 g mol–1

42 E
CBSE
ALLEN
80
Now, no. of moles present in 80 g of benzene = mol = 1.026 mol
78
100
And, no. of moles present in 100 g of toluene = mol = 1.087 mol
92
1.026
Mole fraction of benzene, xb = = 0.486
1.026  1.087
And, mole fraction of toluene, xt = 1 – 0.486 = 0.514
It is given that vapour pressure of pure benzene, p0b = 50.71 mm of Hg
and vapour pressure of pure toluene, p0t = 32.06 mm of Hg
Therefore, partial vapour pressure of benzene,
pt = xt × pt = 0.486 × 50.71 = 24.645 mm of Hg
And, partial vapour pressure of toluene,
p1 = x1 × p1 = 0.514 × 32.06 = 16.479 mm of Hg
Hence, mole fraction of benzene in vapour phase is given by :
pb 24.645 24.645
= = = 0.599 = 0.6
p b  p1 24.645  16.479 41.124
39. The air is a mixture of a number of gases. The major components are oxygen and nitrogen
with approximate proportion of 20% is to 79% by volume at 298 K. The water is in equilibrium
with air at a pressure of 10 atm. At 298 K if the Henry's law constants for oxygen and nitrogen
are 3.30 × 107 mm and 6.51 × 107 mm respectively. Calculate the composition of these gases
in water.
Ans. Percentage of oxygen (O2) in air = 20%
Percentage of nitrogen (N2) in air = 79%
Also, it is given that water is in equilibrium with air at a total pressure of 10 atm, that is,
(10 × 760) mm Hg = 7600 mm Hg
20
Partial pressure of oxygen, p O 2   7600 mm Hg = 1520 mm Hg
100
79
Partial pressure of nitrogen, p N2   7600 mm Hg = 6004 mm of Hg
100
Now, according to Henry's law : p = KHx
For oxygen : p O 2  K H .x O2

p O2 1520 mm of Hg
 xO 2  = (Given KH = 3.30 × 107 mm of Hg = 4.61 × 10–5.
3.30 10 mm of Hg
7
node06\B0BC-BD\Kota\Board Material\Chemistry\Booklets\CBSE\Part-1

KH
For nitrogen, p N2  K H .x N2
p N1 6004 mm Hg
 x N2  = = 9.22 × 10–5
KH 6.5110 mm Hg
7

Hence, the mole fractions of oxygen and nitrogen in water are 4.61 × 10–5 and 9.22 × 10–5
respectively.

E 43
Chemistry
ALLEN
40. Determine the amount of CaCl2 (i = 2.47) dissolved in 2.5 litre of water such that its osmotic
pressure is 0.75 atm at 27°C.
n
Ans. We know that,   i RT
V
w MV
 i RT  w 
MV iRT
 = 0.75 atm
V = 2.5 L
i = 2.47
T = (27 + 273) K = 300 K
Here,
R = 0.082 L atm K–1 mol–1
M = 1 × 40 + 2 × 35.5 = 111 g mol–1
0.75 111 2.5
Therefore, w = = 3.42 g
2.47  0.0821 300
Hence, the required amount of CaCl2 is 3.42 g.
41. Determine the osmotic pressure of a solution prepared by dissolving 25 mg of K2SO4 in 2 liter of
water at 25°C, assuming that it is completely dissociated.
Ans. When K2SO4 is dissolved in water, K+ and SO 24 ions are produced.
K2SO4  2K+ + SO 24
Total number of ions produced = 3
 i=3
Given, w = 25 mg = 0.025 g
V=2L
T = 25°C = (25 + 273) K = 298 K
Also, we know that :
R = 0.0821 L atm K–1 mol–1
M = (2 × 39) + (1 × 32) + (4 × 16) = 174 g mol–1
Applying the following relation.
n
  i RT
v
w 1 0.025 1
RT = 3    0.0821 298 = 5.27 × 10–3 atm
node06\B0BC-BD\Kota\Board Material\Chemistry\Booklets\CBSE\Part-1

=i
Mv 174 2

44 E
CBSE
ALLEN
EXERCISE-3 EXEMPLAR

1. On dissolving sugar in water at room temperature solution feels cool to touch. Under which of
the following cases dissolution of sugar will be most rapid?
(a) Sugar crystals in cold water. (b) Sugar crystals in hot water.
(c) Powdered sugar in cold water. (d) Powdered sugar in hot water.
Ans. (d)
2. Considering the formation, breaking and strength of hydrogen bond, predict which of the
following mixtures will show a positive deviation from Raoult’s law?
(a) Methanol and acetone. (b) Chloroform and acetone.
(c) Nitric acid and water. (d) Phenol and aniline.
Ans. (a)
3. Which of the following aqueous solutions should have the highest boiling point?
(a) 1.0 M NaOH (b) 1.0 M Na2SO4 (c) 1.0 M NH4NO3 (d) 1.0 M KNO3
Ans. (b)
4. In comparison to a 0.01 M solution of glucose, the depression in freezing point of a 0.01 M
MgCl2 solution is _____________.
(a) the same (b) about twice (c) about three times (d) about six times
Ans. (c)
5. An unripe mango placed in a concentrated salt solution to prepare pickle, shrivels because
_____________.
(a) it gains water due to osmosis. (b) it loses water due to reverse osmosis.
(c) it gains water due to reverse osmosis. (d) it loses water due to osmosis.
Ans. (d)
6. At a given temperature, osmotic pressure of a concentrated solution of a substance
_____________.
(a) is higher than that at a dilute solution.
(b) is lower than that of a dilute solution.
(c) is same as that of a dilute solution.
(d) cannot be compared with osmotic pressure of dilute solution.
Ans. (a)
7. Which of the following statements is false?
(a) Two different solutions of sucrose of same molality prepared in different solvents will have
the same depression in freezing point.
The osmotic pressure of a solution is given by the equation  = CRT (where C is the
node06\B0BC-BD\Kota\Board Material\Chemistry\Booklets\CBSE\Part-1

(b)
molarity of the solution).
(c) Decreasing order of osmotic pressure for 0.01 M aqueous solutions of barium chloride,
potassium chloride, acetic acid and sucrose is BaCl2 > KCl > CH3COOH > sucrose.
(d) According to Raoult’s law, the vapour pressure exerted by a volatile component of a
solution is directly proportional to its mole fraction in the solution.
Ans. (a)

E 45
Chemistry
C
N
ALLEN
8.
8 The vaalues of vann’t Hoff facttors for KCll, NaCl and
d K2SO4, resspectively, aare _______
_______.
(a) 2, 2 and 2 (b) 2,, 2 and 3 (c) 1,
1 1 and 2 (d)) 1, 1 and 1
Ans.
A (b)
9.
9 Considder the Fig. 2.1 and maark the correect option.

(a) wwater will move


m from
m side (A) tto side (B) if a pressu
ure lower thhan osmotic pressure is
aapplied on piston
p (B).
(b) wwater will move
m from side (B) too side (A) if a pressurre greater thhan osmotiic pressure is
aapplied on piston
p (B).
(c) wwater will move
m s (B) to side (A) if a pressure equal
from side e to osm
motic pressu
ure is applieed
oon piston (B
B).
(d) wwater will move
m from side (A) too side (B) iff pressure equal to osm
motic pressu
ure is applieed
oon piston (A
A).
Ans.
A (b)
10.
1 We haave three aqqueous soluttions of NaCCl labelled as ‘A’, ‘B’ and ‘C’ wiith concentrrations 0.1M
M,
0.01M
M and 0.001M M, respectiv
vely. The vvalue of van
n’t Hoff facttor for thesee solutions will
w be in thhe
order_______.
(a) iA < iB < iC (b) iA > iB > iC (c) iA = iB = iC (d)) iA < iB > iC
Ans.
A (b)
11.
1 On thee basis of innformation given
g below
w mark the correct
c optio
on.
Inform
mation:
(A) IIn bromoethhane and chloroethane
c e mixture intermolecu
i ular interacttions of A–
–A and B––B
ttype are neaarly same ass A–B type interactionss.
(B) IIn ethanol and
a acetonee mixture A
A–A or B–B
B type intermolecular iinteractionss are stronger
tthan A–B tyype interactiions.
(C) IIn chlorofoorm and accetone mixtture A–A or
o B–B typ
pe intermoolecular inteeractions arre
node06\B0BC-BD\Kota\Board Material\Chemistry\Booklets\CBSE\Part-1

wweaker thann A–B type interactionss.


(a) Sollution (B) and
a (C) will follow Raooult’s law.
(b) Sollution (A) will
w follow Raoult’s
R law
w.
(c) Sollution (B) will
w show neegative deviiation from Raoult’s law.
(d) Sollution (C) will
w show po
ositive devi ation from Raoult’s
R law
w.
Ans.
A (b)

46
6 E
CBSE
ALLEN
12. If two liquids A and B form minimum boiling azeotrope at some specific composition then
_______________.
(a) A–B interactions are stronger than those between A–A or B–B.
(b) vapour pressure of solution decreases because more number of molecules of liquids A and
B can escape from the solution.
(c) vapour pressure of solution decreases because less number of molecules of only one of the
liquids escape from the solution.
(d) A–B interactions are weaker than those between A–A or B–B.
Ans. (d)
13. KH value for Ar(g), CO2(g), HCHO (g) and CH4(g) are 40.39, 1.67, 1.83 × 10–5 and
0.413 respectively.
Arrange these gases in the order of their increasing solubility.
(a) HCHO < CH4 < CO2 < Ar (b) HCHO < CO2 < CH4 < Ar
(c) Ar < CO2 < CH4 < HCHO (d) Ar < CH4 < CO2 < HCHO
Ans. (c)
(SHORT ANSWER TYPE QUESTIONS)
14. Explain why on addition of 1 mol of NaCl to 1 litre of water, the boiling point of water increases,
while addition of 1 mol of methyl alcohol to one litre of water decreases its boiling point.
Ans. NaCl is a non-volatile solute. So, its addition to water lowers the vapour pressure of the water.
Hence, boiling point of water (solution) increases. Whereas methyl alcohol is more volatile than
water.
So, its addition to water increases the total vapour pressure over the solution. It results in the
decreases of boiling point.
15. Why is the vapour pressure of an aqueous solution of glucose lower than that of water?
Ans. In pure liquids, the entire surface of liquid is occupied by the molecules of water. When a
non-volatile solute, e.g., glucose is dissolved in water, the fraction of surface covered by the
solvent molecules gets reduced because some positions are occupied by glucose molecules.
So, number of solvent molecules escaping from the surface is reduced. That is why vapour
pressure of aqueous solution of glucose is reduced.
16. How does sprinkling of salt help in clearing the snow covered roads in hilly areas? Explain the
phenomenon involved in the process.
Ans. When salt is spread over snow covered road, snow starts melting from the surface because
depression of freezing point of water takes place due to addition of salt. It helps in clearing of
node06\B0BC-BD\Kota\Board Material\Chemistry\Booklets\CBSE\Part-1

roads.
Hence, the phenomena is depression in freezing point which helps in clearing the snow covered
roads in hilly areas.
17. Give an example of a material used for making semipermeable membrane for carrying out
reverse osmosis.
Ans. Since pressure required for the reverse osmosis is very high, so a suitable material is used for
making semipermeable membrane. It is generally cellulose acetate placed over suitable support.
E 47
Chemistry
ALLEN
(MATCHING THE COLUMNS)
18. Match the items given in Column I and Column II.
Column I Column II
(A) Saturated solution (1) Solution having same osmotic
pressure at a given temperature as
that of given solution.
(B) Binary solution (2) A solution whose osmotic pressure
is less than that of another.
(C) Isotonic solution (3) Solution with two components.
(D) Hypotonic solution (4) A solution which contains maximum
amount of solute that can be dissolved
in a given amount of solvent at a
given temperature.
(E) Solid solution (5) A solution whose osmotic pressure
is more than that of another.
(F) Hypertonic solution (6) A solution in solid phase.
Ans. A-4, B-3, C-1, D-2, E-6, F-5
19. Match the items given in Column I with the type of solutions given in Column II.
Column I Column II
(A) Soda water (1) A solution of gas in solid
(B) Sugar solution (2) A solution of gas in gas
(C) German silver (3) A solution of solid in liquid
(D) Air (4) A solution of solid in solid
(E) Hydrogen gas in palladium (5) A solution of gas in liquid
(6) A solution of liquid in solid
Ans. A-5, B-3, C-4, D-2, E-1
20. Match the laws given in Column I with expressions given in Column II.
Column I Column II
(A) Raoult’s law (1) Tf = Kfm
node06\B0BC-BD\Kota\Board Material\Chemistry\Booklets\CBSE\Part-1

(B) Henry’s law (2)  = CRT


(C) Elevation of boiling point (3) p = x1p1o  x 2 p o2

(D) Depression in freezing point (4) Tb = Kbm


(E) Osmotic pressure (5) p = KH . x
Ans. A-3, B-5, C-1, D-1, E-2

48 E
CBSE
ALLEN
21. Match the terms given in Column I with expressions given in Column II.
Column I Column II
Number of moles of the solute components
(A) Mass percentage (1)
Volume of solution in litres
Number of moles of a component
(B) Volume percentage (2)
Total number of moles of all the components

(C) Mole fraction (3) Volume of the solute component in solution 100
Total volume of solution
Mass of the solute component in solution
(D) Molality (4) 100
Total mass of the solution
Number of moles of the solute components
(E) Molarity (5)
Mass of solvent in kilograms
Ans. A-4, B-3, C-2, D-5, E-1
(ASSERTION AND REASON TYPE)
Note: In the following questions a statement of assertion followed by a statement of reason is given.
Choose the correct answer out of the following choices.
(a) Both (A) and (R) correct and (R) is correct explanation of (A).
(b) Both (A) and (R) correct and (R) is not correct explanation of (A).
(c) (A) is correct but (R) is wrong.
(d) (A) Assertion is wrong but (R) reason is correct.
22. Assertion (A) : Molarity of a solution in liquid state changes with temperature.
Reason (R) : The volume of a solution changes with change in temperature.
Ans. (a)
23. Assertion (A) : When methyl alcohol is added to water, boiling point of water increases.
Reason (R) : When a non-volatile solute is added to a volatile solvent elevation in boiling point
is observed.
Ans. (d)
24. Assertion (A) : When NaCl is added to water a depression in freezing point is observed.
Reason (R) : The lowering of vapour pressure of a solution causes depression in the freezing
point.
node06\B0BC-BD\Kota\Board Material\Chemistry\Booklets\CBSE\Part-1

Ans. (a)
25. Assertion (A) : When a solution is separated from the pure solvent by a semipermeable
membrane, the solvent molecules pass through it from pure solvent side to the solution side.
Reason (R) : Diffusion of solvent occurs from a region of high concentration solution to a region
of low concentration solution.
Ans. (b)

E 49
Chemistry
ALLEN
EXERCISE-4 MCQ, A/R, CASE-BASED QUESTIONS

(Introduction, Types of solutions, concentration terms)


1. 8 g NaOH is dissolved in one litre of solution, its molarity is:
(a) 0.8 M (b) 0.4 M (c) 0.2 M (d) 0.1 M
2. If 18 g of glucose is present in 1000 g of solvent, the solution is said to be:
(a) 1 molar (b) 0.1 molar (c) 0.5 molar (d) 0.1 molal
3. The molarity of pure water is:
(a) 100 M (b) 55.5 M (c) 50 M (d) 18M
4. The mole fraction of oxygen in a mixture of 7g of nitrogen and 8g of oxygen is:
8
(a) (b) 0.5 (c) 0.25 (d) 1.0
15
5. An example of liquid in solid is:
(a) Bronze (b) Alchol solution (c) Sugar Solution (d) Amalgam
(Solubility, Henry's Law)
6. Which of the following gas will have most solubility in water ?
(a) NH3 (B) H2 (C) O2 (D) He
7. Value of Henry’s constant KH ____________.
(a) increases with increase in temperature. (b) decreases with increase in temperature.
(c) remains constant. (d) first increases then decreases.
8. Increase in temperature solubility of H2 gas in water:
(a) Increase (b) Decrease (c) Remain unchanged (d) None of these
9. Low concentration of oxygen in the blood and tissues of people living at high altitude is due to
____________.
(a) low temperature
(b) low atmospheric pressure
(c) high atmospheric pressure
(d) both low temperature and high atmospheric pressure
10. Maximum amount of a solid solute that can be dissolved in a specified amount of a given liquid
solvent does not depend upon ____________.
(a) Temperature (b) Nature of solute (c) Pressure (d) Nature of solvent
(Vapour Pressure, Raoult's law, Ideal-Non ideal Solutions)
11. Which condition is not satisfied by an ideal solution?
(a) H mixing = 0 (b) V mixing = 0
node06\B0BC-BD\Kota\Board Material\Chemistry\Booklets\CBSE\Part-1

(c) S mixing = 0 (d) Obeyance of Raoult's law


12. Among the following, that does not form an ideal solution is :
(a) C6H6 and C6H5CH3 (b) C2H5Cl and C6H5OH
(c) C6H5Cl and C6H5Br (d) C2H5Br and C2H5I
13. An azeotropic mixture of two liquids has b.p. lower than either of them when it :-
(a) shows a (+ve) deviation from Raoult's law (b) shows no deviation from Raoult's law
(c) shows (+ve) deviation from Henry's law (d) shows (–ve) deviation from Henry's law

50 E
CBSE
ALLEN
14. The mixture that forms maximum boiling azeotrope is :
(a) Water + Nitric acid (b) Ethanol + Water
(c) Acetone + Carbon disulphide (d) Heptane + Octane
15. For a solution of volatile liquids the partial vapour pressure of each component in solution is
directly proportional to –
(a) Molarity (b) Mole fraction (c) Molality (d) Normality
(Colligative Properties: (a) RLVP (b) Elevation of Boiling points
(c) Depression in Freezing Point)
16. Which of the following compounds can be used as antifreeze in automobile radiators ?
(a) Nitrophenol (b) Ethyl alcohol (c) Methyl alcohol (d) Glycol
17. The relative lowering of vapour pressure is equal to the mole fraction of the nonvolatile solute,
This statement was given by :
(a) Raoult (b) Henry (c) Joule (d) Dalton
18. The unit of ebulioscopic constant is _______________.
(a) K kg mol–1 or K (molality)–1 (b) mol kg K–1 or K–1(molality)
(c) kg mol–1 K–1 or K–1(molality)–1 (d) K mol kg–1 or K (molality)
19. Pressure cooker reduces cooking time for food –
(a) Heat is more evenly distributed in the cooking space
(b) Boiling point of water involved in cooking is increased
(c) The higher pressure inside the cooker crushes the food material
(d) Cooking involves chemical changes helped by a rise in temperature
20. The molal elevation constant is the ratio of the elevation in B.P. to –
(a) Molarity (b) Molality
(c) Mole fraction of solute (d) Mole fraction of solvent
(Osmotic Pressure)
21. At constant temperature the osmotic pressure of a solution is :
(a) Directly proportional to the concentration
(b) Inversely proportional to the concentration
(c) Directly proportional to the square of concentration
(d) Directly proportional to the square root of concentration
22. Which inorganic precipitate acts as semipermeable membrane?
(a) Calcium sulphate (b) Barium oxalate
(c) Nickel phosphate (d) Copper ferrocyanide
node06\B0BC-BD\Kota\Board Material\Chemistry\Booklets\CBSE\Part-1

23. If mole fraction of the solvent in solution decreases then :


(a) Vapour pressure of solution increases (b) B. P. decreases
(c) Osmotic pressure increases (d) All are correct
24. The osmotic pressure of a solution increases if :
(a) Temperature is lowered (b) Volume is increased
(c) Number of solute molecules is increased (d) None

E 51
Chemistry
ALLEN
25. In osmosis –
(a) Solvent molecules move from higher concentration to lower concentration
(b) Solvent molecules move from lower to higher concentration
(c) Solute molecules move from higher to lower concentration
(d) Solute molecules move from lower to higher concentration
(Abnormal molar mass, Van’t Hoff factor)
26. Van't Hoff factor i
Normal molecular mass Observed molecular mass
(a)  (b) 
Observed molecular mass Normal molecular mass
(c) Less than one in case of dissociation (d) More than one in case of association+
27. The Van't Hoff factor will be highest for
(a) Sodium chloride (b) Magnesium chloride
(c) Sodium phosphate (d) Urea
28. Which of the following salt has the same value of Van't Hoff factor i as that of K 3 [Fe(CN)6 ]

(a) Al2 (SO 4 )3 (b) NaCl (c) Na 2SO4 (d) Al(NO3 )3


29. The observed osmotic pressure of a solution of benzoic acid in benzene is less than its expected
value because
(a) Benzene is a non-polar solvent
(b) Benzoic acid molecules are associated in benzene
(c) Benzoic acid molecules are dissociated in benzene
(d) Benzoic acid is an organic compound
30. The experimental molecular weight of an electrolyte will always be less than its calculated value
because the value of Van't Hoff factor “i” is
(a) Less than 1 (b) Greater than 1
(c) Equivalent to one (d) Zero

ANSWER KEY
Q. No. 1 2 3 4 5 6 7 8 9 10
node06\B0BC-BD\Kota\Board Material\Chemistry\Booklets\CBSE\Part-1

Ans. c d b b d a a a b c
Q. No. 11 12 13 14 15 16 17 18 19 20
Ans. c b a a b d a a b b
Q. No. 21 22 23 24 25 26 27 28 29 30
Ans. a d c c b a c a b b

52 E
CBSE
ALLEN
ASSERTION AND REASON
Note : In the following questions a statement of assertion followed by a statement of reason is
given. Choose the correct answer out of the following choices.
(a) Both (A) and (R) correct and (R) is correct explanation of (A).
(b) Both (A) and (R) correct and (R) is not correct explanation of (A).
(c) (A) is correct but (R) is wrong.
(d) (A) Assertion is wrong but (R) reason is correct.
1. Assertion : If red blood cells were removed from the body and placed in pure water, pressure
inside the cells increases.
Reason : The concentration of salt content in the cells increases.
2. Assertion : The solubility of a gas in liquid increases with increase of pressure.
Reason : The solubility of a gas in liquid is directly proportional to the pressure of the gas.
3. Assertion : An ideal solution obeys Raoult's law.
Reason : In an ideal solution, solute-solute as well as solvent-solvent interactions are similar to
solute-solvent interactions.
4. Assertion : Acetone and aniline show negative deviations.
Reason : H-bonding between acetone and aniline is stronger than that between acetone-acetone
and aniline-aniline.
5. Assertion (A) : One molar aqueous solution has always higher concentration than one molal.
Reason (R) : The molality of a solution depends upon the density of the solution whereas
molarity does not.
6. Assertion (A) : Aquatic species are more comfortable in cold water rather than in warm water.
Reason (R) : Different gases have different KH values at the same temperature.
7. Assertion (A) : When scuba divers come towards surface, their capillaries get blocked which is
painful and dangerous to life.
Reason (R) : There occurred release of dissolved gases as the pressure decreases and leads to the
formation of bubbles of nitrogen in the blood.
node06\B0BC-BD\Kota\Board Material\Chemistry\Booklets\CBSE\Part-1

8. Assertion (A) : Cryoscopic constant depends on nature of solvent.


Reason (R) : Cryoscopic constant is a universal constant.
9. Assertion (A) : Ethanol and acetone show positive deviation from Raoult's law.
Reason (R) : Pure ethanol molecule show hydrogen bond and on adding acetone hydrogen bond
between ethanol molecules breaks.

E 53
Chemistry
ALLEN
10. Assertion (A) : Nitric acid and water form maximum boiling azeotrope.
Reason (R) : Azeotropes are binary mixture having the same composition in liquid and vapour
phase.
11. Assertion (A) : The vapour pressure of 0.1 M sugar solution is less than that of 0.1 M potassium
chloride solution.
Reason (R) : Lowering of vapour pressure is directly proportional to the number of species
present in the solution.
12. Assertion : The freezing point of solution is always lower than that of pure solvent.
Reason : Non-volatile solute decreases the vapour pressure of the solvent.
13. Assertion (A) : Depression in freezing point is depends upon Kf.
Reason (R) : It depends molality of solution.
14. Assertion (A) : Cryoscopic constant is also known as molal freezing point depression constant (Kf).
Reason (R) : It contains 100g of solvent for 1 molal solution.
15. Assertion (A) : Kf is not unit less.
Reason (R) : Unit of Kf is K kg mol–1.

ANSWER KEY
node06\B0BC-BD\Kota\Board Material\Chemistry\Booklets\CBSE\Part-1

Q. No. 1 2 3 4 5 6 7 8 9 10
Ans. c a a a b b a c a b
Q. No. 11 12 13 14 15
Ans. d a b c a

54 E
CBSE
ALLEN
CASE BASED QUESTIONS
Case-I
Henry was the first to give a quantitative relation between pressure and solubility of a gas in a
solvent which is known as Henry’s law. The law states that at a constant temperature, the
solubility of a gas in a liquid is directly proportional to the pressure of the gas. Dalton, a
contemporary of Henry, also concluded independently that the solubility of a gas in a liquid
solution is a function of partial pressure of the gas. If we use the mole fraction of a gas in the
solution as a measure of its solubility, then it can be said that the mole fraction of gas in the
solution is proportional to the partial pressure of the gas over the solution. The most commonly
used form of Henry’s law states that “the partial pressure of the gas in vapour phase (p) is
proportional to the mole fraction of the gas (x) in the solution” and is expressed as : p = KHx
Answer the following questions :
(a) Why are aquatic species more comfortable in cold water in comparison to warm water?
Ans. Aquatic species are more comfortable in cold water due to the presence of more oxygen.
Solubility of oxygen in water increases with decrease in temperature as solubility of a gas in
given liquid decreases with increase in temperature.
OR
Why soda water bottle kept at room temperature fizzes on opening?
Ans. Soda water bottle kept at room temperature fizzes on opening due to different pressure inside and
outside the bottle. When the bottle is opened to air, the partial pressure of CO2 above the solution
decreases. As a result, solubility decreases and hence CO2 bubbles out.
(b) Why do gases always tend to be less soluble in liquid as the temperature is raised ?
Ans. As a dilution of a gas in liquid is exothermic process therefore, the solubility should decrease
with in increase in temperature.
(c) Explain the following phenomena with the help of Henry’s law.
(i) Painful condition known as bends.
(ii) Feeling of weakness and discomfort in breathing at high altitude.
Ans. (i)
node06\B0BC-BD\Kota\Board Material\Chemistry\Booklets\CBSE\Part-1

Henry's law represents a relation between solubility of gases in liquid and pressure. Scuba
drivers when comes towards surface, the pressure gradually decreases. This reduce pressure
releases the dissolve gas present in blood and leads to formation of bubbles of nitrogen in
the blood.
(ii) At high altitude atmospheric pressure is low as compared to surface which causes difficulty
in breathing. On that condition we feel weakness and discomfort.

E 55
Chemistry
ALLEN
Case - II
When a non-volatile solute is added to a solvent, the freezing point of the formed solution is
always lower than that of pure solvent. This difference in freezing point is known as depression
in freezing point. If Tfo is the freezing point temperature of pure solvent and Tf is the freezing
point temperature of the solution when non-volatile solute is dissolved in it, then depression in
freezing point (Tf) is given by
Tf = Tfo – Tf

For dilute solutions, Tf = Kf m [where, m = molal concentration of the solution]


Answer the following questions :
(a) Why the freezing point of solution is always lower than that of pure solvent ?
Ans. The freezing point of the solution is always lower than that of pure solvent as the vapour pressure
of the solvent decreases in the presence of non-volatile solute.
(b) Define the proportionality constant (Kf)
Ans. Molal freezing point depression constant (Kf) or cryoscopic constant is defined as the depression
in freezing point for 1 molal solution i.e. a solution containing 1 g mole of solute dissolved in
1000 g of solvent.
(c) Calculate the depression in freezing point of 5% glucose in water.
Ans. Given,
wsolute = 5g, Msolute = 180 g mol–1 wsolvent = 95 g
5 1000
Molality of glucose solution =   0.2924
180 95
Tf = Kf × m
2.15
 Tf =  0.2924  4.08 K
0.154
OR
(i) Write the formula relating depression in freezing point with molar mass of solute.
Ans. Depression in freezing point (Tf) = Kf × m
K f  w solute 1000
w solvent  M solute
node06\B0BC-BD\Kota\Board Material\Chemistry\Booklets\CBSE\Part-1

K f  w solute 1000
Msolute =
Tf  w solvent
(ii) Write the unit of Kf.
Ans. The unit of Kf is K kg mol–1.

56 E
CBSE
ALLEN
PREVIOUS YEARS QUESTIONS
SECTION-A (ONE MARK QUESTIONS)
1. Value of Henry's constant KH : [1] (CBSE 2023)
(a) increases with decrease in temperature
(b) decreases with increase in temperature
(c) increases with increase in temperature
(d) remains constant.
Ans. (c)
2. An azeotropic mixture of two liquids will have a boiling point lower than either of the two
liquids when it [1] (CBSE 2023)
(a) shows a negative deviation from Raoult's law
(b) forms an ideal solution
(c) shows a positive deviation from Raoult's law
(d) is saturated
Ans. (c)
3. Solubility of gas in liquid decreases with increase in [1] (CBSE 2023)
(a) Pressure (b) Temperature
(c) Volume (d) Number of solute molecules
Ans. (b)
4. Assertion (A) : Osmotic pressure is a colligative property. [1] (CBSE 2023)
Reason (R) : Osmotic pressure is proportional to the molality.
Ans. (c) (A) is correct but (R) is wrong.
5. Assertion (A): Elevation in boiling point is a colligative property. [1] (CBSE 2023)
Reason (R): The lowering of vapour pressure of solution causes elevation in boiling point.
Ans. (a)
6. Assertion (A) : The enthalpy of mixing mix H is equal to zero for an ideal solution.
Reason (R) : For an ideal solution the interaction between solute and solvent molecules is
stronger than the interactions between solute-solute or solvent-solvent molecules.
[1] (CBSE 2023)
node06\B0BC-BD\Kota\Board Material\Chemistry\Booklets\CBSE\Part-1

Ans. (c)
7. Increasing the temperature of an aqueous solution will cause : [1] (CBSE 2022 Term -I)
(a) Increase in Molarity (b) Increase in Molality
(c) Decrease in Molarity (d) Decrease in Molality
Ans. (c) An increase in temperature increases the volume of the solution and thus, decreases it's
molarity.
E 57
Chemistry
ALLEN
8. Which of the following conditions is correct for an ideal solution ? [1] (CBSE 2022 Term-I)
(a) Hmix = 0 and Vmix = 0 (b) Hmix > 0 and Vmix > 0
(c) Hmix < 0 and Vmix < 0 (d) Hmix > 0 and Vmix < 0
Ans. (a)
9. For determination of molar mass of polymers and proteins, which colligative property is
used ? [1] (CBSE 2022 Term-I)
(a) Relative lowering in vapour pressure (b) Elevation in boiling point
(c) Osmotic pressure (d) Depression in freezing point
Ans. (c) Osmotic pressure
Because this is the only method which takes place at room temperature and all biomolecules are
unstable above room temperature.
10. Pure water boils at 373.15 K and nitric acid boils at 359.15 K. An azeotropic mixture of H2O
and HNO3 boils at 393.55 K. Distilling the azeotropic mixture will cause :
[1] (CBSE 2022 Term -I)
(a) Pure nitric acid to distil over first
(b) Pure water to distil over first.
(c) One of them to distil over with a small amount of the other.
(d) Both of them to distil over in the same composition as that of the mixture being distilled.
Ans. (d) Some liquid on mixing forms azeotropes which are binary mixtures having the same
composition in liquid and vapour phase and boil at constant temperature. The liquid and vapour
have the same composition and no further separation occurs.
11. A 5% (by mass) solution of glucose (molar mass = 180 g mol–1) is isotonic with 1% solution
(by mass) of a substance 'X'. The molar mass of 'X' is : [1] (CBSE 2022 Term -I)
(a) 36 g mol–1 (b) 18 g mol–1 (c) 72 g mol–1 (d) 900 g mol–1
Ans. (a) For isotomic solution
C1 = C2
n1 n 2 w /m w / m2 5 1
  1 1 2  
v v v v 180 x
x = 180/5 = 36 g mol–1.
12. When 2.5g of a non-volatile solute was dissolved in 50 mL of water, it gave boiling point
elevation of 0.52 °C. The molar mass of the solute is (Kb for water = 0.52 Km–1)
node06\B0BC-BD\Kota\Board Material\Chemistry\Booklets\CBSE\Part-1

[1] (CBSE 2022 Term -I)


(a) 100g mol–1 (b) 50g mol–1 (c) 25g mol–1 (d) 75g mol–1
w 2 1000
Ans. (b) Tb = kbm = k b  
M 2 w1
2.5 1000 0.52  2.5 1000
 0.52 = 0.52 ×   w1   50 g mol1
50 w1 50  0.52
58 E
CBSE
ALLEN
13. To increase the solubility of CO2 gas in soft drinks, the bottle is sealed under :
[1] (CBSE 2022 Term -I)
(a) Low pressure (b) High temperature (c) Constant pressure (d) High pressure
Ans. (d)
To increase the solubility of CO2 gas in soft drink the bottle are sealed under low temperature
and high pressure. Because solubility of gas in liquid increases under low temp and high
pressure. (acc. to Henry's law).
14. A solution of a pair of volatile liquids A and B shows negative deviation from Raoult's law.
This is because :- [1] (CBSE 2022 Term -I)
(a) p A  p oA x A and p B  poB x B (b) The intermolecular forces A–A, B–B < A–B
(c) Both Hmixing and Vmixing are positive (d) All of the above
Ans. (b) Properties of Raoult's law of negative deviation.
(i) Hmix < 0
(ii) Vmix < 0
(iii) PT < PAo XA + PBo XB
15. Assertion (A) : Relative lowering in vapour pressure is a colligative property.
Reason (R) : Relative lowering in vapour pressure depends upon mole fraction of pure solvent.
[1] (CBSE 2022 Term-I)
Ans. (c) (A) is correct but (R) is wrong.
Relative lowering in vapour pressure depends on upon number of solute particles.
16. Which of the following analogies is correct? [1] (CBSE 2022 Term -I)
(a) Chloroform-acetone : Positive deviation : : Ethanol H2O : Negative deviation.
(b) p A  p oA  x A : Henry 's law : : p = KH  x : Raoult's law.
(c) PTotal = pA + pB : Non-ideal solution : : PTotal > pA + pB : Ideal solution
(d)  = CRT : Osmotic pressure :: P >  : Reverse osmosis.
Ans. (d)  = CRT : Osmotic pressure : : P >  reverse osmosis.
Chloroform : acetone show negative derivation due to formation of H-Bond between
chloroform and acetone molecules.
Ethanol : H2O show positive derivation due to weak inter molecular force of attraction.
P = KHX is Henry's law & PA = PAo XA is Raoult's law.
Ptotal = PA + PB for ideal solution = Ptotal > PA + PB for Non ideal solution.
node06\B0BC-BD\Kota\Board Material\Chemistry\Booklets\CBSE\Part-1

17. Assertion (A) : Molarity of a solution in liquid state changes with temperature.
Reason (R) : The volume of solution changes with the change in temperature.
[1] (CBSE 2021 Compartment)
Ans. (a) Both (A) and (R) correct and (R) is correct explanation of (A).
18. Identify which liquid will have a higher vapour pressure at 90°C if the boiling points of two
liquids A and B are 140°C and 180°C, respectively. [1] (CBSE 2020)
Ans. (A)
E 59
Chemistry
ALLEN
SECTION-B (TWO MARKS QUESTIONS)
19. The vapour pressure of pure liquid X and pure liquid Y at 25°C are 120 mm Hg and 160 mm
Hg respectively. If equal moles of X and Y are mixed to form an ideal solution, calculate the
vapour pressure of the solution. [2] (CBSE 2023)
Ans. PT  PA0 X A  PB0 X B
1
XA  XB  (As nA = nB)
2
1 1
PT  120   160 
2 2
= 60 +80
= 140 mm Hg
20. For a 5% solution of urea (Molar mass = 60 g/mol), calculate the osmotic pressure at 300 K.
[R = 0.0821 LatmK-1mol-1] : [2] (CBSE 2020)
OR
Visha took two aqueous solutions - one containing 7.5 g of urea (Molar mass = 60 g/mol) and
the other containing 42.75 g of substance Z in 100 g of water, respectively. It was observed that
both the solutions froze at the same temperature. Calculate the molar mass of Z.
Ans.  = CRT (Volume of solution = 100 mL) [½]
n
  RT [½]
v
5 0.0821 300
  [½]
60 0.1
 = 20.5 atm [½]
OR
Tf(urea) = Tf(Z) [½]
w 1000 w 1000
k ƒ  urea   kf  z  [½]
M urea w solvent M z w solvent
7.5 1000 42.75 1000
   [½]
60 100 Mz 100
42.75  60
Mz =  342 g / mol [½]
7.5
21. Give reasons :
(a) Red Blood Cells (RBC) shrink when placed in saline water but swell in distilled water.
node06\B0BC-BD\Kota\Board Material\Chemistry\Booklets\CBSE\Part-1

(b) Aquatic animals are more comfortable in cold water than in warm water.
[2] (CBSE 2019)
Ans. (a) RBC looses water in saline water and absorb water in distilled water due to osmosis. [1]
(b) In water, oxygen is in dissolved state & as temperature rises, the solubility of oxygen
decreases, It means the solubility of oxygen is less in warm, because of this reason
Aquatic animals are more comfortable in cold water than in warm water. [1]

60 E
CBSE
ALLEN
22. Give reasons :
(a) A decrease in temperature is observed on mixing ethanol and acetone.
(b) Potasium chloride solution freezes at a lower temperature than water. [2] (CBSE 2019)
Ans. (a) Due to positive deviation. [1]
(b) Due to more van't hoff factor. [1]
23. Calculate the freezing point of a solution containing 60 g of glucose (Molar mass = 180 g mol–1)
in 250 g of water.
(Kf of water = 1.86 K kg mol–1) [2] (CBSE 2018)
Ans. w1 = 250 g ; w2 = 60g ; M2 = 180g/mol or 180g mol–1
Kf = 1.86 k kg mol–1
Tf = Kf m [1/2]
w 2 (g)  1000 60  1000
Tf = Kf ×  1.86  [1/2]
M 2  w1 (g) 180  250

= 2.48 K [1/2]
Tf = t(solvent) – t(solution) [1/2]
t(solution) = t(solvent) – Tf
= 273.15 – 2.48 = 270.67 K [1]
24. What concentration of nitrogen should be present in a glass of water at room temperature?
Assume a temperature of 25°C, a total pressure of 1 atmosphere and mole fraction of nitrogen in
air of 0.78 [KH for nitrogen = 8.42 × 10–7 M/mm Hg] [2] [CBSE 2018)

Ans. Given PN2 = 1 atm

Applying Henry's law


PN2  K H . X N2

PN2 1
X N2  
KH 8.42  10 7

n(N 2 ) n(N 2 )
Since X N2  
n(N2 )  n(H 2 O) n(H2 O)
node06\B0BC-BD\Kota\Board Material\Chemistry\Booklets\CBSE\Part-1

1000
n(N2) = XN2 . n(H2O) [ n(H2O) = = 55.5 moles]
18

1
= 7
× 55.5 = 6.59 × 107 moles.
8.42  10

Hence concentration of nitrogen in water = 6.59 × 107 mol L–1.

E 61
Chemistry
ALLEN
25. 100 mg of a protein is dissolved in enough water to make 10.0 mL a solution. If this solution
has an osmotic pressure of 13.3 mm Hg at 25°C, what the molar mass of protein ?
(R = 0.0821 L atm mol–1 K–1 and 760 mm Hg = 1 atm) [2] (CBSE 2018)
Ans. Here w = 100 mg = 0.100 = 0.1 g; V = 10.0 mL = 0.01 L
13.3
 = 13.3 mm Hg = atm ; T = 25°C = 25 + 273 = 298 K
760
R = 0.0821 L atm mol–1K–1; M=?
wRT
 Molar mass, M =
V
.1  0.0821  298 .1  0.0821  298  760 1859.4008
M= = =
13.3
 0.01 13.3  0.01 0.133
760
M = 13980.4 g mol–1
SECTION-C (THREE MARKS QUESTIONS
26. When 19.5g of F–CH2–COOH (Molar mass = 78 g mol–1), is dissolved in 500 g of water, the
depression in freezing point at observed to be 1ºC. Calculate the degree of dissociation of
F–CH2–COOH.
[Given : Kf for water = 1.86 K kg mol–1] [3] (CBSE 2023)
Ans. It is given that : w1 = 500 g,
w2 = 19.5 g
Kf = 1.86 K kg mol–1, Tf = 1 K
K f  w 2 1000
We know that : M2  [½]
Tf  w1

1.86 K kg mol1 19.5g 1000g kg 1


= = 72.54 mol–1 [½]
500 g 1K
Therefore, observed molar mass of CH2FCOOH, (M2)obs = 72.54 mol
The calculated molar mass of CH2FCOOH is :
(M2)cal = 14 +19 + 12 + 16 + 16 + 1 = 78 g mol–1
(M 2 )cal 78g mol1
Therefore, van't Hoff factor, i   = 1.0753 [½]
(M 2 )obs 72.54 g mol1
node06\B0BC-BD\Kota\Board Material\Chemistry\Booklets\CBSE\Part-1

Let a be the degree of dissociation of CH2FCOOH


CH2FCOOH 
 CH2FCOO– + H+
Initial conc. C mol L–1 0 0
At equilibrium C (1 – ) C C
Total = C (1 + )

62 E
CBSE
ALLEN
C 1   
 i [½]
C
 i=1+ [½]
  = i – 1 = 1.0753 – 1 = 0.0753
  = 7.53% [½]
–1
27. Calculate the mass of ascorbic acid (Molar mass = 176gmol ) to be dissolved in 75g of acetic
acid, to lower its freezing point by 1.5°C. (Kf = 3.9 K kgmol–1) [3] (CBSE 2020)
Ans. Tf  K f m [1]
w  1000
Tf  K f  [1]
m W
w 1000
1.5  3.9  [1]
176  75
mass of ascorbic acid (w) = 5.08 g
28. A solution containing 1.9 g per 100 mL of KCl (M = 74.5 g mol–1) is isotonic with a solution
containing 3 g per 100 mL of urea (M = 60 g mol–1). Calculate the degree of dissociation of
KCl solution. Assume that both the solutions have same temperature. [3] (CBSE 2019)
Ans. 1(urea) = 2[KCl]
C1RT = iC2RT [1]
n1 n
i 2 (V1 = V2)
V1 V2
3 1.9
 i [1]
60 74.5
i = 1.96
i  1 1.96  1
  = 0.96 or 96% [1]
n 1 2 1
29. 15.0 g of an unknown molecular material is dissolved in 450 g of water. The resulting solution
freezes at –0.34°C. what is the molar mass of the material ? (Kf for water = 1.86 K kg mol–1)
[3] (CBSE 2019)
Ans. Tf = iKfm
Tf = 273 – (–0.34 + 273) = 273 – 272.66
node06\B0BC-BD\Kota\Board Material\Chemistry\Booklets\CBSE\Part-1

= 0.34 K
15
0.34  450 15
0.34 = 1.86 M  
450 1.86  1000 M
1000
1000  1.86
or M = = 182.35 gm/mol
0.34  30

E 63
Chemistry
ALLEN
30. A solution of glycerol (C3H8O3) in water was prepared by dissolving some glycerol in 500 g of
water. This solution has a boiling point of 100.42°C. What mass of glycerol was dissolved to
make this solution? (Kb for water = 0.512 K kg mol–1) [3] (CBSE 2019)
Ans. Tb = Kb × m
wB
100.42 – 100 = 0.512 × 92 = wB = 37.8 gm
500
1000
31. Give reasons for the following :
(a) Measurement of osmotic pressure method is preferred for the determination of molar
masses of macromolecules such as proteins and polymers.
(b) Aquatic animals are more comfortable in cold water than in warm water.
(c) Elevation of boiling point of 1 M KCl solution is nearly double than that of 1M sugar
solution. [3] (CBSE 2018)
Ans. (a) Protein are high molecular mass material, the magnitude of colligative property depends
inversely on the molecular mass and osmotic pressure is the only colligative property
have measurable magnitude. So, the Osmotic pressure method is preferred for the
determination of molar mass of macromolecules as proteins & polymers. [1]
(b) In water, oxygen is in dissolved state & as temperature rises, the solubility of oxygen
decreases, It means the solubility of oxygen is less in warm, because of this reason
Aquatic animals are more comfortable in cold water than in warm water. [1]
(c) The value of Van't Haff factor is twice in 1M KCl than 1M sugar solution due to which
elevation of Boiling point is more.
The solvent is somewhat but KCl is ionic due to which it dissociates completely.
The elevation of B.P. is Tb  i K b m {In both 1 M KCl & 1M sugar solution} [1]
32. What concentration of nitrogen should be present in a glass of water at room temperature?
Assume a temperature of 25°C, a total pressure of 1 atmosphere and mole fraction of nitrogen
in air of 0.78 [KH for nitrogen = 8.42 × 10–7 M/mm Hg] [3] [CBSE 2018)
Ans. Given PN2 = 1 atm
Applying Henry's law
PN2  K H . X N2
PN2 1
X N2  
KH 8.42  10 7
n(N 2 ) n(N 2 )
node06\B0BC-BD\Kota\Board Material\Chemistry\Booklets\CBSE\Part-1

Since X N2  
n(N2 )  n(H 2 O) n(H 2 O)
1000
n(N2) = XN2 . n(H2O) [ n(H2O) = = 55.5 moles]
18
1
= 7
× 55.5 = 6.59 × 107 moles.

8.42 10
Hence concentration of nitrogen in water = 6.59 × 107 mol L–1.

64 E
CBSE
ALLEN
SECTION-D (FOUR MARKS QUESTIONS)
33. Boiling point of liquid solution would be affected by the dissolved solids in the liquid phase. A
soluble solid in solution has the effect of raising its boiling point. The addition of non-volatile
substances to a solvent decreases the vapour pressure and the added solute particles affect the
formation of pure solvent crystals.
According to many researches the increase in boiling point directly correlated to the
concentration of solutes dissolved in the solvent. This phenomenon is expressed as boiling
point elevation and it is useful for several applications such as boil concentration of liquid food
and to find the molar mass of an unknown solute in the solution.
The boiling point elevation is referred as a colligative property and it is proportional to the
molar concentration of the solution (m), along with vapour pressure higher, boiling point
elevation and osmotic pressure. These are physical characteristics of solutions that depend only
on the identity of the solvent and the concentration of the solute. The characters are not
depending on the solute's identity.
The following questions are multiple choice questions. Choose the most appropriate
answer:
(a) Two liquids A and B boil at 145°C and 190°C respectively. Which of them has higher
vapour pressure at 80°C ? [1]
Ans. Liquid A with lower boiling point is more volatile and hence will have higher vapour
pressure.
(b) Define molal elevation constant or ebullioscopic constant ? [1]
Ans. Molal elevation constant is defined as the elevation in boiling point when molality is
equal to one.
(c) What are maximum boiling azeotropes ? Give one example. [2]
Ans. The solutions which show a large positive deviation from Raoult's law form minimum
boiling azeotropes at a specific composition.
Example : Ethanol water mixture.
OR
Why is the elevation in boiling point of water differnet in the following two solutions ?
(i) 0.1 molar NaCl solution
(ii) 0.1 molar sugar solution
Ans. NaCl is an electrolyte. It dissociates to give Na+ and Cl– ions in the solution. Sugar is a
non-electrolyte. It does not dissociate in the solution. Thus, the number of particles is
node06\B0BC-BD\Kota\Board Material\Chemistry\Booklets\CBSE\Part-1

greater in 0.1 M NaCl solution than in 0.1 M sugar solution. Hence elevation in boiling
point is greater in case of NaCl solution.
SECTION-E (FIVE MARKS QUESTIONS
34. (a) State Henry’s law and mention its two applications. [2]
(b) 5% aqueous solution of a non-volatile solute was made and its vapour pressure at 373 K
was found to be 745 mm. Vapour pressure of pure water at this temperature was
760 mm. Calculate the molar mass of solute. [3]
E 65
Chemistry
ALLEN
OR
(a) Give two differences between ideal and non-ideal solutions. [2]
–1
(b) Calculate the amount of NaCl (M = 58·5 g mol ) that must be added to 100 g of water
so that freezing point is depressed by 2 K. Kf for water is 1·86 K/m. [3]
[2 + 3 = 5] (CBSE 2021 Compartment)
Ans. (a) The partial pressure of the gas in vapour phase (P) is proportional to the mole fraction of
the gas (X) in the solution. (This is most common definition) [1]
P = KHX KH = Henry's Constant
Applications : [1]
(1) To increase the solubility of CO2 in soft drinks and soda water, the bottle is
sealed under high pressure.
(2) At high altitudes the partial pressure of oxygen is less than that at the ground level.
This leads to low concentration of oxygen in the blood and tissues of people living
at high altitudes or climbers. Low blood oxygen causes climbers to become weak
and unable to think clearly, symptoms of a condition known as anoxia.
PA0  Ps n B Given:
(b) Raoult's law,  [1]
Ps nA WB(mass of solute) = 5g in 100 g of solution
760  745 5 / m B WA = 95 g mass of solvent) ;
  [1]
745 95 / m B mA (molecular mass of water) = 18 g
mB (molecular mass of solute) = ?
5  18  745
mB =  47.05g / mol [1]
15  95 PAo (V.P. of pure water) = 760 mmHg ;
Ps (V.P. of solution after addition of solute)
= 745 mm Hg
OR
(a) Ideal solution Non-ideal solution
(i) It obeys Raoult's law over the (i) Does not obey Raoult's law over the
entire range of concentration. entire range of concentration.
(ii) mix H = 0 (ii) mix H is not equal to 0.

(b) Tf = i × kf × m [1] NaCl  Na+ + Cl–  i = 2.


w 1000 w1 = 100 g
 Tf = i × kf ×  [½]
node06\B0BC-BD\Kota\Board Material\Chemistry\Booklets\CBSE\Part-1

m w M2 = 58.5 g mol–1
w 1000
2 = 2 × 1.86 ×  [½]
58.5 100
58.5
w =
1.86 10
w = 3.145 g [1]

66 E
CBSE
ALLEN
PRACTICE TEST

SECTION-A
1. Isotonic solutions have same [1]
(a) vapour pressure (b) freezing temperature
(c) osmotic pressure (d) boiling temperature
2. The mixture which shows positive deviation from Raoult's law is :- [1]
(a) Chloroethane + Bromoethane (b) Ethanol + Acetone
(c) Benzene + Toluene (d) Acetone + Chloroform
3. Which of the following statements is correct regarding a solution of two compounds A and B
exhibiting positive deviation from ideal behaviour? [1]
(a) Intermolecular attractive forces between A-A and B-B are stronger than those between A-B.
(b) mix H = 0 at constant T and P
(c) mix V = 0 at constant T and P
(d) Intermolecular attractive forces between A-A and B-B are equal to those between A-B.
4. The density of 2 M aqueous solution of NaOH is 1.28 g/cm3. The molality of the solution is
[Given that molecular mass of NaOH = 40 g mol–1] [1]
(a) 1.20 m (b) 1.56 m (c) 1.67 m (d) 1.32 m
5. For an ideal solution, the correct option is :- [1]
(a) mix S = 0 at constant T and P (b) mix V  0 at constant T and P
(c) mix H = 0 at constant T and P (d) mix G = 0 at constant T and P
Note : In the following questions a statement of assertion followed by a statement of reason is
given. Choose the correct answer out of the following choices.
(a) Both (A) and (R) correct and (R) is correct explanation of (A).
(b) Both (A) and (R) correct and (R) is not correct explanation of (A).
(c) (A) is correct but (R) is wrong.
(d) (A) Assertion is wrong but (R) reason is correct.
6. Assertion (A) : NaCl in water and organic acids in benzene show abnormal molecular mass. [1]
Reason (R) : Abnormal molecular mass is obtained when the substance in the solution undergoes
dissociation or association.
7. Assertion (A) : Osmotic pressure is a colligative property. [1]
Reason (R) : Osmotic pressure of a solution depends on the molar concentration of solute at any
temperature T.
node06\B0BC-BD\Kota\Board Material\Chemistry\Booklets\CBSE\Part-1

SECTION-B
8. State the following ;
(i) Henry’s law about partial pressure of a gas in a mixture. [1]
(ii) Raoult’s law in its general form in reference to solutions. [1]
9. (i) Write the colligative property which is used to find the molecular mass of macromolecules [1]
(ii) In non-ideal solution, what type of deviation shows the formation of minimum boiling
azeotropes? [1]
E 67
Chemistry
ALLEN
10. Define the term 'osmotic pressure'. Describe how the molecular mass of a substance can be
determined on the basis of osmotic pressure measurement. [2]
11. Define the following terms:
(i) Molarity [1]
(ii) Molal elevation constant (Kb) [1]
SECTION-C
12. Calculate the temperature at which a solution containing 54 g of glucose, C6H12O6, in 250 g of
water will freeze. [Kf for water = 1.86 K kg mol–1] [3]
13. A solution containing 8g of a substance in 100 g of diethyl ethane boils at 36.86 °C, whereas
pure ether boils at 35.60 °C. Determine the molecular mass of the solute.
(For ether Kb = 2.02 K kg mol–1) [3]
SECTION-D
14. When a solution does not obey Raoult's law over the entire range of concentration, then it is
called non-ideal solution. The vapour pressure of such a solution is either higher or lower than
that predicted by Raoult's law. If it is higher, the solution exhibits positive deviation and if it is
lower, it exhibits negative deviation from Raoult's law. This variation is due to change in the
strength of the forces of attraction between solute and solvent molecules as compared to pure
solvent or solute. [1+1+2=4]
Answer the following questions :
(a) 10 mL of liquid A was mixed with 10 mL of liquid B. The volume of the resulting solution
was found to be 19.9 mL. What do you conclude ?
OR
Two liquids A and B on mixing produce a warm solution. Which type of deivation from
Raoult's law does it show ?
(b) Why is the vapour pressure of a solution of glucose in water lower than that of water ?
(c) Give reasons :
(a) Cooking is faster in pressure cooker than in cooking pan.
(b) An increase in temperature is observed on mixing chloroform and acetone.
SECTION-E
15. (a) A solution prepared by dissolving 8.95 mg of a gene fragment in 35.0 mL of water has an
node06\B0BC-BD\Kota\Board Material\Chemistry\Booklets\CBSE\Part-1

osmotic pressure of 0.335 torr at 25ºC. Assuming that the gene fragment is a non-electrolyte,
calculate its molar mass.
(b) What mass of NaCl (molar mass = 58.5 g mol-1 be dissolved in 65 g of water to
tower the freezing point by 7.5ºC? The freezing point depression constant, Kf, for water is
1.86 K kg mol-1. Assume van’t Hoff factor for NaCl is 1.87. [2+3=5]

68 E
CBSE
ALLEN
PRACTICE TEST SOLUTIONS
SECTION-A
1. (c)
2. (b)
3. (a)
4. (c)
5. (c)
6. (a)
7. (a)
SECTION-B
8. (i) Henry's law : It states that at constant temperature, the solubility of a gas in a liquid is
directly proportional to the pressure of the gas.
(ii) Raoult’s law : It states that for a solution of volatile liquids, the partial vapour pressure of
each component in the solution is directly proportional to its mole fraction.
9. (i) Osmotic pressure
(ii) Positive deviation from Raouts’ law/ Positive deviation
10. Ans. (i) Osmotic pressure : It is the pressure of the solution column that can prevent the
entry of solvent molecules through a semipermeable membrane, when the solution and
the solvent are separated by the same. It is shown by . It's unit is mm of Hg of
atmosphere.
(ii) Relation between osmotic pressure and molar mass :
We know that V = BRT (Solution equation)

w B RT
or V =
MB

wB
( B = , where wB is mass of the solute and MB is molecular mass of the same)
MB

Thus V MB = wBRT

w B RT
or, MB =
node06\B0BC-BD\Kota\Board Material\Chemistry\Booklets\CBSE\Part-1

V
This is the required relation.
11. (i) Molarity : Molarity (M) is defined as the number of moles of the solute dissolved in one Litre
of solution.
(ii) Molal elevation constant can be defined as the elevation in boiling point produced when one
mole of non-volatile solute is dissolved in 1 kg of the solvent.

E 69
Chemistry
ALLEN
SECTION-C
12. Molecular mass of glucose , MB = 72 + 12 + 96 = 180
K f  w B 1000 1.86  54 1000 100440
Tf =    2.23
MB  w A 180  250 45000
Freezong point of solution = 0 – 2.23 = – 2.23°C
13. We have, mass of solute, w2 = 8g
mass of solvent, w1 = 100g
elevation of boiling point, Tb = 36.86 – 35.60 = 1.26°C
Kb = 2.02 K kg mol–1
 Molecular mass of the solute
1000  w 2  K b 1000  8  2.02 161.6
M2 = =   128.25 g mol 1
Tb  w1 1.26  100 1.26
SECTION-D
14. (a) Due to increase in extent of attraction forces between molecules A and B there is a slight
reduction in volume and mixture will show negative deivation from the Raoult's law.
OR
Warming up of the solution means that the process of mixing is exothermic, i.e.,
Hmixing = –ve. This implies that the solution shows a negative deviation.
(b) A part of the water surface is occupied by non-volatile glucose molecules. This decreases
the effective surface area for the vapourisation of water molecules consequently, the vapour
pressure of a solution of glucose in water is lower than that of water.
(c) (a) Due to increase of pressure in cooker, boiling point of water increases
(b) Due to negative deviation
SECTION-E
15. (a) wB = 8.95 mg = 8.95 × 10 g–3

0.335
 = 0.335 torr = atm , T = 25ºC = 25 + 273 = 298 K.
760
R = 0.0821 L atm K–1 mol–1
V = 35 ml = 35 × 10–3 L
w B RT 8.95 103 g  0.0821L atm K 1mol1  298 K  760
Molar Mass (MB) = 
V 0.335atm  35 103 L
= 14193.29 g mol–1.
(b) Here M2 = 58.5 g mol–1, w1 = 65 g, w2 = ?
Tf = 7.5ºC, i = 1.87, Kf = 1.86 K kg mol–1
node06\B0BC-BD\Kota\Board Material\Chemistry\Booklets\CBSE\Part-1

Using the formula,


Tf = i Kf m
w 2  1000 w  1000
Tf  i K f   7.5  1.87  1.86  2
M 2  w1 58.5  65
7.5  58.5  65 28518.75
 w2    8.199 g
1.87  1.86  1000 3478.2

70 E
CBSE
ALLEN
UNIT-3 : ELECTROCHEMISTRY
UNIT INDEX
• Theory 71-90
• Exercise-1_Intext Questions 91-93
• Exercise-2_NCERT Exercise Questions 94-103
• Exercise-3_Exemplar 104-108
• Exercise-4_MCQ, A/R, Case Based Questions 109-116
• Previous Years Questions 117-131
• Practice Test 132-136
THEORY
INTRODUCTION :
Electrochemistry– The branch of physical chemistry which deals with the relationship between
electrical energy and chemical energy and their conversion of one form to another known as
Electrochemistry.
• The electrochemical cell is a device by which such conversions takes place.
• Electrochemical cell are of following two types:
(1) Galvanic cell: In which chemical energy is converted into electrical energy.
(2) Electrolytic cell: In which electrical energy is converted into chemical energy.
DANIELL CELL:
e–

ZnSO4(aq) CuSO4(aq)
node06\B0BC-BD\Kota\Board Material\Chemistry\Booklets\CBSE\Part-1

Fig. : DANIELL CELL


(i) First Half cell reaction:
Zn(s)  Zn 2 (aq)  2e  (oxidation)
(ii) Second Half cell reaction:
Cu 2 (aq)  2e   Cu(s) (Reduction)

E 71
Chemistry
C
N
ALLEN
Net ceell equation
n–
Zn(s)  Cu 2 (aq)  Zn 2 (aq)  Cu(s)
Cell R
Representattion–
Zn | Znn 2 (aq) || C
Cu 2 (aq) | Cu
u(s)
Anodde Salt Bridgge Cathode

(i) IIn Daniell cell, zinc electrode


e iss dipped in zinc sulph
hate solutioon and copp
per electrodde
ddipped in coopper sulphate solutionn.
(ii) T h oxidation
The electrodde on which n reaction takes
t place known as aanode and the electrodde
oon which reeduction reeaction takees place, kno
own as cath
hode.
(iii) IIn this cell, internal circcuit compleeted by usin
ng salt bridg
ge.
Salt
S Bridge and its fun
nctions:
 Salt brridge is inveerted U-tubee containingg solution of
o inert electtrolyte withh agar-agar or
o gelatine to
t
converrt into semi solid form.
 Ions oof inert electtrolyte do not
n involve iin any chem
mical changee.
 The ellectrolytes arre used in saalt bridge Exx. KCl, KNO
O3, NH4NO
O3.
 If Ag+, Hg2+2, Pb+2 ions are present in a cell then KCl is not used becauuse precipittate of AgC
Cl,
Hg2Cl2, PbCl2 aree formed.
Functions
F
 It connnects the sollution of tw
wo half cell tto complete the circuit.
 It mainntains the ellectrical neu
utrality of thhe solution in order to give continnuous flow or
o generatioon
of currrent.
 If the ssalt bridge is removed then
t voltagee drops to zero.
z
 It prevvents the liquuid -liquid junction
j pottential. Thee potential difference
d w
which arises between tw
wo
solutioons when theey brought in contact w
with each otther.
1.
1 When
n Eext < 1.1 V
(i) E
Electrons flow from Zn
n rod to Cu rod hence current
c flow
ws from Cu tto Zn.
(ii) Z
Zn dissolves at anode and
a copper ddeposits at cathode.
node06\B0BC-BD\Kota\Board Material\Chemistry\Booklets\CBSE\Part-1

72
2 E
 CBSSE
ALLEN
A
2.
2 When n Eext = 1.1 V
(i) N No flow of electrons orr current.
(ii) NNo chemicaal reaction

3.
3 When Eext > 1.1 V
(i) EElectrons flow from Cu u to Zn and current flowws from Znn to Cu.
(ii) Z
Zinc is depoosited at thee zinc electrrode and cop
pper dissolv
ves at coppeer electrode

ZnnSO4 Cu
uSO4

Cell
C Potentiial–
The
T potentiaal differencce between the electroode potential (reductio on potentiall) of the tw wo half cellls
(aanode & catthode) is caalled as cell potential, w
when cell is used in thee circuit.
Electromoti
E ive Force (E EMF)–
The
T potentiaal differencee between th he electrodee potential (reduction
( potential)
p off the two haalf cells wheen
no
n current iss withdrawnn from the cell.
Electrode
E P
Potential–
The
T potentiaal differencce between the electroode and elecctrolyte is called c electtrode potenntial. It is thhe
teendency of an electrode to lose or gain electroon when it isi placed in salt solutioon of its ownn ion.
Standard
S ellectrode pootential  Eocell
c 

When
W the cooncentrationn of all the species invvolved in haalf cell is 1.0 M conceentration, 1 bar pressure
and
a 298 K teemperature then electro d electrode ppotential  E ocell  .
ode potentiaal is known as standard
0
E 0Reductiion = –E Oxidat
tion

Electrode
E pootential are two
t types :
node06\B0BC-BD\Kota\Board Material\Chemistry\Booklets\CBSE\Part-1

1.
1 Oxidaation potenttial–
M 

Oxidation
O
 M n   ne 
 SOP = E oM/ M  n
S (SOP = Staandard Oxid
dation Poten
ntial)
2.
2 Reducction potenntial–
n 
M  ne  
Reducttion
M
 SRP = E oM n / M
S (SRP = Staandard Redu
uction Potential)

E 73
3
Chemistry
ALLEN
Note-1: According to IUPAC convention, standard reduction potential (SRP) now called as standard
electrode potential.
Note-2: Eo depends upon concentration of solution, nature of metal and condition of temperature and
pressure.
THE DIFFERENCE IN BETWEEN ELECTROLYTIC CELL AND GALVANIC CELL
Electrolytic cell Galvanic cell/Voltaic cell
1. In this cell, electrical energy is converted 1. In this cell, chemical energy is
into chemical energy converted into electrical energy
2. The cell reaction is non-spontaneous reduction 2. The cell reaction is spontaneous redox
reaction. (Therefore, G = + ve ; E ocell   ve ) reaction (G = – ve ; E ocell   ve )

3. In this cell, anode and cathode are dipped in 3. In this cell, anode and cathode are dipped
same compartment. in different electrolyte solution and
separated by salt Bridge.
4. In this cell positive electrode is known as 4. In case of galvanic cell, the electrode on
anode and –ve electrode, known as cathode. which oxidation reaction takes place,
known as anode while the electrode on
which reduction reaction takes place,
known as cathode.
ELECTROCHEMICAL SERIES (ECS):
Arrangement of different elements in increasing order of SRP is known as electrochemical
series or activity series.
Maximum –ve
value means
• easy oxidation
• strongest reducing agent
• good metal
Eo=0 H2

Maximum +ve
value means
• easy reduction
• strongest oxidising agent
• good non-metal
node06\B0BC-BD\Kota\Board Material\Chemistry\Booklets\CBSE\Part-1

APPLICATIONS Of ECS:
(i) It is used to determine the relative oxidising and reducing nature of metal.
 If metal have less Eo value then it act as reducing agent.
(ii) It is used to determine the relative displacement power of metal.
 If metal have low Eo value then it displaces the metal from it’s solution having
higher Eo value.

74 E
CBSE
ALLEN
(iii) It is used to determine the Galvanic cell. By determining anode and cathode.
 Also used to calculate to the cell potential of Galvanic cell.
E ocell = E ocathode - E oanode
(iv) It is used to determine the feasibility of the cell.
If E ocell > 0 , cell is feasible.

If E ocell = 0 , cell in equilibrium.

If E ocell < 0 , cell is not feasible.


(v) It is used to determine the free energy change of the reaction.
 r G o = –nFE ocell
(vi) It is used to determine the stability of metal ion in aqueous solution.
  If Eo value of the metal ion decreases; then stability of an element in aqueous
solution increases.
NERNST EQUATION:
Mn+ (aq) + ne–  M(s)
RT [M]
E (M n / M)  E 1(M n /M)  ln n 
nF [M ]
But concentration of solid M is taken as unity.
RT 1
E (M n / M)  E 1(M n /M)  ln n 
nF [M ]
In Daniell cell, the electrode potential for any given concentration of Cu2+ and Zn2+ ions.

2.303RT [Zn 2 ]
E cell  E ocell  log10
nF [Cu 2 ]
where, R = Gas constant = 8.314 J K–1 mol–1
T = 298 K, F = 96500 C mol–1, n = 2

0.0591 [Zn 2 ]
E cell  E ocell  log10
2 [Cu 2 ]

EQUILIBRIUM CONSTANT (KC) FROM NERNST EQUATION :


node06\B0BC-BD\Kota\Board Material\Chemistry\Booklets\CBSE\Part-1

If the circuit in Daniell cell is closed then we note that the reaction
Zn(s) + Cu2+(aq)  Zn2+(aq) + Cu(s)
takes place and as time passes, the concentration of Zn2+ keeps on increasing while the concentration of
Cu2+ keeps on decreasing. At the same time voltage of the cell as read on the voltmeter keeps on
decreasing. After some time, we shall note that there is no change in the concentration of Cu2+ and Zn2+
ions and at the same time, voltmeter gives zero reading.

E 75
Chemistry
ALLEN
This indicates that equilibrium has been attained. In this situation the Nernst equation may be written as:
2.303RT [Product]
E cell  E ocell  log10
nF [Reactant]
2.303RT [Zn 2 ] [Zn 2 ]
0  E o(cell)  log (ECell = 0 and  K C at equilibrium)
2F [Cu 2 ] [Cu 2 ]

or E o(cell) 
2.303RT
2F
log K C  E 0cell  1.1V 

0.0591V 0.0591V
 E o(cell)  log K c  1.1V  log K c
2  2
(1.1V  2)
log K C   37.288
0.0591V
Kc = antilog (37.288)
K C  2  1037 at 298 K
Gibbs Free Energy:
The energy associated with a chemical reaction, that can be used to do work ; known as Gibbs free
energy.
Electrical work done in one second is equal to electrical potential multiplied by total charge passed. If
we want to obtain maximum work from a galvanic cell then charge has to be passed reversibly. The
reversible work done by a galvanic cell is equal to decrease in its Gibbs energy and therefore, if the
emf of the cell is E and nF is the amount of charge passed and rG is the Gibbs energy of the reaction,
then
 rG = –nFEcell
It may be remembered that Ecell is an intensive parameter but rG is an extensive thermodynamic
property and the value depends on n. Thus, if we write the reaction
Zn(s) + Cu2+(aq)  Zn2+(aq) + Cu(s)
rG = – 2FEcell
o
If the concentration of all the reacting species is unity (standard condition), then Ecell = E cell
rGo =  nFE cell
o

rGo = –RT ln KC  rGo = –2.303 RT log KC


MEASUREMENT OF ELECTRODE POTENTIAL :
The potential of individual half-cell cannot be measured. We can measure only the difference between
the two half-cell potentials that gives the emf of the cell.
node06\B0BC-BD\Kota\Board Material\Chemistry\Booklets\CBSE\Part-1

For the determination of single electrode potential, this electrode is connected to any reference
electrode.
• As a reference electrode we use standard hydrogen electrode (SHE)
• Electrode potential of SHE is assumed zero.
o
ESHE =0V
• Cell potential or emf of cell find out by following formula.
E ocell  E ocathode  E oanode

76 E
CBSE
ALLEN
STANDARD HYDROGEN ELECTRODE (SHE) :

H2(g)
at 1 bar

Finely divided
platinum coated
HCl(aq) on platinum foil
1 M H+

• In this cell we can use only platinum or gold wire because they are inert towards air as well as
electrolytes. Hence it is also known as inert electrode.
• Pt or gold electrode dipped in 1.0 M HCl solution.
• Hydrogen gas passed through in cell at 1 bar.

• E ocell  0 V

• If SHE act as anode :


H 2  2e   2H  (oxidation)
(aq.)

Pt | H 2 (g, 1bar)|2H  (aq,1M)


• If SHE act as cathode :
2H  (aq, 1M)|H 2 (g,1bar) | Pt
CONDUCTORS :
Substances or materials which allow electric current to flow through them are called conductors.
Examples - Metals, Aqueous solution of acids, bases and salts, fused salts and impure water etc.
Conductors are of two types -
(i) Metallic conductors or electronic conductors
(ii) Electrolytic conductors or ionic conductors
(i) Metallic conductors : The conductors which conduct electric current by the movement of
electrons without undergoing any chemical change are known as metallic conductors.
Metals - Cu, Ag, Fe, Al etc, non metals - graphite
(ii) Electrolytic conductors : Those substances which conduct the electric current through
ions in aqueous solution or molten state are called electrolytic conductors.
node06\B0BC-BD\Kota\Board Material\Chemistry\Booklets\CBSE\Part-1

• Strong electrolyte : Electrolytes which are completely ionized in aqueous solution


are called strong electrolyte.
Ex. : Salts, strong acid and strong base
• Weak electrolyte :- Electrolytes which are not completely ionized in aqueous
solution are called weak electrolytes.
Ex. : All carbonic acids (except sulphonic acid), CH3COOH, HCN, NH3, amine, etc.

E 77
Chemistry
ALLEN
Factors affecting electrolytic conductance–
(1) Nature of electrolyte
(2) Temperature
(3) Concentration of solution
(4) Size of ions
(5) Solvation of ions
(6) Nature of solvent and its viscosity
Difference between metallic and electrolytic conductor
Metallic conductor Electrolytic conductor
(i) Charge carrier are free electrons Charge carrier are free ions.
Flow of electricity takes place without Flow of electricity takes place by chemical
(ii)
any chemical change changes at electrodes.
(iii) No transfer of matter takes place. Transfer of matter takes place in the form of ions
Resistance is due to inter ionic attraction and
(iv) Resistance is due to vibrations of Kernels
viscosity of medium.
The resistance increases with the increase The resistance decreases with the increase in
(v)
in temperature temperature.
Faraday's laws of electrolysis are not
(iv) Faraday's laws of electrolysis are followed.
followed.
Some Important Terms:
1. Resistance (R) :- Metallic and electrolytic conductors obey ohm's law according to which the
resistance of a conductor is the ratio of the applied potential difference (V) to the current (I)
flowing .
V
R R is expressed in ohms.
I

Relation between resistance (R), length of conductor () and area of cross section (A)

When electric current is passed through any electrolyte then the


resistance (R) of a conductor is directly proportional to its length (l)
node06\B0BC-BD\Kota\Board Material\Chemistry\Booklets\CBSE\Part-1

and inversely proportional to its area of cross section (A)



 R A A
A

R  · 
A
where,  = Resistivity or Specific resistance

78 E
CBSE
ALLEN
2. Specific resistance / Resistivity ():
A
  R.

If  = 1 cm, A = 1 cm2, therefore  = R

 V = A ×  = 1 cm3
Therefore resistance offered by 1 cm3 electrolytic solution is known as resistivity.
3. Conductance (G):
The inverse of resistance, equivalent to conductance
It is the property by virtue of which it favours flow of current. The conductance of a conductor is
1
equal to reciprocal of resistance G 
R

unit of G is mho or ohm –1 or Siemens (S).

Unit of conductance
1 1
 G  
R ohm
    ohm–1  mho  Simens(s)
4. Conductivity or Specific conductance ()
1
It is defined as the reciprocal of specific resistance,  

 R   
A

1
  
   RA

  GG*
Specific conductance = Conductance × Cell constant
Hence specific conductivity of a solution is defined as the conductance offered by 1 cm3 of
electrolytic solution. Unit of  is ohm 1 cm 1 or Scm 1

5. Cell constant (G*) :



G*  ; Its unit is cm–1 or Siemen (S)
A
node06\B0BC-BD\Kota\Board Material\Chemistry\Booklets\CBSE\Part-1

6. Molar Conductivity (m) or  m


If the concentration is expressed in moles per lit then it is called molar conductivity.
or
It is defined as the conductance of all the ions produced by one mole of electrolyte present in the
given volume of solution
  m =  × V

E 79
Chemistry
ALLEN
1
or m =  ×
C
(V = volume of solution containing 1 mole of electrolyte)

or m  [M = Molar concentration]
M
If unit of M is taken in molarity or mol L–1 then
  1000
Important for numerical m 
M
In case of SI unit molar conductivity expressed in Sm2mol–1 and concentration mol. m–3
 L   mol 
because 1 mol. m–3 = 1000  3  × Molarity  
m   L 

 m 
M
(S.m 1 )
m 
(1000 Lm 3 )  Molarity(mol L1 )
(S.m 1 ) 1000(cm3 L–1 )
 m (Scm 2 mol –1 ) 
Molarity(mol L1 )

 Unit of molar conductivity  m  Sm 2 mol1

Note-1 : Conductance of strong as well as weak electrolyte increases with increase in dilution
(decrease in conc.) This is because as dilution increase free movement of ions increase.
Hence G increases.
Note-2 : With increase in dilution, the number of ion per cm3 of solution decrease and since  is the
measures conductance of 1 cm3 of solution. Hence  decreases.
Note-3 : The molar conductance   m  and equivalent conductance   eq  increase with increase in
dilution because they are the product of specific conductivity and volume of solution
containing 1 mole or 1 gm equivalent of electrolyte. With increase in dilution although 
decrease, but the decrease is less than increase in volume therefore,  m and  eq increases.
VARIATION OF MOLAR CONDUCTANCE OF SOLUTION WITH CONCENTRATION FOR
STRONG AND WEAK ELECTROLYTE
1. Weak electrolyte ionise only to a small extant so the conductivity of the these are very low
as compared to strong electrolyte.
node06\B0BC-BD\Kota\Board Material\Chemistry\Booklets\CBSE\Part-1

2. Their conductivity increase with increase of dilution due to increasing of ionisation.


3. ‘KOHLRAUSH’ gives the following relation for the molar conductance of strong
electrolyte at different concentration or dilution.
1
 m   om – AC 2
 m   om – A C ….(i) (It is Debye Huckel Onsager Equation)

80 E
CBSE
ALLEN
 M   CM   C = Molar concentration
 oM   M    = Molar concentration when concentration approaches zero or at  dilution.
(It is known as limiting molar conductivity)
Eq.(i) follows y = –mx + C

for strong electrolyte




4. Weak electrolyte do not completely ionise in aqueous solution on further dilution.


It shows measurable changes hence graph for weak electrolyte obtained as curve

Weak
electrolyte

ARRANGEMENT FOR MEASUREMENT OF RESISTANCE OF A SOLUTION OF AN


ELECTROLYTE

R 1R 4
Unknown resistance R 2 
R3

KOHLRAUSCH LAW OF INDEPENDENT MIGRATION OF IONS


According to Kohlrausch law “The limiting molar conductivity of an electrolyte can be represented as
node06\B0BC-BD\Kota\Board Material\Chemistry\Booklets\CBSE\Part-1

the sum of the individual contributions of the cation and anion of the electrolyte.”
  om  V  o  V  o– 
 V+ = Number of cation
V– = Number of anion
 o  Molar conductivity of cation
 o–  Molar conductivity of anion
E 81
Chemistry
ALLEN
Example
(1)  om( NaCl)  1  oNa   1  oCl–

(2)  om(CaCl2 )  1  oCa 2  2   oCl–

(3)  om Al2 (SO 4 )3  2   oAl3  3  SO


o
–2
4

(4)  om (CH 3COOH)  1  oCH COO–  1  oH


3

(5)  (CH 3COONa)  1 


o
m
o
CH3COO –
 1  oNa 

Applications of Kohlrausch’s law–



0

(1) Used for calculating for weak electrolyte


m

Ex.  0m CH 3COOH  ?
Given  0m CH 3COONa  x
 0m HCl  y
 0m NaCl  z
Ans.  0m CH 3COOH   0m CH 3COO    0m H  ..............(1)
 0m CH 3COONa   0m CH 3COO    0m Na  ..............(2)
 0m HCl   0m H    0m Cl ..............(3)
 0m NaCl   0m Na    0m Cl ..............(4)
Add and subtract  0m NaCl from RHS in equation ..............(1)
 0m CH 3COOH   0m CH 3COO    0m Na    0m H    0m Cl  ( 0m Na    0m Cl )
Thus,  0m CH 3COOH   0m CH 3COONa   0m HCl   0m NaCl
=x+y–z
(2) Degree of dissociation
The degree of dissociation of an electrolyte electron is always equal to the ratio of molar
conductivity of an electrolyte at given concentration (c) to the molar conductivity of an
electrolyte at infinite dilution (  m ).
 m  cm
 o  
m m
(3) For calculation of dissociation constant of weak electrolyte.
node06\B0BC-BD\Kota\Board Material\Chemistry\Booklets\CBSE\Part-1

 CH 3 COO –  H 
CH 3 COOH 
At initial : 1mol 0 0
After dissocation : (1 – )  
1–   
V V V
C(1 – ) C C

82 E
CBSE
ALLEN
[CH 3COO – ][H  ]
 Ka  
[CH 3COOH]
C   C C 2
  Ka   
C(1 – ) (1 – )
C 2
  Ka  
1 –  
ELECTROLYSIS :
The process of chemical decomposition of an electrolyte by the passage of electricity is called
electrolysis or electrolytic dissociation. It is carried out in electrolytic cell where electrical energy is
converted into chemical energy.
For electrolysis two suitable electrodes are immersed in the solution of an electrolyte. When an electric
potential is applied between the electrodes, the positive ions move towards the cathode and negative
ions move towards the anode. The gain of electrons (decrease in oxidation number) means reduction
takes place at the cathode and loss of electrons (increase in oxidation number) means oxidation takes
place at anode.

Inert
electrode

Products of electrolysis– The products formed during electrolysis depend on


(i) Nature of the metal electrodes
(ii) Relative discharge Potential of ions
Examples of electrolysis:
(a) Electrolysis of Molten Sodium Chloride : It produces sodium metal at the cathode and
chlorine gas at the anode. This happens according to the following reactions :
At cathode : 2Cl– (aq)  Cl2(g) + 2e–
At anode : Na+ (aq) + e–  Na(s)
(b) Electrolysis of aqueous solution of sodium chloride (brine) It is more complicated.
Since, the solution is in water, so there are two dissociation reactions producing four
different ions.
NaCl  Na+ + Cl–
H2O  H+ + OH–
At cathode, there are two possible reactions :
node06\B0BC-BD\Kota\Board Material\Chemistry\Booklets\CBSE\Part-1

Na+(aq) + e–  Na(s) ; E ocell  2.71V


1
H+(aq) + e–  H2(g) ; E ocell  0.00 V
2
The reaction with the higher value of Eo is preferred. Thus, the reaction at the cathode
during electrolysis is
1
H+(aq) + e–  H2(g) …..(i)
2

E 83
Chemistry
ALLEN
But H+ (aq) is produced by the dissociation of H2O.
i.e., H2O(l)  H+(aq) + OH–(aq) ……(ii)
Therefore, net reation at the cathode are the sum of eq. (i) and (ii)
1
H2O(l) +e–  H2(g) + OH– ……(iii)
2
At anode, possible oxidation reactions are :
1
Cl–(aq)  Cl2 (g)  e  , E ocell  1.36 V ……(iv)
2
or 2H2O(l)  O2(g) + 4H+(aq) + 4e– , E ocell  1.23 V ……(v)
The reaction at anode with lower value of Eo is preferred but and therefore water should get
oxidised in preference to Cl–(aq). So, due to over potential of oxygen Cl2 (g) is discharged
at anode.
NaCl(aq) 
H 2O
 Na  (aq)  Cl (aq)
At cathode
1
H2O(l) + e–  H2(g) + OH– (aq)
2
At anode
1
Cl– (aq)  Cl2(g) + e–
2
Net reaction
1 1
NaCl (aq) + H2O (l)  Na+(aq) + OH–(aq) + H2(g) + Cl2(g)
2 2
(iii) During electrolysis of sulphuric acid the following reactions are possible.
At anode (i) 2H2O (l)  O2(g) + 4H+(aq) + 4e– ; E ocell  1.23 V

(ii) 2SO 24 (aq)  S2O 24 (aq)  2e  ; E ocell  1.96 V


For dilute sulphuric acid, reaction (i) is preferred but at higher concentration of H2SO4,
reaction (ii) is preferred.
1
At cathode H+ + e–  H 2 (g) ;E ocell  0.00 V
2
node06\B0BC-BD\Kota\Board Material\Chemistry\Booklets\CBSE\Part-1

FARADAY’S LAW OF ELECTROLYSIS:


(a) First law of electrolysis:
Amount of substance deposited or liberated at an electrode is directly proportional to amount of
charge passed through the solution.
WQ W = amount of substance deposited, Q = charge in coulomb
W  ZQ Z = electrochemical equivalent

84 E
 CBSSE
ALLEN
A
• when Q = 1 coulomb, then W = Z
w
• Thus, amounnt of substan
T nce depositedd by 1 couloomb charge is called elecctrochemica
al equivalen
nt.
• Let 1 amperre current iss passed till ‘t’ secondss .
L
Then, Q = I t
T
 W  ZIt
• 1 Faraday = 96500 cou ulomb = Chaarge of one mole electrrons
• OOne faradaay is the ch harge requ
uired to lib
berate or deposit
d onee gram equ uivalent of a
ssubstance at
a correspoonding electtrode.
(b) SSecond law w of electrollysis:
WWhen samee amount off charge is ppassed throu
ugh differen
nt electrolytte solutions connected in
sseries then weight of substances deposited at electrod des are in rratio of theeir equivalent
W E
wweights. i.e. 1 = 1
W2 E 2
BATTERIE
B ES (COMM MERCIAL CELLS) :
A batteryy (which coontains onee or more thhan one eleectrochemiccal cell conn nnected in series) or ceell
that we use as a soource of eleectrical enerrgy is basiccally a galv vanic cell w
which conveerts chemiccal
energy oof the redoxx reaction in nto electricaal energy. For
F a battery y to be of ppractical usee it should be
b
reasonabbly light, compact and itsi voltage sshould not vary
v appreciably duringg its use.
There arre mainly twot types of o batteries :
(i) Primaary batteriess – Non-rech hargeable
(ii) Secondary batteries – Rech hargeable
(ii) Primaary batteriees :
(a) D Dry cell (LLeclanche cell) : Thee most famiiliar type of o battery i s the dry cell
c which is
aacompact ofo Leclanche cell know wn after itss discovererr Leclanchee (fig.). In this cell, thhe
aanode consiists of a zincc container and the cath hode is a graphite rod surrrounded by b
ppowdered MnO
M 2 and carbon.
c The space betw ween the eleectrodes is ffilled with a moist paste
oof NH4Cl annd ZnCl2.
node06\B0BC-BD\Kota\Board Material\Chemistry\Booklets\CBSE\Part-1

F
Fig. : Dry ceell
T y can be wriitten approxximately as follows :
The electrodde reactionss are compleex, but they
Anodee Zn(s)) Zn + 2e
+2 –

Cathoode MnO O2 + NH4+ + e–  MnO(OH)


M + NH3

E 85
5
Chemistry
C
N
ALLEN
IIn the cathoode reactionn, manganesse is reduceed from thee 4+ oxidatiion state to the 3+ statte.
AAmmonia is not liberaated as a gaas but comb Z 2+ to form
bines with Zn m [Zn(NH3)4]2+ ion. Drry
ccells do nott have an inddenfinite liffe as acidic NH4Cl corrodes the ziinc containeer even wheen
nnot in use. Dry
D cells haave a potenttial of appro oximately 1..5 V.
UUses : Radiios, Remotee control, F Flash-lightss etc.
(b) MMercury cell : This is i suitable for low cu urrent devices like heaaring aids, watches,
w ettc.
cconsists of zinc – meercury amallgam as an node and a paste of H HgO and carbon as thhe
ccathode. Thhe electrolyte is a pastte of KOH and ZnO. The T reducinng agent is zinc and thhe
ooxidising aggent is merccury (II) oxiide. The eleectrode reactions for thee cell are giiven below :

Figg. : Mercury
y cell
Anodee : Zn(Hg)) + 2OH 
 ZnO(s) + H2O + 2ee–

Cathoode : HgO + H2O + 2e–  Hg(l) + 2OH–


T
The overall reaction is representedd by
Zn(Hgg) + HgO(s))  ZnO O(s) + Hg(l)
TThe cell pootential is approximate
a ely 1.35 V and remains constannt during itts life as thhe
ooverall reacction does not involvee any ion in solution n whose cooncentration n can changge
dduring its life time.
UUses : Hearring aids, Watches
W etcc.
(iii) SECOONDARY OR O RECHA ARGEABL LE BATTE ERIES
(a) L Lead storaage battery : The mosst importantt secondary ge battery. It
y cell is thee lead storag
cconsists of a lead anod de and a grrid of lead packed witth lead dioxxide as cathhode. A 38%%
ssolution of sulphuric
s accid is used aas an electro
olyte.
Anode Cathode
node06\B0BC-BD\Kota\Board Material\Chemistry\Booklets\CBSE\Part-1

Negativve plates:
lead griids filled
with spongy lead

38% Sulph huric Positive plates:


p
acid solutio
on lead gridss filled
with PbOO2
Fig. The Lead sstorage batttery

86
6 E
 CBSSE
ALLEN
A
WWhen the cell discharg T lead is ooxidised to Pb+2 ion annd
ges, it operaates as a voltaic cell. The
llead plates acquire
a a neegative charrge.
At Anode : Pb(s) 
A Pb+2 + 2e– .....((i)
+2
Pb ions combine witth sulphate iions of solu
ution to form
m insoluble PbSO4
O 24 PbSO4 (Precippitation)
Pb+2 + SO .....((ii)
_________
____________________
__________
________
Overall reacction Pb(s)) + SO 24 
O PbSO4(ss) + 2e–
T
The electronns given by Pb are usedd at cathodee (PbO2 elecctrode)
At Cathode : PbO2(s) + 4H+ + 2e– Pb2++ + 2H2O
A ......(iii)
S 24 P
Pb+2 + SO PbSO4 (Preccipitation) ......(iv)
_________
____________________
__________
________
PbO2(s) + 4H+ + SO 24 + 2e– 
PbSO4(s)) + 2H2O
T
Therefore thhe overall cell reactionn during disccharging
O 24 PbbSO4(s) + 2ee–
At Anode : Pb(s) + SO
A
A O 24 + 2e– PbSO
At Cathode : PbO2(s) + 4H+ + SO  4(s)
( + 2H2O
_______
____________________
__________
___________
Pb(s) + PbO2(s) + 2H2SO4 2PbSO
 (s) + 2H2O

WWhen a pottential slighhtly greater tthan the potential of battery is appplied, the battery
b can be
b
rreacharged. During reccharging thhe cell is op perated as an electrolyytic cell, thhe reaction is
rreversed annd PbSO4(s)) on anode and cathod de is converrted into Pbb and PbO2 respectively.
LLead storagge batteries have
h a potenntial of app
proximately 2V.
U
Uses : Autoomobiles an
nd Power IInverters ettc.
(b) N
Nickel - Caadmium ("N
Ni - Cd") b
batteries :
TThey are ussed in varieety of cordleess appliancces such as telephoness, video cammcorders, annd
ccordless power tools, are
a lightweeight and reechargeable.. The chem mistry of thee cell utilizees
tthe oxidatioon of cadmiuum and the reduction ofo nickel (IIII) oxide unnder basic conditions. A
rrechargeablle nickel-caadmium celll in a jelly y roll arran
ngement annd separated d by a layer
ssoaked in moist
m sodium
m or potassiuum hydroxiide.
node06\B0BC-BD\Kota\Board Material\Chemistry\Booklets\CBSE\Part-1

Fig. : Nick
kel Cadmiu
um Battery

E 87
7
Chemistry
ALLEN
At cathode: [NiO(OH) (s) + H2O() + e–  Ni(OH)2(s) + OH–(aq)] × 2
At anode: Cd(s) + 2 OH–  Cd(OH)2(s) + 2e–
_________________________________________________
Net cell reaction : 2NiO(OH)(s) + Cd(s) + 2H2O ()  2Ni(OH)2 (s) + Cd(OH)2 (s)
OR
Cd (s) + 2Ni(OH)3 (s)  CdO (s) + 2Ni(OH)2 (s) + H2O (l )
As the active masses of all reaction components are independent of their amount. The cell
delivers constant EMF throughout. Nickel cadmium batteries have a potential of
approximately 1.3 V.
Uses : Camcorders, Portable computers etc.
(iii) Fuel Cells
It is possible to make batteries in which the reactants are fed continuously to the electrodes.
Electrical cells that are designated to convert the free energy from the combustion of fuels such
as hydrogen, carbon monoxide or methane directly into electrical energy are called fuel cells.
Water
anode (–) (+) cathode

aqueous
electrolyte
H2 O2

Fig. : H2-O2 Fuel cell


One of the most successful fuel cells uses the reaction of hydrogen with oxygen to form water
(fig.). This cell has been used for electric power in the Appollo space programme. The water
vapours produced were condensed and added to the drinking water supply for the astronauts.
In the cell shown in figure hydrogen and oxygen are bubbled through a porous carbon electrode
into concentrated aqueous sodium hydroxide. Catalysts are incorporated in the electrode. The
electrode reactions are :
At anode : 2[H2(g) + 2OH– (aq)  2H2O() + 2e– ]

At cathode : O2(g) + 2H2O() + 4e– 4OH– (aq)


——————————————————
Net cell reaction : 2H2(g) + O2(g)  2H2O()
node06\B0BC-BD\Kota\Board Material\Chemistry\Booklets\CBSE\Part-1

This cells runs continuously as long as the reactants are supplied. Since fuel cells convert
the energy of a fuel directly to electricity, they are potentially more efficient than the
conventional method of generating electricity on a large scale by burning hydrogen, carbon
fuels or by using nuclear reactor. Fuel cells produce electricity with an efficiency of about
70 % compared to thermal plants whose efficiency is about 40%. Since, fuel cells are
efficient and free from pollution, efforts are being made to produce better commercially
practical fuel cells.

88 E
 CBSSE
ALLEN
A
Main
M feature
es
Name of thhe ode
Ano Catthode Elecctrolyte
cell/Battery
y
(i) Dry cell Zincc Graphite surroun
nded NH44Cl + ZnCl2
by MnO
M 2 and C (toucching anode)
(ii) Mercury cell Zincc/Mercury Pastte of HgO Pastte of KOH annd ZnO
Amalgum and carbon
(iii) Lead storage
e Lea
ad Leadd dioxide H2SO
O4 (38%)
b
battery
(iv) Ni-Cd cell Cad
dmium Nick
kel dioxide KOHH sol.
(v) H2–O2 fuel cell Porous carbon ous carbon
Poro Connc. aq. NaOH
H sol.
con
ntaining catalyysts conttaining cataly
ysts
(H2 passed) (O2 passed)

Uses
U : Emerrgency bacckup powerr, Transporrtation elecctric vehiclee, Primaryy source of electricity
e i
in
Apollo
A spacce programmme.
Advantage
A
(1) IIt has high efficiency
e about
a 70%
(2) IIt is a constaant source of
o energy annd never becomes dead d.
(3) IIt do not cauuse any polllution becauuse the by-p
product is water.
w
CORROSIO
C ON
Corrossion is basiically an ellectrochemiical phenommenon. Thee rusting off iron, the tarnishing of
o
silver, developmeent of a grreen coatinng on coppeer and bron nze are somme of the examples ofo
corrosiion. Corrossion causes enormous damage to o building, bridges, shhips and to
o many other
articles made of irron.

Fig
g.: Corrosioon of iron in i atmosphhere
In corrrosion, a meetal is oxidiised by losss of electron n and formss metal oxid
ns to oxygen de. Corrosioon
of ironn(which is commonly known as rusting) occcurs in presence of w water and oxygen
o (airr).
Althouugh the cheemistry of corrosion
c is complex but derstood thaat at one sp
b it is und pot of an iroon
object oxidation occurs
o and that
t spot behhaves as an anode.
A
Anode : 2Fe(ss) Fee2+ (aq) + 4e 
4 – ; E oFe2 /FFe  0.44V
V 
Electroons releasedd at anode move throough the meetal and go to anotherr spot on th he metal annd
node06\B0BC-BD\Kota\Board Material\Chemistry\Booklets\CBSE\Part-1

+
reducee the oxygenn in presencce of H ionns (which iss believed to able from H2CO3 formeed
t be availab
due to dissolutionn of CO2 in water).
w Thiss spot work
ks as a catho
ode :
C
Cathode : O2(g)) + 4H+ (aqq) + 4e– 
 2H2O () ; E oH /O  H 2O
2 /H
 1.23V 
Here FFe2+ ions moove throughh water on tthe surface of the iron object. (If w nt is saline, it
water presen
will hhelp more in i carrying the currennt in the miniature
m ceell thus forrmed and will
w enhancce
corrosiion). The overall
o he sum of the cathod
reacction of thhe miniaturee cell is th de and anodde
reactioons as follow
ws :

E 89
9
Chemistry
ALLEN
2Fe(s) + O2(g) + 4H+(aq) 2Fe2+(aq) + 2H2O() ; E ocell = 1.67 V)
The Fe2+ ions are further oxidised by atmospheric oxygen to Fe3+ (as Fe2O3) and comes out as
rust in the form of a hydrated iron (III) oxide expressed as Fe2O3, xH2O.
4Fe2+(aq) + O2(g) + 4H2O()  2Fe2O3(s) + 8H+
H+ ions produced in the above reaction help further in rusting.
PREVENTION FROM CORROSION
Water drop
Corrosion of a metal is prevented by applying
protective coating (such as grease, paint, Zn O2(g)
2+

+
bisphenol or metal coatings) on metal surface. In O2 + 4H + 2e¯
Zn Zn +2e¯
2+
2H2O
case of iron this coating is done in two ways : (i)
electrolysis (Cr, Ni and Cd coating) (ii) dipping Iron cathode
iron objects in a molten metal (Zn and Sn Zinc anode
coating). Use of zinc coating to protect iron is Fig. : Galvanised iron. Cathodic protection of iron
called Galvanisation. In galvanising iron, zinc in contact with zinc is provided by galvanization.
being more reactive than iron serves as an anode
and is oxidised magnesium anode

E Zn 2 /Zn  0.76 V and E Fe2 /Fe  0.44 V
o o
 Mg Mg2+ + 2e

Here it is important to note that iron does not rust


even after zinc coating is broken which is not
true in case of tin coating over iron
 o
ESn 2
/Sn 
 0.14 V . Now if the coating is
broken, iron is exposed and iron being more
active than both copper and tin, is corroded. Here
iron corrodes more rapidly than it does in the Iron pipe (cathode)
absence of tin. But tin protects copper in the O2 + 2H2O + 4e– 4OH–
same way as zinc protects iron Fig. : Cathodic protection by connecting a block of
E Cu 2 /Cu  0.34 V .
o
 magnesium to an iron pipe burried underground.
Instead of coating more reactive metal on iron,
the use of such metals is made as sacrificial anode. This method of preventing iron from
corrosion is called CATHODIC PROTECTION.
In this method, a plate of reactive metal (Zn or Mg) is burried beside the iron pipe or tank and
connected to it by wires.
Here iron becomes cathode and more reactive metal becomes anode. The reactive metal anode is
sacrificed to protect the iron. Since these reactive metal plates are oxidised quickly, they are
replaced from time to time which is easy to do.
Air
Rust O2(g)
Fe2O3(s) Fe2
water
The rusting of iorn (Iron in contact with water
node06\B0BC-BD\Kota\Board Material\Chemistry\Booklets\CBSE\Part-1

cathode anode
e¯ forms the anode and in contact with air forms
the cathode. At the anode iron is oxidised to
O2 + 4H+ + 4e¯ 2H2O Fe Fe2+ + 2e¯ Fe2+, and at the cathode oxygen is reduced to

90 E
CBSE
ALLEN
EXERCISE-1 INTEXT QUESTIONS
1. How would you determine the standard electrode potential of the system Mg2+/Mg ?
Ans. The standard electrode potential of Mg2+| Mg can be measured with respect to the standard
hydrogen electrode, represented by Pt(s), H2(g) (1 atm) | H+(aq) (1M).
A cell, consisting of Mg | MgSO4, (aq 1 M) as the anode and the standard hydrogen electrode as
the cathode, is set up.
Mg | Mg2+(aq, 1M)|| H+(aq, 1M)|H2(g, 1 bar), Pt(s)
Then, the emf of the cell is measured and this measured emf is the standard electrode potential
of the magnesium electrode.
E   E R  E L

Here, E R for the standard hydrogen electrode is zero.

Therefore, E   0  E L = E L
2. Can you store copper sulphate solution in a zinc pot ?
Ans. No, Zinc is more reactive than copper. Zinc reacts with copper sulphate and displaces copper
from its salt solution.
Zn + CuSO4  ZnSO4 + Cu
3. Consult the table of the standard electrode potentials and suggest three substances that can
oxidise ferrous ions under suitable conditions?
Ans. Fe2+ Fe3+ + e– ; E ocell  0.77V
Substances which have greater reduction potential than +0.77 V will oxidise Fe2+, e.g., Br2, Cl2
and F2.
4. Calculate the potential of hydrogen electrode in contact with a solution whose pH is 10.
Ans. 2H+ + 2e–  H2
0.0591 PH
Ecell = E ocell – log + 2 2
2 [H ]
E ocell = 0, PH 2 = 1 atm, [H+] = 10–pH = 10–10

0.0591 1
Ecell = – log = –0.591V
2 (10 –10 ) 2
5. Calculate the emf of the cell in which following reaction takes place :
node06\B0BC-BD\Kota\Board Material\Chemistry\Booklets\CBSE\Part-1

Ni(s) + 2Ag+ (0.002 M)  Ni2+ (0.160 M) + 2Ag(s)


Given that E ocell = 1.05 V
Ans. Applying Nernst equation we have:

 0.0591  Ni 2 
E(cell) = E cell  log 2
n  Ag  

E 91
Chemistry
ALLEN
 1.05 
0.0591
log
 0.160 
 0.002 
2
2
0.16
 1.05  0.02955log
0.000004
= 1.05 – 0.02955 log (4 × 104)
= 1.05 – 0.02955 (log 4 + log 10)
= 1.05 – 0.02955 (0.6020 + 4)
= 1.05 – 0.02955 (4.6020)
= 1.05 – 0.14 = 0.91 V
6. The cell in which the following reaction occurs :
2Fe3 (aq.)  2I  (aq.)  2Fe 2 (aq.)  I 2 (s) has E ocell = 0.236 V at 298 K
Calculate the standard Gibbs energy and equilibrium constant of the cell reaction.
Ans. rGo = nFE cell = –2 × 96500 × 0.236 joule mol–1 = –45.55 kJ mol–1
rGo = –2.303 RT log Kc (R = 8.314 J K–1 mol–1)
–45.55 = –2.303 × 8.314 × 298 log Kc
or log Kc = 7.981 Kc = antilog (7.981)
or Kc = 9.6 × 107
7. Why does the conductivity of a solution decrease with dilution ?
Ans. The conductivity of a solution is the conductance of ions present in a unit volume of the solution.
The number of ions (responsible for carrying current) per unit volume decreases when the
solution is diluted. As a result, the conductivity of a solution decreases with dilution.
8. Suggest a way to determine the  0m value of water.
Ans. Applying Kohlrausch’s law of independent migration of ions, the value of water can be
determined as follows :
 0m(H2O)   0H   OH
0

   0H   Cl
0
     Na  
0
OH    Na
0
  0   Cl
0
 

  0m(H2O)   0m(HCl)   0m( NaOH)   0m( NaCl)


Hence, by knowing the values of HCl, NaOH, and NaCl, the value of water can be determined.
9. The molar conductivity of 0.025 mol/L methanoic acid is 46.1 S cm2/mol. Calculate the
degree of dissociation and dissociation constant. Given :  oH  349.6 Scm2mol–1 and
 oHCOO  54.6 Scm2mol–1
node06\B0BC-BD\Kota\Board Material\Chemistry\Booklets\CBSE\Part-1

Ans.  oHCOOH   oH   oHCOO = 349.6 + 54.6 = 404.2

 cm 46.1
= = = 0.114
 m 404.2
o

C 2 0.025× 0.114× 0.114


Ka = = = 3.67 × 10–4
1–  1 – 0.114

92 E
CBSE
ALLEN
10. If a current of 0.5 ampere flows through a metallic wire for 2 hours, then how many electrons
would flow through the wire ?
Ans. I = 0.5 A, t = 2 hours = 2 × 60 × 60 s = 7200 s
Q = It = 0.5A × 7200 s = 3600C
 96500 coulomb are equivalent of 6.023 × 1023 number of electrons

6.023×1023 ×3600
 3600 coulomb are equivalent to = 2.246×1022 electrons
96500
11. Suggest a list of metals that are extracted electrolytically ?
Ans. Alkali metals such as Na, K etc., alkaline earth metals such as Mg, Ca etc. and aluminium.
12. Consider the reaction, Cr2 O72– +14H + + 6e –  2Cr 3+ + 7H 2 O

What is the quantity of electricity in coulombs needed to reduce one mol of Cr2O 72- ?

Ans. To reduce 1 mole of Cr2 O 72 , the required quantity of electricity will be :
6F = 6 × 96487 coulomb = 578922 C
13. Write the chemistry of recharging the lead storage battery, highlighting all the materials that are
involved during recharging.
Ans. lead storage battery consists of a lead anode, a grid of lead packed with lead oxide (PbO2) as the
cathode, and a 38% solution of sulphuric acid (H2SO4) as an electrolyte.
When the battery is in use, the following cell reactions take place:
At Anode : Pb(s) + SO42– (aq.) PbSO4(s) + 2e–
At Cathode : PbO2(s) + 4H+ (aq.) + SO4–2 (aq.) + 2e– PbSO4(s) + 2H2O()

———————————————————————––––––––
Net cell reaction : Pb(s) + PbO2(s) + 2H2SO4(aq.) 2PbSO4(s) + 2H2O()

When a battery is charged, the reverse of all these reactions takes place.
Hence, on charging, PbSO4(s) present at the anode and cathode is converted into Pb(s) and
PbO2(s) respectively.
14. Suggest two materials other than hydrogen that can be used as fuels in fuel cells.
Ans. Methane and methanol.
node06\B0BC-BD\Kota\Board Material\Chemistry\Booklets\CBSE\Part-1

E 93
Chemistry
ALLEN
EXERCISE-2 NCERT EXERCISE
1. Arrange the following metals in the order in which they displace each other from the solution of
their salts.
Al, Cu, Fe, Mg and Zn
Ans. The following is the order in which the given metals displace each other from the solution of
their salts.
Mg, Al, Zn, Fe, Cu
2. Given the standard electrode potentials,
K+/K = –2.93V, Ag+/Ag = 0.80 V
Hg2+/Hg = 0.79V
Mg2+/Mg = –2.37 V, Cr3+/Cr = – 0.74V
Arrange these metals in their increasing order of reducing power.
Ans. The lower the reduction potential, the higher is the reducing power.
The given standard electrode potentials increase in the order of
K+/K < Mg2+/Mg < Cr3+/Cr < Hg2+/Hg < Ag+/Ag.
Hence, the reducing power of the given metals increase in the following order :
Ag < Hg < Cr < Mg < K
3. Depict the galvanic cell in which the reaction Zn(s) + 2Ag+(aq)   Zn2+(aq) + 2Ag(s) takes
place. Further show:
(i) Which of the electrode is negatively charged ?
(ii) The carriers of the current in the cell ?
(iii) Individual reaction at each electrode.
Ans. The galvanic cell in which the given reaction takes place is depicted as :
Zn(s) Zn 2 (aq) Ag  (aq) Ag(s)
(i) Zn electrode (anode) is negatively charged.
(ii) Ions are carriers of current in the cell and in the external circuit, current will flow from silver
to zinc.
(iii) The reaction taking place at the anode is given by,
2
Zn (s) 
 Zn (aq)  2e 
The reaction taking place at the cathode is given by,

2Ag (aq)  2e  
 2Ag (s)
4. Calculate the standard cell potentials of galvanic cell in which the following reactions take place :
node06\B0BC-BD\Kota\Board Material\Chemistry\Booklets\CBSE\Part-1

(i) 2Cr(s) + 3Cd2+(aq) 


 2Cr3+(aq) + 3Cd(s)
E 1Cr3 /Cr =–0.74 V ; E 1Cd2 /Cd = –0.40 V
(ii) Fe2+(aq) + Ag+(aq) 
 Fe3+(aq) + Ag(s)
E 1Fe3 /Fe2 = 0.77 V ; E 1Ag  /Ag = 0.80 V

Calculate the rG1 and equilibrium constant of the reactions.

94 E
CBSE
ALLEN
Ans. (i) The galvanic cell of the given reaction is depicted as :
Cr(s) Cr 3 (aq) Cd 2 (aq) Cd(s)

Now, the standard cell potential is


E 1cell  E 1R  E 1L
= –0.40 – (–0.74) = +0.34 V
 r G 1  nFE 1cell
In the given equation,
n=6
F = 96500 C mol–1
E 1cell  0.34V

Then,  r G 1 = –6 × 96500 C mol–1 × 0.34 V


= – 196860 CV mol–1
= –196860 J mol–1
= –196.86 kJ mol–1
Again, rG1 = –RT ln Kc

 rG1 = –2.303 RT log Kc

rG 196.86 103


 log Kc = – = = 34.50
2.303RT 2.303  8.314  298
 Kc = antilog (34.50) = 3.16 × 1034
(ii) E 1Fe3 /Fe2 = 0.77 V

E 1Ag  /Ag = 0.80 V

The galvanic cell of the given reaction is depicted as :


Fe2 (aq) Fe3 (aq) Ag  (aq) Ag(s)

Now, the standard cell potential is


E 1cell  E 1R  E 1L
= 0.80 – 0.77 = 0.03 V
node06\B0BC-BD\Kota\Board Material\Chemistry\Booklets\CBSE\Part-1

Here, n = 1,
Then,  r G 1  nFE 1cell
= –1 × 96500 C mol–1 × 0.03 V.
= –2895 J mol–1
= –2.895 kJ mol–1

E 95
Chemistry
ALLEN
Again,  r G 1 = –2.303 RT Iog Kc

 r G 1 2.895 103
 log K = = = 0.5073
2.303RT 2.303  8.314  298
 Kc = antilog (0.5073) = 3.2 (approximately)
5. Write the Nernst equation and emf of the following cells at 298K :
(i) Mg(s) Mg 2 (0.001M) Cu 2  0.0001M  Cu(s)

(ii) Fe(s) Fe2 (0.001M) H  1M  H 2 (g) (1bar) Pt(s)

(iii) Sn(s) Sn 2 (0.050M) H   0.020 M  H 2 (g) (1bar) Pt(s)

(iv) Pt(s) Br2 (l) Br   0.010 M  H   0.030M  H 2 (g) (1bar) Pt(s)


Ans. (i) For the given reaction, the Nernst equation can be given as :
0.0591 [Mg 2 ]
E cell  E 1cell  log
n [Cu 2 ]

= 0.34   2.36  
0.0591 .001
log
2 .0001
0.0591
= 2.7  log10
2
= 2.7 – 0.02955
= 2.67 (approximately)
(ii) For the given reaction, the Nernst equation can be given as :
0.0591 [Fe 2 ]
E cell  E 1cell  log  2
n [H ]

= 0   0.44  
0.0591 0.001
log 2
2 1
= 0.44 – 0.02955(–3)
= 0.52865
= 0.53 V (approximately)
(iii) For the given reaction, the Nernst equation can be given as :
0.0591 [Sn 2 ]
E cell  E 1
cell  log  2
n [H ]
node06\B0BC-BD\Kota\Board Material\Chemistry\Booklets\CBSE\Part-1

= 0   0.14  
0.0591 0.050
log
 0.020 
2
2

= 0.14 – 0.0295 × log 125


= 0.14 – 0.062
= 0.078 V
= 0.08 V (approximately)
96 E
CBSE
ALLEN
(iv) For the given reaction, the Nernst equation can be given as :
0.0591 1
E cell  E 1cell  log
n [Br ] [H  ]2
 2

0.0591 1
=  0  1.09   log
 0.010   0.030 
2 2
2
1
= 1.09  0.02955  log
0.00000009
1
= 1.09  0.02955  log
9 108
= 1.09 – 0.02955 × log(1.11 × 107)
= –1.09 – 0.02955 (0.0453 + 7)
= –1.09 – 0.208
= –1.298 V
6. In the button cells widely used in watches and other devices the following reaction takes place :
Zn(s) + Ag2O(s) + H2(O) (l) 
 Zn2+(aq) + 2Ag(s) + 2OH–(aq)
Determine rG1 and E1 for the reaction.
2
Ans. Zn (s) 
 Zn (aq)  2e  ;E 1  0.76V
Ag 2 O(s)  H 2 O(  )  2e  
 2Ag (g)  2OH  (aq) ; E 1  0.344 V
_______________________________________________________
2 
Zn (s)  Ag 2 O(s)  H 2 O(f ) 
 Zn (aq)  2Ag (s)  2OH (aq) ; E 1  1.104 V
 E1 = 1.104 V
We know that,
 r G 1  nFE 1
= –2 × 96487 × 1.104
= –213043.296 J
= –213.04 kJ
7. Define conductivity and molar conductivity for the solution of an electrolyte. Discuss their
variation with concentration.
Ans. The conductivity of a solution at any given concentration is the conductance (G) of one unit
volume of solution kept between two platinum electrodes with the unit area of cross-section and
at a distance of unit length. The conductivity of a solution is defined as the conductance of a
node06\B0BC-BD\Kota\Board Material\Chemistry\Booklets\CBSE\Part-1

solution of 1 cm of length and area of cross section 1 cm2. The inverse of resistivity is called
conductivity.
A
i.e., G    .1   (Since A = 1, = 1)

Conductivity always decrease with a decrease in concentration, both for weak and strong
electrolytes. This is because the number of ions per unit volume that carry the current in a
solution decreases with a decrease in concentration.

E 97
Chemistry
ALLEN
Molar conductivity : Molar conductivity of a solution at a given concentration is the
conductance of volume V of a solution containing 1 mole of the electrolyte kept between two
electrodes with the area of cross - section A and distance of unit length is l.
A
m   =

Now,  = 1 and A = V (volume containing 1 mole of an electrolyte).

  m = V
Molar conductivity increases with a decrease in concentration. This is because the total volume V
of the solution containing one mole of an electrolyte also increases on dilution.
The variation of  m with c for strong and weak electrolytes is shown in the following plot :

KCl(strong electrolyte)
(S cm mol )
–1
2
m

CH3COOH(weak electrolyte)

–1 1/2
C(mol L )

8. The conductivity of 0.20 M solution of KCl at 298 K is 0.0248 Scm–1. Calculate its molar
conductivity.
Ans. Given, = 0.0248 S cm–1; c = 0.20 M
 Molar conductivity,
 1000 0.0248 1000
m  = = 124 Scm2mol–1
c 0.2
9. The resistance of a conductivity cell containing 0.001M KCl solution at 298 K is 1500 . What
is the cell constant if conductivity of 0.001 M KCl solution at 298 K is 0.146 × 10–3 S cm–1.
Ans. Given, Conductivity,  = 0.146 × 10–3 S cm–1
Resistance, R = 1500
 Cell constant =×R
= 0.146 × 10–3 × 1500 = 0.219 cm–1
node06\B0BC-BD\Kota\Board Material\Chemistry\Booklets\CBSE\Part-1

10. The conductivity of sodium chloride at 298 K has been determined at different concentrations
and the results are given below :
Concentration/ M 0.001 0.010 0.020 0.050 0.100

102 × /S m–1 1.237 11.85 23.15 55.53 106.74

Calculate  m for all concentrations and draw a plot between  m and C½. Find the value of  0m .

98 E
CBSE
ALLEN
Ans. Given,
 = 1.237 × 10–2 S m–1, c = 0.001 M
Then,  = 1.237 × 10–4 S cm–1, c½ = 0.0316 M1/2
 1.237 104 S cm 1 1000 cm3
 m  = 1
 = 123.7 S cm2 mol–1
c 0.001mol L L
Given,
 = 1185 × 10–2 S m–1, c = 0.010 M
Then,  = 11.85 × 10–4 S cm–1, c½ = 0.1 M1/2
 11.85 104 Scm 1 1000 cm3
 m  = 1
 = 118.5 S cm2 mol–1
c 0.010 mol L L
Given,
 = 23.15 × 10–2 S m–1, c = 0.020 M
Then, k = 23.15 × 10–4 S cm–1, c1/2 = 0.1414 M1/2
 23.15 104 Scm 1 1000 cm3
 m  = 1
 = 115.8 cm2 mol–1
c 0.020 mol L L
Given,
k = 55.53 × 10–2 S m–1, c = 0.050 M
Then, k = 55.53 × 10–4 S cm–1, c1/2 = 0.2236 M1/2
 55.53 104 Scm 1 1000 cm 2
 m  = 1
 = 111.11 S cm2 mol–1
c 0.050 mol L L
Given,
 = 106.74 × 10–2 Sm–1, c = 0.100 M
Then,  = 106.74 × 10–4 S cm–1, c1/2 = 0.3162 M1/2
 106.74 104 1000 cm3
 m  = 1
 = 106.74 S cm2 mol–1
c 0.100 mol L L
Now, we have the following data.
C1/ 2 0.0316 0.1 0.1414 0.2236 0.3162
m(S cm mol )
2 –2
123.7 118.5 115.8 111.1 106.74
126.0
124.0
(0.0316.123.7)
122.0
m(S cm2 mol–-1)

120.0
(0.1, 118.5)
118.0
node06\B0BC-BD\Kota\Board Material\Chemistry\Booklets\CBSE\Part-1

116.0
114.0 (0.1414, 115.8)
112.0
110.0 (0.2236, 111.1)
108.0
(0.3162, 106.741)
106.0
0 0.05 0.1 0.15 0.2 0.25 0.3 0.35
1
2
C

Since the line interrupts  m at 124.0 S cm2 mol–1,  0m = 124.0 S cm2 mol–1

E 99
Chemistry
ALLEN
11. Conductivity of 0.00241 M acetic acid is 7.896 × 10–5 S cm–1. Calculate its molar conductivity.
If  0m for acetic acid is 390.5 S cm2 mol–1, what is its dissociation constant?
Ans. Given,  = 7.896 × 10 –5 S cm–1
C = 0.00241 mol L–1
Then, molar conductivity,
 7.896 105 Scm 1 1000 cm3
m  = 1
 = 32.76 S cm2 mol–1
c 0.00241mol L L
Again,  0m = 390.5 S cm2 mol–1

 m 32.76S cm 2 mol1
Now,    = 0.084
 0m 390.5Scm 2 mol1
 Dissociation constant,

Ka 
c 2
=
 0.00241mol L   0.084 
1 2

= 1.86 × 10–5 mol L–1


1    1  0.084 
12. How much charge is required for the following reductions :
(i) 1 mol of Al3+ to Al.
(ii) 1 mol of Cu2+ to Cu.
(iii) 1 mol of MnO4– to Mn2+
Ans. (i) Al3+ + 3e– 
 Al
 Required charge = 3F
= 3 × 96500 C = 289461 C
(ii) Cu2+ + 2e– 
 Cu
 Required charge = 2F
= 2 × 96500 C = 192974 C
(iii) MnO–4 
 Mn2+
i.e., Mn7+ + 5e– 
 Mn2+
 Required charge = 5F
= 5 × 96500 C = 482435 C
13. How much electricity in terms of Faraday is required to produce
(i) 20.0 g of Ca from molten CaCl2.
node06\B0BC-BD\Kota\Board Material\Chemistry\Booklets\CBSE\Part-1

(ii) 40.0 g of Al from molten Al2O3.


Ans. (i) According to the question,
Ca 2  2e  
 Ca
40g

Electricity required to produce 40 g of calcium = 2F


2  20
Therefore, electricity required to produce 20 g of calcium = F = 1F
40
100 E
CBSE
ALLEN
(ii) According to the question,
Al3  3e  
 Al
27g

Electricity required to produce 27 g of Al = 3F


3  40
Therefore, electricity required to produce 40 g of Al = F = 4.44 F
27
14. How much electricity is required in coulomb for the oxidation of
(i) 1 mol of H2O to O2.
(ii) 1 mol of FeO to Fe2O3.
Ans. (i) According to the question,
1
H 2 O 
 H 2  O2
2
Now, we can write :
1
O 2 
 O 2  2e 
2
Electricity required for the oxidation of 1 mol of H2O to O2 = 2F = 2 × 96500 C = 192974 C
(ii) According to the question,
Fe 2 
 Fe3  e 1
Electricity required for the oxidation of 1 mol of FeO to Fe2O3 = 1 F = 96500 C
15. A solution of Ni(NO3)2 is electrolysed between platinum electrodes using a current of
5 amperes for 20 minutes. What mass of Ni is deposited at the cathode?
Ans. Given,
Current = 5A
Time = 20 × 60 = 1200 s
 Charge = current × time
= 5 × 1200 = 6000 C
According to the reaction,
Ni 2 (aq)  2e  
 Ni (s)
58.7g

Nickel deposited by 2 × 96500 C = 58.71 g


58.71 6000
node06\B0BC-BD\Kota\Board Material\Chemistry\Booklets\CBSE\Part-1

Therefore, nickel deposited by 6000 C = g = 1.825 g


2  96500
Hence, 1.825 g of nickel will be deposited at the cathode.
16. Three electrolytic cells A, B, C containing solutions of ZnSO4, AgNO3 and CuSO4, respectively
are connected in series. A steady current of 1.5 amperes was passed through them until 1.45 g of
silver deposited at the cathode of cell B. How long did the current flow? What mass of copper
and zinc were deposited?

E 101
Chemistry
ALLEN
Ans. According to the reaction :

Ag (aq)  e  
 Ag  s 
108g

i.e., 108 g of Ag is deposited by 96500 C.


96500 1.45
Therefore, 1.45 g of Ag will be deposited by = C = 1295.43 C
108
Given, Current = 1.5A
1295.60
 Time = s = 863.73 s = 864 s = 14.40 min
1.5
Again,
2
Cu (aq)  2e  
 Cu (s)
63.5g

i.e., 2 × 96500 C of charge deposit = 63.5 g of Cu


63.5  1295.60
Therefore, 1295.60 C of charge will deposit = g = 0.426 g of Cu
2  96500
2
Given, Zn (aq)  2e  
 Zn (s)
65.4g

i.e., 2 × 96500 C of charge deposit = 65.4 g of Zn


65.4  1295.60
Therefore, 1295.60 C of charge will deposit = g = 0.439 g of Zn
2  96500
17. Using the standard electrode potentials given in electrochemical series predict if the reaction
between the following is feasible :
(i) Fe3+(aq) and I–(aq)
(ii) Ag+(aq) and Cu(s)
(iii) Fe3+(aq) and Br–(aq)
(iv) Ag(s) and Fe3+(aq)
(v) Br2(aq) and Fe2+(aq).
Ans. (i) Fe3+(aq) + e– Fe2+(aq) × 2 ; E° = +0.77 V

2l–(aq) I2(s) + 2e ; E° = -0.54V

2Fe3+(aq) + 2I–(aq) 2Fe2+(aq) + I2(s) ; E° = +0.23 V


node06\B0BC-BD\Kota\Board Material\Chemistry\Booklets\CBSE\Part-1

Since E1 for the overall reaction is positive, the reaction between Fe3(aq)

and I–(aq) is feasible.

(ii) Ag
+
(aq) +e

Ag(s) ×2 ; E° = +0.80 V

Cu(s) Cu
2+
+ 2e ;
(aq)
E° = -0.34V

2Ag+(aq) + Cu(s) 2Ag(s) + Cu


2+
(aq)
; E° = +0.46 V

Since E1 for the overall reaction is positive, the reaction between Ag (aq)

and Cu(s) is feasible.

102 E
CBSE
ALLEN
(iii) Fe3+(aq) + e– Fe2+(aq) × 2 ; E° = +0.77 V

2Br–(aq) Br2(l)+ 2e –
; E° = -1.09 V

2Fe3+(aq) + 2Br–(aq) 2Fe2+(aq) + Br2(l) ; E° = -0.32 V

Since E1 for the overall reaction is negative, the reaction between Fe3(aq)

and Br(aq) is not feasible.

(iv) Ag(s) Ag+(aq) + e– ; E° = -0.80 V

Fe3+(aq) + e– Fe2+(aq) ; E° = +0.77 V

Ag(s) + Fe3+(aq) Ag
+
(aq) + Fe
2+
(aq)
; E° = -0.03 V

Since E1 E for the overall reaction is negative, the reaction between Ag(s) and Fe3(aq)

is not
feasible.
(v) Br2(aq) + 2e– 2Br–(aq) ; E° = +1.09 V

Fe2+(aq) Fe
3+
(aq) +e × 2 ;

E° = -0.77 V
2+
Br2(aq) + 2Fe (aq)

2Br (aq) + 2Fe
3+
(aq)
; E° = +0.32 V

Since E1 for the overall reaction is positive, the reaction between Br2(aq) and Fe(aq)
2
is feasible.

18. Predict the products of electrolysis in each to the following :


(i) An aqueous solution of AgNO3 with silver electrodes.
(ii) An aqueous solution of AgNO3 with platinum electrodes.
(iii) A dilute solution of H2SO4 with platinum electrodes.
(iv) An aqueous solution of CuCl2 with platinum electrodes.
Ans.   C P

AgNO3 Ag Ag+
AgNO3 Ag O2
H2SO4 H2 O2
CuCl2 Cu Cl2
node06\B0BC-BD\Kota\Board Material\Chemistry\Booklets\CBSE\Part-1

E 103
Chemistry
ALLEN
EXERCISE-3 EXEMPLAR
1. Which cell will measure standard electrode potential of copper electrode ?
(a) Pt(s) | H2(g, 0.1bar) | H+(aq, 1M) || Cu2+(aq, 1M) | Cu
(b) Pt(s) | H2(g, 1bar) | H+(aq, 1M) || Cu2+(aq, 2M) | Cu
(c) Pt(s) | H2(g, 1bar) | H+(aq, 1M) || Cu2+(aq, 1M) | Cu
(d) Pt(s) | H2(g, 0.1 bar) | H+ (aq, 0.1 M) || Cu2+(aq, 1M) | Cu
Ans. (c)
2. Electrode potential for Mg electrode varies according to the equation
0.059 1
E Mg2 /Mg  E 1Mg 2 /Mg  log . The graph of E Mg2 /Mg :
2 [Mg 2  ]
EMg2+/Mg

EMg2+/Mg
(a) (b)

log[Mg2+]  log[Mg2+] 
EMg2+/Mg
EMg2+/Mg

(c) (d)

log[Mg2+]  log[Mg2+] 

Ans. (c)
3. Which of the statements about solutions of electrolytes is not correct ?
(a) Conductivity of solution depends upon size of ions
(b) Conductivity depends upon viscosity of solution
(c) Conductivity does not depend upon solvation of ions present in solution
(d) Conductivity of solution increases with temperature
Ans. (c)
4. Using the data given below find out the strongest reducing agent :
E 1Cr O2 /Cr3  1.33V ; E 1Cl   1.36 V
2 7 2 /Cl

E 1MnO /Mn 2  1.51V ; E 1Cr3 /Cr   0.74 V


4

(a) Cl– (c) Cr3+ (d) Mn2+


node06\B0BC-BD\Kota\Board Material\Chemistry\Booklets\CBSE\Part-1

(b) Cr
Ans. (b)
5. While charging the lead storage battery...............
(a) PbSO4 anode is reduced to Pb (b) PbSO4 cathode is reduced to Pb
(c) PbSO4 cathode is oxidised to Pb (d) PbSO4 anode is oxidised to PbO2
Ans. (a)

104 E
CBSE
ALLEN
6.  om ( NH4OH) is equal to ................

(a)  om ( NH4OH)   om ( NH4Cl)   o(HCl) (b)  om ( NH4Cl)   om ( NaOH)   o( NaCl)

(c)  om ( NH4Cl)   om ( NaCl)   o( NaOH) (d)  om ( NaOH)   om ( NaCl)   o( NH4Cl)


Ans. (b)
7. In the electrolysis of aqueous solution chloride solution which of the half cell reaction will occur
at anode ?
(a) Na+(aq) + e–  Na(s) ; E 1cell  2.71 V

(b) 2H2O (l)  O2(g) + 4H+(aq) + 4e– ; E 1cell  1.23V


1
(c) H+(aq) + e–  H 2 (g) ; E 1cell  0.00 V
2
1
(d) Cl–(aq)  Cl2 (g)  e  ; E 1cell  1.36V
2
Ans. (d)
(SHORT ANSWER TYPE QUESTIONS)
8. Value of standard electrode potential for the oxidation of Cl– ions is more positive than that of
water, even then in the electrolysis of aqueous sodium chloride, why is Cl– oxidised at anode
instead of water ?
Ans. Under the condition of electrolysis of aqueous solution chloride, oxidation of water at anode
requires over potential. So, Cl– is oxidised at anode instead of water.
Possible oxidation half cell reactions occurring at anode are :
1
Cl–(aq)  Cl2 (g)  e  ; E 1cell  1.36 V
2
2H2O (l)  O2(g) + 4H+(aq) + 4e– ; E 1cell  1.23 V

Species having lower E 1cell cell undergo oxidation first than the higher value but oxidation of H2O
to O2 is kinetically so slow that it needs some overvoltage.
9. How will the pH of brine (aq. NaCl solution) be affected when it is electrolysed ?
Ans. The pH of the solution will rise as NaOH is formed in the electrolytic cell.
Chemical reaction occurring at cell when aqueous solution is electrolysed are as follows :
NaCl (aq) Na+(aq) + Cl–(aq)
node06\B0BC-BD\Kota\Board Material\Chemistry\Booklets\CBSE\Part-1

1
Cathode H2O(l) + e–  H 2 (g)  OH  (aq)
2
1
Anode Cl– (aq)  Cl2 (g)  e 
2
1 1
Net reaction NaCl (aq) + H2O (l)  Na+(aq) + OH–(aq) + H 2  Cl2
2 2

E 105
Chemistry
ALLEN
10. Solutions of two electrolytes A and B are diluted. The  m of 'B' increases 1.5 times while that of
A increases 25 times. Which of the two is a strong electrolyte ? Justify your answer.
Ans. Strong electrolytes dissociate almost completely even on high concentration. Therefore,
concentration of such solutions remain almost same on dilution. Electrolyte 'B' is stronger than
'A' because in 'B' the number of ions remains the same on dilution, but only interionic attraction
decreases.
Therefore,  m increases only 1.5 times. While in case of weak electrolyte on dilution, number of
constituents ions increases.
11. Consider a cell given below
Cu | Cu2+ || Cl– | Cl2, Pt
Write the reactions that occurs at anode and cathode.
Ans. Cell reaction represented in the question is composed of two half cell reactions. These reactions
are as follows :
At anode Cu  Cu2+ + 2e–
At cathode Cl2 + 2e–  2Cl–
Copper is getting oxidised at anode. Cl2 is getting reduced at cathode.
(MATCHING THE COLUMNS)
12. Match the terms given in Column I with the units given in Column II.
Column-I  Column-II
(A)  ^m  (1)  S cm–1
(B)  Ecell  (2)  m–1
(C)  (3)  S cm2 mol–1
(D)  G  (4)  V
Ans. A-3, B-4, C-1, D-2
13. Match the terms given in Column I with the items given in Column II.
Column-I  Column-II
(A)  ^m  (1) Intensive property
(B) E 1cell   (2) Depends on number of ions/volume

(C)  (3) Extensive property


(D) r Gcell  (4) Increases with dilution
Ans. A-4, B-1, C-2, D-3
14. Match the items of Column I and Column II.
node06\B0BC-BD\Kota\Board Material\Chemistry\Booklets\CBSE\Part-1

Column-I  Column-II
(A)  Lead storage battery  (1)  Maximum efficiency
(B)  Mercury cell  (2)  Prevented by galvanisation
(C)  Fuel cell (3)  Gives steady potential
(D)  Rusting  (4)  Pb is anode, PbO2 is cathode
Ans. A-4, B-3, C-1, D-2
106 E
CBSE
ALLEN
15. Match the items of Column I and Column II.
Column-I  Column-II
(A)  (1)  1 × t

(B)  ^m  (2) ^m / ^m


o

(C)  (3) 
C
(D)  Q  (4) G*
R
Ans. A-4, B-3, C-2, D-1
16. Match the items of Column I and Column II.
Column-I  Column-II
(A)  Lechlanche cell (1)  Cell reaction 2H2 + O2  2H2O

(B)  Ni-Cd cell  (2)  Does not involve any ion in solution and is used in
hearing aids.
(C)  Fuel cell (3)  Rechargable
(D)  Mercury cell  (4)  Reaction at anode, Zn  Zn2+ + 2e–

    (5)  Converts energy of combustion into electrical energy

Ans. A-4, B-3, C-1&5, D-2


17. Match the items of Column I and Column II on the basis of data given below :
E 1F /F  2.87 V, E 1Li /Li  3.5 V
2

E 1Au3 /Au  1.4 V, E 1Br / Br   1.09 V


2

Column-I  Column-II
(A)  F2 (1)  Metal is the strongest reducing agent
(B)  Li  (2)  Metal ion which is the weakest oxidising agent
(C)  Au3+ (3)  Non-metal which is the best oxidising agent
(D)  Br–  (4)  Unreactive metal
node06\B0BC-BD\Kota\Board Material\Chemistry\Booklets\CBSE\Part-1

(E)  Au  (5)  Anion that can be oxidised by Au3+


(F)  Li+  (6)  Anion which is the weakest reducing agent
(G)  F–  (7)  Metal ion which is an oxidising agent
Ans. A-3, B-1, C-7, D-5. E-4, F-2, G-6

E 107
Chemistry
ALLEN
ASSERTION AND REASON
In the following questions a statement of assertion (A) followed by a statement of reason (R) is
given. Choose the correct answer out of the following choices :
(a) Both (A) and (R) correct and (R) is correct explanation of (A).
(b) Both (A) and (R) correct and (R) is not correct explanation of (A).
(c) (A) is correct but (R) is wrong.
(d) (A) Assertion is wrong but (R) reason is correct.
18. Assertion (A) : Cu is less reactive than hydrogen.
Reason (R) : E
Cu 2 /Cu
is negative.
Ans. (c)
19. Assertion (A) : Ecell should have a positive value for the cell to function.
Reason (R) :Ecathode < Eanode
Ans. (c)
20. Assertion (A) : Conductivity of all electrolytes decreases on dilution.
Reason (R) : On dilution number of ions per unit volume decreases.
Ans. (a)
21. Assertion (A) :  m for weak electrolytes shows a sharp increase when the electrolytic solution is
diluted.
Reason (R) : For weak electrolytes degree of dissociation increase with dilution of solution.
Ans. (a)
22. Assertion (A) : Mercury cell does not give steady potential.
Reason (R) : In the cell reaction, ions are not involved in solution.
Ans. (d)
23. Assertion (A) : Electrolysis of NaCl solution gives chlorine at anode instead of O2.
Reason (R) : Formation of oxygen at anode required over voltage.
Ans. (a)
24. Assertion (A) : For measuring resistance of an ionic solution an AC source is used.
Reason (R) : Concentration of ionic solution will charge if DC source is used.
Ans. (a)
25. Assertion (A) : Current stops flowing when Ecell = 0.
Reason (R) : Equilibrium of the cell reaction is attained.
Ans. (a)
node06\B0BC-BD\Kota\Board Material\Chemistry\Booklets\CBSE\Part-1

26. Assertion (A) : E Ag  /Ag increase with increase in concentration of Ag+ ions.

Reason (R) : E Ag  /Ag has a positive value.

Ans. (b)
27. Assertion (A) : Copper sulphate can be stored in zinc vessel.
Reason (R) : Zinc is less reactive than copper.
Ans. (d)
108 E
CBSE
ALLEN
EXERCISE-4 MCQ, A/R, CASE-BASED QUESTIONS
(Electrolyte and Electrolysis)
1. Electrolyte can conduct electricity because
(a) Their molecules contain unpaired electrons, which are mobile
(b)Their molecules contain loosely held electrons which get free under the influence of voltage
(c) The molecules break up into ions when a voltage is applied
(d) The molecules are broken up into ions when the electrolyte is fused or is dissolved in the
solvent
2. Which one of the following metals could not be obtained on electrolysis of aqueous solution of
its salts ?
(a) Ag (b) Mg (c) Cu (d) Cr
3. Electrolysis involves oxidation and reduction respectively at
(a) Anode and cathode (b) Cathode and anode
(c) At both the electrodes (d) None of the above
4. During the electrolysis of an electrolyte, the number of ions produced, is directly proportional to the
(a) Time consumed
(b) Electro chemical equivalent of electrolysis
(c) Quantity of electricity passed
(d) Mass of electrons
5. During the electrolysis of fused NaCl, which reaction occurs at anode
(a) Chloride ions are oxidized (b) Chloride ions are reduced
(c) Sodium ions are oxidised (d) Sodium ions are reduced
6. On electrolysing a solution of dilute H2SO4 between platinum electrodes, the gas evolved at the
anode is
(a) SO2 (b) SO3 (c) O2 (d) H2
7. In electrolysis of aqueous copper sulphate, the gas at anode and cathode is
(a) O2 and H2 (b) SO2 and H2 (c) H2 and O2 (d) SO3 and O2
8. Use of electrolysis is
(a) Electroplating (b) Electro refining
(c) (a) and (b) both (d) None of these
9. Electrolysis of molten anhydrous calcium chloride produces
(a) Calcium (b) Phosphorus (c) Sulphur (d) Sodium
10. Which of the following liberate hydrogen on reaction with dilute H2SO4?
(a) Fe (b) Cu (c) Al (d) Hg
node06\B0BC-BD\Kota\Board Material\Chemistry\Booklets\CBSE\Part-1

(Faraday’s law of Electrolytes)


11. Amount of electricity that can deposit 108 gm of silver from AgNO3 solution is
(a) 1 ampere (b) 1 coulomb
(c) 1 faraday (d) None of the above
12. The atomic weight of Al is 27. When a current of 5 Faradays is passed through a solution of
Al+++ ions, the weight of Al deposited is
(a) 27 gm (b) 36 gm (c) 45 gm (d) 39 gm

E 109
Chemistry
ALLEN
13. What weight of copper will be deposited by passing 2 Faradays of electricity through a cupric
salt (Atomic weight of Cu = 63.5)
(a) 2.0 gm (b) 3.175 gm (c) 63.5 gm (d) 127.0 gm
14. Unit of Faraday is
(a) Ampere (b) Coulomb
–1
(c) Coulomb mole (d) Coulomb sec–1
15. The desired amount of charge for obtaining one mole of Al from Al3+
96500 96500
(a) 3 × 96500 C (b) 96500 C (c) C (d) C
3 2
16. One Faraday of electricity when passed through a solution of copper sulphate deposits
(a) 1 mole of Cu (b) 1 gm atom of Cu
(c) 1 molecule of Cu (d) 1 gm equivalent of Cu
17. When 1 coulomb of charge is passed through electrolyte solution, then the mass deposited is
equal to
(a) Equivalent weight (b) Atomic weight
(c) Electrochemical equivalent (d) Chemical equivalent
18. According to the first law of Faraday, the weight of a substance discharge at the electrode is
Z
(a) W = ZQ (b) W = eF (c) W = It (d) W = ZI
F
19. Total charge on 1 mole of a monovalent metal ion is equal to
(a) 9.65 × 104 Coulomb (b) 6.28 × 1018 Coulomb
(c) 1.6 × 10–19 Coulomb (d) None of these
20. A certain quantity of electricity is passed through an aqueous solution of AgNO3 and cupric salt
solution connected in series. The amount of Ag deposited is 1.08 gm, the amount of copper
deposited is (atomic weight of Cu = 63.5; Ag = 108)
(a) 0.6454 g (b) 6.354 g (c) 0.3177 g (d) 3.177 g
(Conductor and Conductance)
21. The conductivity of strong electrolyte is
(a) Increase on dilution slightly
(b) Decrease on dilution
(c) Does not change with dilution
(d) Depend upon density of electrolytes itself
22. Conductivity of a solution is directly proportional to
(a) Dilution (b) Number of ions
(c) Current density (d) Volume of the solution
node06\B0BC-BD\Kota\Board Material\Chemistry\Booklets\CBSE\Part-1

23. The unit of molar conductivity is


(a) –1 cm–2 mol–1 (b)  cm–2 mol–1
(c) –1 cm2 mol–1 (d)  cm2 mol
24. The unit ohm–1 is used for
(a) Molar conductivity (b) Equivalent conductivity
(c) Specific conductivity (d) Conductivity

110 E
CBSE
ALLEN
(Cell constant and electrochemical cell)
25. Which of the following reaction is used to make a fuel cell
(a) Cd(s)  2Ni(OH)3 (s)  CdO(s)  2Ni(OH)  H 2 O(l)
(b) Pb(s)  PbO 2 (s)  2H 2SO 4 (aq)  2PbSO 4 (s)  2H 2 O(l)
(c) 2H 2 (g)  O 2 (g)  2H 2 O(l)
(d) 2Fe(s)  O 2 (g)  4H  (aq)  2Fe 2 (aq)  2H 2 O(l)
26. When lead storage battery is charged
(a) PbO2 is dissolved (b) H2SO4 is regenerated
(c) PbSO4 is deposited on lead electrode (d) Lead is deposited on lead electrode
27. In electrolysis of dilute H2SO4 using platinum electrodes
(a) H2 is evolved at cathode (b) NH3 is produced at anode
(c) Cl2 is obtained at cathode (d) O2 is produced
28. For cell reaction, Zn + Cu  Zn + Cu, cell representation is
2+ 2+

(a) Zn | Zn2+||Cu2+|Cu (b) Cu | Cu2+ ||Zn2+| Zn


(c) Cu | Zn2+ || Zn | Cu2+ (d) Cu2+ | Zn || Zn2+ | Cu
29. A cell from the following which converts electrical energy into chemical energy
(a) Dry cell (b) Electrochemical cell
(c) Electrolytic cell (d) None of these
30. Hydrogen–oxygen fuel cells are used in space–craft to supply
(a) Power for heat and light (b) Power for pressure
(c) Oxygen (d) Water
 2
31. The cell reaction Cu  2Ag  Cu  Ag is best represented by
(a) Cu(s) | Cu+2(aq) || Ag+(aq) | Ag(s) (b) Pt | Cu+2 || Ag+(aq) | Ag(s)
(c) Cu+2 | Cu || Pt | Ag (d) None of the above representations
32. Zn (s) | Zn 2 (aq) | | Cu 2  (aq) | Cu (s) is
(anode) (cathode)

(a) Weston cell (b) Daniel cell (c) Calomel cell (d) Faraday cell
33. The anode half-reaction occurring during the discharge of a lead storage battery is
(a) Pb(s) + SO2 + O2  PbSO4(s)
(b) Pb(s) + SO42– (aq)  PbSO4(s) + 2e–
(c) PbO2(s) + 4H+(aq) + 2e– + SO42–(aq)  PbSO4(s) + 2H2O
node06\B0BC-BD\Kota\Board Material\Chemistry\Booklets\CBSE\Part-1

(d) Pb2+(aq) + SO42–(aq)  PbSO4(s)


34. The unit of cell constant is
(a) ohm–1 cm–1 (b) ohm cm (c) cm (d) cm–1
35. In galvanic cell, the salt bridge is used to
(a) Complete the circuit (b) Reduce the electric resistance in the cell
(c) Separate cathode from anode (d) Carry salts for the chemical reaction
E 111
Chemistry
ALLEN
(Electrode potential and Nernst equation)
36. The standard electrode potentials of Zn 2 / Zn and Ag  / Ag are –0.763V and +0.799V
respectively. The standard potential of the cell is
(a) 1.56 V (b) 0.036 V (c) – 1.562 V (d) 0.799
37. The standard oxidation potential of zinc and silver in water at 298 K are
Zn(s)  Zn2+ + 2e– ; E = 0.76 V
Ag(s)  Ag2+ + 2e– ; E = –0.80 V
Which of the following reactions actually take place
(a) Zn(s) + 2Ag+(aq)  Zn+2(aq) + 2Ag(s) (b) Zn+2(aq) + 2Ag(s)  2Ag+(aq) + Zn(s)
(c) Zn(s) + Ag(s)  Zn+2(aq) + Ag+(aq) (d) Zn+2 (aq) + Ag+(aq)  Zn(s) + Ag(s)
38. The correct representation of Nernst's equation is
0.0591 0.0591
(a) E M n /M  E o M n /M  log (M n  ) (b) E M n /M  E o M n /M  log (M n  )
n n
n
(c) E M n /M  E o M n /M  log (M n  ) (d) None of the above
0.0591
39. 2H+(aq) + 2e–  H2(g). The standard electrode potential for the above reaction is (in volts)
(a) 0 (b) + 1 (c) – 1 (d) None of these
40. EMF of a cell whose half cells are given below is
Mg2+ + 2e–  Mg(s) ; E = –2.37 V
Cu2+ + 2e–  Cu(s) ; E = +0.33 V
(a) – 2.03 V (b) 1.36 V (c) 2.7 V (d) 2.03 V

ANSWER KEY
Q. No. 1 2 3 4 5 6 7 8 9 10
Ans. d b a c a c a c a c
Q. No. 11 12 13 14 15 16 17 18 19 20
node06\B0BC-BD\Kota\Board Material\Chemistry\Booklets\CBSE\Part-1

Ans. c c c c a d c a a c
Q. No. 21 22 23 24 25 26 27 28 29 30
Ans. a b c d c c a a c b
Q. No. 31 32 33 34 35 36 37 38 39 40
Ans. a b b d a a a a a c

112 E
CBSE
ALLEN
ASSERTION AND REASON
Directions : Following questions have Assertion (A) followed by Reason (R). Answer them
according to following options :
(a) Both (A) and (R) correct and (R) is correct explanation of (A).
(b) Both (A) and (R) correct and (R) is not correct explanation of (A).
(c) (A) is correct but (R) is wrong.
(d) (A) Assertion is wrong but (R) reason is correct.
1. Assertion (A) : Conductivity always increases with decrease in concentration for strong and
weak electrolytes.
Reason (R) : Number of ions per unit volume decreases on dilution.
2. Assertion (A) : Conductivity of pure water is 3.5 × 10–5 S m–1.
Reason (R) : High amounts of hydrogen and hydroxyl ions are present in water.
3. Assertion (A) : Pure water conducts electricity.
Reason (R) : Pure water exists in unionised form.
4. Assertion (A) : Solid NaCl does not conduct electricity.
Reason (R) : Solid NaCl has no free ions.
5. Assertion (A) : Mercury cells give a constant voltage throughout its life.
Reason (R) : Electrolyte KOH is not involved in the reaction.
6. Assertion (A) : 96500 C charge is required for the reduction of one mole of silver ions.
Reason (R) : The amount of electricity (or charge) required for oxidation or reduction depends
on the stoichiometry of the electrode reaction.
7. Assertion (A) : Electrolysis of an aqueous solution of KI gives I2 at the anode but that of KF
gives O2 at the anode not F2.
Reason (R) : I– ions have much lower oxidation potential than water while F– ions have much
higher oxidation potential than water.
8. Assertion : Sodium ions are discharged in preference to hydrogen ions at mercury electrode.
Reason : The nature of cathode can affect the order of discharge of cations.
9. Assertion : Galvanised iron does not rust.
node06\B0BC-BD\Kota\Board Material\Chemistry\Booklets\CBSE\Part-1

Reason : Zinc has a more negative electrode potential than iron.


10. Assertion : A reaction is spontaneous if Ecell = +ve.
Reason : For E ocell = +ve, G is always –ve.

11. Assertion (A) : Fuel cell is a galvanic cell.


Reason (R) : It convert the energy of combustion fuel into electrical energy.

E 113
Chemistry
ALLEN
12. Assertion (A) : Efficiency of fuel cell is 70% as compared to thermal plants.
Reason (R) : Thermal plants have 40% efficiency.
13. Assertion (A) : Fuel cell has no such importance of uses.
Reason (R) : They used in automobiles.
14. Assertion (A) : H2 – O2 fuel cell gives a constant voltage throughout its life.
Reason (R) : In this fuel cell, H2 reacts with OH– ions yet the overall concentration of OH– ions
does not change.

ANSWER KEY
node06\B0BC-BD\Kota\Board Material\Chemistry\Booklets\CBSE\Part-1

Q. No. 1 2 3 4 5 6 7 8 9 10
Ans. d c d a c a c a a a
Q. No. 11 12 13 14
Ans. a b d a

114 E
CBSE
ALLEN
CASE BASED QUESTIONS
Case I
Production of electricity by thermal plants is not a very efficient method and is a major source
of pollution. In such plants, the chemical energy (heat of combustion) of fossil fuels (coal, gas
or oil) is first used for converting water into high pressure steam. This is then used to run a
turbine to produce electricity. A galvanic cell directly converts chemical energy into electricity
and is highly efficient.
Galvanic cells that are designed to convert the energy of combustion of fuels like hydrogen,
methane, methanol, etc. directly into electrical energy arc called fuel cells.
One of the most successful fuel cells uses the reaction of hydrogen with oxygen to form water.
The cell was used for providing electrical power in the Apollo space programme. The water
vapours produced during the reaction were condensed and added to the drinking water supply
for the astronauts. In the cell, hydrogen and oxygen are bubbled through porous carbon
electrodes into concentrated aqueous sodium hydroxide solution. Catalysts like finely divided
platinum or palladium metal are incorporated into the electrodes for increasing the rate of
electrode reactions.
Answer the following questions :
(a) How does fuel cell operate ?
Ans. In fuel cell, chemical energy of the fuel is converted into the electrical energy.
OR
What is the basis of obtaining electrical energy in a fuel cell ?
Ans. In a fuel cell the chemical energy of fuel is converted into the electrical energy.
(b) Which cells were used in Apollo space programme ? What was the product used for ?
Ans. H2 — O2 fuel cell. The product H2O was used for drinking by the astronauts.
(c) (a) State two advantages of H2 — O2 fuel cell over an ordinary cell.
(b) One can utilise the electrochemical principles in preserving fossil fuels. Explain.
Ans. (a) (i) they do not cause any pollution.
(ii) they have high efficiency of 60-70%.
(b) Hydrogen provides an ideal alternate, as on combustion it forms only water. It is a
renewable and non-polluting source of energy. We can use it in place of fossil fuels
such as coal, petroleum etc.
Case II
node06\B0BC-BD\Kota\Board Material\Chemistry\Booklets\CBSE\Part-1

Molar conductivity of a solution is the conductance of solution containing one mole of


electrolyte, kept between two electrodes having unit length between them and large cross
sectional area so as to contain the electrolyte. In other words, molar conductivity is the
conductance of the electrolytic solution kept between the electrodes of a conductivity cell at
unit distance but having area of cross section large enough to accommodate sufficient volume
of solution that contains one mole of the electrolyte. It is denoted by m.

E 115
Chemistry
ALLEN
Answer the following questions :
(a) Write the mathematical expression for molar conductivity.
Ans. The mathematical expression for molar conductivity is given as :
  1000
m 
M
Where,  = Molar conductivity of solution
 = Conductivity of solution
M = Molarity of the solution
(b) What are the units of molar conductivity m ?
Ans. The unit of molar conductivity m are –1m2 mol–1 or S m2mol–1
(c) Conductivity of 0.00241 M acetic acid is 7.896 × 10–5 Scm–1. Calculate its molar
conductivity in this solution.
Ans. Given k = 7.896 × 10–5 Scm–1
Molarity (M) = 0.00241 M
  1000
Molar conductivity m =
M
7.896  105 Scm 1  1000
=
0.00241 M

  m  32.76 Scm 2 mol1


OR
(i) How does molar conductivity varies with concentration for strong as well as weak
electrolytes ?
Ans. Molar conductivity increases with decrease in concentration. This is because, the total
volume (V) of solution containing one mole of electrolyte increases.
(ii) Give the graphical representation of variation of conductivity with concentration for
weak electrolytes ?
Ans. The graphical representation of variation of molar conductivity with concentration for
weak electrolyte is given as
node06\B0BC-BD\Kota\Board Material\Chemistry\Booklets\CBSE\Part-1

0

Variation of m with for weak electrolyte

116 E
CBSE
ALLEN
PREVIOUS YEARS QUESTIONS
SECTION-A (ONE MARK QUESTIONS)
1. Which of the following solutions of KCl will have the highest value of molar conductivity ?
[1] (CBSE 2023)
(a) 0.01 M (b) 1 M (c) 0.5 M (d) 0.1 M
Ans. (a)
2. Assertion (A) : Conductivity decreases with decrease in concentration of electrolyte.
Reason (R) : Number of ions per unit volume that carry the current in a solution decreases on
dilution. [1] (CBSE 2023)
Ans. (a)
3. Assertion (A) : Molar conductivity decreases with increase in concentration.
Reason (R) : When concentration approaches zero, the molar conductivity is known as limiting
molar conductivity. [1] (CBSE 2023)
Ans. (b)
4. Predict the change in molar conductivity with decrease in concentration.
(CBSE 2021 Compartment)
Ans. Molar conductivity increases with decrease in concentration. Because the total volume (V) of the
solution containing one mole of electrolyte also increases.
5. Write the name of the cell which is generally used in inverters. Write the reactions taking place at
the anode and the cathode of this cell. [1] (CBSE 2020)
Ans. Lead storage battery
Anode : Pb(s) + SO42–(aq.) PbSO4(s) + 2e– [1/2]
Cathode : PbO2 + SO42–(aq) + 4H+ + 2e– PbSO4(s) + 2H2O() [1/2]

6. What is meant by ‘limiting molar conductivity’? [1] (CBSE 2019)


Ans. Limiting molar conductivity: The molar conductivity of a solution at infinite dilution is called
limiting molar conductivity and is represented by the symbol 0m .

7.
node06\B0BC-BD\Kota\Board Material\Chemistry\Booklets\CBSE\Part-1

Express the relation between the conductivity and the molar conductivity of a solution.
[1] (CBSE 2018)
Ans. The relationship between conductivity () and molar conductivity of a solution is given by

m 
C
where C is concentration of solution.

E 117
Chemistry
ALLEN
SECTION-B (TWO MARKS QUESTIONS)
8. (a) Give reasons : [2] (CBSE 2023)
(i) Mercury cell delivers a constant potential during its life time.
(ii) In the experimental determination of electrolytic conductance, Direct Current (DC) is
not used.
OR
(b) Define fuel cell with an example. What advantage do the fuel cells have over primary and
secondary batteries ?
Ans. (a) Because as the overall reaction does not involve any ion in solution.
(b) Because passing direct current (DC) changes the composition of the solution.
OR
The galvanic cell that are designed to convert the energy of combustion of fuel cell like
hydrogen, methane, methanol directly into electrical energy eg. H2 – O2 fuel cell.
(i) These cell never becomes dead due to the continuous supply
(ii) These are highly efficient.
9. Define conductivity for the solution of an electrolyte. Why does the conductivity of a solution
decreases with dilution? [2] (CBSE 2022 Term - II)
Ans. Specific conductance/ Conductivity () :-
1
It is defined as the reciprocal of specific resistance,  =


R =  
A
 1 
 .
 R A
  GG*
Specific conductance = Conductance × Cell constant
Hence specific conductivity of a solution is defined as the conductance offered by 1 cm3 of
electrolytic solution. Unit of  is ohm 1 cm 1
The number of ions (responsible for carrying current) decreases when the solution is diluted. As a
result, the conductivity of a solution decreases with dilution.
10. Predict the products of electrolysis in each of the following :
node06\B0BC-BD\Kota\Board Material\Chemistry\Booklets\CBSE\Part-1

(a) An aqueous solution of CuSO4 with Pt electrode


(b) An aqueous solution of AgNO3 with Pt electrode [2] (CBSE 2021 Compartment)
OR
How many seconds does it require to reduce 3 moles of Fe3+ to Fe2+ with 2 A current ?
(Given : 1 F = 96500 C mol–1)

118 E
CBSE
ALLEN
Ans. (a) At anode : 2H2O() O2(g) + 4H+(aq.) + 4e–

At Cathode : Cu+2 + 2e– Cu(s)


Bubbles of O2 gas are formed at Anode & Copper deposited at Cathode.
(b) At Cathode : Ag+(aq.) + e– Ag(s) ; Eocell = 0.80 V
1
H+(aq.) + e–  H 2 (g); Eocell = 0V
2
The value of E cell is high (Ag ions have low discharge potential than H+ ions) then Ag+ ions
o +

deposited at cathode.
At Anode : Pt electrodes are inert, OH– ions have low discharge potential will be discharged in
preference to NO3– ions. OH– ions will de compose to give oxygen.
4OH–(aq.) H2O() + O2(g) + 4e–
OR
Given, Current = 2.0 Ampere
3Fe3+ + 3e–3Fe2+
Charge required = 3 Faraday = 3 × 96500 C = 289500 C
We know, Charge = Current × Time
289500 144750
or Time = = 144750 sec. = min . = 2412.5 min.
2 60
2412.5
= sec. = 40.20 sec.
60
11. Define the following terms: [2] (CBSE 2020)
(i) Molar conductivity (  m )
(ii) Secondary batteries
Ans. (i) Moalr conductivity,  m , is defined by k/c where c is the concentration. Conductivity
decreases but molar conductivity increases with decrease in concentration. It increases
slowly with decrease in concentration for strong electrolytes while the increase is very
steep for weak electrolytes in very dilute solutions.
(ii) A secondary cell after use can be recharged by passing current through it in the opposite
direction so that it can be used again. A good secondary cell can undergo a large number of
discharging and charging cycles.
12. The standard electrode potential (E°) for Daniel cell is +1.1 V. Calculate the G° for the reaction.
Zn (s) + Cu2+ (aq)  Zn2+ (aq) + Cu (s) [2] (CBSE 2019)
(1F = 96500 Cmol–1).
node06\B0BC-BD\Kota\Board Material\Chemistry\Booklets\CBSE\Part-1

Ans. Zn (s) + Cu2+ (aq)  Zn2+ (aq) + Cu (s)


E° = 1.1V
G° = –nFE°
= –2 × 96500 × (l.l)
= –212300 J

E 119
Chemistry
ALLEN
13. Express the relation among cell constant, resistance of the solution in the cell and conductivity of
the solution. How is molar conductivity of a solution related to its conductivity ? [2] [CBSE 2018]
OR
The molar conductivity of a 1.5 M solution of an electrolyte is found to be 138.9 S cm2mol–1.
Calculate the conductivity of this solution.
1 1   1000
Ans.  =   m 
 R A C
OR
  1000 138.9 1.5
 m =  = = 0.20835 Scm–1
C 1000
SECTION-C (THREE MARKS QUESTIONS)
14. (a) Calculate rG° and log KC for the following cell :
Ni(s) + 2Ag+(aq)  Ni2+(aq) + 2Ag(s)
Given that E ocell = 1.05 V, 1F = 96,500 Cmol–1 [3] (CBSE 2022 Term - II)
OR
(b) Calculate the e.m.f. of the following cell at 298 K.
Fe(s) | Fe2+(0.001 M) || H+(0.01 M) | H2(g) (1 bar) | Pt(s)
Given that E ocell = +0.44 V
[log 2 = 0.3010, log 3 = 0.4771, log 10 = 1]
Ans. (a) rG° = –nF E ocell
= –2 × 96500 × 1.05
rG° = –202.65 kJ mol–1 [½]
rG° = –2.303 RT log KC [½]
 r G o 202.65
log K C  
2.303RT 2.303  8.314  298
log KC = 35.54 Kc = antilog (35.54) = 3.47 × 1035
OR
0.0591 [Fe 2 ]
(b) E cell  E ocell  log  2
node06\B0BC-BD\Kota\Board Material\Chemistry\Booklets\CBSE\Part-1

2 [H ]
0.0591 0.001
 0.44V  log
2 (0.01) 2
0.0591
 0.44V  log10  0.41V
2

120 E
CBSE
ALLEN
15. Conductivity of 0.002 M acetic acid is 8 × 10–5 S cm–1. Calculate its molar conductivity and degree of
dissociation () if  om for acetic acid is 390.5 S cm2 mol–1. [3] (CBSE 2021 Compartment)

Ans.  = 8 × 10–5 Scm–1


C = 0.002 M or 0.002 mol L–1.
1000 1000
Molar conductivity (m) =    8 105   40 Scm2 mol–1
C 0.002
m 40
Degree of dissociation () =   0.102
 m 390.5
0

C 2 0.002mol L   0.102 
1 2
0.002mol L1  0.0104
Dissociation constant (Ka) =  
1  1  0.102 0.898
Ka = 2.31 × 10–5 mol L–1.
16. (a) Calculate Ecell  0.261V for the following reaction at 298 K:

2Cr(s) + 3Fe2+(0.01M)  2Cr3+(0.01M) + 3Fe(s)


Given : Ecell = 0.261V
(b) Using the E° values of A and B, predict which one is better for coating the surface of iron
[E°(Fe2+/Fe) = –0.44 V] to prevent corrosion and why ?
Given : E°(A2+/A) = –2.37 V : E°(B2+/B) = –0.14V [3] (CBSE 2020)
0.0591 [Cr 3 ]2
Ans. (a) E cell  E 0
 log [1/2]
[Fe 2 ]3
cell
n

0.0591 [0.01]2
0.261V  E 0cell  log [1/2]
6 [0.01]3
0.0591
0.261V  E 0cell  log100 [1/2]
6
E 0cell  0.261  0.0197
= 0.2807 V [1/2]
(b) A because low value of SRP [1]
17. Calculate E ocell and rG0 for the following reaction at 25°C :
A2+ + B+  A3+ + B
node06\B0BC-BD\Kota\Board Material\Chemistry\Booklets\CBSE\Part-1

Given : Kc = 1010, 1 F = 96500 C mol–1 [3] (CBSE 2020)


Ans. A2+ + B+  A3+ + B (n = 1)
Kc = 1010 F = 96500C/mol T = 25ºC = 298K
Go = ? Eo = ? R = 8.314J/K/mol
Go = –2.303RT log Kc [1]

E 121
Chemistry
ALLEN
Go = –2.303 × 8.314 J/K/mol × 298K × log 1010.
 Go = –57058.4 J/mol or –57.0584 kJ/mol
Go = –57058.4 J/mol = –nFEo = –1 × 96500 × Eo [1]
57058.4
 Eo   0.591V [1]
96500
18. Calculate the time to deposit 1.5 g of silver at cathode when a current of 1.5 A was passed
through the solution of AgNO3. (Molar mass of Ag = 108 g mol–1, 1 F = 96500 C mol–1)
[3] (CBSE 2019)
Ans. Wt. of Ag = 1.5g Current = i = 1.5amp
Molecular mass = 108 g/mol F = 96500 C/mol
n = number of electron transferred
MIt
W [1]
nF
M  n  F 1.5  1  96500
 t  [1]
MI 108  1.5
= 893.51 s or 14.89 min [1]
19. The electrical resistance of a column of 0.05 M NaOH solution of diameter 1 cm and
length 50 cm is 5.55 × 103 ohm. Calculate its resistivity, conductivity and molar conductivity.
[3] (CBSE 2019)
 RA 5.55  10 3   0.785cm 2
Ans. R =  or  = = = 87.135 cm
A  50cm
1  1 
Conductivity = =  Scm–1 = 0.01148 Scm–1
  87.135 
 1000 3 –1
Molar conductivity m = cm L
C
0.01148Scm 1  1000cm 3 L1
= 1
= 229.6 Scm2mol–1
0.05molL
20. The resistance of a conductivity cell containing 0.001 M KCl solution at 298 K is 1500 . What
is the cell constant if the conductivity 0.001 M KCl solution at 298 K is 0.146 × 10–3 S cm–1?
[3] (CBSE 2019)
Ans. Here conductivity () = 0.146 × 10–3 S cm–1
resistance (R) = 1500 
node06\B0BC-BD\Kota\Board Material\Chemistry\Booklets\CBSE\Part-1

conductivity
Cell constant = = conductivity × Resistance
conductance
1
=×R [ conductance = ]
resistance
cell constant

Resistance
Cell constant = 0.146 × 10–3 × 1500 = 0.219 cm–1

122 E
CBSE
ALLEN
21. (a) The cell in which the following reaction occurs :
2 Fe3+(aq) + 2 I–(aq)  2Fe2+(aq) + I2(s)
has Eocell = 0.236 V at 298 K, Calculate the standard Gibbs energy of the cell reaction.
(Given: 1 F = 96,500 C mol–1)
(b) How many electrons flow through a metallic wire if a current of 0.5 A is passed for
2 hours ? (Given : 1 F = 96,500 C mol–1) [3] (CBSE 2018)
Ans. (a)  G0 = –nFE0cell [1/2]
n=2
G0 = –2 × 96500 C/mol × 0.236 V
= –45548 J/mol
= – 45.548 kJ/mol [1]
(b) Q = l t = 0.5 × 2 × 60 × 60 [1/2]
= 3600 C
96500 C = 6.023 × 1023 electrons
3600 C = 2.25 × 1022 electrons [1]
22. Depict the galvanic cell in which the reaction
Zn (s) + 2Ag+ (aq) Zn2+ (aq) + 2 Ag (s)
takes place. Further indicate what are the carriers of the current inside and outside the cell. State
the reaction at each electrode. [3] (CBSE 2018)
Ans. The cell reaction is
Zn (s) + 2Ag+ (aq) Zn2+ (aq) + 2 Ag (s)
The cell is represented as, Zn(s) | Zn2+ (aq) || Ag+ (aq) | Ag(s)
SECTION-D (FOUR MARKS QUESTIONS)
23. Case Based Question:- [1+1+2=4]
The lead-acid battery represnts the oldest rechargeable battery technology. Lead acid batteries
can be found in a wide variety of applications including small-scale power storage such as UPS
systems, ignition power sources for automobiles, along with large, grid-scale power systems. The
spongy lead act as the anode and lead dioxide as the cthode. Aqueous sulphuric acid is used as an
electrolyte. The half-reactions during discharging of lead storage cells are :
node06\B0BC-BD\Kota\Board Material\Chemistry\Booklets\CBSE\Part-1

Anode : Pb(s) + SO 24  (aq) PbSO4(s) + 2e–

Cathode : PbO2(s) + 4H+(aq) + SO 24 (aq) + 2e–  PbSO4(s) + 2H2O


There is no safey way of disposal and these batteries end - up in landfills, Lead and sulphuric
acid are extremely hazardous and pollute soil, water as well as air. Irrespective of the
environmental challenges it poses, lead-acid batteries have remained an important source of
energy.
E 123
Chemistry
ALLEN
Designing green and sustainable battery systems as alternatives to conventional means remains
relevant. Fuel cells are seen as the future source of energy. Hydrogen is considered a green fuel.
Problem with fuel cells at present is the storage of hydrogen. Currently, ammonia and methanol
are being used as a source of hydrogen for fuel cell. These are obtained industrially, so add to the
environmental issues.
If the problem of storage of hydrogen is overcome, is it still a "green fuel?" Despite being the
most abundant element in the Universe, hydrogen does not exist on its own so needs to be
extracted from the water using electrolysis or separated from carbon fossil fuels. Both of these
processes require a significant amount of energy which is currently more than that gained from
the hydrogen itself. In addition, this extraction typically requires the use of fossil fuels. More
research is being conducted in this field to sovle these problems. Despite the problem of no good
means to extract Hydrogen. It is a uniquely abundant and renewable source of energy, perfect for
our future zero-carbon needs.
Answer the following questions :
(a) How many coulombs have been transferred from anode to cathode in order to consume one
mole of sulphuric acid during the discharging of lead storage cell?
Ans. 2 mol e– (or 2F) have been transferred from anode to cathode to consume 2 mol of H2SO4
therefore, one mole H2SO4 requires oen faraday of electricity or 96500 coulombs.
(b) How much work can be extracted by using lead storage cell if each cell delivers about
2.0 V of voltage ? (1 F = 96500 C)
Ans. wmax = –nFE* = –2 × 96500 × 2.0 = 386000 J of work can be extracted using lead storage
cell when the cell is in use.
(c) Do you agree with the statement - "Hydrogen is a green fuel." Give your comments for and
against this statement and justify your views.
Ans. Both yes and no should be accepted as correct answers depending upon what explanation
is provided.
Yes, Hydrogen is a fuel that on combustion gives water as a byproduct. There are no
carbon emissions and no pollutions caused.
However, at prsent the means to obtain hydrogen are electrolysis of water which use
electricity obtained from fossil fuels and increase carbon emission.
Inspite of the problems faced today in the extraction of hydrogen, we cannot disagree on
the fact that hydrogen is a clean source of energy. Further research can help in finding
solutions and greens ways like using solar energy for extraction of hydrogen. [2]
node06\B0BC-BD\Kota\Board Material\Chemistry\Booklets\CBSE\Part-1

No, It is true that Hydrogen is a fuel that on combustion gives water as a byproduct. There
are no carbon emissions and no pollutions caused.
However, at present the means to obtain hydrogen are electrolysis of water which use
electricity obtained from fossil fuels and increase carbon emissions.
Hydrogen is no doubt a green fuel, but the process of extraction is not green as of today. At
present, looking at the process of extraction, hydrogen is not a green fuel. [2]

124 E
CBSE
ALLEN
OR
Imagine you are a member of an agency funding scientific research. Which of the following
projects will you fund and why ?
(i) Safe recycling of lead batteries
(ii) Extraction of hydrogen
Ans. (i) Lead batteries are currently the most important and widely used batteries. These are
rechargeable. The problem is waste management which needs research and awareness.
Currently, these are being thrown into landfills and there is no safe method of disposal or
recycling. Researching into safe method of disposal will reduce the pollution and health
hazards caused to a great extent.
(ii) Fuel cell is a clean source of enegy. Hydrogen undergoes combustion to produce water.
The need of the hour is green fuel and hydrogen is a clean fuel. The current problem is
obtaining hydrogen. Research that goes into this area will help solve the problem of
pollution and will be a sustainable solution.
SECTION-E (FIVE MARKS QUESTIONS)
24. (a) Conductivity of 2 × 10–3 M methanoic acid is 8 × 10–5 S cm–1. Calculate its molar
conductivity and degree of dissociation if  on for methanoic acid is 404 S cm2 mol–1.

(b) Calculate the rG° and log Kc for the given reaction at 298 K : [5] (CBSE-2023)

Ni (s)  2Ag (aq.)  Ni (aq.)

2
 2Ag (s)

Given : E oNi2 /N  0.25V, E oAg /Ag  0.80V


i

1F = 96500 C mol–1.
Ans. (a) k = 8 × 10–5 Scm–1
1000 1000
m  k   8 10 –5   40Scm 2 mol –1 [1]
C 2 10 –3

 40
  0m   0.099  0.1 [1]
 m 404

(b) rG° = –nF E ocell [½]

= –2 × 96500 × 1.05 [½]


node06\B0BC-BD\Kota\Board Material\Chemistry\Booklets\CBSE\Part-1

rG° = –202.65 kJ mol–1 [½]


0.0591
E ocell  log k c [½]
n
0.0591
1.05  log k c [½]
2
log KC = 35.54 [½]

E 125
Chemistry
ALLEN
25. (a) Calculate rG° for the reaction
Zn(s) + Cu2+(aq)  Zn2+(aq) + Cu(s)
Given: Eocell for Zn2+/Zn = –0.76 and

Eocell for Cu+2/Cu = +0.34 V


R = 8.314 JK–1mol–1
F = 96500 mol–1
(b) Give two advantages of fuel cell. [3 + 2 = 5] (CBSE 2020)
OR
(a) Out of the following pairs, predict with reason which pair will allow greater conduction of
electricity : [3]
(i) Silver wire at 30°C or silver wire at 60° C.
(ii) 0.1 M CH3COOH solution or 1 M CH3COOH solution.
(iii) KCl solution at 20°C or KCl solution at 50°C.
(b) Give two points of difference between electrochemical and electrolytic cells. [2]
Ans. (a) Eocell = E 2  E  2 [½]
Cu /Cu Zn /Zn

= 0.34 – (–0.76)
= 1.10 V [1]
rGº = –nF Eocell [½]
= –2×1.10×96500
= –212300 J/mol or –212.3 kJ/mol [1]
(b) (i) Pollution free (ii) High efficiency [2]
OR
(a) (i) Silver wire at 30ºC because as temperature decreases, resistance decreases so
conduction increases. [1]
(ii) 0.1 M CH3COOH, because on dilution degree of ionization increases hence
conduction increases. [1]
(iii) KCl solution at 50ºC, because at high temperature mobility of ions increases and
hence conductance increases. [1]
(b) [2]
node06\B0BC-BD\Kota\Board Material\Chemistry\Booklets\CBSE\Part-1

Electrochemical Electrolytic
(1) Anode –ve Anode +ve
Cathode +ve Cathode –ve
(2) Convert chemical Convert electrical
Energy to electrical energy Energy of chemical energy

126 E
CBSE
ALLEN
26. (a) How many moles of mercury will be produced by electrolysing 1.0 M Hg (NO3)2 solution
with a current of 2.00 A for 3 hours? [Hg = 200.6 g mol-1]
(b) A voltaic cell is set up at 25ºC with the following half-cells Al3+ (0.001 M) and Ni2+
(0.50 M). Write an equation for the reaction that occurs when the cell generates an electric
current and determine the cell potential. (Give Eº Ni2 /Ni  0.25V, Eº Al3 /Al  1.66 V )
[5] (CBSE 2020)
Ans. (a) Time = 3 hours
= 3 × 60 × 60 = 10800 Sec.
Current = 2A
Charge = Current × time
= 2 × 10800 = 21600 C
According to the questions
deposited mass of mercury
W = ZIt
E
  I  t  I  t  21600C
96487
200.6
W  21600  22.45g
2  96487
22.45
Number of mole of Hg =
200
Number of moles of mercury = 0.11 mole
(b) Given : E0Ni2 /Ni  0.25V, E0Al3 /Al  1.66 V
Halfcell equations are
(Al  Al3+ + 3e – )  2 (at Anode)
(Ni 2+ + 2e-  Ni)  2 (at Cathode)
2Al + 3Ni 2+  2Al3  3Ni Over all reaction
The cell may be represented as
Al | Al3+ || Ni2+ | Ni,
E ocell  E oright  E left
o

= (–0.25) – (–1.66) = –0.25 + 1.66



node06\B0BC-BD\Kota\Board Material\Chemistry\Booklets\CBSE\Part-1

E ocell = 1.41 V.
27. (a) The conductivity of 0.001 mol L–1 acetic acid is 4.95 × 10–5 S cm–1. Calculate the
dissociation constant if  0m for acetic acid is 390.5 S cm2 mol–1. [5] (CBSE 2019)
(b) Write Nernst equation for the reaction at 25°C :
2Al(s) + 3Cu2+(aq) 2Al3+(aq) + 3Cu(s)
(c) What are secondary batteries ? Given an example.

E 127
Chemistry
ALLEN
OR
(a) Represent the cell in which the following reaction takes place : [5]
2Al(s) + 3Ni2+(0.1 M) 2Al3+(0.01 M) + 3Ni(s)
Calculate its emf if E 0cell  1.41V
(b) How does molar conductivity vary with increase in concentration for strong electrolyte and
weak electrolyte ? How can you obtain limiting molar conductivity  0m for weak
electrolyte ?
 4.95  103 Scm 2 1000 cm 3
Ans. (a) m   1
  49.5Scm 2 mol –1 [1]
c 0.001 mol L L

m 49.5 Scm 2 mol 1


   0.126 [1]
 0m 390.5Scm 2 mol 1

0.001 mol L-2   0.126 


2
c 2
Ka    1.81106 mol L1 [1]
1    1  0.126

(If K = c2, then K = 1.6 × 10–5 mol L–1)


2
 Al3 
log  
 0.059
(b) E cell  E cell  3
[1]
6 Cu 
2 
 
(c) Batteries which are rechargeable [1]
Example- Lead storage, Ni-Cd batteries (Or any other one example)
OR
(a) Al(s) | Al3+(0.01 M) || Ni2+(0.1 M) | Ni(s)
2
 Al3 
log  
 0.059
E cell   E cell   3
[1]
6  Ni2 
 

E  cell   1.41V 
0.059
log
0.01 2
[1]
6 0.13
E(cell) = 1.4198 V or E(cell) = 1.42 V [1]
node06\B0BC-BD\Kota\Board Material\Chemistry\Booklets\CBSE\Part-1

(b) m decreases with increase in concentration for both strong & weak electrolyte  0m can be
obtained for weak electrolyte by applying Kohlrausch law. [2]
28. (a) Write the cell reaction and calculate the e.m.f. of the following cell at 298 K:
Sn (s) | Sn2+ (0.004 M) || H+ (0.020 M) | H2(g) (1 bar) | Pt (s)
(Given : E 0 2   0.14V )
Sn /Sn

128 E
CBSE
ALLEN
(b) Give reasons :
(i) On the basis of Eº values, O2 gas should be liberated at anode but it is Cl2 gas which is
liberated in the electrolysis of aqueous NaCl.
(ii) Conductivity of CH3COOH decreases on dilution. [3+2 = 5] (CBSE 2019)
OR
(a) For the reaction, 2AgCl(s) + H2 (g) (1 atm)  2Ag (s) + 2H+ (0.1 M) + 2Cl– (0.1 M),
GO = – 43600 J at 25°C.
Calculate the e.m.f. of the cell. [log 10–n = –n]
(b) Define fuel cell & write its two advantages.
Ans. (a) Sn(s) |Sn2+ (0.004 M) || H+(0.020 M)| H2(g), 1 bar |Pt(s)
(given  E 0  0.14V )
Sn 2  /Sn

Reaction at Anode  Sn(s)  Sn 2  (aq)  2e  [1/2]

Reaction at Cathode  2H  (aq)  2e   H 2 (g) [1/2]


__________________
Overall reaction  Sn(s) + 2H  (aq)  Sn 2  (aq)  H 2 (g) [1/2]

Ecell  (0.14) V  0.14 V [1/2]

0.0591 [Sn 2 ]
Ecell  Ecell  log  2 [1/2]
2 [H ]
0.0591 0.004
 0.14  log  0.11045V [1/2]
2 [0.020]2
(b) (i) E° value of O2 is higher Cl2 but O2 will be liberated from H2O only when higher voltage
is applied. NaCl  Na+ + Cl–
H2O  H+ + OH– [1]

Due to over potential of O2


(ii) Conductivity changes as the concentration of electrolyte changes.
The conductivity will always decrease with decrease in the concentration because the no. of ions
per unit volume carrying current decreases on dilution. Hence, the conductivity of CH3COOH
decreases on dilution. [1]
node06\B0BC-BD\Kota\Board Material\Chemistry\Booklets\CBSE\Part-1

OR
(a) 2AgCl (s) + H2(g) (1 atm)  2Ag(s) + 2H+(0.1 M) + 2Cl–(0.1M)

G    nF E cell [1/2]

G (43600)
E cell    0.23V [1/2]
nF 2  96500

E 129
Chemistry
ALLEN
0.0591
 0.23  log (0.1)2 (0.1)2 [1/2]
2
0.0591
 0.23  log (0.0001) [1/2]
2
0.0591
 0.23  log (10)4
2
= 0.23 + 0.1182 = 0.3482 V
E cell  0.35 V [1]

(b) The are galvanic cell that are designed to convert the energy of combustion of fuel like
hydrogen, methane, methanol directly into electrical energy. eg. : H2 – O2 Fuel cell [1]
Two advantages of fuel cells 
(i) These cells never becomes dead because due to the continuous supply of fuels. [1/2]
(ii) Do not cause only pollution like thermal plant. [1/2]
29. Two half-cell reactions of an electrochemical cells are given below :
MnO–4(aq) + 8H+(aq) + 5e–,  Mn2+(aq) + 4H2O(), E° = + 1.51 V

Sn2+(aq)  Sn4+(aq) + 2e–, E° = –0.15 V


Construct the redox equation from the two half-cell reactions and predict if this reaction favours
formation of reactants or product shown in the equation. [3 + 2 = 5] (CBSE 2018)
Ans. At anode (oxidation) :
Sn2+ (aq)  Sn4+ (aq) + 2e–, E0 = –0.15 V
At cathode (reduction) :
MnO–4(aq) + 8H+(aq) + 5e– Mn2+(aq) + 4H2O(), E0 = +1.51V

Thus over all redox equation is


5Sn2+(aq)+2MnO–4(aq)+16H+(aq) 5 Sn4+(aq)+2Mn2+(aq) + 8H2O()

E0cell = E0cathode – E0anode = 1.51 – 0.15 = 1.36 V


Hence the reaction favours formation of product.
30. A copper-silver cell is set up. The copper ion concentration in it is 0.10 M. The concentration of
silver ion is not known. The cell potential measured 0.422 V. Determine the concentration of
silver ion in the cell.
node06\B0BC-BD\Kota\Board Material\Chemistry\Booklets\CBSE\Part-1

Given : E 0Ag  / Ag   0.80 V, E Cu


0
2
/Cu
  0.34V [5] (CBSE 2018)

Ans. Half-cell reactions :


At cathode : 2Ag+ (aq) + 2e– 2Ag(s) ;
At anode : Cu(s)  Cu2+ (aq) + 2e–
The cell reaction is Cu(s) + 2Ag+(aq)  Cu2+(aq) + 2Ag(s)

130 E
CBSE
ALLEN
The cell can be represented as
Cu(s) | Cu2+ (aq) || Ag+(aq) | Ag(s)
 E0cell = E0cathod – E0anode
= 0.80 – 0.34 = 0.46 V
0.0591 [Cu 2 ]
 E cell  E 0cell  log
n [Ag  ]2
0.0591 0.1
0.422  0.46  log
2 [Ag  ]2
0.0591
0.422 – 0.46 = – (log 0.1 – log[Ag+]2)
2
0.038  2
 = [log 0.1 – log[Ag+]2]
0.0591
1.286 = –1– log [Ag+]2 = 2.286 = –2log[Ag+]
log [Ag+]= –1.143; [Ag+] = antilog (–1.143) = 7.2 × 10–2 M
node06\B0BC-BD\Kota\Board Material\Chemistry\Booklets\CBSE\Part-1

E 131
Chemistry
ALLEN
PRACTICE TEST
SECTION-A
1. The number of Faradays (F) required to produce 20 g of calcium from molten CaCl2 (Atomic
mass of Ca = 40 g mol–1) is :
(a) 4 (b) 1 (c) 2 (d) 3
2. On electrolysis of dil. sulphuric acid using Platinum (Pt) electrode, the product obtained at anode
will be:
(a) SO2 gas (b) Hydrogen gas (c) Oxygen gas (d) H2S gas
3. For the cell reaction
2Fe3+ (aq) + 2I–(aq)  2Fe2+(aq) + I2(aq)
E
cell  0.24V at 298 K. The standard Gibbs energy  r G

 
of the cell reaction is :

[Given that Faraday constant F = 96500 C mol–1]


(a) – 46.32 kJ mol–1 (b) – 23.16 kJ mol–1 (c) 46.32 kJ mol–1 (d) 23.16 kJ mol–1
4. A button cell used in watches function as following
Zn(s) + Ag2O(s) + H2O()  2Ag(s) +
Zn2+(aq) + 2OH–(aq)
If half cell potentials are
Zn2+(aq) + 2e–  Zn(s); E° = –0.76V
Ag2O(s) + H2O() + 2e–  2Ag(s) + 2OH–(aq);
E° = 0.34V
The cell potential will be :-
(a) 1.34 V (b) 1.10 V (c) 0.42 V (d) 0.84 V
5. Limiting molar conductivity of NH4OH

i.e.  

m NH 4OH   is equal to:-
     
(a)  m NH4OH    m NH4Cl   m HCl (b)  m NH4Cl   m NaOH    m NaCl
     
(c)  m NH4Cl   m NaCl   m NaOH  (d)  m NaOH    m NaCl   m NH4Cl
6. Assertion (A) : Electrolytes used in fuel cells.
Reason (R) : Cont. KOH (aq) solution cannot be used as electrolyte.
node06\B0BC-BD\Kota\Board Material\Chemistry\Booklets\CBSE\Part-1

7. Assertion (A) : H2O2 fuel cell has many advantages.


Reason (R) : It causes pollution.
SECTION-B
8. Calculate rGº for the reaction [2]
Mg(s) + Cu2+ (aq) Mg2+ (aq) + Cu (s)
Given : Eºcell = + 2.71 V, 1F = 96500 C mol–1

132 E
CBSE
ALLEN
9. On the basis of Eº values, O2 gas should be liberated at anode but it is Cl2 gas which is liberated
in the electrolysis of aqueous NaCl. Explain. [2]
–1 –5 –1
10. The conductivity of 0.001 mol L solution of CH3COOH is 3.905 × 10 S cm . Calculate its
molar conductivity and degree of dissociations().
Given °(H+) = 349.6 S cm2 mol–1 and °(CH3COO–)= 40.9 S cm2 mol–1. [2]
11. How does molar conductivity vary with increase in concentration for strong electrolyte and weak
electrolyte? How can you obtain limiting molar conductivity  0m for weak electrolyte? [2]
SECTION-C
12. Calculate the emf of the following cell at 25°C: [3]
Ag (s) | Ag+ (10–3 M)|| Cu2+ (10–1 M)| Cu(s)
Given E° = + 0.46 and log 10n = n.
13. (a) Calculate Ecell for the following reaction at 298 K:
2Cr(s) + 3Fe2+(0.01M)  2Cr3+(0.01M) + 3Fe(s)
Given : Ecell = 0.261V
(b) Using the E° values of A and B, predict which one is better for coating the surface of iron
[ E 0Fe2 Fe = –0.44 V] to prevent corrosion and why ?

Given : E 0A 2 A = –2.37 V : E°(B2+/B) = –0.14V


SECTION-D
14. Any battery (have one or more than one cell connected in series) or cell that we use as a source of
electrical energy is basically a galvanic cell where the chemical energy of the redox reaction is
converted into electrical energy. However, for a battery to be of practical use it should be
reasonably light, compact and its voltage should not vary appreciably during its use. These are
further classified as primary and secondary cell. [1+1+2=4]
(a) What are primary cells ?
OR
What are secondary cells ?
(b) Describe the composition of anode and cathode in a mercury cell ?
(c) Write the electrode reactions for mercury cell :
(i) At anode (ii) At cathode
SECTION-E
15. (a) For the reaction [5]
2AgCl(s) + H2 (g) (1 atm)  2Ag (s) + 2H+ (0.1 M) + 2Cl (0.1 M),

 GO = – 43600 J at 25°C.


node06\B0BC-BD\Kota\Board Material\Chemistry\Booklets\CBSE\Part-1

Calculate the e.m.f. of the cell.


[log 10–n = –n]
(b) Calculate the potential for half-cell containing
0.10 M K2 Cr2O7 (aq), 0.20 M Cr3+ (aq) and 1.0 × 10-4 M H+ (aq)
The half-cell reaction is Cr2 O72 (aq)  14 H  (aq)  6 e   2 Cr 3 (aq)  7 H 2 O (l ), and the
standard electrode potential is given as Eº = 1.33 V.

E 133
Chemistry
ALLEN
PRACTICE TEST SOLUTIONS
SECTION-A
1. (b)
2. (c)
3. (a)
4. (b)
5. (b)
6. (c)
7. (c)
SECTION-B
8. G° = – nFE°
= –2 × 96500 × (2.71) = –523030 J
9. E° value of O2 is higher Cl2 but O2 will be liberated from H2O only when higher voltage is
applied. NaCl
 Na+ + Cl¯
 H+ + OH¯
H2O 

Due to over potential of O2


10.  cm =  × 1000/C

= 3.905 × 10–5 × 1000/0.001


= 39.05 S cm2/mol
CH3COOH  CH3COO¯ + H+

 oCH3COOH   0CH COO   0H 


3

= 40.9 + 349.6
 oCH3COOH = 390.5 S cm /mol
2

Degree of dissociation
m
node06\B0BC-BD\Kota\Board Material\Chemistry\Booklets\CBSE\Part-1


 0m

= 39.05/390.5
= 0.1
11. Decreases with increase in concentration for both strong & weak electrolyte  0m can be obtained
for weak electrolyte by applying Kohlrausch law.
134 E
CBSE
ALLEN
SECTION-C
+ –3 2+ –1
12. Ag (s) | Ag (10 M) || Cu (10 M) | Cu(s)
E°cell, = +0.46 V
Cell reaction is Cu2+ + 2Ag(s)  Cu + 2Ag
0.059 [Ag  ]2
Ecell = E°cell – log
2 [Cu 2 ]
0.059
Ecell = 0.46 – log105
2
= 0.46 + .147
= .6075 V

0.059 [Cr 3 ]2
13. (a) E cell  E 0cell log [1/2]
n [Fe 2  ]3

0.059 [0.01]2
0.261V  E 0cell log [1/2]
6 [0.01]3
0.059
0.261V  E 0cell log100 [1/2]
6
E 0cell  0.261  0.0197

= 0.2807 V [1/2]
(b) A because low value of SRP [1]
SECTION-D
14. (a) Primary cells are those cells which are not rechargeable i.e. products cannot change
back into the reactants.
OR
Secondary cells are those cells which can be recharged again once used, by passing
direct current through them.
(b) Mercury cell : It consists of zinc mercury amalgam as anode, a paste of HgO and
carbon black is used as cathode. The electrolyte is a paste of KOH and ZnO.
(c) (i) At anode :
node06\B0BC-BD\Kota\Board Material\Chemistry\Booklets\CBSE\Part-1

Zn (amalgam) + 2OH–  ZnO(s) + H2O (l) + 2e–


(ii) At cathode :
HgO(s) + H2O + 2e–  Hg(l) + 2OH–
The net reaction :
Zn(amalgam) + HgO(s)  ZnO(s) + Hg(l)

E 135
Chemistry
ALLEN
SECTION-E
15. (a) 2AgCl (s) + H2(g) (1 atm)  2Ag(s) + 2H+ (0.1 M) + 2Cl¯(0.1M)

G   nF E cell

G (43600)
E cell    0.23V
nF 2  96500
0.0591
 0.23  log (0.1) 2 (0.1) 2
2
0.0591
 0.23  log (0.0001)
2
0.0591
 0.23  log (10) 4
2
= 0.23 + 0.1182 = 0.3482 V

E cell  0.35 V

(b) The cell reaction for the cell:

Cr2 O72 (aq)  14 H  (aq)  6 e   2 Cr 3 (aq)  7H 2 O (l )

0.059 [Cr 3 (aq)]2


E  Eº Cell  log 
n [H (aq)]14 [Cr2 O72 ]

0.059 (0.2) 2
E Cell  1.33  log
1.0 104  (0.1)
14
6

ECell = 1.33 – 0.556 = 0.774

node06\B0BC-BD\Kota\Board Material\Chemistry\Booklets\CBSE\Part-1

136 E
CBSE
ALLEN
UNIT-4 : CHEMICAL KINETICS
UNIT INDEX
• Theory 137-151
• Exercise-1_Intext Questions 152-154
• Exercise-2_NCERT Exercise Questions 155-170
• Exercise-3_Exemplar 171-177
• Exercise-4_MCQ, A/R, Case Based Questions 178-187
• Previous Years Questions 188-199
• Practice Test 200-205
THEORY
INTRODUCTION :
CHEMICAL KINETICS : Chemical kinetics is the branch of physical chemistry which deals
with the study of rates of reactions, factor affecting the rates of reaction and their mechanisms,
called as CHEMICAL KINETICS.
Rate of Reaction :
The change in concentration of reactants or products in a unit time is known as speed or
rate of reaction.

uct
Prod
Concentration

Rea
cta nt

Time

Change in concentration of reactant or product


Rate of reaction =
Time taken for this change
Explanation of rate of reaction :
Consider a hypothetical reaction in which one mole of the reactant R produces one mole of
product P.
R (reactant)  P(Product)
node06\B0BC-BD\Kota\Board Material\Chemistry\Booklets\CBSE\Part-1

If [R1]and [P1] are the concentration of R and P at time t1 and [R2] and [P2] are their
concentration at time t2 respectively.
  t = t2 –t1
[R] = [R2] – [R1] [ ] = moler concentration
 [P] = [P2] – [P1]

E 137
Chemistry
ALLEN
(i) Rate of reaction or Rate of disappearance of

Δ Δ
Decrease in concentration of R [R]
R= = –
Time taken t
(ii) Rate of reaction or Rate of appearance of
Increase in concentration of product P
P=
Time taken
[P]
=+
t
for the change in concentration of reactants '–ve' sign is used and for the change in concentration
of products '+ve' sign is used.
Unit of rate of reaction :
Changein concentration of reactant or product
Rate =
Time taken
concentration mol L–1
Rate =  mol L–1s–1 (only for solution phase)
time sec.
Rate = atm sec–1 (only for gas phase)
TYPES OF REACTION
(i) Average rate of reaction (ravg)
(ii) Instantaneous rate of reaction (rinst.)
Average rate (ravg.) : The rate of reaction measured over a long time interval is called average
rate of reaction.

A
C1
Concentration

ravg.of reaction with respect to reactant


of reactant

C]
C2
Reactant

t

O t1 t2 time
   
[C] [C  C1 ]
or ravg. = –  2
t  t 2  t1 
Instantaneous Rate : When the rate of reaction is determined at particular time
(when limit t  0) then it is called instantaneous rate.
dx
It is equal to = 
node06\B0BC-BD\Kota\Board Material\Chemistry\Booklets\CBSE\Part-1

dt
C dC
Rate of reaction (Rinst.) = limit  
t  0 t dt d[c]
rinst = – = –slope
Instantaneous rate can be determined by drawing a tangent at C B dt
time t on a curve (fig.) between concentration and time.
The slope of this tangent gives the actual
rate this is known as instantaneous Rate. t time

138 E
CBSE
ALLEN
The graph of ravg. & rinst. w.r.t. reactants & products.
d[P]
rinst= dt = slope
[R]0
(P)

–[R] [R22]–[R
{[R ]–[R11]]} d[P]
[R1] ravg.= t = – (t2–t1)
Concentration of reactants

[P2] dt

Concen tration of product


[R]
d[R] [P]
[R2] –d[R]
rinst = dt = – slope [P1] [P] [P2]–[P1]
t t rav = t = (t –t )
2 1

dt
t1 t2 t t1 t2 t
Time Time

Instantaneous and average rate of a reaction


Note-1 : The instantaneous rate of reaction can be expressed in stoichiometry terms.
e.g For a general reaction A  B
Rate of reaction = rate of disappear of A = Rate of formation of B
[A] d[A]
Rate = – 
t dt
Δ[B] d[B]
=+ =+
Δt dt

d[A] d[B]
Rate = – =+
dt dt

Note-2 : For expressing the rate of such a reaction when stoichiometry coefficient of
reactants or products are not equal to one ; then rate of disappearance of any of the
reactants and rate of appearance of products is devided by their respective stoichiometric
coefficient.
eg.-1 2HI  H 2  I 2

Rate = Rate of disappearance of HI = Rate of formation of H2 = Rate of formation of I2


1  d[HI]  d[H 2 ] d[I ]
or Rate =    2
2  dt  dt dt
node06\B0BC-BD\Kota\Board Material\Chemistry\Booklets\CBSE\Part-1

 Rate of consumption of HI is twice the rate of formation of H2 or I2 to make them equal, the term
[HI] is divided by 2 then rate of reaction is given as above.
eg.-2 N2 + 3H2  2NH3
d[N 2 ] 1 d[H 2 ] 1 d[NH 3 ]
Rate =   
dt 3 dt 2 dt

E 139
Chemistry
ALLEN
RATE LAW AND RATE CONSTANT OF A REACTION
Rate law (law of mass action) :
According to rate law "The rate of homogeneous chemical reaction is proportional to active
mass of the reacting substances which take part in the reaction.
eg. 1 A + B  C + D
A/c to Rate law Rate  [A] [B]
eg. 2 aA + bB  cC + dD
a,b,c,d are stoichiometric coefficients of reactants and products.

  The rate expression rate  [A]x [B]y ....(i)

x, y can have any numerical value.


x, y may or may not be equal to the stoichiometric coefficient of reactant.
Eq.(1) can be written as
Rate = k[A]x [B]y ....(ii)

d[R]
or   k[A]x [B]y . ...(iii)
dt

Eq. (3) is known as differential rate equation.


where k = Rate constant.
eg.1 2NO(g) + O2 (g)  2NO2(g)
Rate = k [NO]2 [O2]
d[R]
differential form   = k [NO]2 [O2]
dt
Elementary step in reactions : The reaction which is completed in single step.
Complex reaction :The reaction which is completed in the two or more than two steps. A complex
reaction consist of many elementary steps. The rate constant of all the steps may not be similar.
Rate determining step : The slowest elementary step in a complex reaction which determines reaction
rate is called rate determining step (r.d.s.).
Order of reaction :
node06\B0BC-BD\Kota\Board Material\Chemistry\Booklets\CBSE\Part-1

The sum of powers of the molar concentration of the reactants in the rate law expression is called the
order of that chemical reaction.
Order of reaction can be 0,1,2,3 ... and even a fraction.
eg. Rate  [A]x [B]y (experimental)
Rate = k[A]x [B]y

140 E
CBSE
ALLEN
over all order of reaction n = x + y
order of reaction w.r.t A = x
order of reaction w.r.t B = y
Molecularity of a reaction :
The number of reacting species (atom, ions or molecules) taking part in an elementary
reaction which must collide simultaneously in order to bring about a chemical reaction is
called molecularity of a reaction.
OR
The total number of molecules, atoms, ion taking part in the chemical reaction is known as
molecularity of reaction.
If one, two, three molecules take part in the reaction then the reaction are known as unimolecular,
bimolecular, trimolecular respectively.
eg. H2O2  H2O + O2 unimolecular
NH4NO2  N2 + 2H2O unimolecular
H2 + I2  2HI bimolecular
2HI  H2 + I2 bimolecular
2NO + O2  2NO2 trimolecular
Pseudo first order reaction :
Such reaction whose order of reaction is one but molecularity are different then it is called
as pseudo first order reaction
eg. Inversion of cane sugar :

C12H22O11 + H 2 O 
H
 C6 H 12 O6  C6 H 12 O6
excess Glucos e Fructose

Rate  [C12H22O11]
  order of reaction = 1
Molecularity = 1 + 1 = 2
e.g. Hydrolysis of ester :
node06\B0BC-BD\Kota\Board Material\Chemistry\Booklets\CBSE\Part-1

O
+
H
CH3–C–OC2H5 + H2O CH3COOH + C2H5OH
(excess)

Rate  [CH3COOC2H5]
order of reaction = 1
Molecularity = 1 + 1 = 2
E 141
Chemistry
ALLEN
Difference between molecularity and order of reaction :
S.No. Order of reaction Molecularity of reaction
1. It is the number of molecules, atoms or ions It is the number of molecules,
whose concentration change during the atom or ion taking part in the
reaction reaction according to
stoichiometric equation.
2. It is an experimental value. It is an theoretical value.
3. It is not necessary that it should be whole It is always whole number.
number.
4. It may be zero even in fraction. It is never zero.
5. It is used for the establishment of It is not used for the
mechanism of the reaction by R.D.S. determination of mechanism of
the reaction.
Units of rate constant k :
aA + bB  Product
Rate = k[A]x [B]y
x + y = n  order of reaction
Rate concentration 1
k= x y
=  ; ([A] =[B])
[A] [B] time (concentration) n
(concentration)1 n
k=
time
(mole L–1 )1 n
=  mol1–n Ln–1s–1
sec.
k = mol1-n Ln-1s-1

n = order of the reaction


Reaction Order Units of Rate const.
Zero order 0 mol L–1s –1
First order 1 s –1
Second order 2 mol –1 L s –1
node06\B0BC-BD\Kota\Board Material\Chemistry\Booklets\CBSE\Part-1

INTEGRATED RATE EQUATION FOR ZERO ORDER REACTION


Zero order reaction means that the rate of the reaction is proportional to zero power of the
concentration of reactant.
Consider the following reaction
R  P
(Reactant) (Product)

142 E
CBSE
ALLEN
According to Rate law for zero order reaction
d[R]
Rate =  [R]0
dt
d[R]
Rate = – = k[R]° (k = rate const for zero order reaction)
dt
d[R]
or  =k×1
dt
or d[R] = –kdt .....(1)
integrate both sides-

 d[R]  k  dt
[R] = –kt + C .....(2)
(C = constant of integration)
At t = 0, [R] = [R]0
When [R]0 is initial concentration of the reaction
Substituting in equation (2)
[R]0 = – k × 0 + C
 C = [R]0
[R]0
Put the value of C in equation (2)
Slope = –k
[R]= – k t + [R]0 (important for graph) [R]

y –mx c
t
[R]0 – [R]
or k= ...(3)
t

x x
 k=  x  kt
t
Amount of reactant that will change in product.
t
For a zero order reaction, Rate of reaction is euqal to rate
constant.
Rate = k[R]0
node06\B0BC-BD\Kota\Board Material\Chemistry\Booklets\CBSE\Part-1

Rate = k + 0 × [R]0
Rate
Example of zero order reactions :
1130K
(i) 
2NH3(g)  
 N2(g) + 3H2(g)
Pt
Concentration
Au
(ii) 
2HI(g)   H2(g) + I2(g)

h

(iii) H2 + Cl2  
 2HCl
E 143
Chemistry
ALLEN
Half life of a reaction : (t1/2)
The half life of a reaction is the time in which the concentration of a reactant is reduced to
one half of its initial concentration. It is represented as (t1/2).
Half life expression for zero order reactions :
From rate equation of zero order reaction
[R]0  [R]
k ...(1)
t t1/2
[R]0  [R] 1/2 k
or t= ...(2)
k
[R]0 concentration  
for half life time put t = t1/2 , [R] =
2
in Eq. (2)
[R]0  [R]0
t1/2 =
2
[R]0
t 1/ 2 = ...(3)
2K
equation (3) is half life Eq. for zero order reaction.
  It is clear from Eq. (3) that t1/2 for zero order reaction is directly proportional to the rate
constant.
Integrated rate equation for first order reaction :
First order reaction means the rate of the reaction is proportional to the first power of the
concentration of the reactant R.
R  P
(Reactant) (Product)
According to rate law for first order reaction.
d[R]
Rate =   [R]
dt
d[R]
Rate =   k[R]
dt
d[R]
or   k[R] ..... (1) (k = Rate const. for first order reaction)
node06\B0BC-BD\Kota\Board Material\Chemistry\Booklets\CBSE\Part-1

dt
equation (1) is differential rate equation for first order reaction
d[R]
  kdt
[R]
d[R]
 kdt ..... (2)
[R]

144 E
CBSE
ALLEN
Integrate both sides.
d[R]
 [R]  k  dt

n[R] = –kt + C .......(3) (C = const. of integration)

At t = 0, [R] = [R]0, Put in equation (3) ln[R]0


 n[R]0 = – k × O + C –k
ln[R]
C = n [R]0 ..... (4)

Put the value of C in equation (3) t

n[R] = -kt + n[R]0 ..... (5) (Important for graph)

or kt = n[R]0 – n[R]

[R]0
kt = n
[R]

1 [R]0 k/2.303
k = n
t [R]

2.303 [R]0 [R] kt t


k= log10  log 0  ......(6)
t [R] [R] 2.303

Rate = k[R]'
Rate
2.303 a
or k= log10
t (a – x)

equation (6) is integrated rate expression for first order Concentration


expression for first order reaction.
Rate expression of first order reaction at two different time.

 n[R] = -kt + n[R]0 ......(1)

At time t1, from equation (1)


node06\B0BC-BD\Kota\Board Material\Chemistry\Booklets\CBSE\Part-1

n [R1] = –kt1 + n[R]0 ......(2)

At time t2 from equation (2)

n[R2] = –kt2 + n[R]0 ......(3)

where [R]1 and [R]2 are the concentrations of the reactants at time t1 and t2 respectively.

E 145
Chemistry
ALLEN
Subtracting (3) from (2)
n[R1] – n[R2] = (–kt1) – (–kt2)

[R1 ]
n  k(t 2  t1 )
[R 2 ]

1 [R]1
k= n .......(4)
t 2 – t 1 [R]2

Example of first order reactions :


(i) C2 H 4  H 2 
 C 2 H 6 (g) Rate = k[C2H4] 2H 2 O 2  2H 2 O  O 2
(g) (g) H
CH 3 COOC 2 H 5  H 2 O   CH 3 COOH
excess
(ii) All radioactive reactions  C2 H 5 OH
eg. 226
Ra 
 42 He  86
222
Rn Rate = k[Ra] Rate  k[CH 3 COOC2 H 5 ]
88
2N 2 O  2N 2  O 2
(iii) Decomposition of N2O5 
C12 H 22 O11  H 2 O 
H
 C6 H12 O6  C6 H12 O6
 1
N 2 O5   2NO 2  O 2 Rate = k[N2O5] Sucrose excess glucose Fructose

(g) (g) 2 Rate  k[C12 H 22 O11 ]


(g)
Half life expression for first order reaction :
2.303 [R]0
k= log10
t [R]
2.303 [R]0
or t= log10 ..... (1)
k [R]
for half life Put t = t½ and [R] = [R]0/2 in equation (i)
2.303 [R]0
t1/2 = log
k [R]0/2
2.303 2.303
= log10 2 =  0.3010 t1/2
k k
0
.
6 k
9
3
t1
=

..... (2) Concentration


2

Eq. (2) is half life expression of first order reaction.


Rate expression for first order reaction in gas phase :
Let us consider a typical first order gas phase reaction -
node06\B0BC-BD\Kota\Board Material\Chemistry\Booklets\CBSE\Part-1

n[R]0
A 
 B  C
(g) (g) (g) Slope = –k

Let Pi be the initial pressure of A and Pt the total pressure at time n[R]
t.
Integrated rate equation for such a reaction can be derived as O t
follows.

146 E
CBSE
ALLEN
If x atm be the decrease in pressure of A a time t and one mol each of B and C is being Formed,
the increase in pressure of B and C will also be x atm each.

A 
 B  C
(g) (g) (g)
Initial pressure :
at t = 0 Pi atm 0 atm 0 atm
At time t : (Pi – x) x x
If PA, PB, PC are the partial pressure of A, B and C respectively and Pt is total pressure.
 Pt = PA + PB + PC
= (Pi – x) + x + x
Pt = Pi + x
 x = (Pt – Pi)
 PA = Pi – x

 PA = Pi – (Pt – Pi) = Pi – Pt + Pi

PA  2Pi  Pt

2.303 [R]0
 for first order reaction k  log10
t [R]
2.303 P
 k log10 i
t PA

2.303 Pi
 k log10 .........(1)
t 2Pi  Pt

equation (1) is rate expression for 1st order typical reaction in gas phases.
FACTOR AFFECTING REACTION RATE :
(1) Concentration : If concentration of the reactant increases then rate of the reaction increase
because rate is proportional to molar concentration of reactant.
(2) Surface area : Surface of reactant increases then number of mole per unit volume
increases. Hence, the rate of the reaction increase.
eg.-1 Powdered sugar dissolved faster then that of sugar crystal.
node06\B0BC-BD\Kota\Board Material\Chemistry\Booklets\CBSE\Part-1

eg.-2 Pulverised coal burn faster then that of wooden charcoal.


(3) Exposure to radiation : The photochemical reaction start only in the presence of sun light.
If intensity of light increases then rate of the reaction increases.
This kind of reaction come in category of zero order
eg. CH4 + Cl2 
h
 CH3Cl + HCl

E 147
Chemistry
ALLEN
(4) Temperature : On increasing temperature the rate of reaction increases due to increase in
kinetic energy thereby increase in effective number of collisions.
(5) Catalyst : It increases the rate of reaction by decreasing the activation energy and
providing the alternative path.
TEMPERATURE DEPENDENCE OF THE RATE OF REACTION :
For a chemical reaction with rise in temperature by 10°, the rate constant is nearly is doubled.
Eg. : In decomposition of N2O5, the time taken for half of the original amount to decompose is
12 minutes at 50°C, 5 hrs at 25°C and 10 days at 0°C.
The effect of temperature is usually expressed in terms of temperature coefficient.
k (t 10)
Temperature coefficient = = 2 or 3
kt
where, kt = rate constant at t°C ; kt+10 = rate constant at (t+10)°C
Ratio of rate constant with temperature difference of 10°C is known as temperature coefficient.
ARRHENIUS EQUATION :
The temperature dependence of rate of a chemical reaction is expressed by Arrhenius equation,
k  Ae  Ea /RT
where, k = rate constant
A is the Arrenius factor / frequency factor / pre-exponential factor
(It is a constant specific to a particular reaction)
R = Gas constant ; Ea = Activation energy
So Arrhenius equation is a quantitative relationship between rate constant and temperature.
ARRHENIUS THEORY :
H H
H2 + I2   2HI
I I
reactant activated complex Product
(unstable Intermediate)
Note :
1. Activation energy :
It is extra amount of energy given to the reactant to cross the threshold energy barrier.
Ea = ET – EP
For eg. Reactant A – B and C react and form product A + BC which can be represented as
node06\B0BC-BD\Kota\Board Material\Chemistry\Booklets\CBSE\Part-1

A – B + C  [A ....... B ....... C] A + B – C


reactant molecules Activated complex Product molecules
(most unstable)
2. Threshold energy :
It is the minimum energy required by the reactant to convert into product.
Threshold energy = Activation energy + Average potential energy

148 E
 CBSSE
ALLEN
A
3.
3 Effecttive collision
ns :
Those collisions which
w lead to
t the formaation of pro
oduct molecu
ules.

Maxw well Boltzmaann Distrib bution Currve :


At a particular tem
mperature T,
T all the moolecules of reactants do
d not have same kinetiic energy buut
the fraaction of molecules
m haaving particcular kineticc energies at
a a particulular temperaature remainns
constaant.
The ddistribution of kinetic energy maay be desccribed by plotting
p thee fraction ofo moleculees
(NE / NT) with a given
g kineticc energy (E)). This is caalled Maxwell's distribuution of eneergies.

NE  Number off molecules with energyy E


NT  Total numbber of moleccules.
The peak of the curve reprresents the kinetic eneergy possesssed by thee maximum m fraction ofo
molecuules, that is known as most
m probabble kinetic energy.
e
On raiising temperrature to t + 10, the maaximum of the curve sh
hifts to highher energy value
v and thhe
curve spreads to right such that there is a greateer proportioon of moleccules with much higher
energies.
node06\B0BC-BD\Kota\Board Material\Chemistry\Booklets\CBSE\Part-1

Rise iin temperatture of the substance increases the fraction n of moleccules that collides
c with
energies more thaan Ea. It is clear
c from tthe diagram
m that in the curve at (t + 10), the area
a showinng
the fraaction of moolecules having energyy equal to or o greater th
han activatiion energy gets doubleed
and heence, doublee the rate off reaction.
Accordding to Arrhhenius equaation on incrrease in tem mperature orr decrease inn activation
n energy willl
result iin an increaase in the rate of reactioon and an ex
xponential energy
e in thhe rate constant.

k = Ae –Ea / RT
R

E 14
49
Chemistry
ALLEN
Take log both sides
n K = n A + ln e–Ea/RT
Ea
n K = n A – lne (logee = 1)
RT
Ea
n k = n A –
RT
Ea
n k = – + lnA ..... (2)
RT
equation (2) is equation of y = mx + c

E
slope = – a E a = –(slope) × RT
RT
lnA

n k slope = –Ea/R

1/T  
(ii) Determination of Ea by mathematical method
E
  n k = – a + nA ..... (1)
RT
If k1 and k2 are the values of rate constant at temp T1 and T2 respectively
Since A is constant for a given reaction
E
n k1 = – a + nA ..... (2)
RT1
Ea
n k2 = – + nA ..... (3)
RT2
Subtracting equation (2) from (3)
Ea Ea
n k2 – n k1 = –
RT1 RT2
 k  Ea  1 1 
n  2  =   
k
 1 R  T1 T2 
 k  Ea  T2  T1 
n  2  =  
 k1  R  T1T2 
node06\B0BC-BD\Kota\Board Material\Chemistry\Booklets\CBSE\Part-1

k  Ea  T2  T1 
or log10  2     .... (4)
 k1  2.303R  T1T2 
Effect of catalyst :
A catalyst is a substance which increases the rate of a reaction without itself undergoing any
permanent chemical change.
Catalyst provides an alternate path or reaction mechanism by reducing the activation energy
between reactants and products hence lowering the potential energy barrier.
150 E
 CBSSE
ALLEN
A
Lowerr the value of activatio
on energy, faster will be the ratee of a reacti
tion.
Reactiion path
with catalyst
c reaction patth
without cataalys

Potential energy
Energgy of Energgy of activation
activaation withoout catalyst
with catalyst

Reacttant
Productts

R
Reaction coorddinate
Characteris
C stics of a Caatalyst :
• A smaall amount of o the catalyyst can catallyse a large amount of reactants.
r
• A cataalyst does noot alter Gibb bs energy, G of a reacction.
• Catalyyst does nott change the equilibriuum constantt of a reaction rather, it helps in attaining thhe
equilibbrium fasterr, i.e., it caatalyses the forward ass well as th he backwarrd reactions to the sam me
extent so that the equilibrium m state remaains same bu ut is reached
d earlier.
• It catallyses the sppontaneous reactions
r buut does not catalyse
c nonn-spontaneoous reaction
ns.
COLLISIO
C ON THEOR RY OF CHE EMICAL R REACTION NS :
Collisiion theory is based on kinetic
k theoory of gases.
Accordding to thiss theory, thee reactant m molecules are assumed d to be hardd spheres annd reaction is
postulaated to occuur when mo olecules colllide with eaach other. The
T number of collision ns per seconnd
per unnit volume ofo the reactio on mixture is known ass collision frequency
f (Z
Z).
For a bbimolecularr elementary y reaction
A + B  Productts
Rate = ZAB e Ea /RTT
where ZAB  Colllision frequ
uency of reaactants, A and B
e  Ea /RT Frraction of molecules
m w
with energiess equal to or greater thaan Ea.
Collisiion theory depends on n that theree is a collission betweeen the reacctant moleccules. Due tot
collisioon the old bonds
b breakk up and wiith the resu ult, new arraangement taakes place, forming neew
bond aand give prooducts.
The coollisions in which
w moleecules collidde with suffficient kineetic energy (threshold d energy) annd
properr orientatioon, so as to o facilitate bbreaking of bonds betw ween reactinng species and
a formatioon
of neww bonds to formf ve collision
produccts are calledd as effectiv ns.
Formation
F oof methanool from bro omoethane depends up pon the oriientation off reactant molecules
m :
node06\B0BC-BD\Kota\Board Material\Chemistry\Booklets\CBSE\Part-1

E 15
51
Chemistry
ALLEN
EXERCISE-1 INTEXT QUESTIONS
1. For the reaction R   P, the concentration of a reactant changes from 0.03 M to 0.02 M in
25 minutes. Calculate the average rate of reaction using units of time both in minutes and seconds.
[R]
Ans. Average rate of reaction = 
t

=
[R 2 ]  [R1 ] =
0.02  0.03 M min 1
t 2  t1 25

=
0.02  0.03 M min 1
25
= 4 × 10 M Min–1
–4

4  104
= Ms 1
60
= 6.67 × 10–6 M s–1
2. In a reaction, 2A   Products, the concentration of A decreases from 0.5 mol L–1 to
0.4 mol L–1 in 10 minutes. Calculate the rate during this interval?
1 [A] 1 [A]2  [A]1
Ans. Average rete =  
2 t 2 t 2  t1

1 [0.4  0.5] 1 0.1


= 
2 10 2 10
= 0.005 mol L–1 min–1 = 5 × 10–3 M min–1
3.  Product; the rate law is given by, r = k[A]1/2[B]2. What is the order
For a reaction, A + B 
of the reaction?
1 1
Ans. The order of the reaction,  2  2 = 2.5.
2 2
4. The conversion of molecules X to Y follows second order kinetics. If concentration of X is
increased to three times how will it affect the rate of formation of Y?
Ans. The order of reaction is defined as the sum of the powers of concentrations in the rate law.
The rate of second order reaction can be expressed as rate = k[A]2
The reaction X 
 Y follows second order kinetics.
Therefore, the rate equation for this reaction will be:
Rate = k[X]2 .....(1)
node06\B0BC-BD\Kota\Board Material\Chemistry\Booklets\CBSE\Part-1

–1
Let [X] = a mol L , then equation (1) can be written as :
Rate = k [a]2 = ka2
If the concentration of X is increased to three times, then [X] = 3a mol L–1
Now, the rate equation will be :
Rate = k [3a]2 = 9(ka2)
Hence, the rate of formation will increase by 9 times.
152 E
CBSE
ALLEN
5. A first order reaction has a rate constant 1.15 × 10–3 s–1. How long will 5g of this reactant take to
reduce to 3g ?
Ans. We know that for a 1st order reaction,
2.303 [R]0 2.303 5
t log  3
log
k [R] 1.15 10 3
2.303
=  0.2219  444.38s or 444 s (approx)
1.15 103
6. Time required to decompose SO2Cl2 to half of its initial amount is 60 minutes. If the
decomposition is a first order reaction, calculate the rate constant of the reaction.
0.693
Ans. We know that for a 1st order reaction, t1/2  .
k
It is given that t1/2 = 60 min
0.693 0.693
 k = = 0.01155 min–1 = 1.155 × 10–2 min–1
t1/ 2 60

1.155 102
= sec1  0.01925  102 s 1
60
= 1.925 × 10–4 s–1
7. What will be the effect of temperature on rate constant?
Ans. The rate constant of a reaction is nearly doubled with a 10°C rise in temperature. However, the
exact dependence of the rate of a chemical reaction on temperature is given by Arrhenius
equation, k = Ae–Ea/RT
Where, A is the Arrhenius factor or the frequency factor
T is the temperature
R is the gas constant
Ea is the activation energy
8. The rate of the chemical reaction doubles for an increase of 10 K in an absolute temperature from
298 K. Calculate Ea.
Ans. It is given that
T1 = 298 K
T2 = (298 + 10) K = 308 K
We also know that the rate of the reaction doubles when temperature is increased by 10°.
node06\B0BC-BD\Kota\Board Material\Chemistry\Booklets\CBSE\Part-1

Therefore, let us take the value of k1= k and that k2 = 2k


Also, R = 8.314 J K–1 mol–1
Now, substituting these values in the equation :
k2 E a  T2  T1 
log   
k1 2.303R  T1T2 

E 153
Chemistry
ALLEN
We get :
2k Ea  10  Ea  10 
log     log 2 
k 2.303  8.314  298  308  2.303  8.314  298  308 

2.303  8.314  298  308  log 2


 Ea 
10
= 52897.78 J mol–1 = 52.9 kJ mol–1
9. The activation energy for the reaction 2HI(g)   H2(g) + I2(g) is 209.5 kJ mol–1 at 581 K.
Calculate the fraction of molecules of reactant having energy equal to or greater than activation
energy?
Ans. The fraction of molecules of reactants having energy equal to or greater than activation energy is
given as :
x = e–Ea/RT
 In x = –Ea/RT
E a
 log x 
2.303RT

209500 J mol1
 log x   18.8323
2.303  8.314JK 1mol1  581
Now, x = Anti log (–18.8323)
= 1.471 × 10–19

node06\B0BC-BD\Kota\Board Material\Chemistry\Booklets\CBSE\Part-1

154 E
CBSE
ALLEN
EXERCISE-2 NCERT EXERCISE
1. From the rate expression for the following reactions, determine their order of reaction and the
dimensions of the rate constants :
(i)  N2O(g) ; Rate = k[NO]2
3NO(g) 
(ii)  2H2O(l) + I3– ; Rate = k[H2O2][I–]
H2O2(aq) + 3I– (aq) + 2H+ 
 CH4(g)  CO(g) ; Rate = k[CH3CHO]3/2
(iii) CH3CHO(g) 
 C2H4(g) + HCl(g) ; Rate = k[C2H5Cl]
(iv) C2H5Cl(g) 
Ans. (i) Given rate = k[NO]2
Therefore, order of the reaction = 2
Rate mol L1s 1 mol L1s 1
Dimension of k    = mol–1 L s –1
 NO  mol L 
2 1 2 2 2
mol L

(ii) Give rate = k[H2O][I–]


Therefore, order of the reaction = 2.
Rate molL1s 1
Dimension of k   = L mol–1 s –1
[H 2 O2 ][I ] (molL )  molL 
 1 1

(iii) Given rate = k[CH3CHO]3/2


3
Therefore, order of reaction =
2
Rate mol L1s 1 mol L1s 1 1

1
2 1
Dimension of k = 3
 3
 3 3
= L mol s .
2

 CH3CHO  mol L 
1 2 
2
mol L2 2

(iv) Given rate = k[C2H5Cl]


Therefore, order of the reaction = 1.
Rate molL1s 1
Dimension of k =  = s–1
 2 5 
C H Cl mol L1

2. For the reaction : 2A + B   A2B the rate = k [A][B]2 with k = 2.0 × 10–6 mol–2L2s–1.
Calculate the initial rate of the reaction when [A] = 0.1mol L–1. [B] = 0.2 mol L–1 . Calculate the
rate of reaction after [A] is reduced to 0.06 mol L–1.
Ans. The initial rate of the reaction is
Rate = k[A][B]2 = (2.0 × 10–6 mol–2 L2 s–1) (0.1 mol L–1) (0.2 mol L–1)2 = 8.0 × 10–9 mol L–1 s–1
when [A] is reduced from 0.1 mol L–1 to 0.06 mol–1, the concentration of A reacted
node06\B0BC-BD\Kota\Board Material\Chemistry\Booklets\CBSE\Part-1

= (0.1 – 0.06) mol L–1 = 0.04 mol L–1


1
Therefore, concentration of B reacted =  0.04 mol L1  0.02 mol L1
2
Then, concentration of B available, [B] = (0.2 – 0.02) mol L–1 = 0.18 mol L–1
After [A] is reduced to 0.06 mol L–1, the rate of the reaction is given by.
Rate = k [A][B]2 = (2.0 × 10–6 mol–2 L2 s–1) (0.06 mol L–1) (0.18 mol L–1)2 = 3.88 × 10–9 mol L–1 s–1.

E 155
Chemistry
ALLEN
3. The decomposition of NH3 on platinum surface is zero order reaction. What are the rate of
production of N2 and H2 if k = 2.5 × 10–4 mol litre–1 s–1.
Ans. 2NH3  N2 + 3H2.
1 d[NH 3 ] d[N 2 ] 1 d[H 2 ]
Rate of reaction (r) = –  
2 dt dt 3 dt
Rate (r) = k[NH3]° = k ( zero order reaction)
= 2.5 × 10–4
d[N 2 ]
 = r = 2.5 × 10–4 mol lit–1s–1
dt
d[H 2 ]
= 3r = 3 × 2.5 × 10–4 = 7.5 × 10–4 mol lit–1sec–1
dt
4. The decomposition of dimethyl ether leads to the formation of CH4, H2 and CO and the reaction
rate is given by:
Rate = k[CH3OCH3]3/2
The rate of reaction is followed by increase in pressure in a closed vessel, so the rate can also be
expressed in terms of the partial presure of dimethyl ether i.e.,
Rate = k[PCH3OCH3 ]3/2
If the pressure is measured in bar and time in minutes, then what are the units of rate and rate
constants?
Ans. CH3OCH3 
 CH4 + CO + H2
Rate = k[CH3OCH3]3/2 = k[PCH3OCH3 ]3/2
unit of rate = bar min–1
Rate bar min 1
Unit of K   / 2
= bar–1/2 min–1
[PCH3OCH3 ]3/ 2 bar
5. Mention the factors that affect the rate of a chemical reaction.
Ans. The important factors on which the rate of a chemical reaction depends are
(i) Nature of the reacting species.
(ii) Concentration of the reacting species.
(iii) Temperature at which a reaction proceeds.
(iv) Surface area of the reactants.
(v) Presence of a catalyst.
6. A reaction is of second order with respect to a reactant. How is the rate of reaction affected if the
concentration of the reactant is (i) doubled (ii) reduced to half ?
node06\B0BC-BD\Kota\Board Material\Chemistry\Booklets\CBSE\Part-1

Ans. Given rate (r0 )  K[A]2


Therefore, the rate of reaction would increase by 4 times.
(i) If [A] is doubled : r1 = k[2A]2 r1 = 4r0
Therefore, the rate of reaction would increase by 4 times.
2
A 1
(ii) If [A] is reduced to half : r2  k    r2  r0
2 4
156 E
CBSE
ALLEN
7. What is the effect of temperature on the rate constant of a reaction? How can this effect of
temperature effect on rate constant be represented quantitatively?
Ans. The rate constant is nearly doubled with a rise in temperature by 100C for a chemical reaction.
The temperature effect on the rate constant can be represented quantitatively by Arrhenius
equation, k  Ae Ea / RT
k 2 E a  T2  T1 
or 2.303log   
k1 R  T1T2 
where, K is the rate constant at temperature
A is the Arrhenius parameter,
R is the gas constant,
T is the temperature,
and Ea is the energy of activation which is always positive.
8. In a pseudo first order hydrolysis of ester in water, the following results were obtained :
t/s  0  30  60  90
[Ester] / mol L–1  0.55  0.31  0.17  0.085

(i) Calculate the average rate of reaction between the time interval 30 to 60 seconds.
(ii) Calculate the pseudo first order rate constant for the hydrolysis of ester.
Ans. (i) Average rate of reaction between the time interval 30 to 60 seconds
d[Ester] (0.17  0.31) (0.14)
=   = 4.67 × 10–3 mol L–1 s–1
dt 60  30 30
(ii) For a pseudo first order reaction,
2.303 [R ]
k log 0
t [R]
2.303 0.55
For t = 30 s, k1 = log = 1.911 × 10–2 s–1
30 0.31
2.303 0.55
For t = 60 s, k2 = log = 1.957 × 10–2 s–1
60 0.17
2.303 0.55
For t = 90 s, k 3  log = 2.075 × 10–2 s–1
90 0.085
k1  k 2  k 3
Then, average rate constant, k 
3
1.91110   1.957 10    2.075 10 
node06\B0BC-BD\Kota\Board Material\Chemistry\Booklets\CBSE\Part-1

2 2 2

 = 1.98 × 10–2 s–1


3
9. A reaction is first order in A and second order in B :
(i) Write the differential rate equation.
(ii) How is the rate affected on increasing the concentration of B three times ?
(iii) How is the rate affected when the concentration of both A and B are doubled ?

E 157
Chemistry
ALLEN
Ans. (i) Rate = k [A]1[B]2
(ii) r0 = k [A]1[B]2
r1 = k [A]1[3B]2
r1 = 9 × r0
(iii) r0 = k [A1][B]2
r2 = k [2A]1[2B]2
r2 = 8 × r0
10. In a reaction between A and B, the initial rate of reaction (r0) was measured for different initial
concentrations A and B as given below :
A/mol L–1  0.20  0.20  0.40
B/mol L–1  0.30  0.10  0.05
r0/mol L–1 s–1  5.07 × 10–5  5.07 × 10–5  1.43 × 10–4
What is the order of the reaction with respect to A and B?
Ans. Let the order of the reaction with respect to A be x and with respect to B be y.
Therefore, r0 = k [A]x [B]y
5.07 × 10–5 = k [0.20]x [0.30]y ... (i)
5.07 × 10–5 = k [0.20]x [0.10]y ... (ii)
1.43 × 10–4 = k [0.40]x [0.05]y ... (iii)
Dividing equation (i) by (ii), we obtain
5.07  105 k[0.20]x [0.30]y

5.07  105 k[0.20]x [0.10]y
0 y
[0.30]y  0.30   0.30 
 l y
      y = 0
[0.10]   0.10   0.10  

Dividing equation (iii) by (i), we obtain


1.43  104 k[0.40]x [0.05]y

5.07  105 k[0.20]x [0.30]y

1.43  104 [0.40]x  Since y  0 


  [0.05]y  [0.30]y  1
5.07  105 [0.20]x  
node06\B0BC-BD\Kota\Board Material\Chemistry\Booklets\CBSE\Part-1

 2.821 = 2x
 log 2.821 = x log 2 (Taking log on both sides)
log 2.821
 x  1.496  1.5
log 2
Hence, the order of the reaction with respect to A is 1.5 and with respect to B is zero.

158 E
CBSE
ALLEN
11. The following results have been obtained during the kinetic studies of the reaction :
2A + B  C + D
Experiment  A/mol L–1  B/mol L–1  Initial rate of formation
of D/mol L–1 min–1
I  0.1  0.1  6.0 × 10–3
II  0.3  0.2  7.2 × 10–2
III  0.3  0.4  2.88 × 10–1
IV  0.4  0.1  2.40 × 10–2
Determine the rate law and the rate constant for the reaction.
Ans. Let the order of the reaction with respect to A be x and with respect to B be y.
Therefore, rate of the reaction is given by,
Rate = k[A]x [B]y
According to the question,
6.0 × 10–3 = k[0.1]x [0.1]y ...(i)
7.2 × 10–2 = k[0.3]x [0.2]y ...(ii)
–1 x y
2.88 × 10 = k[0.3] [0.4] ...(iii)
2.40 × 10–2 = k[0.4]x [0.1]y ...(iv)
Dividing equation (iv) by (i), we obtain
2.4 102 k[0.4]x [0.1]y

6.0  103 k[0.1]x [0.1]y
x
[0.4]x  0.4 
   4  4  x  1
1
 4 x
4 x

[0.1]  0.1 
Dividing equation (iii) by (ii), we obtain
2.88 101 k[0.3]x [0.4]y

7.2  102 k[0.3]x [0.2]y
y
 0.4 
 4  42 2 2 y2
y 2 y

 0.2 
Therefore, the rate law is
Rate
Rate = k[A] [B]  k  2
[A][B]2

From experiment I, we obtain


node06\B0BC-BD\Kota\Board Material\Chemistry\Booklets\CBSE\Part-1

6.0  103 mol L1 min 1


k  6.0 L2 mol2 min 1
 0.1mol L  0.1mol L 
1 1 2

From experiment II, we obtain


7.2 102 mol L1 min 1
k = 6.0 L2 mol–2 min–1
 0.3mol L  0.2 mol L 
1 1 2

E 159
Chemistry
ALLEN
From exp. III, we obtain,
2.88 101 m ol L1 min 1
k = 6.0 L2 mol–2 min–1.
 0.3mol L  0.4mol L 
1 1 2

From exp. IV, we obtain,


2.40 102 mol L1 min 1
k = 6.0 L2 mol–2 min–1.
(0.4 mol L1 ) (0.1mol L1 ) 2
Therefore, rate constant,
k = 6.0 L2 mol–2 min–1.
12. The reaction between A and B is first order with respect to A and zero order with respect to B.
Fill in the blanks in the following table :
Experiment  [A]/mol L–1  [B]/ mol L–1  Initial rate/
mol L–1 min–1
I  0.1  0.1  2.0 × 10–2
II  ......  0.2  4.0 × 10–2
III  0.4  0.4  ......
IV  ......  0.2  2.0 × 10–2
Ans. The given reaction is of the first order with respect to A and zero order with respect to B.
Therefore, the rate of the reaction is given by,
Rate = k [A]1 [B]0
Rate = k [A]
From experiment I, we obtain
2.0 × 10–2 mol L–1 min–1 = k (0.1 mol L–1)
k = 0.2 min–1
From experiment II, we obtain
4.0 × 10–2 mol L–1 min–1 = 0.2 min–1 [A]
[A] = [0.2] mol L–1
From experiment III, we obtain
Rate = 0.2 min–1 × 0.4 mol L–1 = 0.08 mol L–1 min–1
From experiment IV, we obtain
2.0 × 10–2 mol L–1 min–1 = 0.2 min–1 [A]
[A] = 0.1 mol L–1
13. Calculate the half-life of a first order reaction from their rate constants given below :-
(i) 200 s-1 (ii) 2 min-1 (iii) 4 year-1
node06\B0BC-BD\Kota\Board Material\Chemistry\Booklets\CBSE\Part-1

0.693 0.693
Ans. (a) t1/2 = = 1
= 3.4 × 10–3 sec (approx.)
k 200s
0.693 0.693
(b) t1/2 = = = 0.35 min (approx.)
k 2 min 1
0.693 0.693
(c) t1/2 = = 1
= 1.73 × 10–1 year (approx.)
k 4 year

160 E
CBSE
ALLEN
14
14. The half-life for radioactive decay of C is 5730 years. An archaeological artifact containing
14
wood had only 80% of the C found in a living tree. Estimate the age of the sample.
0.693 0.693
Ans. Here, k   years 1
t2 5730

2.303 a
  t = log
k (a  x)

t1/2 of 14C = 5730 yr ;


 Also, a = 100, (a - x) = 80
2.303  5730 100
t= log = 19042.12 × 0.097 = 1847 years
0.693 80
15. The experimental data for decomposition of N2O5

[2N2O5 
 4NO2+O2] in gas phase at 318 K are given below :

t / (s)  0  400  800  1200  1600  2000  2400  2800  3200

102 × [N2O5] / mol L–1  1.63  1.36  1.14  0.93  0.78  0.64  0.53  0.43  0.35

(i) Plot [N2O5] against t.


(ii) Find the half - life period for the reaction.
(iii) Draw a graph between log [N2O5] and t.
(iv) What is the rate law?
(v) Calculate the rate constant.
(vi) Calculate the half - life period from k and compare it with (ii).

1.8
1.6

1.4
Ans. (i)
10 –[N2O5](mol L )
-1

1.2

1.0

0.8
2

0.6
0.4
node06\B0BC-BD\Kota\Board Material\Chemistry\Booklets\CBSE\Part-1

0.2
0
0 400 800 1200 1600 2000 2400 2800 3200 3600
t(s)

1.630 102
(ii) Time corresponding to the concentration, molL1  81.5 mol L1 is the half life. From
2
the graph the half life is obtained as 1450 s.

E 161
Chemistry
ALLEN
(iii) t(s) 102 × [N2O5]/mol L–1 log[N2O5]
0 1.63 –1.79 -1.6

400 1.36 –1.87 -1.7

-1.8
800 1.14 –1.94

log[N2O5]
-1.9

1200 0.93 –2.03 -2.0

-2.1
1600 0.78 –2.11 -2.2

2000 0.64 –2.19 -2.3


-2.4
2400 0.53 –2.28 -2.5
0 400 800 1200 1600 2000 2400 2800 3200 3600
2800 0.43 –2.37 t(s)

3200 0.35 –2.46


(iv) The given reaction of the first order as the plot, log [N2O5] v/s t, is a straight line.
Therefore, the rate law of the reaction is
Rate = k [N2O5]
(v) From the plot, log [N2O5] v/s t, we obtain
2.46  (1.79) 0.67
Slope = 
3200  0 3200
k
Again ,slope of the line of the plot log [N2O5] v/s t is given by 
2.303
Therefore, we obtain,
k 0.67
 
2.303 3200
 k = 4.82 × 10–4 s–1
(vi) Half-life is given by ,
0.639 0.693
t1/2   s = 1437.75 sec  1438 sec
k 4.82 104
This value, 1438 s, is very close to the value that was obtained from the graph.
16. The rate constant for a first order reaction is 60 s–1. How much time will it take to reduce the
initial concentration of the reactant to its 1/16th value ?
Ans. It is known that,
2.303 a
node06\B0BC-BD\Kota\Board Material\Chemistry\Booklets\CBSE\Part-1

t log
k (a  x)
1
If a = 1 then (a – x) 
16
2.303 1
t 1
log = 0.0462 s = 4.62 × 10–2 sec (approx)
60s 1/16

162 E
CBSE
ALLEN
17. During nuclear explosion, one of the products is 90Sr with half-life of 28.1 years. If l g of 90Sr
was absorbed in the bones of a newly born baby instead of calcium, how much of it will remain
after 10 years and 60 years if it is not lost metabolically.
Ans. Here,
0.693 0.693 1
k  y
t1/2 28.1
It is known that,
2.303 [R]0 2.303 1 2.303
t log  10 
0.693
log  10 
0.693
  log[R]
k [R] [R]
28.1 28.1
10  0.693
 log[R]    [R]  anti log (0.107)  0.7816 g
2.303  28.1
 anti log(1.8929)  0.7814 g
Therefore, 0.7816 g of 90Sr will remain after 10 years.
Again,
2.303 [R]0 2.303 1 60  0.693
t log  60  log  log[R]  
k [R] 0.693 [R] 2.303  28.1
28.1
 [R] = antilog (–0.6425) = 0.2278 g
Therefore, 0.2278 g of 90Sr will remain after 60 years.
18. For a first order reaction, show that time required for 99% completion is twice the time required
for the completion of 90% of reaction.
Ans. For a first order reaction, the time required for 99% completion is
2.303 100 2.303 2.303
t1  log  log100  2 
k 100  99 k k
For a first order reaction, the time required for 90% completion is
2.303 100 2.303 2.303
t2  log  log10 
k 100  90 k k
Therefore, t1 = 2t2
Hence, the time required for 99% completion of a first order reaction is twice the time required
for the completion of 90% of the reaction.
19. A first order reaction takes 40 min for 30% decomposition. Calculate t1/2.
Ans. For a first order reaction,
2.303 [R]0
t log
node06\B0BC-BD\Kota\Board Material\Chemistry\Booklets\CBSE\Part-1

k [R]
2.303 100 2.303 10
k log  log  8.918 103 min 1
40 min 100  30 40 min 7
Therefore, t1/2 of the decomposition reaction is
0.693 0.693
t1/2   min  77.7 min (approx.)
k 8.918 103

E 163
Chemistry
ALLEN
20. For the decomposition of azoisopropane to hexane and nitrogen at 543 K, the following data are
obtained.
t(sec)  P(mm of Hg)
0  35.0
360  54.0
720  63.0
Calculate the rate constant.
Ans. The decomposition of azoisopropane to hexane and nitrogen at 543 K is represented by the
following equation.
(CH3)2 CHN = NCH (CH3)2(g)  N2(g) + C6H14(g)
At t = 0 Pi 0 0
At t = t PA=Pi– x PB = x PC = x
After time, t, total pressure, Pt = (Pi – x) + x + x
 Pt = Pi + x  x = Pt – Pi
Therefore, PA = Pi – x = Pi – (Pt – Pi) = 2Pi – Pt
For a first order reaction,
2.303 P 2.303 Pi
k log i  log
t Pi  p t 2Pi  p t
2.303 35.0
When t = 360 s, k  log  2.175 103 s 1
360s (2  35.0  54.0)
2.303 35.0
When t = 720 s, k  log  2.235 103 s 1
720s 2  35.0  63.0
Hence, the average value of rate constant is

k
 2.175 10    2.235 10  s
3 3
1
 2.205 103 s 1
2
21. The following data were obtained during the first order thermal decomposition of SO2Cl2 at a
constant volume.
SO2Cl2(g) 
 SO2(g) + Cl2(g)
Experiment  Times/s–1  Total pressure/atm
node06\B0BC-BD\Kota\Board Material\Chemistry\Booklets\CBSE\Part-1

1  0  0.5
2  100  0.6
Calculate the rate of the reaction when total pressure is 0.65 atm.
Ans. The thermal decomposition of SO2Cl2 at a constant volume is represented by the following
equation.

164 E
CBSE
ALLEN
SO2Cl2(g)  SO2(g) + Cl2(g)
At t = 0 Pi 0 0
At t = t PA=Pi– x PB = x PC = x
After time, t, total pressure, Pt = (Pi – x) + x + x
 Pt = Pi + x  x = Pt – Pi
Therefore, PA = Pi – x = Pi – (Pt – Pi) = 2Pi – Pt
For a first order reaction,
2.303 P 2.303 Pi
k log i  log
t Pi  p t 2Pi  p t
2.303 0.5
When t = 100 s, k  log  2.231103 s 1
100s 2  0.5  0.6
When Pt = 0.65 atm, Pi + x = 0.65
x = 0.65 – Pi = 0.65 – 0.5 = 0.15 atm
Therefore, when the total pressure is 0.65 atm, Pressure of SOCl2 is
pSO2Cl2  Pi  x = 0.5 – 0.15 = 0.35 atm
Therefore, the rate of equation, when total pressure is 0.65 atm, is given by,
 
Rate = k pSOCl2 = (2.23 × 10–3 s–1) (0.35 atm) = 7.8 × 10–4 atm s–1
22. The rate constant for the decomposition of N2O5 at various temperature is given below :
T/°C  0  20  40  60  80
105 × k/s–1  0.0787  1.70  25.7  178  2140
Draw a graph between In k and l/T and calculate the values of A and Ea.
Predict the rate constant at 30° and 50°C.
Ans. For the given data, we obtain
T/°C  0  20  40  60  80
T/K  273  293  313  333  353
1
/ K –1   3.66 × 10–3  3.41 × 10–3  3.19 × 10–3  3.0 × 10–3  2.83 × 10–3
T
105 × k/s–1 0.0787  1.70  25.7  178  2140
In k  –7.147  –4.075  –1.359  –0.577  3.063
4
3
node06\B0BC-BD\Kota\Board Material\Chemistry\Booklets\CBSE\Part-1

2
1
0
–1
–2
In k

–3
–4
–5
–6
–7
–8
2.7 2.8 2.9 3.0 3.1 3.2 3.3 3.4 3.5 3.6 3.7 3.8

10 × 1 (K )
3 –1

E 165
Chemistry
ALLEN
Slope of the line,
y 2  y1
 12.301K
x 2  x1
Ea
According to Arrhenius equations, Slope  
R
 Ea = –Slope × R = –(–12.301 K) × (8.314 JK–1 mol–1) = 102.27 kJ mol–1
Ea
Again, In k = In A 
RT
Ea
In A = In k 
RT
When T = 273 K,
In k = –7.147
102.27 103
Then, In A  7.147   37.911
8.314  273
Therefore, A = 2.91 × 1016
When T = 30 + 273 K = 303 K,
l
= 0.0033 K = 3.3 × 10–3 K
T
l
Then, at =3.3 × 10–3K,
T
In k = –2.8
Therefore, k = 6.08 × 10–2 s–1
Again, when T = 50 + 273 K = 323 K,
l
= 0.0031 K = 3.1 × 10–3 K
T
l
Then, at = 3.1 × 10–3 K,
T
102.27
In k = 37.911   3.1103
8.314
= 37.911 – 0.0381 = 37.8729
node06\B0BC-BD\Kota\Board Material\Chemistry\Booklets\CBSE\Part-1

Therefore, k = 0.38s–1
23. The rate constant for decomposition of hydrocarbons is 2.418 × 10–5 s–1 at 546 K. If the energy of
activation is 179.9 kJ/mol, what will be the value of pre-exponential factor.
Ans. According to the Arrhenius equation,
Ea
k  Ae  Ea /RT  In k  In A 
RT

166 E
CBSE
ALLEN
Ea
 log k  log A 
2.303RT

Ea
 log A  log k 
2.303RT

179.9 103 J mol1


 log  2.418 105 s 1   
2.303  8.314 Jk 1 mol1  546 K

= (0.3855 – 5) + 17.2082 = 12.5917


Therefore, A = antilog (12.5917) = 3.9 × 1012 s–1
24. Consider a certain reaction A  Products with k = 2.0 × 10–2 s–1. Calculate the concentration of
A remaining after 100s if the initial concentration of A is 1.0 mol L–1.
Ans. Since the unit of k is s–1, the given reaction is a first order reaction.
2.303 [A]0
Therefore, k  log
t [A]
2.303 1.0
 2.0 102 s 1  log 
100s [A]

2.303
  2.0 102 s 1    log[A] 
100s

2.0 102 100


  log[A]  
2.303

 2.0  102  100  1


  [A]  anti log    0.135 mol L
 2.303 
Hence, the remaining concentration of A is 0.135 mol L–1.
25. Sucrose decomposes in acid solution into glucose and fructose according to the first order rate
law, with t1/2 = 3.00 hours. What fraction of sample of sucrose remains after 8 hours?
2.303 [R]0
Ans. For a first order reaction, k  log
t [R]
It is given that, t1/2 = 3.00 hours
0.693 0.693 1
Therefore, k   h  0.231h 1
node06\B0BC-BD\Kota\Board Material\Chemistry\Booklets\CBSE\Part-1

t1/2 3

2.303 [R]0
Then, 0.231h 1  log
8h [R]

[R]0 0.231h 1  8h
 log 
[R] 2.303

E 167
Chemistry
ALLEN
[R]0
  anti log  0.8024 
[R]
[R]0
  6.3445
[R]
[R]
  0.1576  0.158
[R]0
Hence, the fraction of sample of sucrose that remains after 8 hours is 0.158.
26. The decomposition of hydrocarbon follows the equation k = (4.5 × 1011 s–1) e–28000 K/T. Calculate
Ea.
Ans. The given equation is k = (4.5 × 1011 s–1) e–28000 K/T ...(i)
Arrhenius equation is given by, k  Ae Ea /RT ...(ii)
From equation (i) and (ii), we obtain
E a 28000 K

RT T
 Ea = R × 28000 K = 8.314 J K–1 mol–1 × 28000 K = 232792 J mol–1 = 232.792 kJ mol–1
27. The rate constant for the first order decomposition of H2O2 is given by the following equation :
log k = 14.34 – 1.25 × 104 K/T
Calculate Ea for this reaction and at what temperature will its half - period be 256 minutes ?
Ans. Arrhenius equation is given by,
E Ea
k  Ae Ea /RT  In k  InA  a  log k  log A  ...(i)
RT 2.303RT
The given equation is, log k = 14.34 – 1.25 × 104 K/T ...(ii)
From equation (i) and (ii), we obtain
Ea 1.25 104 K

2.303RT T
 Ea = 1.25 × 104 K × 2.303 × R = 1.25 × 104 K × 2.303 × 8.314 J K–1 mol–1
    = 239339.3 J mol–1 = 239.34 kJ mol–1
Also, when t1/2 = 256 minutes,
0.693 0.693
k  = 2.707 × 10–3 min–1 = 4.51 × 10–5s–1
t1/2 256
It is also given that, log k = 14.34 – 1.25 × 104 K/T
1.25 104 K
 log  4.5110   14.34 
5

T
node06\B0BC-BD\Kota\Board Material\Chemistry\Booklets\CBSE\Part-1

 1.25 104 K  1.25 104 K


0.654  5  14.34   14.34  4.346 
T T
1.25 104 K
  18.686
T
1.25 104 K
 T  668.95K  669K
18.686
168 E
CBSE
ALLEN
28. The decomposition of A into product has value of k as 4.5 × 103 s–1 at 10°C and energy of
activation 60 kJ mol–1. At what temperature would k be 1.5 × 104 s–1 ?
k2 E a  T2  T1 
Ans. From Arrhenius equation, we obtain, log   
k1 2.303R  T1T2 

1.5  104 6.0  104 J mol1  T2  283 


log  1 1  
4.5 10 3
2.303  8.314 JK mol  283T2 

 T  283 
 0.5229  3133.627  2 
 283T2  
0.5229  283T2
  T2  283
3133.627

0.9528 T2 = 283
T2 = 297.019 K = 297 K = 24° C
Hence, k would be 1.5 × 104 s–1 at 24° C
29. The time required for 10% completion of a first order reaction at 298K is equal to that required
for its 25% completion at 308 K. If the value of A is 4 × 1010 s–1. Calculate k at 318 K and Ea.
2.303 a
Ans. For a first order reaction, t  log
k ax
2.303 100 0.1054
At 298 K, t1  log 
k1 90 k1
2.303 100 0.2877
At 308 K, t 2  log 
k2 75 k2
According to the question, t = t'
0.1054 0.2877 k
   2  2.73
k1 k 2` k1
From Arrhenius equation, we obtain
k2 E a  T2  T1  Ea  308  298 
log     log  2.73   
k1 2.303R  T1T2  2.303  8.314  298  308 

2.303  8.314  298  308  log  0.4361


Ea  = 76640.26 J mol–1 = 76.64 kJ mol–1
308  298
To calculate k at 318 K,
node06\B0BC-BD\Kota\Board Material\Chemistry\Booklets\CBSE\Part-1

It is given that, A = 4 × 1010 s–1, T = 318 K


Again, from Arrhenius equation, we obtain
Ea 76.6 103
log k  log A   log  4 10  
10

2.303R T 2.303  8.314  318


= (0.6021 + 10) – 12.5876 = – 1.9855
Therefore, k = Antilog (– 1.9855) = 1.05 × 10–2 s–1
E 169
Chemistry
ALLEN
30. The rate of a reaction quadruples when the temperature changes from 293 K to 313 K. Calculate
the energy of activation of the reaction assuming that it does not change with temperature.
Ans. From Arrhenius equation, we obtain,
k2 E a  T2  T1 
log   
k1 2.303R  T1T2 
It is given that, k2 = 4k1
T1 = 293 K ; T2 = 313 K
4k1 Ea  313  293 
Therefore, log   
k1 2.303  8.314  293  313 
20  E a
 0.6021 
2.303  8.314  293  313
0.6021 2.303  8.314  293  313
 Ea  = 52863.33 J mol–1 = 52.86 kJ mol–1
20
Hence, the required energy of activation is 52.86 kJ mol–1

node06\B0BC-BD\Kota\Board Material\Chemistry\Booklets\CBSE\Part-1

170 E
CBSE
ALLEN
EXERCISE-3 EXEMPLAR
1. The role of a catalyst is to change ………..
(a) Gibbs energy of reaction (b) enthalpy of reaction
(c) activation of energy (d) equilibrium constant
Ans. (c)
2. Consider a first order gas phase decomposition reaction given below :
A(g)  B(g) +C(g)
The initial pressure of the system before decomposition of A was p1. After lapse of time ‘t’ total
pressure of the system increased by x units and became ‘pt’. The rate constant k for the reaction
is given as …………..
2.303 p 2.303 pi
(a) k  log i (b) k  log
t pi  x t 2pi  p t
2.303 pi 2.303 p
(c) k= log (d) k = log i
t 2pi  p t t pi  x
Ans. (b)
3. According to Arrhenius equation rate constant k is equal to A e  Ea /RT . Which of the following
1
options represents the graph of ln k vs ?
T

ln k ln k
(a) (b)

1/T  1/T 

ln k ln k
(c) (d)

1/T  1/T 
Ans. (a)
4. Consider the Arrhenius equation given below and mark the correct option :
Ea

k = Ae RT
(a) Rate constant increases exponentially with increasing activation energy and decreasing
temperature
node06\B0BC-BD\Kota\Board Material\Chemistry\Booklets\CBSE\Part-1

(b) Rate constant decreases exponentially with increasing activation energy and decreasing
temperature
(c) Rate constant increases exponentially with decreasing activation energy and decreasing
temperature
(d) Rate constant increase exponentially with decreasing activation energy and increasing
temperature
Ans. (d)

E 171
Chemistry
ALLEN
5. Which of the following statements is not correct about order of a reaction?
(a) The order of a reaction can be a fractional number
(b) Order of a reaction is experimentally determined quantity
(c) The order of a reaction is always equal to the sum of the stoichiometric coefficients of
reactants in the balanced chemical equation for a reaction
(d) The order of a reaction is the sum of the powers of molar concentration of the reactants in the
rate law expression
Ans. (c)
6. Which of the following expressions is correct for the rate of reaction given below ?
5Br– (aq) + BrO3 (aq) + 6H+ (aq)  3Br2(aq) + 3H2O (l)

[Br  ] [H  ] [Br  ] 6 [H  ] [Br  ] 5 [H  ] [Br  ] [H  ]


(a) 5 (b)  (c)  (d) 6 j
t t t 5 t t 6 t t t
Ans. (c)
7. Which of the following graphs represents exothermic reaction ?
Activated complex
Energy 

(I) Reactants

Products

Reaction coordinate 

Activated complex
 
Energy 

  Products
(II)
Reactants
Reaction coordinate 

Activated complex
Energy 

 
node06\B0BC-BD\Kota\Board Material\Chemistry\Booklets\CBSE\Part-1

Reactants
 
(III)
Products

Reaction coordinate 
 

(a) Only (I) (b) Only (II) (c) Only (III) (d) (I) and (II)
Ans. (a)
172 E
CBSE
ALLEN
8. A first order reaction is 50% completed in 1.26 × 1014 s. How much time would it take for 100%
completion?
(a) 1.26 × 1015 s (b) 2.52 × 1014 s
(c) 2.52 × 1028 s (d) Infinite
Ans. (d)
9. Compound ‘A’ and ‘B’ react according to the following chemical equation.
A(g) + 2B(g)  2C(g)
Concentration of either ‘A’ or ‘B’ were changed keeping the concentrations of one of the
reactants constant and rates were measured as a function of initial concentration. Following
results were obtained. Choose the correct option for the rate equations for this reaction :

Experiment  Initial Initial Initial


concentration concentration of concentration of
of [B]/mol L–1  [C]/mol L–1
[A]/mol
L–1 

1  0.30  0.30  0.10

2  0.30  0.60  0.40

3  0.60  0.30  0.20

(a) Rate = k [A]2 [B] (b) Rate = k[A] [B]2


(c) Rate = k [A] [B] (d) Rate = k [A]2 [B]
Ans. (b)
10. Which of the following statement is not correct for the catalyst?
(a) It catalyses the forward and backward reactions to the same extent
(b) It alerts G of the reaction
(c) It is a substance that does not change the equilibrium constant of a reaction
(d) It provides an alternate mechanism by reducing activation energy between reactants and products
Ans. (b)
11. The value of rate constant of a pseudo first order reaction …………..
node06\B0BC-BD\Kota\Board Material\Chemistry\Booklets\CBSE\Part-1

(a) depends on the concentration of reactants present in small amount


(b) depends on the concentration of reactants present in excess
(c) is independent of the concentration of reactants
(d) depends only on temperature
Ans. (a)

E 173
Chemistry
ALLEN
(SHORT ANSWER TYPE QUESTIONS)
12. Write the rate equation for the reaction 2A + B  C if the order of the reaction is zero.
Ans. For reaction 2A + B C if the rate of reaction is zero then it can be represented as
Rate = k[A]0 [B]0 = k
i.e., rate of reaction is independent of concentration of A and B.
13. In a reaction if the concentration of reactant A is tripled, the rate of reaction become twenty
seven times. What is the order of the reaction ?
Ans. Rate of any elementary reaction can be represented as
r = k[A]n
After changing concentration to its triple value A = 3A, r becomes 27r
27r  k[3A]n

r k[A]n

27r k[3A]n
n 3 n
1 1  1  1 
       
27  3  3 3
Hence, n = 3
Order of reaction is three.
14. Oxygen is available in plenty in air yet fuels do not burn by themselves at room temperature.
Explain.
Ans. For combustion reactions, activation energy of fuels is very high at room temperature. So, fuels
do not burn by themselves at room temperature.
15. What is the probability of reaction with molecularity higher than three very rate ?
Ans. According to collision theory, we know that to complete any chemical reaction there must be
effective collision between reactant particles and they must have minimum sufficient energy. The
probability of more than three molecules colliding simultaneously is very small. Hence,
possibility of molecularity being three is very low.
16. Why does the rate of any reaction generally decreases during the course of the reaction ?
Ans. The rate of a reaction depends on the concentration of the reactants. As the reaction proceeds in
forward direction, concentration of reactant decreases and that of products increases. So, the rate
node06\B0BC-BD\Kota\Board Material\Chemistry\Booklets\CBSE\Part-1

of reaction generally decreases during the course of reaction.


17. Why can’t molecularity of any reaction be equal to zero ?
Ans. Molecularity of the reaction is the number of molecules taking part in an elementary step. For
this we require at least a single molecule leading to the value of minimum molecularity of one.
Hence, molecularity of any reaction can never be equal to zero.

174 E
CBSE
ALLEN
(MATCHING THE COLUMN)
18. Match the graph given in Column I with the order of reaction given in Column II. More than one
item in Column I may link to the same item of Column II.
Column-I  Column-II
(A) (1)   First order
Rate 

Concentration    
(B) (2)  Zero order
Rate 

Concentration    
(C)  
Conc. 

Time   
(D)  
log(Conc.) 

Time   
Ans. A-1, B-2, C-2, D-1
19. Match the statements given in Column I and Column II.
Column-I  Column-II
(A)  Catalyst alters therate of reaction  (1)  Cannot be fraction or zero
(B)  Molecularity  (2)  Proper orientation is not there
(C)  Second half-life of first order reaction  (3)  By lowering the activation energy
node06\B0BC-BD\Kota\Board Material\Chemistry\Booklets\CBSE\Part-1

(D) e Ea /RT   (4)  Is same as the first


(E)  Energetically favourable reactions are (5)  Total probability is one
sometimes slow 
(F)  Area under the Maxwell, Boltzmann (6)  Refers to the fraction of molecules with energy
curve is constant  equal to or greater than activation energy

Ans. A-3, B-1, C-4, D-6, E-2, F-5


E 175
Chemistry
ALLEN
20. Match the items of Column I and Column II

Column-I  Column-II

(A)  Diamond  (1)   Short interval of time

(B)  Instataneous rate  (2)  Ordinarily rate of conversion is imperceptible

(C)  Average rate  (3)  Long duration of time

Ans. A-3, B-1, C-4, D-6, E-2, F-5


21. Match the items of Column I and Column II

Column-I  Column-II

(A)  Mathematical expression for rate (1)   Rate constant


of reaction 

(B)  Rate of reaction for zero order (2)  Rate law


reaction is equal to 

(C)  Units of rate constant for zero (3)  Order of slowest step
order reaction is same as that of  

(D)  Order of a complex reaction is (4)  Rate of reaction


determined by 

Ans. A-2, B-1, C-4, D-3


(ASSERTION AND REASON)
In the following questions a statement of Assertion (A) followed by a statement of Reason (R) is
given. Choose the correct answer out of the following choices :
(a) Both (A) and (R) correct and (R) is correct explanation of (A).
(b) Both (A) and (R) correct and (R) is not correct explanation of (A).
(c) (A) is correct but (R) is wrong.
(d) (A) Assertion is wrong but (R) reason is correct.
22. Assertion (A) : Order of the reaction can be zero or fractional.
Reason (R) : We cannot determine order from balanced chemical equation.
node06\B0BC-BD\Kota\Board Material\Chemistry\Booklets\CBSE\Part-1

Ans. (b)
23. Assertion (A) : Order and molecularity are same.
Reason (R) : Order is determined experimentally and molecularity is the sum of the
stoichiometric coefficient of rate determining elementary step.
Ans. (d)
176 E
CBSE
ALLEN
24. Assertion (A) : The enthalpy of reaction remains constant in the presence of a catalyst.
Reason (R) : A catalyst participating in the reaction forms different activated complex and
lowers down the activation energy but the difference in energy of reactant and product remains
the same.
Ans. (a)
25. Assertion (A) : All collision of reactant molecules lead to product formation.
Reason (R) : Only those collisions in which molecules have correct orientation and sufficient
kinetic energy lead to compound formation.
Ans. (d)
26. Assertion (A) : Rate constant determined from Arrhenius equation are fairly accurate for simple
as well as complex molecules.
Reason (R) : Reactant molecules undergo chemical change irrespective of their orientation
during collision.
Ans. (c)
node06\B0BC-BD\Kota\Board Material\Chemistry\Booklets\CBSE\Part-1

E 177
Chemistry
ALLEN
EXERCISE-4 MCQ, A/R, CASE BASED QUESTIONS
(Rate of Reaction)
1. The rate of a chemical reaction
(a) Increases as the reaction proceeds
(b) Decreases as the reaction proceeds
(c) May increase or decrease during the reaction
(d) Remains constant as the reaction proceeds
2. The rate of a reaction that not involve gases is not dependent on
(a) Pressure (b) Temperature (c) Concentration (d) Catalyst
3. The rate of chemical reaction at constant temperature is proportional to
(a) The amount of products formed
(b) The product of masses of the reactants
(c) The product of the molar concentration of the reactants
(d) The mean free path of the reaction
4. For reaction 2A + B  products, the active mass of B is kept constant and that of A is doubled.
The rate of reaction will then
(a) Increase 2 times (b) Increase 4 times
(c) Decrease 2 times (d) Decrease 4 times
5. A catalyst increases the rate of reaction because it
(a) Increases the activation energy
(b) Decreases the energy barrier for reaction
(c) Decreases the collision diameter
(d) Increases the temperature coefficient
 dc 
6. The term    in a rate equation refers to the
 dt 
(a) Concentration of the reactant
(b) Decrease in concentration of the reactant with time
(c) Increase in concentration of the reactant with time
(d) Velocity constant of the reaction
7. For a given reaction 3A + B  C + D the rate of reaction can be represented by
1 d[A] d[B]  d[C] d[D]
node06\B0BC-BD\Kota\Board Material\Chemistry\Booklets\CBSE\Part-1

(a)    
3 dt dt dt dt
1 d[A] d[C]
(b)    K[A]m [B]n
3 dt dt
1 d[A] d[C]
(c)    K[A]n [B]m
3 dt dt
(d) None of these
178 E
CBSE
ALLEN
8. For the reaction N 2  3H 2  2NH 3
[NH 3 ] [H 2 ]
if  2 104 mol l1s 1 , the value of would be
t t
(a) 1104 mol l1s 1 (b) 3 104 mol l1s 1

(c) 4 104 mol l1s 1 (d) 6 104 mol l1s 1


9. An increase in temperature by 10°C, generally increases the rate of a reaction by
(a) 2 times (b) 10 times (c) 9 times (d) 100 times
10. Rate of reaction
(a) Decreases with increase in temperature
(b) Increases with increase in temperature
(c) May increase or decrease with increase in temperature
(d) Does not depend on temperature
11. A first order reaction complete its 10% in 20 minutes then time required to complete its 19% is
(a) 30 minutes (b) 40 minutes (c) 50 minutes (d) 38 minutes
(Rate Laws and Rate Constant)
12. The rate law for reaction will be
(a) Rate = K[A] [B] (b) Rate = K[A][2B]
[C][D]2
(c) Rate = K[A] [B]2 (d) Rate  K
[A][B]2
d[A] d[B]
13. A + 2B  C + D. If   5 104 mol l1s 1 1, then  is
dt dt
(a) 2.5 104 mol l1s 1 (b) 5.0 104 mol l1s 1

(c) 2.5 10  3mol l1s 1 (d) 1.0 103 mol l1s 1
14. The data for the reaction A + B  C is

Exp. [A]0 [B]0 Initial rate

(1) 0.012 0.035 0.10


(2) 0.024 0.070 0.80
node06\B0BC-BD\Kota\Board Material\Chemistry\Booklets\CBSE\Part-1

(3) 0.024 0.035 0.10


(4) 0.012 0.070 0.80
The rate law corresponds to the above data is
(a) Rate = k [B]3 (b) Rate = k[B]4
(c) Rate = k[A] [B]3 (d) Rate = k[A]2 [B]2

E 179
Chemistry
ALLEN
15. For a reactions A + B  product, it was found that rate of reaction increases four times if
concentration of ‘A’ is doubled, but the rate of reaction remains unaffected. If concentration of
‘B’ is doubled. Hence, the rate law for the reaction is
(a) rate = k[A] [B] (b) rate = k[A]2
(c) rate = k[A]2 [B]1 (d) rate = k[A]2 [B]2
16. The rate constant of a reaction depends on
(a) Temperature (b) Mass (c) Weight (d) Time
17. The rate of reaction is determined by slow step reaction. The step is called
(a) Reaction rate (b) Activation step
(c) Rate determining step (d) None of the above
18. If the concentration is expressed in moles per litre, the unit of the rate constant for a first order
reaction is
(a) mole litre–1 sec–1 (b) mole litre–1
(c) sec–1 (d) mole–1 litre–1 sec–1
19. A zero order reaction is one whose rate is independent of
(a) Temperature of the reaction
(b) The concentrations of the reactants
(c) The concentration of the products
(d) The material of the vessel in which the reaction is carried out
20. The unit of rate constant for a zero order reaction is
(a) litre sec–1 (b) litre mole–1 sec–1
(c) mole litre–1 sec–1 (d) mole sec–1
21. For a chemical reaction A B it is found that the rate of reaction doubles, when the
concentration of A is increased four times. The order in A for this reaction is
(a) Two (b) One (c) Half (d) Zero
22. Which one of the following formula represents a first order reaction
x 1  1 1
(a) K  (b) K    2
t 2t  (a  x) a 
2

2.303 a 1 x
(c) K  log10 (d) K 
t (a  x) t a(a  x)
node06\B0BC-BD\Kota\Board Material\Chemistry\Booklets\CBSE\Part-1

23. A first order reaction which is 30% complete in 30 minutes has a half-life period of
(a) 24.2 min (b) 58.2 min (c) 102.2 min (d) 120.2 min
24. The reaction 2N2O5 ֖ 2NO2 + O2 follows first order kinetics. Hence, the molecularity of the
reaction is
(a) Unimolecular (b) Pseudo–unimolecular
(c) Bimolecular (d) None of the above

180 E
 CBSSE
ALLEN
A
25.
2 Catalyyst decompo
osition of hy
ydrogen perroxide is a ........ order reaction
r
(a) Firrst (b) Seecond (c) Third
T (d)) Zero
26.
2 The haalf life of a first
f order reaction
r is
(a) Inddependent ofo the initial concentratiion of the reeactant
(b) Dirrectly propoortional to thhe initial cooncentrationn of the reacctants
(c) Invversely propportional to the initial cconcentratioon of the reaactant
(d) Dirrectly propoortional to thhe square oof the initial concentratiion of the reeactant
27.
2 A firstt order reacttion requirees 30 minutees for 50% completion. The time rrequired to complete thhe
reactioon by 75% will
w be
(a) 45 minutes (b) 155 minutes (c) 60
6 minutes (d)) None of th
hese
28.
2 Inversiion of canessugar in dilu ute acid (coonversion innto glucose and fructosee) is a
(a) Unnimolecular reaction (b) Bimolecula
B r reaction
(c) Triimolecular reaction
r (d) Pseudo-unim
P molecular rereaction
29.
2 The haalf life perio
od of a first order reactiion
0.6693 0.693
0 2.303 0.303
(a) (b) (c)) (d))
t K t K1
30.
3 Half liife period t1/2 for first order
o reactioon is
1.303log 2 2.303log 2 9
(a) K (b) (c)) (d))
K K K
(Colliision theoryy and Arrh henius equa ations)
31.
3 Accordding to Arrh henius theorry, the activvation energ gy is
(a) Thee energy it should
s posssess so that it can enterr into an effeective collission
(b) The energy wh hich the mo olecule shouuld possess in order to undergo
u reaaction
(c) Thee energy it has
h to acquiire further sso that it can n enter into a effective collison
(d) The energy gaained by thee molecules on colliding with another molecul ule
32.
3 The ennergy of acttivation is
(a) Thee energy associated with the activaated molecu ules
(b) Threshold eneergy – energ gy of normaal moleculess
(c) Thrreshold eneergy + energ gy of normaal moleculess
(d) Ennergy of prodducts – enerrgy of reacttants
33.
3 Whichh of the folloowing plotss is in accorddance with the Arrheniius equationn ?

(a) (b)
node06\B0BC-BD\Kota\Board Material\Chemistry\Booklets\CBSE\Part-1

(c) (d)

E 18
81
Chemistry
ALLEN
34. The minimum energy a molecule should possess in order to enter into a fruitful collision is
known as
(a) Reaction energy (b) Collision energy
(c) Activation energy (d) Threshold energy
35. Activation energy is given by the formula
K2 E a  T2  T1  K1 E a  T2  T1 
(a) log    (b) log   
K1 2.303R  T1T2  K2 2.303R  T1T2 

K1 E a  T1  T2 
(c) log    (d) None of these
K2 2.303R  T1T2 
36. Relation between rate constant and temperature by Arrhenius equation is
Ea Ea
(a) log e A  log e K  (b) log K  A
RT RT
Ea
(c) log e K  log e A  (d) log A  RT ln E a  ln K
RT 2
37. Collision theory is applicable to
(a) First order reactions (b) Zero order reactions
(c) Bimolecular reactions (d) Intra molecular reactions

ANSWER KEY
Q. No. 1 2 3 4 5 6 7 8 9 10
Ans. b a c b b b a b a c
Q. No. 11 12 13 14 15 16 17 18 19 20
node06\B0BC-BD\Kota\Board Material\Chemistry\Booklets\CBSE\Part-1

Ans. b c a a b a c c b c
Q. No. 21 22 23 24 25 26 27 28 29 30
Ans. c c b c a a c d b c
Q. No. 31 32 33 34 35 36 37  
Ans. c b a d a a c

182 E
CBSE
ALLEN
ASSERTION AND REASON

Directions : In the following questions, an Assertion (A) is followed by a corresponding


Reason (R) Use the following keys to choose the appropriate answer.
(a) Both (A) and (R) correct and (R) is correct explanation of (A).
(b) Both (A) and (R) correct and (R) is not correct explanation of (A).
(c) (A) is correct but (R) is wrong.
(d) (A) Assertion is wrong but (R) reason is correct.
1. Assertion (A) : Order of the reaction can be zero or fractional.
Reason (R) :We cannot determine order from balanced chemical equation.
2. Assertion (A) : Order and molecularity are same.
Reason (R) : Order is determined experimentally and molecularity is the sum of the
stoichiometric coefficient of rate determining elementary step.
3. Assertion (A) : The enthalpy of reaction remains constant in the presence of a catalyst.
Reason (R) : A catalyst participating in the reaction forms different activated complex and
lowers down the activation energy but the difference in energy of reactant and product remains
the same.
4. Assertion (A) : All collision of reactant molecules lead to product formation.
Reason (R) : Only those collisions in which molecules have correct orientation and sufficient
kinetic energy lead to compound formation .
5. Assertion (A) : Rate constants determined from Arrhenius equation are fairly accurate for simple
as well as complex molecules.
Reason (R) : Reactant molecules undergo chemical change irrespective of their orientation
during collision.
6. Assertion (A) : [R ] is multiplied with –1 to make the rate of the reaction a positive quantity.
[R]
Reason (R) : [R] is a negative quantity in the expression, rate of disappearance of R 
t
7. Assertion (A) : Some zero order reactions may have order in fractions.
Reason (R) : Order cannot be determined from balanced chemical equation.
node06\B0BC-BD\Kota\Board Material\Chemistry\Booklets\CBSE\Part-1

8. Assertion (A) : Decomposition of gaseous ammonia on a hot platinum surface is a zero order
reaction at high pressure.
Reason (R) : At high pressure, the metal surface gets saturated with gas molecules.
9. Assertion (A) : A catalyst increases the rate of reaction.
Reason (R) : Catalyst also changes the equilibrium constant.

E 183
Chemistry
ALLEN
10. Assertion (A) : A positive catalyst increases the rate of reaction.
Reason (R) : A catalyst increases the rate of reaction by making available a new and more
efficient mechanism.
11. Assertion (A) : Order of reaction can be zero or fractional.
Reason (R) : We cannot determine order from balanced chemical equation.
12. Assertion (A) : Rate of reaction is a measure of change in concentration of reactant with respect
to time.
Reason (R) : Rate of reaction is a measure of change in concentration of product with respect to
time.
13. Assertion (A) : For a reaction: P + 2Q  products, Rate = k[P]1/2 [Q]1 so the order of reaction
is 1.5
Reason (R) : Order of reaction is the sum of stoichiometric coefficients of the reactants.
14. Assertion (A) : The unit of k is independent of order of reaction.
Reason (R) : The unit of k is moles L–1 s–1.
15. Assertion (A) : Reactions can occur at different speeds.
Reason (R) : Rate of reaction is also called speed of reaction.

ANSWER KEY
node06\B0BC-BD\Kota\Board Material\Chemistry\Booklets\CBSE\Part-1

Q. No. 1 2 3 4 5 6 7 8 9 10
Ans. b d a d c a d a c a
Q. No. 11 12 13 14 15
Ans. b b c d b

184 E
CBSE
ALLEN
CASE BASED QUESTIONS
Case-I
Most of the chemical reactions are accelerated by increase in temperature T. At a particular
temperature T, all the molecules of reactants do not have the same kinetic energy but the
fractions of molecules having particular kinetic energies at a particular temperature remain
constant. The distribution of kinetic energy may be described by plotting the fraction of
molecules (NE/NT) with a given kinetic energy (E). This is called Maxwell's Boltzmann
distribution of energies. Here, NE is the number of molecules with energy E and NT is the total
number of molecules.
The temperature dependence of rate of a chemical reaction is expressed by Arrhenius equation,
k  Ae Ea RT
Answer the following questions :
(a) Define effective collision ?
Ans. Those collisions which lead to the formation of products are called effective collisions.
OR
What is the effect of catalyst on the rate of reaction ?
Ans. A catalyst increases the rate of reaction by lowering down the activation energy. Itprovides
an alternative path to the reaction.
(b) According to the Arrhenius, rate fo a reaction increases with increse in temperature. Give
reason ?
Ans. Rate of a reaction increases with increase in temperature because kinetic energy of the
molecules increases, number of molecules possessing activation energy increases, i.e. the
number of effective collisions increases.
(c) What do you understand by the rate law and rate constant of a reaction? Identify the order
of a reaction if the units of its rate constant are:
(i) L–1 mol s–1 (ii) L mol–1 s–1
Ans. The effect of concentration on the rate reaction was first studied by Guldberg and Wage in
1867. They put forward the following result, called the Law of Mass Action.
For a general reaction.
aA + AB  Products.
Rate  [A]a [B]b or Rate = k [A]a [B]b
It is observed that the rate of this reaction may not depend upon all the ‘a’ concentration
term of A and all the ‘b’ concentration term of B. Suppose, experimentally the rate of the
reaction is found to depend upon ‘’ concentration term of A and ‘’ concentration term of
node06\B0BC-BD\Kota\Board Material\Chemistry\Booklets\CBSE\Part-1

B. Then,

Rate  [A] [B]or Rate = k [A] [B]


 where A and B are the molar concentrations of A and B respectively and k is a constant
called velocity constant or rate constant. The above expression is called Rate Law.
(i) Zero order
(ii) Second order

E 185
Chemistry
ALLEN
Case-II
The rate of a reaction, which may also be called its velocity or speed, can be defined with relation
to the concentration of any of the reacting substances, or to that of any product of the reaction. If
the species chosen is a reactant which has a concentration c at time t the rate is
–dc/dt, while the rate with reference to a product having a concentration x at time t is dx/dt. Any
concentration units may be used for expressing rate; thus, if moles per liter are employed for
concentration and seconds for the time, the units for die rate are moles liter–1 sec–1. For gas
reactions pressure units are sometimes used in place of concentration, so that legitimate units for
the rate would be (mm Hg) sec–1 and atm sec–1.
The order of a reaction concerns the dependence of the rate upon the concentrations of reacting
substances; thus, if the rate is found experimentally to be proportional to the th power of the
concentration of one of the reactants A, to the th power of the concentration of a second reactant
B, and so forth, via.,
rate = kCA CB …..(i)
the overall order of the reaction is simply
n =  +  + ……. …..(ii)
Such a reaction is said to be of the  order with respect to the substance A, the th order with
th

respect to B and so on...


Answer the following questions :
(a) Give the example of first order reaction.
Ans. All natural and artificial radioactive decay of unstable nuclei takes place by first order
kinetics.
88 Ra  2 He  86 Rn
226 4 222

Hydogenation of ethene and decomposition of N2O5 and N2O are also the examples of first
order reactions.
(b) Plot the graph between log [R]0 / [R] vs time (t) for a first order reaction.
Ans. For first order reaction,
1 [R]0
k = ln
t [R]
2.303 [R]0
R= log
t [R]
2.303 [R]0
t= log
k [R]
k
If we plot a graph between log [R]0 / [R] vs t, we get a straight line with the slope =
2.303
as shown below :
node06\B0BC-BD\Kota\Board Material\Chemistry\Booklets\CBSE\Part-1
1log[R]0 / [R] 

Slope= k/2.303

Time 

186 E
CBSE
ALLEN
(c) (a) Derive the general expression for half-life of a first action.
(b) The decomposition of NH3 on platinum surface is a zero order. What would be the
rates of production of N2 and H2 if k = 2.5 × 10–4 mol–1 Ls–1 ?
Ans. (a) Half-life of first order reaction :
For first order reaction, we have
2.303 [A]0 2.303 [A]0
k log  t log
t [A] k [A]
1
when t = t1/2 then [A] = [A 0 ]
2
2.303 [A] 2.303
 t1/2  log 1 0  log 2
k 2  A 0 k
2.303  0.3010 0.693
t1/2  
k k
This shows that half-life period for first order reaction does not depend upon initial
concentration of the reactants.
(b)  N2(g) + 3H2(g)
2NH3(g) 
1 [NH3 ] [N 2 ] 1 [H 2 ]
rate of reaction = – .  
2 t t 3 t
for zero order reaction,
rate of reaction = K = 2.5 × 10–4 mol L–1 s–1
[N 2 ]
= 2.5 × 10–4 mol L–1 s–1
t
[H 2 ]
= 3 k = 3 × 2.5 × 10–4 = 7.5 × 10–9 mol L–1 s–1
t
OR
For an elementary reaction :
2A + B  3C
The rate of appearance of C at time 't' is 1.3 × 10–4 mol–1 s–1. Calculate at this time.
(i) Rate of reaction ?
(ii) Rate of disappearance of A.
1 d[x]
Ans. (i) Rate =
3 dt
node06\B0BC-BD\Kota\Board Material\Chemistry\Booklets\CBSE\Part-1

1
= 1.3 104
3
= 0.43 × 10–4 mol L–1 s–1
dA 2 d[x]
(ii)   
dt 3 dt
= 8.87 × 10–4 mol L–1 s–1

E 187
Chemistry
ALLEN
PREVIOUS YEARS QUESTIONS
SECTION-A (ONE MARK QUESTIONS)
1. The slope in the plot of ln[R] vs. time for a first order reaction is [1] (CBSE 2023)
k –k
(a) (b) –k (c) (d) +k
2.303 2.303
Ans. (b)
[R]o
2. The slope in the plot of log vs. time for a first order reaction is [1] (CBSE 2023)
[R]
K K
(a) (b) +k (c) (d) –k
2.303 2.303
Ans. (a)
d[B]
3. For the reaction 3A  2B, rate of reaction  is equal to [1] (CBSE 2023)
dt
3 d[A] 2 d[A] 1 d[A] 2d[A]
(a) (b) (c) (d) 
2 dt 3 dt 3 dt dt
Ans. (b)
d[A]
4. For the reaction 2A  3B, rate of reaction  is equal to [1] (CBSE 2023)
dt
3 d[B] 2 d[B] 1 d[B] 2d[B]
(a) (b) (c) (d) 
2 dt 3 dt 3 dt dt
Ans. (b)
5. A biochemical reaction was carried out in the absence of enzyme and the rate of reaction was
found to be 10–6 min–1. If the same reaction is now carried out in the presence of enzyme, then
the Ea for the reaction will be : [1] (CBSE 2021 Compartment)
(a) same (b) Greater than 10–6 min–1
(c) Lower than 10–6 min–1 (d) Data insufficient, Ea cannot be predicted
Ans. (d)
6. Will the rate constant of the reaction depend upon T if the Ea (activation energy) of the reaction is
zero ? [1] (CBSE 2020)
Ans. No
7. Define ‘order of a reaction’. [1] (CBSE 2020)
node06\B0BC-BD\Kota\Board Material\Chemistry\Booklets\CBSE\Part-1

Ans. Order of reaction : Order of a reaction is the sum of the powers of the concentration terms
in the experimentally determined rate equation.
SECTION-B (TWO MARKS QUESTIONS)
8. (a) The conversion of molecule A to B followed second order kinetics. If concentration of
A increased to three times, how will it a affect the rate of formation of B?
(b) Define Psuedo first order reaction with an example. [2] (CBSE 2023)
188 E
CBSE
ALLEN
Ans. (a) R= K[A]2
Concn of A increased three times than rate
R1 = k[3A]2
= 9k[A]2
So, R1 = 9R
(b) The reaction which are Bimolecular but there order is one, are called Psuedo first order
reaction.

C12 H 22 O11  H 2 O 
H
 C6 H12 O6  C6 H12 O6
Rate = K[C12H22O11]
9. Define rate constant and give the mathematical relation between rate constant and half–life
period for a first order reaction. [2](CBSE 2021 Compartment)
Ans. Rate constant :
In a chemical reaction –
n1A + n2B  m1C + m2D
according to law of mass action

Rate  k  A  1  B
n n2

but according to rate law (experimental concept)


Rate = k[A]x[B]y
if [A] = [B] = 1 mol/L
then, Rate = k
Rate of reaction at unit concentration of reactants is called as rate constant or specific reaction rate.
Rate constant does not depend on concentration of reactant but it depends on temperature and
catalyst. [1]
Half-life Period for first order reaction : The time in which half of the initial amount of
reactant is consumed.
At t = t1/2 ; x = a/2 ; a–x = a/2
1  a  n2 2.303 0.693 0.693
t1/2 = n   or t1/2 = = (log2) or t1/2  k [1]
k a/2 k k k t1/2
node06\B0BC-BD\Kota\Board Material\Chemistry\Booklets\CBSE\Part-1

Half-life period for first order reaction is independent of the initial concentration of reactant.
10. Assertion (A) : The molecularity of the reaction H2 + Br2  2HBr
Reason (R) : Two molecules of the reactants are involved in the given elementary reaction.
[2] (CBSE 2020)
Ans. (i) Both Assertion (A) and Reason (R) are correct statements, and Reason (R) is the correct
explanation of the Assertion (A).

E 189
Chemistry
ALLEN
11. Analyse the given graph, drawn between concentration of reactant vs. time :
1.6

Concentration of reactant
0.8

0.4
0.2
0 10 20 30
time 

(a) Predict the order of reaction.


(b) Theoretically, can the concentration of the reactant reduce to zero after infinite time?
Explain. [1 × 2 = 2] (CBSE 2020)
st
Ans. (a) I order
(b) No, due to exponential relation / the curve never touches the x-axis.
12. (a)For a reaction A + B  P, the rate law is given by,
r = k[A]1/2 [B]2.
What is the order of this reaction ?
(b) A first order reaction is found to have a rate constant k = 5.5 × 10–14 s–1. Find the half-life
of the reaction. [2] (CBSE 2020)
Ans. (a) 2.5
0.693 0.693
(b) t1/2 =  = 0.126 × 1014sec.
k 5.5 1014
13. Define order of reaction. Predict the order of reaction in the given graphs :

(a) t1/2 (b) t1/2

[R]0 [R]0
where [R]0 is the initial concentration of reactant and t1/2 is half-life. [2] (CBSE 2019)
Ans. It is defined as the sum of powers to which the concentration terms are raised in the rate law
equation.
(a) First order (b) zero order
Pt
14. For a reaction : 2NH3(g)   N2(g) + 3H2(g)
Rate = k
(i) Write the order and molecularity of this reaction.
(ii) Write the unit of k. [2] (CBSE 2019)
node06\B0BC-BD\Kota\Board Material\Chemistry\Booklets\CBSE\Part-1

Ans. (i) zero order , bimolecular [1]


(ii) mol L–1 s–1 [1]
15. What is the effect of adding a catalyst on :
(a) Activation energy (Ea), and
(b) Gibbs energy (G) of a reaction ? [2] (CBSE 2018)
Ans. (a) Decreases [1]
(b) No change [1]
190 E
CBSE
ALLEN
SECTION-C (THREE MARKS QUESTIONS)
16. A first order reaction is 50% complete in 30 minutes at 300 K and in 10 minutes at 320 K.
Calculate activation energy (Ea) for the reaction. [3] (CBSE 2023)
[R = 8.314 J K–1mol–1]
[Given : log2 = 0.3010, log3 = 0.4771, log4 = 0.6021]
2.303 100
Ans. T1 = 300 K, K1  log [½]
t1 100 – 50
2.303
 log 2
30
2.303
  0.3010
30
= 0.0767 × 0.3010 = 0.0231
2.303
T2 = 320 k, K1  log 2 [½]
t2
2.303
 log 2
10
 0.2303 × 0.3010  0.0693
K2 E a  T2 – T1 
log    [½]
K1 2.303R  T1T2 

 0.0693  Ea  320 – 300 


log   [½]
 0.0231  2.303  8.314  320  300 

20 E a
log 3
1838121.6
[½]
876967.82
Ea 
20
= 43848.40 j mol–1
= 43.848 kj mol–1 [½]
17. A first order reaction has a rate constant 4.9 × 10–3 s–1. How long will 4 g of this reactant take to
reduce to 3 g ? [log 4 = 0·6020, log 3 = 0·4771. [3] (CBSE 2021_Compartment)
OR
node06\B0BC-BD\Kota\Board Material\Chemistry\Booklets\CBSE\Part-1

The hydrolysis of sucrose is represented by the following chemical equation :


C12H22O11 + H2O (excess)  C6H12O6 + C6H12O6
For the above equation, predict
(a) molecularity, (b) order, and
(c) rate of reaction in terms of all the reactants and products. [1 × 3 = 3]

E 191
Chemistry
ALLEN
2.303 [R]0
Sol. t  log [½]
k [R]
2.303 4
 3 1
log   [½]
4.9 10 s 3
= 0.47 × 103 (log 4 – log 3} = 0.47 × 103 (0.6020 – 0.4771) [½]
3 3
= 0.47 × 10 × 0.125 = 0.0587 × 10 = 58.7 sec [½]
OR
(a) 2
(b) 1
d[C12 H 22 O11 ] d[H 2 O] d(C6 H12 O6 ) d(C6 H12O6 )
(c) Rate     
dt dt dt dt
18. How will the rate of the reaction of affected when [3] (CBSE 2020)
(a) Surface area of the reactant is reduced,
(b) Catalyst is added in a reversible reaction, and
(c) Temperature of the reaction is increased ?
Ans. (a) Decreases (b) Increases (c) Increases
19. The rate of a reaction increases four times when the temperature changes from 300 K to 320 K.
Calculate the energy of activation of the reaction, assuming that it does not change with
temperature. (R = 8.314 J K-1 mol-1) [3] (CBSE 2020)
Ans. Here k2 = 4 k1, T1 = 300 K, T2 = 320 K, Ea = ?
k2 Ea  T2  T1 
 log   
k1 2.303R  T1 T2 
4k1 Ea  320  300 
log 
k1 2.303R  300  320 
Ea  20 
log 4 
2.303R  96000 
Ea  1 
2 log 2 
2.303R  4800 
Ea 1
2  0.3010  
2.303  8.314 4800
 Ea = 0.6020 × 2.303 × 8.314 × 4800 =55327.581 J mol–1
node06\B0BC-BD\Kota\Board Material\Chemistry\Booklets\CBSE\Part-1

20. The thermal decomposition HCO2H is a first order with a rate constant of 2.4 × 10–3 s–1 at a
certain temperature. Calculate how long will it take for 3/4 of initial quantity of HCO2H to
decompose.
(log 0.25 = – 0.6021). [3] (CBSE 2020)
Ans. The first order reaction is
2.303 [A]0
t log
K [A]
192 E
CBSE
ALLEN
3
where [A]0  1, [A] , log 0.25   0.6021, k  2.4 103
4
2.303
t [log(A)0  log(A) 2 ]
2.4 103
2.303
t [log1  log 25]
2.4 103
2.303
t [10  (0.6021)] = 578 s
2.4 103
21. (a) List the factors that determine the rate of a chemical reaction.
(b) The half-life for decay of radioactive 14C is 5730 years. archaeological artifact containing
wood had only 80% of the 14C activity found in a living tree. Calculate the age of the
artefact. [3] (CBSE 2019)
Ans. (a) The important factors on which the rate of a chemical reaction depends are
(i) nature of the reacting species.
(ii) concentration of the reacting species.
(iii) temperature at which a reaction proceeds.
(iv) surface area of the reactants.
(v) presence of a catalyst.
(b) Radioactive decay follows first order kinetics. Let [A]0 = 100
 [A] = 100 × 80% = 80
0.693 0.693 –1
Decay constant (K) =  y
t1/2 5730
2.303 [A ] 2.303 100
t= log 0  log
K [A] 0.693 / 5730 80
2.303  5730
=  log1.25
0.693
2.303  5730
=  0.0969
0.693
1278.7108
= = 1845.18  1845 year
0.693
22. The following data were obtained for the reaction :
A + 2B 
C
node06\B0BC-BD\Kota\Board Material\Chemistry\Booklets\CBSE\Part-1

Experiment  [A]/M  [B]/M  Initial rate of formation of C/M min–1


1 0.2  0.3  4.2 × 10–2
2  0.1  0.1  6.0 × 10–3
3  0.4  0.3  1.68 × 10–1
4  0.1  0.4  2.40 × 10–2

E 193
Chemistry
ALLEN
(a) Find the order of reaction with respect to A and B.
(b) Write the rate law and overall order of reaction.
(c) Calculate the rate constant (k). [3] (CBSE 2019)
p q
Ans. Rate = k[A] [B]
On solving,
(a) Order with respect to A = 2, B = 1 [1]
(b) Rate = k[A]2[B]1 ; overall order = 3 [1]
(c) Experiment-1 4.2 × 10–2 = k(0.2)2(0.3); k = 3.5
Experiment-2 6.2 × 10–3 = k(0.1)2(0.1); k = 6 [1]
23. The rate constant for the first order decomposition of H2O2 is given by the following equation:
1.0 104 K
log k  14.2 
T
Calculate Ea for this reaction and rate constant k if its half-life period be 200 minutes.
(Given : R = 8.314 JK–1 mole–1) [3] (CBSE 2018)
Ans. log k = log A – Ea /2.303RT [1]
Ea / 2.303 RT = 1 × 104 k/ T
Ea = 1.0 × 104 × 2.303 × 8.314
= 191471.4 J/mol [1]
t1/2 = 0.693/ k
k = 0.693/200
= 0.0034 min–1 / 3.4 × 10–3 min–1 [1]
SECTION-D (FOUR MARKS QUESTIONS)
24. Read the passage given below and answer the questions that follow.
Are there nuclear reactions going on in our bodies?
There are nuclear reactions constantly occurring in our bodies, but there are very few of them
compared to the chemical reactions, and they do not affect our bodies much. All of the physical
processes that take place to keep a human body running are chemical processes. Nuclear
reactions can lead to chemical damage, which the body may notice and try to fix.
The nuclear reaction occurring in our bodies is radioactive decay. This is the change of a less
stable nucleus to a more stable nucleus. Every atom has either a stable nucleus or an unstable
nucleus, depending on how big it is and on the ratio of protons to neutrons. The ratio of neutrons
node06\B0BC-BD\Kota\Board Material\Chemistry\Booklets\CBSE\Part-1

to protons in a stable nucleus is thus around 1:1 for small nuclei (Z < 20). Nuclei with too many
neutrons, too few neutrons, or that are simply too big are unstable. They eventually transform to
a stable form through radioactive decay. Wherever there are atoms with unstable nuclei
(radioactive atoms), there are nuclear reactions occurring naturally. The interesting thing is that
there are small amounts of radioactive atoms everywhere: in your chair, in the ground, in the
food you eat, and yes, in your body.

194 E
CBSE
ALLEN
The most common natural radioactive isotopes in humans are carbon-14 and potassium-40.
Chemically, these isotopes behave exactly like stable carbon and potassium. For this reason, the
body uses carbon-14 and potassium-40 just like it does normal carbon and potassium; building
them into the different parts of the cells, without knowing that they are radioactive. In time,
carbon-14 atoms decay to stable nitrogen atoms and potassium-40 atoms decay to stable calcium
atoms. Chemicals in the body that relied on having a carbon-14 atom or potassium-40 atom in a
certain spot will suddenly have a nitrogen or calcium atom. Such a change damages the chemical.
Normally, such changes are so rare, that the body can repair the damage or filter away the
damaged chemicals.
The natural occurrence of carbon-14 decay in the body is the core principle behind carbon dating.
As long as a person is alive and still eating, every carbon-14 atom that decays into a nitrogen
atom is replaced on average with a new carbon-14 atom. But once a person dies, he stops
replacing the decaying carbon-14 atoms. Slowly the carbon-14 atoms decay to nitrogen without
being replaced, so that there is less and less carbon-14 in a dead body. The rate at which carbon-
14 decays is constant and follows first order kinetics. It has a half - life of nearly 6000 years, so
by measuring the relative amount of carbon-14 in a bone, archeologists can calculate when the
person died. All living organisms consume carbon, so carbon dating can be used to date any
living organism, and any object made from a living organism. Bones, wood, leather, and even
paper can be accurately dated, as long as they first existed within the last 60,000 years. This is all
because of the fact that nuclear reactions naturally occur in living organisms.
Answer the following questions : [1+1+2=4]
(a) Why is Carbon -14 radioactive while Carbon -12 not? (Atomic number of Carbon: 6)
Ans. Ratio of neutrons to protons is 2.3 : 1 which is not the stable ratio of 1:1
(b) Suppose an organism has 20 g of Carbon -14 at its time of death. Approximately how much
Carbon -14 remains after 10,320 years? (Given antilog 0.517 = 3.289)
Ans. t = 2.303/ k log (Co/Ct)
Co = 20 g Ct = ?
t = 10320 years k = 0.693/6000 (half-life given in passage)
substituting in equation:
10320 = 2.303 / (0.693/6000) log 20/ Ct
0.517 = log 20 / Ct anlilog (0.517) = 20/Ct
3.289 = 20/Ct
Ct = 6.17 g
node06\B0BC-BD\Kota\Board Material\Chemistry\Booklets\CBSE\Part-1

(c) Approximately how old is a fossil with 12 g of Carbon -14 if it initially possessed 32 g of
Carbon -14? (Given log 2.667 = 0.4260)
Ans. t = 2.303/ k log (Co/Ct)
Co = 32 g Ct = 12
t = ? k = 0.693/6000 (half life given in passage)
substituting in equation:
E 195
Chemistry
ALLEN
t = 2.303 / (0.693 / 6000) log 32/ 12
t = 2.303 × 60000 / 0.693 log 2.667
t = 2.303 × 6000 × 0.4260 / 0.693
= 8494 years
OR
(i) Researchers have uncovered the youngest known dinosaur bone, dating around
65 million years ago. How was the age of this fossil estimated?
(ii) Which are the two most common radioactive decays happening in human body?
Ans. (i) Age of fossils can be estimated by C-14 decay. All living organisms have C-14 which
decays without being replaced back once the organism dies.
(ii) Carbon-14 atoms decay to stable nitrogen atoms and potassium-40 atoms decay to
stable calcium
SECTION-E (FIVE MARKS QUESTIONS)
25. Read the following passage and answer the questions that follow :
The rate of reaction is concerned with decrease in concentration of reactants or increase in the
concentration of products per unit time. It can be expressed as instantaneous rate at a particular
instant of time and average rate over a large interval of time. A number of factors such as
temperature, concentration of reactants, catalyst affect the rate of reaction. Mathematical
representation of rate of a reaction is given by rate law : Rate = k[A]x [B]y x and y indicate how
sensitive the rate is to the change in concentration of A and B. Sum of x + y gives the overall
order of a reaction.
When a sequence of elementary reactions gives us the products, the reactions are called complex
reactions. Molecularity and order of an elementary reaction are same. Zero order reactions are
relatively uncommon but they occur under special conditions. All natural and artificial
radioactive decay of unstable nuclei take place by first order kinetics.
(a) What is the effect of temperature on the rate constant of a reaction ?
(b) For a reaction A + B Product, the rate law is given by, Rate = k[A]2 [B]1/2. What is the
order of the reaction ?
(c) How order and molecularity are different for complex reactions ?
(d) A first order reaction has a rate constant 2 × 10–3 s–1. How long will 6g of this reactant take
to reduce to 2g ? [1+1+1+2]
OR
14
The half life for radioactive decay of C is 6930 years. An archaeological artifact containing
wood had only 75% of the 14C found in a living tree. Find the age of the sample.
node06\B0BC-BD\Kota\Board Material\Chemistry\Booklets\CBSE\Part-1

[log 4 = 6021; log 3 = 0.4771; log 2 = 0.3010 log 10 = 1] [1+1+1+2]


Ans. (a) With increase in temperature effective no. of collision, increases due to increase in kinetic
energy of molecules. Hence, Rate of reactions increase. [1]
(b) Rate = k[A]2 [B]1/2 [½]
1
Order = 2 + Order(n)  2.5 [½]
2

196 E
CBSE
ALLEN
(c) Order is determined by show step of the reaction and molecularity determined by
elementary step of the reaction.
2.303  R 0
(d) k= log [1]
t R 
2.303 6
 t= 3
log [½]
2 10 2
t = 549.38 sec. [½]
OR
0.693
t1/2  [1]
k
0.693
 k = [1]
6930
k = 10–4 year–1 [1]
2.303 100
t= 4 1
log [1]
10 year 75

t  496 years [1]

26. (a) For a reaction A + B  P, the rate is given by


Rate = k [A]2 [B]
(i) How is the rate of reaction affected if the concentration of A is doubled?
(ii) What is the overall order of reaction if B is present in large excess?
(b) A first order reaction taken 23.1 minutes for 50% completion. Calculate the time required
for 75% completion of this reaction.
(Given : log 2 = 0.301, log 3 = 0.4771, log 4 = 0.6021) [5](CBSE 2020)
Ans. (a) (i) Rate increases by 4 times [1]
(ii) 2nd order [1]
(b) Reaction is 50% completed in 23.1 min i.e. Half-life is 23.1 min
0.693 0.693
 k    0.03 min 1    [1]
t1/ 2 23.1
2.303 [A ]
k log 0 [1]
t [A]
node06\B0BC-BD\Kota\Board Material\Chemistry\Booklets\CBSE\Part-1

2.303 100
0.03min 1  log [1]
t 25
2.303
0.03  log 4
t
2.303 1.3866
t  0.6021  = 46.22 min [1]
0.03 0.03

E 197
Chemistry
ALLEN
27. (a) Explain the following terms:
(i) Order a reaction
(ii) Molecularity of a reaction
(b) The rate of a reaction increases four times when the temperature changes from 300 K to
320 K. Calculate the energy of activation of the reaction, assuming that it does not change
with temperature. (R = 8.314 J K–1 mol–1) [5] (CBSE 2019)
Ans. (a) (i) Order of a reaction: The sum of powers of the concentration of the reactants in the
experimental rate law expression is called the order of that chemical reaction.
(ii) Molecularity of a reaction: The number of reacting species (atoms, ions or
molecules) taking part in an elementary reaction which must collide simultaneously
in order to bring about a chemical reaction is called molecularity of a reaction.
(b) Here k2 = 4 k1, T1 = 300 K, T2 = 320 K, Ea = ?
k2 E a  T2  T1 
 log   
k1 2.303R  T1 T2 
4k1 E a  320  300 
log 
k1 2.303R  300  320 
E a  20 
log 4 
2.303R  96000 
Ea  1 
2 log 2 
2.303R  4800 
Ea 1
2  0.3010  
2.303  8.314 4800
 Ea = 0.6020 × 2.303 × 8.314 × 4800 = 55327.581 J mol–1
28. (a) Explain the following terms:
(i) Rate of a reaction
(ii) Activation energy of a reaction
(b) The decomposition of phosphine, PH3, proceeds according to the following equation:
4 PH3 (g)  P4 (g) + 6 H2 (g)
It is found that the reaction follow the following rate equation:
Rate = k [PH3]
The half-life of PH3 is 37.9 s at 120ºC.
(i) How much time is required for 3/4th of PH3 to decompose?
node06\B0BC-BD\Kota\Board Material\Chemistry\Booklets\CBSE\Part-1

(ii) What fraction of the original sample of PH3 remains behind after 1 minute?
[5] (CBSE 2018)
Ans. (a) (i) Rate of reaction: The rate of a reaction can be defined as the change in concentration
of a reactant of product in unit time.
(ii) Activation energy of a reaction: The minimum extra amount of energy absorbed by
the reactant molecules so that their energy becomes equal to threshold value is called
activation energy. It is denoted by Ea.

198 E
CBSE
ALLEN
(b) (i) Here t1/2 = 37.9 s
Let initial concentration = a
3
 x a
4
Using the formula
2.303 a
t log
k ax
2.303
a  0.693 
t  0.693 log  k  
t1/2 3  t1/2 
a a
4
t1/2 a
t  2.303  log
0.693 1
a
4
2.303  t1/2 2.303  t1/2  log 2
t  log 4 
0.693 0.693
2.303  37.9  2  0.3010
t [ t1/2 = 37.9s]
0.693
52.544
  75.82 sec.
0.693
(ii) Here t = 1 min = 60s and t1/2 = 37.9s
Using the formula
2.303 [A]0 0.693
t log and k 
k [A]t t1/ 2
2.303
[A]0
 t  0.693 log
t1/2 [A]1
t1/ 2 [A]0
t  2.303   log
0.693 [A]t
2.303  37.9 [A]0
1 log
0.693 [A]1
[A]0 1 0.693 0.693
node06\B0BC-BD\Kota\Board Material\Chemistry\Booklets\CBSE\Part-1

 log    0.0079
[A]t 2.303  37.9 87.2837
[A]0
 anti  log (0.0079)  1.018
[A]t

E 199
Chemistry
ALLEN
PRACTICE TEST
SECTION-A
1. The half-life for a zero order reaction having 0.02 M initial concentration of reactant is 100 s.
The rate constant (in mol L–1 s–1) for the reaction is
(a) 1.0 × 10–4 (b) 2.0 × 10–4 (c) 2.0 × 10–3 (d) 1.0 × 10–2
2. In collision theory of chemical reaction, ZAB represents
(a) the fraction of molecules with energies greater than Ea
(b) the collision frequency of reactants, A and B
(c) steric factor
(d) the fraction of molecules with energies equal to Ea
3. When initial concentration of a reactant is doubled in a reaction, its half-life period is not
affected. The order of the reaction is :-
(a) First (b) Second
(c) More than zero but less than first (d) Zero
4. A first order reaction has a rate constant of 2.303×10–3 s–1. The time required for 40g of this
reactant to reduce to 10 g will be-
[Given that log10 2=0.3010]
(a) 230.3 s (b) 301 s (c) 2000 s (d) 602 s
5. For the chemical reaction
N2(g) + 3H2(g)  2NH3(g)
the correct option is :

1 d H2  1 d  NH 3 
d N  d NH3 
  2  2 
(a)   (b) dt dt
3 dt 2 dt
d N  1 d NH3  d H  d NH3 
  2  3  2  2 
(c) dt 2 dt (d) dt dt

6. Assertion (A) : All collisions of reactant molecules lead to product formation.


Reason (R) : Only those collisions in which molecules have correct orientation and sufficient
kinetic energy lead to compound formation.
7. Assertion (A) : Rate constant determined from Arrhenius equation are fairly accurate for simple
as well as complex molecules.
Reason (R) : Reactant molecules undergo chemical change irrespective of their orientation
during collision.
SECTION-B
node06\B0BC-BD\Kota\Board Material\Chemistry\Booklets\CBSE\Part-1

8. For a chemical reaction R P, the variation in the concentration (R) vs. time (t) plot is given as

(R)

t
(i) Predict the order of the reaction. (ii) What is the slope of the curve ? [2]

200 E
CBSE
ALLEN
9. Define the following :
(a) Elementary step in a reaction.
(b) Rate of a reaction [2]
10. A reaction is of second order with respect to a reactant. How is its rate affected if the
concentration of the reactant is (i) doubled (ii) reduced to half ? [2]
11. For the reaction, 2N2O5(g)  4NO2(g) + O2(g),
the rate of formation of NO2 (g) is 2.8 × 10–3 M s–1. Calculate the rate of disappearance of
N2O5(g). [2]
SECTION-C
12. A first order reaction is 50% completed in 40 minutes at 300 K and in 20 minutes at 320 K.
Calculate the activation energy of the reaction. (Given : log 2 = 0.3010, log 4 = 0.6021,
R = 8.314 JK–1 mol–1). [3]
13. The following data were obtained during the first order thermal decomposition of SO2Cl2 at a
constant volume :
SO2Cl2(g) SO2 (g) + Cl2 (g)

Experiment Time/s 1 Total pressure/atm


1 0 0.4
2 100 0.7

Calculate the rate constant.


(Given log 4 = 0.6021, log 2 = 0.3010) [3]
SECTION-D
14. The half-life of a reaction is the time in which the concentration of a reactant is reduced to one
half of its initial concentration. It is represented as t1/2.
t1/2 for a zero order reaction is directly proportional to the initial concentration of the reactants
and inversely proportional to the rate constant.
For a first order reaction, half-life period is constant, i.e. it is independent of initial concentration
of the reacting species. The half-life of a first order equation is readily calculated from the rate
constant and vice versa.
node06\B0BC-BD\Kota\Board Material\Chemistry\Booklets\CBSE\Part-1

For zero order reaction t1/2  [R]0. For first order reaction t1/2 is independent of [R]0.
Answer the following questions : [1+1+2=4]
(a) Define half-life of a reaction (t1/2) ?
OR
On what factors t1/2 of a first order reaction depends ?

E 201
Chemistry
ALLEN
(b) t1/2 of the reaction increases with increase in initial concentration. what si the order of
reaction ?
(c) (i) Answer the following question on the basis of curve for the first order reaction .
A P
Derive the relationship between half life of a first order reaction and its rate constant.
(ii) Calculate the half life of a first order reaction whose rate constant is 200 s–1.
SECTION-E
15. For the hydrolysis of methyl acetate in aqueous solution, the following results were obtained :
t/s 0 30 60
[CH 3COOCH3 ]mol L1 0.60 0.30 0.15

(i) Show that it follows pseudo first order reaction as the concentration of water remains
constant.
(ii) Calculate the average rate of reaction between the time interval 30 to 60 seconds
[5]

node06\B0BC-BD\Kota\Board Material\Chemistry\Booklets\CBSE\Part-1

202 E
CBSE
ALLEN
PRACTICE TEST SOLUTIONS
SECTION-A
1. (a)
2. (b)
3. (a)
4. (d)
5. (c)
6. (c)
7. (c)
SECTION-B
8. (i) Zero order (ii) Slope = – K
9. (a) Elementary step in a reaction: The study of reactions have three main aspects.
(i) Whether the reaction at all takes place or not i.e. spontaneity of a reaction.
(ii) If it is spontaneous, then to what extent the reaction takes place before equilibrium is attained.
(iii) Rates of speeds of reactions.
(b) Rate of reaction : The rate of a reaction is the change in the concentration of any one of
the reactants or products per unit time.

10. Rate = K[A]2


(i) 4 times (ii) 1/4 times
11. Given Reaction :
2N2O5(g)  4NO2(g) + O2(g),
[NO 2 ]
(given   2.8 103 M / sec. or M sec–1)
t
1 [N 2 O5 ] 1 [NO 2 ] [O 2 ]
Rate of the Reaction    
2 t 4 t t
1 [N 2 O5 ] 1 [NO 2 ]
 
2 t 4 t
[N 2 O5 ] 2 [NO 2 ] 1
    2.8 103 M sec–1
t 4 t 2
[N 2 O5 ]
  1.4 103 M sec–1 or M/sec.
t
node06\B0BC-BD\Kota\Board Material\Chemistry\Booklets\CBSE\Part-1

SECTION-C
2.303 100 2.303
12. T1 = 300 K ; K1  log  log 2  0.0575  0.3010  0.0173 [1/2]
t1 100  50 40
2.303 2.303
T2 = 320 K ; K 2  log 2  log 2  0.1151 0.3010  0.0346 [1/2]
t2 20

E 203
Chemistry
ALLEN
K2 E a  T2  T1 
log  [1]
K1 2.303R  T1T2 

 0.0346  Ea  320  300 


log    
 0.173  2.303  8.314  320  300 
20E a 0.3010  2.303  8.314  320  300
log2 =  Ea  [1/2]
2.303  8.314  320  300 20
Ea = 27663.79 J mol–1 = 27.663 kJ mol–1 or kJ/mol [1/2]
13. The thermal decomposition of SO2Cl2 at a constant volume is represented by the following
equation.
SO2Cl2(g)  SO2(g) + Cl2(g)
At t = 0 Pi 0 0
At t = t PA=Pi– x PB = x PC = x
After time, t, total pressure, Pt = (Pi – x) + x + x
 Pt = Pi + x  x = Pt – Pi
Therefore, PA = Pi – x = Pi – (Pt – Pi) = 2Pi – Pt
For a first order reaction,
2.303 P 2.303 Pi
k log i  log
t Pi  p t 2Pi  p t
2.303 0.4
When t = 100 s, k  log  1.386 102 s–1
100s 2  0.4  0.7
SECTION-D
14. (a) Half life is the time in which 50% of the reaction is completed. [1]
OR
t1/2 of a first order reaction depends on the rate constant k.
0.693
t1/2 =
k
(b) Zero order reaction [1]
(c) (i) For a first order reaction : [1]
2.303 [R]0
t= log
k [R]
node06\B0BC-BD\Kota\Board Material\Chemistry\Booklets\CBSE\Part-1

[R]0
At t1/2, [R] =
2
2.303
t1/2 = log 2
k
0.693
t1/2 =
k

204 E
CBSE
ALLEN
(ii) Here rate constant [1]
–1
k = 200 s
 Half life of a first order reaction is :
0.693 0.693
t1/2  
k 200
= 3.46 × 10–3 sec.
SECTION-E
15. (i) A0 = 0.60 A = 0.30 when t = 30 s
2.303 [A ]
k log 0 [1/2]
t [A]
2.303 0.60
k log [1/2]
30 0.30
2.303 2.303
k log 2   0.3010
30 30
0.693
k  0.231 s 1 [1/2]
30
When A0 = 0.60 A = 0.15 when t = 60s
2.303 0.60
k log [1/2]
t 0.15
2.303 0.60
k log [1/2]
60 0.15
2.303 2.303
k log 4   0.6021
60 60
1.3866
k  0.231s 1 [1/2]
60
As for both cases k is approximately same reaction is of pseudo first order
Change in concentration
(ii) Average rate during the interval 30 – 60 sec = [1/2]
Change in time
0.15  0.30
 [1/2]
60  30
0.15
  0.005 mol L1S1
node06\B0BC-BD\Kota\Board Material\Chemistry\Booklets\CBSE\Part-1

[1]
30

E 205
Chemistry
ALLEN
UNIT-8 : d & f-BLOCK
UNIT INDEX
• Theory 206-219
• Exercise-1_Intext Questions 220-221
• Exercise-2_NCERT Exercise Questions 222-234
• Exercise-3_Exemplar 235-239
• Exercise-4_MCQ, A/R, Case Based Questions 240-246
• Previous Years Questions 247-257
• Practice Test 258-261

THEORY
INTRODUCTION :
(i) d-block elements are often called as ‘transition elements because their position in periodic table
is between the s-block and p-block elements.
(ii) Their properties are transitional between the highly reactive metallic elements of s-block (which
form ionic compounds) and the elements of p-block (which form covalent compounds).
(iii) Typically the transition elements have an incompletely filled d-orbital. A transition element may
be defined as the element whose atom in ground state or in one of common oxidation states, has
partly filled d-sub shell i.e. having electrons between 1 to 9.
(iv) Group 12 (the zinc group) elements have completely filled d-orbitals in the ground state as well
as in their common oxidation states. Therefore, they are not regarded as transition elements.
(v) The general electronic configuration of d-block elements is (n–1) d1–10 ns1–2, where n is the outer
most shell. However, palladium does not follow this general electronic configuration. It has
electron configuration [Kr]36 4d10 5s0.
(vi) Zn, Cd and Hg are involved in transition series but they are called non transition elements due to
completely filled d-orbitals.

1st Series
Sc Ti V Cr Mn Fe Co Ni Cu Zn
Z 21 22 23 24 25 26 27 28 29 30
4s 2 2 2 1 2 2 2 2 1 2
3d 1 2 3 5 5 6 7 8 10 10

2nd Series
Y Zr Nb Mo Tc Ru Rh Pd Ag Cd
Z 39 40 41 42 43 44 45 46 47 48
node06\B0BC-BD\Kota\Board Material\Chemistry\Booklets\CBSE\Part-1

5s 2 2 1 1 1 1 1 0 1 2
4d 1 2 4 5 6 7 8 10 10 10

3rd Series
La Hf Ta W Re Os Ir Pt Au Hg
Z 57 72 73 74 75 76 77 78 79 80
6s 2 2 2 2 2 2 2 1 1 2
5d 1 2 3 4 5 6 7 9 10 10

206 E
 CBSSE
ALLEN
A
GENERAL
G L TRENDS IN THE CHEMISTR
C RY OF TRA
ANSITION
N ELEMEN
NTS.
(A
A) Atomiic and Ionicc Radii
In trannsition metaals, left to riight net nucclear chargee increases due
d to poor shielding effect.
e Due to
t
this, thhe atomic annd ionic rad
dii for transsition elemeents for a giiven series sshow a decreasing trennd
for firsst five elem
ments and theen becomess almost con
nstant for neext five elem
ments of thee series.

(A
A) Metalllic characteer :
(i) IIn d-block elements
e th
he penultimaate shell off electrons is expandingg. Thus they have manny
pphysical andd chemical properties iin common.
(ii) N
Nearly all the
t transitio
on elementss display ty
ypical metalllic propertties such ass high tensiile
sstrength, duuctility, mallleability, hi gh thermal and electriccal conductiivity and metallic lustre.
(iii) M
Most transittion elemen
nts have ccpp/hcp type sttructures.
(iv) T
The transitioon elementss are very m
much hard and have low
w volatility.
(v) C
Cr, Mo andd W have maximum
m nuumber of un
npaired elecctrons and ttherefore, th
hese are verry
hhard metals and have maximum
m ennthalpies off atomisation in their reespective peeriod.
(vi) T
The metals with very high
h enthalppy of atomissation tend to
t be noble in their reacctions.
(vii) T
The metalss of the 4d
d and 5d sseries havee greater en
nthalpies oof atomisatiion than thhe
ccorrespondiing elemen
nts of the 3d and thiis is an im
mportant faactor indicaating for thhe
ooccurrence of much more
m frequuent metal--metal bond
ding in com
mpounds of
o the heavvy
ttransition metals.
m
(B
B) MELT
TING AND
D BOILING
G POINTS
node06\B0BC-BD\Kota\Board Material\Chemistry\Booklets\CBSE\Part-1

(i) M
Melting andd boiling po
oint of d-bloock elements is more th
han that of s block elem
ments.
R
Reason : Sttronger metallic bond fformed by present
p unpaaired d-elecctrons.
(ii) IIn Zn, Cd, and
a Hg therre is no unppaired electtron presentt in d-orbitaal, hence du
ue to absencce
oof covalent bond, meltiing and boiiling point are
a very low
w in series. ((Volatile metals
m Zn, Cd,
H
Hg)

E 20
07
Chemistry
C
N
ALLEN
(iii) IIn 3d series,, the melting
g points of m
metals rise to um at d5 exccept for ano
t a maximu omalous valuue
oof Mn and Fall
F regularly
y as the atom
mic number increases.

(iv) A
As the num
mber of unp
paired d-eleectron increeases, the number of ccovalent bond and bonnd
eenergy betw
ween the attoms is exppected to in
ncrease up to
t Cr familly where eaach of the d-
d
oorbital has only
o unpaireed electron and the opp
portunity for covalent ssharing is grreatest.

(v) M
Mn and Tc have comp
paratively loow melting
g point, due to weak m
metallic bon
nd because of
o
sstable Half filled (d5) configurationn.

(vi) L
Lowest mellting point Hg
H (– 38°C)) ; Highest melting
m poin
nt W (3400 °C)

(D
D) Ionisaation energiies or Ionissation enthaalpies :

(i) T
They have high
h IE as compared
c too group 1 an
nd group 2 metals
m due tto their high
h zeff.

(ii) T
The first ioonisation en
nergy of Znn, Cd, and Hg
H are very
y high becaause of theiir fully filleed
((n–1) d10 nss2 configuration.

E)
(E Variab
ble oxidatioon states :

(i) The transition metals exhibit


T e a larrge numberr of oxidatio
on states. T
These differeent oxidatioon
sstates are reelated to the electronic configuratio
on of their atoms.
a

(ii) T
The existencce of the traansition elem
ments in diffferent oxidaation states iis due to thee participatioon
oof inner (n – 1) d-electrons in adddition to outter ns-electrrons becausse, the energ
gies of the ns
n
aand (n – 1) d-sub-shellss are nearlyy same.
node06\B0BC-BD\Kota\Board Material\Chemistry\Booklets\CBSE\Part-1

(iii) IIn general oxidation statte of d-blockk element iss given as minimum
m oxiidation statee = number of
o
'nns' electronns maximum
m oxidationn state = number
n of 'ns' electroons + unpaaired '(n–1))d'
eelectron

(iv) D
Different oxxidation staates of firsst transition mmon oness are in bold
n series (thee most com
ttypes).

20
08 E
CBSE
ALLEN
Outer
Element electronic Oxidation states
configuration
1 2
Sc 3d 4s +3
2 2
Ti 3d 4s +2, +3, +4
3 2
V 3d 4s +2, +3, +4, +5
5 1
Cr 3d 4s +2, +3, +4, +5, +6
5 2
Mn 3d 4s +2, +3, +4, +5, +6, +7
6 2
Fe 3d 4s +2, +3, +4, +5, +6
7 2
Co 3d 4s +2, +3, +4
8 2
Ni 3d 4s +2, +3, +4
10 1
Cu 3d 4s +1, +2
10 2
Zn 3d 4s +2

(v) Highest oxidation state in d-block = +8 (Os, Ru)


(vi) Transition elements show zero oxidation state in metal carbonyls e.g. [Ni(CO)4],
[Fe (CO)5]
(vii) Relative stability of oxidation state :
Sc+2 < Sc+3
Ti+3 < Ti+4 inert gas stable configuration
Mn+6 < Mn+7
V+4 < V+5
Mn+3 < Mn+2 d5 stable configuration

Cr+2 < Cr+3 d3 is more stable than d4 in aq. solution

Ni+2 > Ni+4 On the basis of sum of first four I.P's.


Pt+2 < Pt+4

(viii) On moving in a period stability of higher oxidation state decreases


e.g. VO3– > CrO42– > MnO4–
(ix) On moving down the group stability of higher oxidation state increases
6 6 6
e.g. CrO3  MoO3  WO3
node06\B0BC-BD\Kota\Board Material\Chemistry\Booklets\CBSE\Part-1

7 7 7
MnO4–  TcO4–  Re O4–

(F) ELECTRODE POTENTIALS :


In addition to ionisation enthalpy, the other factors such as enthalpy of sublimation, hydration
enthalpy etc. determine the stability of a particular oxidation state in solution. This can be
explained in terms of their electrode potential values. The oxidation potential of a metal involves
the following process:
M(s)  M+(aq) + e–
E 209
Chemistry
ALLEN
This process actually takes place in the following three steps as given in following flowchart :
H
M(s) M+(aq)

(i) Hsub. Hhyd.. (iii)


I.E.
M(g) M+(g)
(ii)
The oxidation potential which gives the tendency of the overall change to occur, depends upon
the net effect of these three steps. The overall energy change is H = Hsub + HI.E. + Hhyd
If SRP is +ve that means reduction will be easy.
Some important examples :
+2
1. E 0M+2 /M +ve only for Cu among 3d elements because hydration energy of Cu is not enough to
compensate for sublimation energy, IE1 and IE2 for Cu.
2. E 0M+3 /M+2  

(a) E 0Cr 3 / Cr 2 = –ve


that means Cr+2 acts as a reducing agent because Cr+3 has t2g half filled stable
configuration.
(b) E 0Mn3 / Mn2 = +ve
that means Mn+3 acts as an oxidising agent due to stability of half filled (d5)
configuration.
(G) TRENDS IN STABILITY OF HIGHER OXIDATION STATES
The highest oxidation states are shown generally among halides and oxides of transition
elements.
(a) In halides of transition elements
(i) The transition elements react with halogens at high temperatures to form transition metal
halides.
(ii) Since fluorine is the most electronegative element, the transition metals show highest
oxidation states with fluorine. The highest oxidation states are found in TiX4 (tetrahalides,
X = F, Cl, Br and I), VF5 and CrF6.
(iii) The +7 oxidation state for Mn is not shown by simple halides. However, MnO3F is known
in which the oxidation state of Mn is +7.
(iv) The tendency of fluorine to stabilise the highest oxidation state is due to either higher lattice
enthalpy as in case of CoF3 or higher bond enthalpy terms for the higher covalent
node06\B0BC-BD\Kota\Board Material\Chemistry\Booklets\CBSE\Part-1

compounds. eg. VF5 and CrF6.


(v) Fluorides are relatively unstable in their low oxidation states. For example, vanadium form
only VX2 (X = Cl, Br or I) and copper can form CuX (X = Cl, Br, I). On the other hand all
copper (II) halides are known except the iodide. This is because, Cu2+ oxidises I– to I2.
2Cu2+ + 4I–  Cu2I2(s) + I2

210 E
CBSE
ALLEN
(vi) It has been observed that many copper (I) compounds are unstable in aqueous solution and
they undergo disproportionation
2Cu+  Cu2+ + Cu
Copper in +2 oxidation state is more stable than in +1 oxidation state. This can be
explained on the basis of much larger negative hydration enthalpy (Hhyd) of Cu2+ (aq) than
Cu+, which is sufficiently high to compensate second ionisation enthalpy of copper.
(b) IN METAL OXIDES AND OXOCATIONS.
(i) The ability of oxygen to stabilize the highest oxidation state is demonstrated in their oxides.
The highest oxidation states in their oxides concides with the group number. For example,
the highest oxidation state of scandium of group 3 is +3 in its oxides, Sc2O3 whereas the
highest oxidation state of manganese of group 7 is +7, in Mn2O7.
(ii) Besides the oxides, oxocation of the metals also stabilise higher oxidation states. For
example, VV as VO2+, VIV as VO2+ and TiIV as TiO2+.
(iii) It may be noted that the ability of oxygen to stabilise these high oxidation states exceeds
that of fluorine. For example, manganese forms highest fluoride as MnF4 whereas the
highest oxide is Mn2O7. This is due to the fact that oxygen has ability to form multiple
bonds to metals.
(iv) The transition elements in the +2 and +3 oxidation states mostly form ionic bonds whereas
with higher oxidation states, the bonds are essentially covalent e.g. in MnO4– all bonds are
covalent. In these higher oxides the acidic character is predominant. Thus CrO3 gives
H2CrO4 and H2Cr2O7 and Mn2O7 gives HMnO4. V2O5 is, however amphoteric though

mainly acidic and with alkalies as well as acids gives VO43– and VO2+ respectively.

(H) MAGNETIC PROPERTIES


When a magnetic field is applied to substances mainly two type of magnetic behaviour is
observed.
(i) Paramagnetic substances. The substances which are attracted by magnetic field are called
paramagnetic substances and this character arises due to the presence of unpaired electrons
in the atomic orbitals.
(ii) Diamagnetic substances. The substances which are repelled by magnetic field are called
node06\B0BC-BD\Kota\Board Material\Chemistry\Booklets\CBSE\Part-1

diamagnetic substances and this character arises due to the presence of paired electrons in the
atomic orbitals.
Most of the transition metal ions or their compounds have unpaired electrons in d-sub-shell
(from configuration d1 to d9) and therefore, they give rise to paramagnetic character.
(iii) The magnetic moment arise only from the spin of the electrons. This can be calculated
from the relation  = n (n  2) B.M. n = Number of unpaired electrons

E 211
Chemistry
ALLEN
Table : Calculated and Observed Magnetic Moments (BM)
Ion Configuration Unpaired Magnetic moment
electron(s) Calculated Observed
Sc3+ 3d0 0 0 0
Ti3+ 3d1 1 1.73 1.75
Ti2+ 3d2 2 2.84 2.76
V2+ 3d3 3 3.87 3.86
Cr2+ 3d4 4 4.90 4.80
Mn2+ 3d5 5 5.92 5.96
Fe2+ 3d6 4 4.90 5.3 – 5.5
Co2+ 3d7 3 3.87 4.4 – 5.2
Ni2+ 3d8 2 2.84 2.9 – 3.4
Cu2+ 3d9 1 1.73 1.8 – 2.2
Zn2+ 3d10 0 0

(I) FORMATION OF INTERSTITIAL COMPOUNDS


Transition metals form interstitial compounds with elements such as hydrogen, boron, carbon and
nitrogen.
The small atoms of these non-metallic elements (H, B, C, N, etc.) get trapped in vacant spaces of
the lattices of the transition metal atoms as shown below.
e.g. TiC, Fe3H, TiH1.70, VH0.56, Mn4N etc.
Properties :
(1) They are generally non-stoichiometric compound.
(2) Neither typically ionic nor typically covalent
(3) Harder than pure metal
(4) High melting point than pure metal
(5) They retain metallic conductivity
(6) They are chemically inert
Trapped atoms in
Transition metal atoms
interstitial sites
node06\B0BC-BD\Kota\Board Material\Chemistry\Booklets\CBSE\Part-1

Figure showing formation of interstitial compounds


(J) CATALYTIC PROPERTIES
d-block metals or their compound act as catalyst in many reaction due to :
(i) Possibility of variable oxidation state
(ii) Presence of free valencies over metal surface (adsorption power)

212 E
CBSE
ALLEN
(iii) Presence of vacant d-orbital
e.g. Iron (III) catalyses the reaction between iodide and per sulphate ions.
3
2I   S2 O82 
Fe
 I2  2SO42

Explanation :
2Fe 3  2I   2Fe 2  I2

2Fe 2  S2 O82  2Fe 3  2SO42

Other examples of catalyst are : Finely divided iron (in Haber process), nickel (in catalytic
hydrogenation)
(K) ALLOY FORMATION :
(1) Alloys are homogeneous solid solutions in which the atoms of one metal are distributed
randomly among the atoms of the other metal.
(2) The alloys are generally formed by those atoms which have metallic radii within about 15% of
each other.
(3) Transition metals form a large number of alloys because transition metals are quite similar
in size and therefore the atoms of one metal can substitute the atoms of other metal in its
crystal lattice.
(4) The alloys so formed are very hard and have often high melting points.
(5) 1. Bronze - Cu + Sn
2. Brass - Cu (60%) +Zn (40%)
3. Gun metal - (Cu + Zn + Sn)
4. German Silver - Cu (25-30%) + Zn (25-30%) + Ni (40-50%)
5. Nichrome - (Ni + Cr + Fe)
6. Alloys of steel
(a) Chromium steel Cr (2-4%)
(b) Nickel Steel Ni (3-5%)
(c) Stainless steel Cr (12-14%) & Ni (2-4%) Cr forms oxide layer & Protects Iron
From Rusting
(d) Invar Ni (36%)
(6) Amalgm is a semisolid alloy which is formed by mixing a metal with Hg. Fe, CO, Ni do
not form amalgam due to large difference in size.
COMPOUNDS OF d-BLOCK ELEMENTS :
(I) POTASSIUM PERMANGANATE (KMnO4) :
node06\B0BC-BD\Kota\Board Material\Chemistry\Booklets\CBSE\Part-1

(a) Preparation
It is prepared by fusing pyrolusite (MnO2) either with KOH or K2CO3 in presence of
atmospheric oxygen or any other oxidising agent such as KNO3. The produce the dark
green K2MnO4 which disproportionates in a neutral or acidic solution to give permagnate.
2MnO2 + 4KOH + O2  2K2MnO4 + 2H2O
3MnO 4  4H   2MnO 4  MnO 2  2H 2O

E 213
Chemistry
ALLEN
Commercially it is prepared by fusion of MnO2 with KOH followed by electrolytic oxidation
of manganate.
Fused with KOH
–2
MnO2 
oxidised with air or KNO
 MnO43

Electrolytic oxidation in
MnO42– (green) 
alkaline solution
MnO4– (purple)
In the laboratory, a manganese(II) ion salt is oxidised by peroxodisulphate to
permanganate
2Mn2+ + 5S2O82– + 8H2O  2MnO4– + 10SO42– + 16H+
(b) Physical property : It is purple coloured crystalline compound. It is moderately soluble in
water at room temperature.
(c) Chemical property:
 Effect of heating 2KMnO4  513 K
 K2MnO4 + MnO2 + O2
 On treatment with concentrated H2SO4 (KMnO4 is taken in excess), it forms manganese
heptoxide which decomposes explosively on heating.
KMnO4 
Conc.H2 SO4
 Mn2 O7  an explosive 
 Potassium permanganate acts as an oxidising agent in weak alkaline, neutral or acidic
solutions.
acidified KMnO4 (purple)
Neutral or weak alkaline KMnO4
H2S S
Br– BrO3–
SO2 SO4–2
I– IO3–
NO2– NO3–
S2O32– SO4–2
Fe+2 Fe3+
Mn+2 MnO2
Cl– Cl2
Br– Br2 MnO2

I I2
H2C2O4 CO2+H2O

Mn+2 (colourless)

Note :- Hydrochloric acid is not used for acidification of KMnO4 because KMnO4 oxidises HCl
to Cl2.
(II) POTASSIUM DICHROMATE (K2Cr2O7) :
(a) Preparation : The chromite ore is fused with sodium carbonate in presence of air.
4FeCr2O4O (chromite ore) + 8Na2CO3 + 7O2 Fusion
 8Na2CrO4 + 2Fe2O3 + 8CO2
The yellow solution of Na2CrO4 is filtered and acidified with sulphuric acid to give a
solution from which orange sodium dichromate (Na2Cr2O7) can be crystallised.
2Na2CrO4 + H2SO4  Na2Cr2O7 + Na2SO4 + H2O
node06\B0BC-BD\Kota\Board Material\Chemistry\Booklets\CBSE\Part-1

Sodium dichromate is more soluble than potassium dichromate so K2Cr2O7 is prepared by


treating solution of Na2Cr2O7 with KCl.
Na2Cr2O7 + 2KCl  K2Cr2O7 + 2 NaCl
  K2Cr2O7 is preferred over Na2Cr2O7 as a primary standard in volumetric analysis (titration)
because Na2Cr2O7 is hygroscopic in nature but K2Cr2O7 is not.
H
  
CrO 42   Cr2 O72
 
OH

Yellow Orange
214 E
CBSE
ALLEN
(b) Physical property : It is orange-red coloured crystalline compound. It is moderately
soluble in cold water but freely soluble in hot water. It melts at 398°C.
(c) Chemical property :
 Effect of heating : On strongly heating, it decomposes with liberating oxygen.
3
2K2Cr2O7  2K2CrO4 + Cr2O3 + O2
2

In acidic medium K2Cr2O7 acts as an oxidising agent.


Acidified K2Cr2O7 (Orange)

H2S S
SO2 SO4–2

NO2– NO3–
SO3–2 SO4–2
Sn+2 Sn+4
+2
Fe Fe+3
Br– Br2
I– I2
C2H5OH CH3COOH

Cr+3
(Green)

1. Chromyl chloride test (used to identify ionic chlorides)


NaCl + K2Cr2O7 + H2SO4 CrO2Cl2 (Red orange vapour)
Passed in NaOH solution
Na2CrO4 (Yellow) 
Add (CH3COO)2 Pb
 PbCrO4 (yellow ppt)
2. Acidified K2Cr2O7 solution reacts with H2O2 in a etheral solution to give a deep blue
solution due to the formation of CrO5.
Cr2O72– + 2H+ + 4H2O2  2CrO5 + 5H2O
node06\B0BC-BD\Kota\Board Material\Chemistry\Booklets\CBSE\Part-1

E 215
Chemistry
ALLEN
f-BLOCK ELEMENTS

INNER TRANSITION ELEMENTS


The elements in which the last electron enters in (n – 2)f orbitals are called inner transition
elements or f-block elements.
Position in the Periodic Table
f-block elements consist of two series Lanthanoids (Ce58 – Lu71), Actinoids (Th90 – Lr103)
These 28 elements are present in 3rd / IIIrd B group and placed separately in lower part of periodic
table.
LANTHANOIDES (Rare Earths or Lanthanones)
(i) Lanthanoides are reactive elements so do not found in free state in nature.
(ii) Most important minerals for lighter Lanthanoides are – Monazite, cerites and orthite and
for heavier lanthanoids – Gadolinite and Xenotime
(a) Electronic configuration
(i) The general configuration of lanthanoides may be given as [Xe] 4f1–145s25p65d0–16s2.
Outer electronic configuration
Atomic No. Element Symbol
Atomic +3 ion

58 Cerium Ce 4f15d16s2 4f1


59 Praseodymium Pr 4f3 6s2 4f2
60. Neodymium Nd 4f4 6s2 4f3
61. Promethium Pm 4f5 6s2 4f4
62. Samarium Sm 4f6 6s2 4f5
63. Europium Eu 4f7 6s2 4f6
64. Gadolinium Gd 4f7 5d1 6s2 4f7
65. Terbium Tb 4f9 6s2 4f8
66. Dysprosium Dy 4f10 6s2 4f9
67. Holmium Ho 4f11 6s2 4f10
68. Erbium Er 4f12 6s2 4f11
69. Thulium Tm 4f13 6s2 4f12
70. Ytterbium Yb 4f14 6s2 4f13
71. Lutecium Lu 4f14 5d1 6s2 4f14
node06\B0BC-BD\Kota\Board Material\Chemistry\Booklets\CBSE\Part-1

(ii) It is to be noted that filling of 4f orbitals in the atoms is not regular. A 5d electron appears in
gadolinium (Z = 64) with an outer electronic configuration of 4f75d16s2 (and not 4f86s2).
This is because the 4f and 5d electrons are at about the same potential energy and that the
atoms have a tendency to retain stable half filled configuration.
(iii) On the other hand, the filling of f-orbitals is regular in tripositive ions.
(iv) Pm is the only synthetic radioactive lanthanoide.

216 E
CBSE
ALLEN
(b) Oxidation states
(i) In lanthanoides +3 oxidation state is most common.
(ii) Ce, Tb, Nd, Dy, Pr also exhibit +4 oxidation state.
(iii) Sm, Eu, Tm, and Yb, also exhibit +2 oxidation state.
(iv) Ce+4 is a strong oxidising agent while Eu+2 is a strong reducing agent.
(c) Magnetic Properties
(i) In tripositive lanthanoide ions the number of unpaired electrons regularly increases from
lanthanum to Gadolinium (0 to 7) and then continuously decreases upto lutecium (7 to 0).
(ii) lanthanum and lutecium ions are diamagnetic, while all other tripositive lanthanoide ions
are paramagnetic. (Exception – Neodymium is the most paramagnetic lanthanoide).
(iii) Ce+4 and Yb+2 are also diamagnetic ions.
(d) Colour
(i) The lanthanoid ions have unpaired electrons in their 4f orbitals. Thus these ions absorbs
visible region of light and undergo f–f transition and hence exhibit colour.
(ii) The colour exhibited depends on the number of unpaired electrons in the 4f orbitals.
(iii) The ions often with 4f n configuration have similar colour to those ions having 4f 14 n
configuration.
(iv) lanthanoide ions having 4f 0 , 4f 14 are colourless.
(e) Lanthanoide Contraction
(i) In the lanthanoide series with increasing atomic number, there is a progressive decrease in
the size from lanthanum to lutecium or from La+3 to Lu+3. This contraction in size is known
as lanthanoide contraction.
(ii) The general electronic configuration of these elements is 4f1–145s2p6d0–16s2. Due to very
poor shielding effect of 4f electrons, they exert very little screening effect on the outermost
6s2 electrons.
Hence with increasing atomic number, the enhanced nuclear charge leads to contraction in
the size of atoms and ions.
APPLICATIONS OF LANTHANOIDE CONTRACTION
 Atomic size : The ionic radii of Zr+4 is about 9% more than Ti+4. Similar trend is not
maintained on passing from the second to third transition series. The ionic radius of
Hf +4, instead of increasing (because of inclusion of one more electronic shell), decreases
node06\B0BC-BD\Kota\Board Material\Chemistry\Booklets\CBSE\Part-1

(or is virtually equal to Zr +4) as a consequence of the lanthanoid contraction.


This explains the close similarities between the members of the second and third transition
series than between the elements of the first and second series.
 Basic nature of hydroxide : Basic nature of hydroxide regularly decreases.
 Similarity in properties of lanthanoide : Due to small difference in size the chemical
properties of lanthanoids are similar and their seperation is difficult.
E 217
Chemistry
ALLEN
(f) Important reactions of Lanthanoides
Ln2O3 H2

heated with S with halogens


Ln2S3 Ln LnX3

2773K
with C
LnN Ln(OH)3 + H2

LnC2, Ln3C, Ln2C3


Misch metal is an alloy of lanthanoides (Mainly cerium) and iron.
ACTINIDES OR (5f - SERIES)
(i) The elements in which the last electron enters 5f-orbitals of (n – 2)th shell are known as
actinides.
(ii) The man-made eleven elements Np93 – Lr103 are placed beyond uranium in the periodic
table and are collectively called trans-uranic elements.
(iii) Th, Pa and U first three actinides are natural elements.
(a) Electronic Configuration
The general configuration of actinides may be given as [Rn] 5f0–14 6d0,1,2,7s2.
Outer Electronic Configuration
Atomic No. Elements Symbol
Atomic +3 ion

90 Thorium Th 6d27s2 5f1


91 Proactenium Pa 5f26d17s2 5f2
92 Uranium U 5f36d17s2 5f3
93 Neptunium Np 5f46d17s2 5f4
94 Plutonium Pu 5f66d07s2 5f5
95 Americium Am 5f76d07s2 5f6
96 Curium Cm 5f76d17s2 5f7
97 Berkellium Bk 5f96d07s2 5f8
98 Californium Cf 5f106d07s2 5f9
99 Einstenium Es 5f116d07s2 5f10
100 Fermium Fm 5f126d07s2 5f11
node06\B0BC-BD\Kota\Board Material\Chemistry\Booklets\CBSE\Part-1

101 Mandelevium Md 5f136d07s2 5f12


102 Nobellium No 5f146d07s2 5f13
103 Lowrencium Lr 5f146d17s2 5f14

(b) Oxidation states


(i) In actinoids +3 oxidation state is the most common.
(ii) In actinides there is a greater range of oxidation states which is due to very less energy
difference between 5f, 6d and 7s energy levels.
218 E
CBSE
ALLEN
(iii) Highest oxidation states in the actinoids is +7 exhibited by Np93 & Pu94, it is unstable.
(iv) Highest stable oxidation state is +6 shown by 92U .
Other Properties
 Physical appearance : Acitinides are silvery white metals.
 Density : All the actinides except thorium and americium have high densities.
 Colour : Actinide ions are generally coloured. The colour of actinide ions depends upon
the number of 5f-electrons. The ions containing no unpaired 5f-electrons (exactly full filled
f-subshell) are colourless, as expected.
 Ionisation energies : Ionisation energies values of early actinides are lower than early
lanthanoids.
 Electropositivie character : All the known actinide metals are highly electropositive.
They resemble lanthanoide series in this respect.
 Melting and Boiling properties : They have high melting and boiling points. They do
not follow regular gradation of melting or boiling points with increase in atomic number.
 Magnetic properties : The actinide elements are paramagnetic due to the presence of
unpaired electrons.
 Radioactive nature : All the actinoids are radiaoactive in nature.
 Actinide contraction : The size of atom/cation decrease regularly along the actinoids
series. The steady decrease in ionic radii with increase in atomic number is referred to as
actinide contraction. This is due to poor shielding of 5f-electrons.
Comparison of Lanthanoides and Actinides
Points of Resemblance :
(i) Both lanthanoids and actinides show a common oxidation state of +3.
(ii) Both are electropositive and act as strong reducing agents.
(iii) Cations with unpaired electrons in both of them are paramagnetic.
(iv) Most of the cations of lanthanoides and actinoids are coloured.
(v) Both of them show a steady decrease in their ionic radii along the series. Thus,
lanthanoides show lanthanoide contraction and actinides show actinide contraction.
POINTS OF DISTINCTION
Lathanides Actinides
1. Besides the most common oxidation state of +3 Besides the most common oxidation state of
lanthanides show +2 and +4 oxidation states in +3, actinides show +4, +5 and +6 oxidation
case of certain elements. states in case of certain elements.
2. Lanthanides have less tendency towards Actinides have a stronger tendency towards
complex formation. complex formation.
node06\B0BC-BD\Kota\Board Material\Chemistry\Booklets\CBSE\Part-1

3. Except promethium, they are non radioactive. All the actinides are radioactive.
4. Oxides and hydroxide of lanthanides are less basic. Oxides and hydroxides of actinides are more basic

Some important uses of actinides are as follows –


Thorium : Thorium is used in atomic reactors as fuel rods and in the treatment of cancer.
Uranium : Uranium is used as nuclear fuel. Its salts are used in glass industry
(for imparting green colour). textile industry and also in medicines.
Plutonium : Plutonium is used as fuel for atomic reactors as well as in atomic bombs.

E 219
Chemistry
ALLEN
EXERCISE-1 INTEXT QUESTIONS
10
1. Silver atom has completely filled d orbitals (4d ) in its ground state. How can you say that it is a
transition element ?
10 1
Ans. Ag has a completely filled 4d orbital (4d 5s ) in its ground state. Now, silver displays two
oxidation states (+1 and +2). In the +1 oxidation state, an electron is removed from the
s-orbital. However, in the +2 oxidation state, an electron is removed from the d-orbital.
9
Thus, the d-orbital now becomes incomplete (4d ). Hence, it is a transition element.
2. In the series Sc (Z = 21) to Zn (Z = 30), the enthalpy of atomization of zinc is the lowest,
–1
i.e., 126 kJ mol . Why?
Ans. The extent of metallic bonding an element undergoes decides the enthalpy of atomization. The
more extensive the metallic bonding of an element, the more will be its enthalpy of atomization.
10 2
In all transition metals (except Zn, electronic configuration: 3d 4s ), there are some unpaired
electrons that account for their stronger metallic bonding. Due to the absence of these unpaired
electrons, the inter-atomic electronic bonding is the weakest in Zn and as a result, it has the least
enthalpy of atomization.
3. Which of the 3d series of the transition metals exhibits the largest number of oxidation states and
why ?
5 2
Ans. Mn (Z = 25) = 3d 4s
Mn has the maximum number of unpaired electrons present in the d-subshell (5 electrons).
Hence, Mn exhibits the largest number of oxidation states, ranging from +2 to +7.
2+
4. The E(M /M) value for copper is positive (+0.34V). What is possibly the reason for this?
(Hint: consider its high energy and low energy )
2+
Ans. The E(M /M) value of a metal depends on the energy changes involved in the following:
1. Sublimation : The energy required for converting one mole of an atom from the solid state
to the gaseous state.
M(s)  M(g) sH(Sublimation energy)
2. Ionization : The energy required to take out electrons from one mole of atoms in the
gaseous state.
2+
M(g)  M (g) iH(Ionization energy)
3. Hydration : The energy released when one mole of ions are hydrated.
2+ 2+
M (g)  M (aq) hydH(Hydration energy)
Now, copper has a high energy of atomization and low hydration energy.
node06\B0BC-BD\Kota\Board Material\Chemistry\Booklets\CBSE\Part-1

2+
Hence, the E(M /M) value for copper is positive.
5. How would you account for the irregular variation of ionization enthalpies (first and second) in
the first series of the transition elements?
Ans. Ionization enthalpies are found to increase in the given series due to a continuous filling of the
inner d-orbitals. The irregular variations of ionization enthalpies can be attributed to the extra
0 5 10
stability of configurations such as d , d , d . Since these states are exceptionally stable, their
ionization enthalpies are very high.

220 E
CBSE
ALLEN
In case of first ionization energy, Cr has low ionization energy. This is because after losing one
5
electron, it attains the stable configuration (3d ). On the other hand, Zn has exceptionally high
first ionization energy as an electron has to be removed from stable and fully-filled orbitals
10 2
(3d 4s ).
Second ionization energies are higher than the first since it becomes difficult to remove an
electron when an electron has already been taken out. Also, elements like Cr and Cu have
exceptionally high second ionization energies as after losing the first electron, they have attained
+ 5 + 10
the stable configuration (Cr :3d and Cu :3d ). Hence, taking out one electron more from this
stable configuration will require a lot of energy.
6. Why is the highest oxidation state of a metal exhibited in its oxide or fluoride only?
Ans. Both oxide and fluoride ions are highly electronegative and have a very small size. Due to these
properties, they are able to oxidize the metal to its highest oxidation state.
2+ 2+
7. Which is a stronger reducing agent Cr or Fe and why ?
2+ 2+
Ans. The following reactions are involved when Cr and Fe act as reducing agents.
2+
Cr  Cr3+
3+
Fe2+  Fe
The E oCr3 /Cr 2 value is –0.41 V and E oFe3 /Fe2 is +0.77 V. This means that Cr2+ can be easily
oxidized to Cr3+, but Fe2+ does not get oxidized to Fe3+ easily. Therefore, Cr2+ is a better reducing
agent that Fe3+.
8. Calculate the ‘spin only’ magnetic moment of M2aq  ion (Z = 27).

Ans. Z = 27
7 2
 [Ar] 3d 4s
2+ 7
 M = [Ar] 3d
7
3d =
 
i.e., 3 unpaired electrons
 n=3
 n(n  2)  µ  3(3  2)  µ
 15  µ µ  4 BM
+
9. Explain why Cu ion is not stable in aqueous solutions?
2+ +
Ans. In an aqueous medium, Cu is more stable than Cu . This is because although energy is required
node06\B0BC-BD\Kota\Board Material\Chemistry\Booklets\CBSE\Part-1

+ 2+ 2+
to remove one electron from Cu to Cu , high hydration energy of Cu compensates for it.
+ 2+
Therefore, Cu ion in an aqueous solution is unstable. It disproportionate to give Cu and Cu.
10. Actinoid contraction is greater from element to element than lanthanoide contraction. Why?
Ans. In actinoids, 5f orbitals are filled. These 5f orbitals have a poorer shielding effect than 4f orbitals
(in lanthanoides). Thus, the effective nuclear charge experienced by electrons in valence shells in
case of actinides is much more that that experienced by lanthanoides. Hence, the size contraction
in actinides is greater as compared to that in lanthanoides.
E 221
Chemistry
ALLEN
EXERCISE-2 NCERT EXERCISE
1. Write down the electronic configuration of :
3+ 3+ + 4+
(i) Cr (ii) Pm (iii) Cu (iv) Ce
2+ 2+ 2+ 4+
(v) Co (vi) Lu (vii) Mn (viii) Th
3+ 2 2 6 2 6 3
Ans. (i) Cr : 1s 2s 2p 3s 3p 3d
3+ 2 2 6 2 6 10 2 6 10 2 6 4
(ii) Pm : 1s 2s 2p 3s 3p 3d 4s 4p 4d 5s 5p 4f
+1 2 2 6 2 6 10
(iii) Cu :1s 2s 2p 3s 3p 3d
4+ 2 2 6 2 6 10 2 6 10 2 6
(iv) Ce : 1s 2s 2p 3s 3p 3d 4s 4p 4d 5s 5p
2+ 2 2 6 2 6 7
(v) Co : 1s 2s 2p 3s 3p 3d
2+ 2 2 6 2 6 10 2 6 10 2 6 14 1
(vi) Lu : 1s 2s 2p 3s 3p 3d 4s 4p 4d 5s 5p 4f 5d
2+ 2 2 6 2 6 5
(vii) Mn :1s 2s 2p 3s 3p 3d
4+ 2 2 6 2 6 10 2 6 10 14 2 6 10 2 6
(viii) Th : 1s 2s 2p 3s 3p 3d 4s 4p 4d 4f 5s 5p 5d 6s 6p
2+ 2+
2. Why are Mn compounds more stable than Fe towards oxidation to their +3 state ?
2+ 18 5
Ans. Electronic configuration of Mn is [Ar] 3d . (half filled stability)
2+ 18 6
Electronic configuration of Fe is [Ar] 3d . (After losing one electron gain half filled stability).
3. Explain briefly how +2 state becomes more and more stable in the first half of the first row
transition elements with increasing atomic number ?
Ans. The oxidation states displayed by the first half of the first row of tranition metals are given in the
table below.
Oxidation state
Sc Ti V Cr Mn
+2 +2 +2 +2
+3 +3 +3 +3 +3
+4 +4 +4 +4
+5 +5 +6
+6 +7
It can be easily observed that except Sc, all others metals display +2 oxidation state. Also, on
moving from Sc to Mn, the atomic number increases from 21 to 25. This means the number of
electron in the 3d-orbital also increase from 1 to 5.
node06\B0BC-BD\Kota\Board Material\Chemistry\Booklets\CBSE\Part-1

1
Sc (+2) = d
2
Ti (+2) = d
3
V (+2) = d
4
Cr (+2) = d
5
Mn (+2) = d

222 E
CBSE
ALLEN
+2 oxidation state is attained by the loss of the two 4s electrons by these metals. Since the
number of d electrons in (+2) state also increase from Ti(+2) to Mn(+2), the stability of +2 state
5
increases (as d-orbital is becoming more and more half-filled). Mn(+2) has d electrons
(that is half-filled d shell, which is highly stable).
4. To what extent do the electronic configurations decide the stability of oxidation states in the first
series of the transition elements ? Illustrate your answer with examples.
Ans. The elements in the first - half of the transition series exhibit many oxidation states with Mn
exhibiting maximum number of oxidation states (+2 to +7). The stability of +2 oxidation state
increases with the increase in atomic number. This happens as more electrons are getting filled in
the d-orbital.
5. What may be the stable oxidation state of the transition element with the following d electron
3 5 8 4
configurations in the ground state of their atoms : 3d , 3d , 3d and 3d ?
Ans. Electronic configuration in
Stable oxidation states 
ground state
(i) 3d3 (Vanadium) +2, +3, +4 and +5
(ii) 3d5 (Chromium) +3, +4, +6
(iii) 3d3 (Manganese) +2, +4, +6, +7
(iv) 3d8 (Cobalt) +2, +3
(v) 3d4 There is no 3 deconfiguration
in ground state
6. Name the oxometal anions of the first series of the transition metals in which the metal exhibits
the oxidation state equal to its group number.
Ans. (i) Vanadate, VO 3 Oxidation state of V is +5.

(ii) Chromate. CrO24  Oxidation state of Cr is +6

(iii) Permanganate, MnO 4 Oxidation state of Mn is +7.


7. What is lanthanoid contraction ? What are the consequences of lanthanoid contraction ?
Ans. As we move along the lanthanoid series, the atomic number increases gradually by one. This
means that the number of electrons and protons present in an atom also increases by one. As
electrons are being added to the same shell, the effective nuclear charge increases. This happens
node06\B0BC-BD\Kota\Board Material\Chemistry\Booklets\CBSE\Part-1

because the increase in nuclear attraction due to the addition of proton is more pronounced than
the increase in the interelectronic repulsions due to the addition of electron. Also, with the
increase in atomic number, the number of electrons in the 4f orbital also increases. The 4f
electrons have poor shielding effect. Therefore, the effective nuclear charge experienced by the
outer electrons increases. Consequently, the attraction of the nucleus for the outermost electrons
increases. This results in a steady decrease in the size of lanthanoids with the increase in the
atomic number. This is termed as lanthanoid contraction.

E 223
Chemistry
ALLEN
Consequences of lanthanoid contraction
(i) There is similarity in the properties of second and third transition series.
(ii) Separation of lanthanoids is possible due to lanthanoide contraction.
(iii) It is due to lanthanoide contraction that there is variation in the basic strength of
lanthanoide hydroxides. (Basic strength decreases from La (OH)3 to Lu (OH)3.)
8. What are the characteristics of the transition elements and why are they called transition
elements? Which of the d-block elements may not be regarded as the transition elements?
Ans. Transition elements are those elements in which the atoms or ions (in stable oxidation state)
contain partially filled d-orbital. These elements lie in the d-block and show a transition of
properties between s-block and p-block. Therefore, these are called transition elements.
Elements such as Zn, Cd and Hg cannot be classified as transition elements because these have
completely filled d-subshell.
9. In what way is the electronic configuration of the transition elements different from that of the
non-transition elements?
Ans. Transition metals have a partially filled d-orbital. Therefore, the electronic configuration of
1–10 0–2
transition elements is (n – 1) d ns .
The non-transition elements either do not have a d-orbital or have a fully filled d-orbital.
1–2 2 1–6
therefore, the electronic configuration of non-transition elements is ns or ns np .
10. What are the different oxidation states exhibited by the lanthanoids?
Ans. In the lanthanoide series, +3 oxidation state is most common i.e., Ln (III) compounds are
predominant. However, +2 and +4 oxidation states can also be found in the solution or in solid
compounds.
11. Explain giving reasons :
(i) Transition metals and many of their compounds show paramagnetic behaviour.
(ii) The enthalpies of atomisation of the transition metals are high.
(iii) The transition metals generally form coloured compounds.
(iv) Transition metals and their many compounds act as good catalyst.
Ans. (i) Transition metals show paramagnetic behaviour. Paramagnetism arises due to the presence
of unpaired electrons with each electron having a magnetic moment associated with its spin
angular momentum and orbital angular momentum.
(ii) Transition elements have high effective nuclear charge and a large number of valence
electrons. Therefore, they form very strong metallic bonds. As a result, the enthalpy of
atomization of transition metals is high.
node06\B0BC-BD\Kota\Board Material\Chemistry\Booklets\CBSE\Part-1

(iii) Most of the complexes of transition metals are coloured. This is because of the absorption
of radiation from visible light region to promote an electron from one of the d-orbitals to
another. In the presence of ligands, the d-orbitals split up into two sets of orbitals having
different energies. Therefore, the transition of electrons can take place from one set to
another. The energy required for these transitions is quite small and falls in the visible
region of radiation. The ions of transition metals absorb the radiation of a particular
wavelength and the rest is reflected, imparting colour to the solution.
224 E
CBSE
ALLEN
(iv) The catalytic activity of the transition elements can be explained by two basic facts.
(a) Owing to their ability to show variable oxidation states and form complexes,
transition metals form unstable intermediate compounds. Thus, they provide a new
path with lower activation energy, Ea, for the reaction.
(b) Transition metals also provide a suitable surface for the reactions to occur.
12. What are interstitial compounds? Why are such compounds well known for transition metals?
Ans. Transition metals are large in size and contain lots of interstitial sites. Transition elements can
trap atoms of other elements (that have small atomic size), such as H, C, N, in the interstitial sites
of their crystal lattices. The resulting compounds are called interstitial compounds.
13. How is the variability in oxidation states of transition metals different from that of the non-
transition metals ? Illustrate with examples.
Ans. In transition elements, the oxidation state can vary from +1 to the highest oxidation state by
removing all its valence electrons. Also, in transition elements, the oxidation states differ by
2+ 3+ + 2+
1 (Fe and Fe ; Cu and Cu ). In non-transition elements, the oxidation states differ by 2, for
example, +2 and +4 or + 3 and +5, etc.
14. Describe the preparation of potassium dichromate from iron chromite ore. What is the effect of
increasing pH on a solution of potassium dichromate ?
Ans. Potassium dichromate is prepared from chromite ore (FeCr2O4) in the following steps.
Step (1) : Preparation of sodium chromate
4FeCr2O4 + 16NaOH + 7O2 8Na2CrO4 + 2Fe2O3 + 8H2O
Step (2) : Conversion of sodium chromate into sodium dichromate
2Na2CrO4 + conc. H2SO4 Na2Cr2O7 + Na2SO4 + H2O
Step (3) : Conversion of sodium dichromate to potassium dichromate
Na2Cr2O7 + 2KCl  K2Cr2O7 + 2NaCl
Potassium dichromate being less soluble than sodium dichromate is obtained in the form of
orange coloured crystals and can be removed by filtration.
The dichromate ion ( Cr2 O 27  ) exists in equilibrium with chromate ( CrO 24 ) ion at pH 4. However,
by changing the pH, they can be interconverted
2CrO24  2H  
pH  4
 Cr2 O 27   H2O
Chromate ion(Yellow) Dichromate ion(Orange)

On increasing pH, Cr2 O 27   2OH  


 2CrO 24   H 2 O
15. Describe the oxidation action of potassium dichromate and write the ionic equations for its
node06\B0BC-BD\Kota\Board Material\Chemistry\Booklets\CBSE\Part-1

reaction with :
(i) iodide (ii) iron(II) solution and (iii) H2S
Ans. K2Cr2O7 acts as a very strong oxidising agent in the acidic medium.
K2Cr2O7 + 4H2SO4 K2SO4 + Cr2 (SO4)3 + 4H2O + 3[O]
K2Cr2O7 takes up electrons to get reduced and acts as an oxidising agent. The reaction of
K2Cr2O7 with iodide ion, iron (II) solution and H2S are given below.

E 225
Chemistry
ALLEN
(i) K2Cr2O7 oxidizes iodide to iodine.
+ – 3+
Cr 2 O 27  + 14H + 6e 2Cr + 7H2O
– –
[2I  I2 + 2e ] × 3
– + 3+
Cr 2 O 27  + 6I + 14H 2Cr + 3I2 + 7 H2O
(ii) K2Cr2O7 oxidizes iron (II) solution to iron (III) solution i.e., ferrous ions to ferric ions.
+ – 3+
Cr 2 O 27  + 14H + 6e 2Cr + 7H2O
2+ 3+ –
[Fe  Fe + e ] × 6
+ 2+ 3+
Cr 2 O 27  + 14H + 6Fe  2Cr + 3I2 + 7H2O
(iii) K2Cr2O7 oxidizes H2S to sulphur.
+ – 3+
Cr 2 O 27  + 14H + 6e 2Cr + 7H2O
+ –
[H2S  S + 2H + 2e ] × 3
+ 3+
Cr 2 O 27  + 3H2S + 8H  2Cr + 3S + 7H2O
16. Describe the preparation of potassium permanganate. How does the acidified permanganate
solution react with (i) iron(II) ions (ii) SO2 (iii) oxalic acid ?
Write the ionic equations for the reactions.
Ans. Potassium permanganate can be prepared from pyrolusite (MnO2). The ore is fused with KOH in
the presence of either atmospheric oxygen or an oxidising agent, such as KNO3 or KClO4, to give
K2MnO4.
2MnO 2  4KOH  O2 
heat
 2K 2 MnO 4  2H 2 O
 Green 
The green mass can be extracted with water and then oxidized either electrolytically or by
passing chlorine/ozone into the solution.
Electrolytic oxidation
+
K2MnO4  2K + MnO 24 
+ –
H2O  H + OH
At anode, manganate ions are oxidized to permanganate ions.
MnO 24   MnO 4  e 
Green Purple
node06\B0BC-BD\Kota\Board Material\Chemistry\Booklets\CBSE\Part-1

Oxidation by chlorine
2K2MnO4 + Cl2  2KMnO4 + 2KCl
2– – –
2MnO4 + Cl2  2MnO4 + 2Cl
Oxidation by ozone
2K2MnO4 + O3 + H2O  2KMnO4 + 2KOH + O2
2– –
2 MnO4 + O3 + H2O  2MnO 24  + 2OH +O2

226 E
CBSE
ALLEN
(i) Acidified KMnO4 solution oxidizes Fe (II) ions to Fe (III) ions i.e., ferrous to ferric ions.
+ – 2+
MnO 4 + 8H + 5e  Mn + 4H2O
2+ 3+ –
[Fe  Fe + e ] × 5
2+ + 2+ 3+
MnO 4 + 5Fe + 8H  Mn + 5Fe + 4H2O
(ii) Acidified potassium permanganate oxidizes SO2 to sulphuric acid.
+ – 2+
MnO 4 + 6H + 5e  Mn + 3H2O] × 2
+ –
2H2O + 2SO2 + O2  4H + 2SO24  + 2e ] × 5
2+ +
2MnO 4 + 10SO2 + 5O2 + 4H2O  2Mn + 10 SO 24  + 8H
(iii) Acidified potassium permanganate oxidizes oxalic acid to carbon dioxide.
+ – 2+
[MnO4 + 8H + 5e  Mn + 4H2O] × 2
2– –
[C2O4 2CO2 + 2e ] × 5
+ 2+
2MnO 4 + 5 C 2 O 27  + 16H  2Mn + 10CO2 + 8H2O
2+ 3+ 2+
17. For M /M and M /M systems, the E values for some metals are as follows :
2+
Cr /Cr – 0.9 V
3 2+
Cr /Cr – 0.4V
2+
Mn /Mn – 1.2V
3+ 2+
Mn /Mn + 1.5 V
2+
Fe /Fe – 0.4 V
3+ 2+
Fe /Fe + 0.8V
Use this data to comment upon :
3+ 3+ 3+
(i) The stability of Fe in acid solution as compared to that of Cr and Mn and
(ii) The ease with which iron can be oxidised as compared to a similar process for either
chromium or manganese metal.
3+ 2+ 3+ 2+
Ans. (i) The E value for Fe /Fe is higher than that for Cr /Cr and lower than that for
3+ 2+ 3+ 2+ 3+ 2+
Mn /Mn . So, the reduction of Fe to Fe is easier than the reduction of Cr to Cr , but
3+ 2+ 3+ 3+
not as easy as the reduction of Mn to Mn . Hence, Fe is more stable than Mn , but less
3+
stable than Cr . These metal ions can be arranged in the increasing order of their stability
3+ 3+ 3+
node06\B0BC-BD\Kota\Board Material\Chemistry\Booklets\CBSE\Part-1

as: Mn < Fe < Cr


(ii) The reduction potentials for the given pairs increase in the following order.
2+ 2+ 2+
Mn /Mn<Cr /Cr < Fe /Fe
2+ 2+
So, the oxidation of Fe to Fe is not a easy as the oxidation of Cr to Cr and the oxidation
2+
of Mn to Mn . Thus these metals can be arranged in the increasing order of their ability to
get oxidised : Fe < Cr < Mn

E 227
Chemistry
ALLEN
3+ 3+ + 3+ 2+
18. Predict which of the following will be coloured in aqueous solution? Ti , V , Cu , Sc , Mn ,
3+ 2+
Fe and Co . Give reasons for each.
Ans. Only the ions that have unpaired electrons in d-orbital will be coloured. The ions in which
d-orbital is empty or fully filled will be colourless.
Electronic Configuration
Element Atomic Number Ionic State
in ionic state
Ti 22 Ti3+ [Ar]3d2
V 23 V3+ [Ar]3d2
Cu 29 Cu3+ [Ar]3d10
Sc 21 Sc3+ [Ar]
Mn 25 Mn3+ [Ar]3d5
Fe 26 Fe3+ [Ar]3d5
Co 27 Co3+ [Ar]3d7
19. Compare the stability of +2 oxidation state for the elements of the first transition series.
Ans. Sc +3
Ti +1 +2 +3 +4
V +1 +2 +3 +4 +5
Cr +1 +2 +3 +4 +5 +6
Mn +1 +2 +3 +4 +5 +6 +7
Fe +1 +2 +3 +4 +5 +6
Co +1 +2 +3 +4 +5
Ni +1 +2 +3 +4
Cu +1 +2
Zn +2

The relative stability of the +2 oxidation state increases on moving from top to bottom. This is
because on moving from top to bottom, it become more and more difficult to remove the third
electron from the d-orbital.
20. Compare the chemistry of actinoids with that of the lanthanoids with special reference to :
(i) electronic configuration (ii) atomic and ionic sizes
(iii) oxidation state (iv) chemical reactivity.
54
Ans. (i) Electronic configuration : The general electronic configuration for lanthanoids is [Xe]
1-14 0-1 2 86 1-14 0-1 2
4f 5d 6s and that for actinoids is [Rn] 5f 6d 7s .
node06\B0BC-BD\Kota\Board Material\Chemistry\Booklets\CBSE\Part-1

(ii) Atomic and Ionic sizes : Similar to lanthanoids, actinoids also exhibit actinoid contraction
(overall decrease in atomic and ionic radii). The contraction is greater due to the poor
shielding effect of 5f orbitals.
(iii) Oxidation states : The principal oxidation state of lanthanoids is (+3). However,
sometimes we also encounter oxidation states of +2 and +4. This is because of extra
stability of fully-filled and half-filled orbitals. Actinoids exhibit a greater range of
oxidation states. This is because the 5f, 6d, and 7s levels are of comparable energies.
228 E
CBSE
ALLEN
(iv) Chemical reactivity : In the lanthanoide series, the earlier members of the series are more
reactive. They have reactivity that is comparable to Ca. With an increase in the atomic
number, the lanthanoides start behaving similar to Al. Actinoids, on the other hand, are
highly reactive metals, especially when they are finely divided.
21. How would you account for the following :
4 2+
(i) Of the d species, Cr is strongly reducing while manganese(III) is strongly oxidising.
(ii) Cobalt(II) is stable in aqueous solution but in the presence of complexing reagents it is
easily oxidised.
1
(iii) The d configuration is very unstable in ions.
2+ 4
Ans. (i) Cr is strongly reducing in nature. It has a d configuration. While acting as a reducting
3+ 3 3
agent, it gets oxidized to Cr (electronic configuration, d ). This d configuration can be
3+ 4
written as t 32g configuration, which is a more stable configuration. In the case of Mn (d ),
2+ 5
it acts as an oxidizing agent and gets reduced to Mn (d ). This has an exactly half- filled
d-orbital and is highly stable.
(ii) Co(II) is stable is aqueous solutions. However, in the presence of strong field complexing
rd
reagents, it is oxidized to Co(III). Although the 3 ionization energy for Co is high, but the
higher amount of crystal field stabilization energy (CFSE) released in the presence of
strong field ligands overcomes this ionization energy.
1 0
(iii) The ions in d configuration tend to lose one more electron to get into stable d
configuration. Also, the hydration or lattice energy is more than sufficient to remove the
only electron present in the d-orbital of these ions. Therefore, they act as reducing agents.
22. What is meant by 'disproportionation' ? Give two examples of disproportionation reaction in
aqueous solution.
Ans. It is found that sometimes a relatively less stable oxidation state undergoes an oxidation -
reduction reaction in which it is simultaneously oxidised and reduced. This is called
disproportionation.
For example,
(1) 3CrO34  8H  
 2CrO24  Cr 3  4H 2 O
Cr(V) Cr(VI) Cr(III)

Cr(V) is oxidized to Cr(VI) and reduced to Cr(III).


(2) 3MnO24  4H  
 2MnO 4  MnO2  2H 2 O
Mn(VI) Mn(VII) Mn(IV)
node06\B0BC-BD\Kota\Board Material\Chemistry\Booklets\CBSE\Part-1

Mn (VI) is oxidized to Mn (VII) and reduced to Mn (IV).


23. Which metal in the first series of transition metals exhibits +1 oxidation state most frequently and
why?
Ans. In the first transition series, Cu exhibits +1 oxidation state very frequently. It is because Cu (+1)
10
has an electronic configuration of [Ar] 3d . The completely filled d-orbital makes it highly
stable.

E 229
Chemistry
ALLEN
3+ 3+ 3+
24. Calculate the number of unpaired electrons in the following gaseous ions: Mn , Cr , V and
3+
Ti . Which one of these is the most stable in aqueous solution ?
Ans. Gaseous ions Number of uppaired electrons
3+ 4
(i) Mn , [Ar]3d 4
(ii) Cr3+, [Ar] 3d3 3
(iii) V3+, [Ar]3d2 2
(iv) Ti3+[Ar] 3d1 1
3+
Cr is the most stable in aqueous solutions owing to a t 32g half filled configuraion.
25. Give examples and suggest reasons for the following features of the tranistion metal chemistry :
(i) The lowest oxide of transition metal is basic, the highest is amphoteric/acidic.
(ii) A transition metal exhibits highest oxidation state in oxides and fluorides.
(iii) The highest oxidation state is exhibited in oxoanions of a metal.
Ans. (i) In the case of a lower oxide of a transition metal, the metal atom has a low oxidation state.
This means that some of the valence electrons of the metal atom are not involved in
bonding. As a result, it can donate electrons and behave as a base.
On the other hand, in the case of a higher oxide of a transition metal, the metal atom has a
high oxidation state. This means that the valence electrons are involved in bonding and so,
they are unavailable. There is also a high effective nuclear charge.
As a result, it can accept electrons and behave as an acid.
II
For example, Mn O is basic and Mn 2VII O 7 is acidic.
(ii) Oxygen and fluorine act as strong oxidising agents because of their high electronegatives
and small sizes. Hence, they bring out the highest oxidation states from the transition
metals. In other words, a transition metal exhibits higher oxidation states in oxides and
fluorides. For example, in OsF6 and V2O5, the oxidation states of Os and V are +6 and +5
respectively.
(iii) Oxygen is a strong oxidising agent due to its high electronegativity and small size. So, oxo-
anions of a metal have the highest oxidation state. For example, in MnO 4 , the oxidation
state of Mn is +7.
26. What are alloys? Name an important alloy which contains some of the lanthanoid metals.
Mention its uses.
Ans. An alloy is a solid of two or more elements in a metallic matrix. It can either be a partial solid
solution or a complete solid solution. Alloys are usually found to possess different physical
properties than those of the component elements.
node06\B0BC-BD\Kota\Board Material\Chemistry\Booklets\CBSE\Part-1

An important alloy of lathanoids is Mischmetal. It contains lanthanoids (94-95%), iron (5%), and
traces of S, C, Si, Ca and Al.
Uses:
(1) Mischmetal is used in cigarettes and gas lighters.
(2) It is used in flame throwing tanks.
(3) It is used in tracer bullets and shells.

230 E
CBSE
ALLEN
27. What are inner transition elements ? Decide which of the following atomic numbers are the
atomic numbers of the inner transition elements : 29, 59, 74, 95, 102, 104.
Ans. Inner transition elements are those elements in which the last electron enters the f-orbital .The
elements in which the 4f and and 5f orbitals are progressively filled are called f-block elements.
Among the given atomic numbers, the atomic numbers of the inner transition elements are 59, 95
and 102.
28. The chemistry of the actinoid elements is not so smooth as that of the Lanthanoids. Justify this
statement by giving some examples from the oxidation state of these elements.
Ans. Lanthanoids primarily show three oxidation states (+2, +3, +4). Among these oxidation states,
+3 state is the most common. Lanthanoids display a limited number of oxidation states because
the energy difference between 4f, 5d and 6s orbitals is quite large. On the other hand, the energy
difference between 5f, 6d, and 7s orbitals is very less. Hence actinoids display a large number of
oxidation states. For example, uranium and plutonium display +3, +4, +5, and +7. The most
common oxidation state in case of actionoids is also +3.
29. Which is the last elements in the series of the actinoids ? Write the electronic configuration of
this element. Comment on the possible oxidation state of this element.
Ans. The last element in the actinoid series is lawrencium, Lr. its atomic number is 103 and its
14 1 2
electronic configuration is [Rn] 5f 6d 7s . The most common oxidation state displayed by it is
14
+3; because after losing 3 electrons it attains stable f configuration.
3+
30. Use Hund's rule to derive the electronic configuration of Ce ion and calculate its magnetic
moment on the basis of 'spin - only' formula.
3+ 2 2 6 2 6 10 2 6 10 2 6 1
Ans. Ce : 1s 2s 2p 3s 3p 3d 4s 4p 4d 5s 5p 4f
Magnetic moment can be calculated as :
  n n  2
Where, n = number of unpaired electrons.
3+
In Ce , n = 1
Therefore,   1 1  2   3 = 1.732 BM
31. Name the members of the lanthanoid series which exhibit +4 oxidation state and those which
exhibit +2 oxidation state. Try to correlate this type of behaviour with the electronic
configuration of these elements.
Ans. The lanthanoides that exhibit + 2 and +4 states are shown in the given table . The atomic number
of these elements are given in the parenthesis.
+2 +4
node06\B0BC-BD\Kota\Board Material\Chemistry\Booklets\CBSE\Part-1

Nd (60) Ce (58)
Sm (62) Pr (59)
Eu (63) Nd (60)
Tm (69) Tb (65)
Yb (70) Dy (66)

E 231
Chemistry
ALLEN
4+
Ce after forming Ce attains a stable electronic configuration of [Xe].
4+ 7
Tb after forming Tb attains a stable electronic configuration of [Xe] 4f
2+ 7
Eu after forming Eu attains a stable electronic configuration of [Xe] 4f
2+ 14
Yb after forming Yb attains a stable electronic configuration of [Xe] 4f
32. Write the electronic configuration of the elements with the atomic numbers 61, 91, 101, and 109.
Ans. Atomic Number Electronic Configuration
61 [Xe]54 4f5 5d0 6s2
91 [Rn]86 5f2 6d1 7s2
101 [Rn]86 5f13 5d0 7s2
109 [Rn]86 5f14 6d7 7s2
33. Compare the general characteristics of the first series of the transition metals with those of the
second and third series metals in the respective vertical columns. Give special emphasis on the
following points :
(i) electronic configurations,
(ii) oxidation states,
(iii) ionisation enthalpies, and
(iv) atomic sizes.
st nd rd
Ans. (i) In the 1 , 2 and 3 transition series, the 3d, 4d and 5d orbitals are respectively filled.
We know that elements in the same vertical column generally have similar electronic
configurations.
In the first transition series, two elements show unusual electronic configurations :
5 1
Cr(24) = 3d 4s
10 1
Cu(29) = 3d 4s
Similarly, there are exceptions in the second transition series. These are :
4 1
Nb(41) = 4d 5s
5 1
Mo(42) = 4d 5s
6 1
Tc (43) = 4d 5s
7 1
Ru(44) = 4d 5s
8 1
Rh(45) = 4d 5s
10 0
Pd(46) = 4d 5s
node06\B0BC-BD\Kota\Board Material\Chemistry\Booklets\CBSE\Part-1

10 1
Ag(47) = 4d 5s
There are some exceptions in the third transition series as well. These are :
9 1
Pt(78) = 5d 6s
10 1
Au(79) = 5d 6s
As a result of these exceptions, it happens many times that the electronic configurations of
the elements present in the same group are dissimilar.
232 E
CBSE
ALLEN
(ii) In each of the three transition series the number of oxidation states shown by the elements
is the maximum in the middle and the minimum at the extreme ends.
However, +2 and +3 oxidation states are quite stable for all elements present in the first
transition series. All metals present in the first transition series form stable compounds in
the +2 and +3 oxidation states. The stability of the +2 and +3 oxidation states decreases in
the second and the third transition series, wherein higher oxidation states are more
important.
4 3
 II   III  3
For example  Fe  CN 6  , Co  NH 3 6  ,  Ti  H 2 O 6  are stable complexes, but no
   
such complexes are known for the second and third transition series such as Mo, W, Rh, In.
They form complexes in which their oxidation states are high. For example : WCl6, ReF7,
RuO4, etc.
(iii) In each of the transition series, the first ionisation enthalpy increases from left to right.
However, there are some exceptions. The first ionisation enthalpies of the third transition
series are higher than those of the first and second transition series. This occurs due to the
poor shielding effect of 4f electrons in the third transition series.
Certain elements in the second transition series have higher first ionisation enthalpies than
elements corresponding to the same vertical column in the first transition series. There are
nd
also elements in the 2 transition series whose first ionisation enthalpies are lower than
st
those of the elements corresponding to the same vertical column in the 1 transition series.
(iv) Atomic size generally decreases from left to right across a period. Now, among the three
transition series, atomic sizes of the elements in the second transition series are greater than
those of the element corresponding to the same vertical column in the first transition series.
However, the atomic sizes of the element in the third transition series are virtually the same
as those of the corresponding members in the second transition series. This is due to
lanthanoid contraction.
34. Write down the number of 3d electrons in each of the following ions:
2+ 2+ 3+ 2+ 2+ 3+ 2+ 2+ 2+
Ti , V , Cr , Mn , Fe , Fe , Co , Ni and Cu
Indicate how would you expect the five 3d orbitals to be occupied for these hydrated ions
(octahedral).
Ans. Metal ion Number of d-electrons Filling of d-orbitals
Ti2+ 2 t 22g

V2+ 3 t 32g

Cr3+ 3 t 32g

Mn2+ 5 t 32g e2g


node06\B0BC-BD\Kota\Board Material\Chemistry\Booklets\CBSE\Part-1

4 2
Fe2+ 6 t 2g eg

Fe3+ 5 t 32g e2g

Co2+ 7 t 52g e2g

Ni2+ 8 t 62g e2g

Cu2+ 9 t 62g e3g

E 233
Chemistry
ALLEN
35. What can be inferred from the magnetic moment values of the following complex species?
Example Magnetic Moment (BM)
K4[Mn(CN)6] 2.2
2+
[Fe(H2O)6] 5.3

K2[MnCl4] 5.9

Ans. Magnetic moment () is given as   n  n  2  BM

For value n = 1,   1 1  2   3  1.732 BM

For value n = 2,   2  2  2   8  2.83 BM

For value n = 3,   3  3  2   15  3.87 BM

For value n = 4,   4  4  2   24  4.899 BM

For value n = 5,   5  5  2   35  5.92 BM

(i) K4[Mn(CN)6]
For transition metals, the magnetic moment is calculated from the spin - only formula.
Therefore,
= n  n + 2  = 2.2 BM
We can see from the above calculation that the given value is closest to n = 1. Also, in this
complex, Mn in in the +2 oxidation state. This means that Mn has 5 electrons in the
d-orbital.

Hence, we can say that CN is a strong field ligand that cause the pairing of electrons.
(ii) [Fe(H2O)6]2+
= n  n + 2  = 5.3 BM
We can see from the above calculation that the given value is closest to n = 4, Also, in this
complex, Fe is in the +2 oxidation state. This means that Fe has 6 electrons in the
d-orbital.
Hence, we can say that H2O is a weak field ligand and does not cause the pairing of
electrons.
(iii) K2[MnCl4]
node06\B0BC-BD\Kota\Board Material\Chemistry\Booklets\CBSE\Part-1

= n  n + 2  = 5.9 BM
We can see from the above calculation that the given value is closest to n = 5. Also, in this
complex, Mn is in the +2 oxidation state. This means that Mn has 5 electrons in the
d-orbital.

Hence, we can say that Cl is a weak field ligand and does not cause the pairing of
electrons.
234 E
CBSE
ALLEN
EXERCISE-3 EXEMPLAR
1. Electronic configuration of a transition element X in +3 oxidation state is [Ar]3d5. What is its
atomic number ?
(a) 25 (b) 26 (c) 27 (d) 24
Ans. (b)
2. The electronic configuration of Cu(II) is 3d9 where as that of Cu(I) is 3d10. Which of the
following is correct ?
(a) Cu(II) is more stable
(b) Cu(II) is less stable
(c) Cu(I) and Cu(II) are equally stable
(d) Stability of Cu(I) and Cu(II) depends on nature of copper salts
Ans. (a)
3. Metallic radii of some transition elements are given below. Which of these elements will have
highest density ?
Element Fe Co Ni Cu
Metallic radii/pm 126 125 125 128
(a) Fe (b) Ni (c) Co (d) Cu
Ans. (d)
4. On addition of small amount of KMnO4 to concentrated H2SO4, a green oily compound is
obtained which is highly explosive in nature. Identify the compound from the following :
(a) Mn2O7 (b) MnO2 (c) MnSO4 (d) Mn2O3
Ans. (a)
5. Which of the following oxidation state is common for all lanthanoids ?
(a) +2 (b) +3 (c) +4 (d) +5
Ans. (b)
6. Which of the following reactions are disproportionation reactions ?
(i) Cu+  Cu2+ + Cu
(ii) 3MnO 4  4H   2MnO 4  MnO 2  2H 2O
(iii) 2KMnO4  K2MnO4 + MnO2 + O2
(iv) 2MnO 4  3Mn 2  2H 2O  5MnO2 + 4H+
(a) (i) (b) (i), (ii) and (iii)
(c) (ii), (iii) and (iv) (d) (i) and (iv)
Ans. (a)
7. When KMnO4 solution is added to oxalic acid solution, the decolourisation is slow in the
beginning but becomes instantaneous after some time because :
(a) CO2 is formed as the product (b) reaction is exothermic

(c) MnO 4 catalyses the reaction (d) Mn2+ acts as autocatalyst
Ans. (d)
8. There are 14 elements in actinoid series, Which of the following elements does not belong to this
node06\B0BC-BD\Kota\Board Material\Chemistry\Booklets\CBSE\Part-1

series ?
(a) U (b) Np (c) Tm (d) Fm
Ans. (c)
9. KMnO4 acts as an oxidising agent in acidic medium. The number of moles of KMnO4 that wil be
needed to react with one mole of sulphide ions in acidic solution is :
2 3 4 1
(a) (b) (c) (d)
5 5 5 5
Ans. (a)
E 235
Chemistry
ALLEN
10. Which of the following is amphoteric oxide ?
Mn2O7, CrO3, Cr2O3, CrO, V2O5, V2O4
(a) V2O5, Cr2O3 (b) Mn2O7, CrO3 (c) CrO, V2O5 (d) V2O5, V2O4
Ans. (a)
11. Gadolinium belongs to 4f series. Its atomic number is 64. Which of the following is the correct
electronic configuration of gadolinium ?
(a) [Xe]4f75d16s2 (b) [Xe]4f65d26s2
(c) [Xe]4f86d2 (d) [Xe]4f95s1
Ans. (a)
12. The magnetic moment is associated with its spin angular momentum and orbital angular
momentum. Spin only magnetic moment value of Cr3+ ion is:
(a) 2.87 BM (b) 3.87 BM (c) 3.47 BM (d) 3.57 BM
Ans. (b)
13. KMnO4 acts as an oxidising agent in alkaline medium. When alkaline KMnO4 is treated with KI,
iodide ion is oxidised to ………….
(a) I2 (b) IO– (c) IO3 (d) IO 4
Ans. (c)
14. Which of the following statements is not correct ?
(a) Copper liberates hydrogen from acids
(b) In its higher oxidation states, manganese forms stable compounds with oxygen and fluorine
(c) Mn3+ and Co3+ are oxidising agents in aqueous solution
(d) Ti2+ and Cr2+ are reducing agents in aqueous solution
Ans. (a)
15. Highest oxidation state of manganese in fluoride is +4 (MnF4) but highest oxidation state in
oxides is +7 (Mn2O7) because :
(a) fluorine is more electronegative than oxygen
(b) fluorine does not possess d orbitals
(c) fluorine stabilises lower oxidation state
(d) in covalent compounds, fluorine can form single bond only while oxygen forms double bond
Ans. (d)
16. Although zirconium belongs to 4d transition series and hafnium to 5d transition series even then
they show similar physical and chemical properties because ………….
(a) both belong to d-block
(b) both have same number of electrons
(c) both have similar atomic radius
(d) both belong to the same group of the Periodic Table
Ans. (c)
node06\B0BC-BD\Kota\Board Material\Chemistry\Booklets\CBSE\Part-1

17. Why is HCl not used to make the medium acidic in oxidation reactions of KMnO4 in acidic
medium ?
(a) Both HCl and KMnO4 act as oxidising agent
(b) KMnO4 oxidises HCl into Cl2 which is also an oxidising agent
(c) KMnO4 is a weaker oxidising agent than HCl
(d) KMnO4 acts as a reducing agent in the presence of HCl
Ans. (b)
236 E
CBSE
ALLEN
(MATCHING THE COLUMNS)
18. Match the catalysts given in Column I with the processes given in Column II.
Column-I Column-II
(Catalyst) (Process)
(A) Ni in the presence of hydrogen (1) Ziegler-Natta catalyst
(B) Cu2Cl2 (2) Contact process
(C) V2O5 (3) Vegetable oil to ghee
(D) Finely divided iron (4) Sandmeyer reaction
(5) Haber’s process
(6) Decomposition of KClO3
Ans. A-3, B-4, C-02, D-5, E-1
19. Match the compounds/elements given in Column I with uses given in Column II.
Column-I Column-II
(Compound/element) (Use)
(A) Lanthanoid oxide (1) Production of iron alloy
(B) Lanthanoid (2) Television screen
(C) Misch metal (3) Petroleum cracking
(D) Magnesium based alloy is (4) Lanthanoid metal + iron
constituent of
(E) Mixed oxides of lanthanoids are (5) Bullets
employed
(6) X-ray screen
Ans. A-2, B-1, C-4, D-5, E-3
20. Match the properties given in Column-I with the metals given in Column-II.
Column-I Column-II
(Property) (Metal)
(A) An element which can show +8 (1) Mn
oxidation state
node06\B0BC-BD\Kota\Board Material\Chemistry\Booklets\CBSE\Part-1

(B) 3d block element that can show (2) Cr


upto +7 oxidation state
(C) 3d block element with highest (3) Os
melting point
(4) Fe
Ans. A-3, B-1, C-2
E 237
Chemistry
ALLEN
21. Match the statements given in Column-I with the oxidation states given in Column-II.
Column-I Column-II
(A) Oxidation state of Mn in MnO2 (1) +2
is
(B) Most stable oxidation state of (2) +3
Mn is
(C) Most stable oxidation state of (3) +4
Mn in oxides is
(D) Characteristic oxidation state of (4) +5
lanthanoid is
(5) +7
Ans. A-3, B-1, C-5, D-2
22. Match the solutions given in Column-I and the colours given in Column-II.
Column-I Column-II
(Aqueous solution of salt) (Colour)
(A) FeSO4.7H2O (1) Green
(B) NiCl2.4H2O (2) Light pink
(C) MnCl2.4H2O (3) Blue
(D) CoCl2.6H2O (4) Pale green
(5) Pink
(6) Colourless
Ans. A-4, B-1, C-2, D-5, E-6
23. Match the property given in Column-I with the element given in Column-II.
Column-I Column-II
(Property) (Element)
(A) Lanthanoid which shows +4 (1) Pm
oxidation state
(B) Lanthanoid which can show +2 (2) Ce
oxidation state
(C) Radioactive lanthanoid (3) Lu
7
(D) Lanthanoid which has 4f (4) Eu
electronic configuration in +3
node06\B0BC-BD\Kota\Board Material\Chemistry\Booklets\CBSE\Part-1

oxidation state
(E) Lanthanoid which has 4f14 (5) Gd
electronic configuration in +3
oxidation state
(6) Dy
Ans. A-2, B-4, C-1, D-5, E-3

238 E
CBSE
ALLEN
24. Match the properties given in Column-I with the metals given in Column-II.
Column-I Column-II
(Property) (Metal)
(A) Element with highest second (1) Co
ionisation enthalpy
(B) Element with highest third (2) Cr
ionisation enthalpy
(C) M in M(CO)6 is (3) Cu
(D) Element with highest heat of (4) Zn
atomisation
(5) Ni
Ans. A-3, B-4, C-2, D-1
(ASSERTION AND REASON TYPE)
Note: In the following questions a statement of assertion followed by a statement of reason is given.
Choose the correct answer out of the following choices.
(a) Both (A) and (R) correct and (R) is correct explanation of (A).
(b) Both (A) and (R) correct and (R) is not correct explanation of (A).
(c) (A) is correct but (R) is wrong.
(d) (A) Assertion is wrong but (R) reason is correct.
25. Assertion (A) : Cu2+ iodide is not known.
Reason (R) : Cu2+ oxidises I– to iodine.
Ans. (a)
26. Assertion (A) : Separation of Zr and Hf is difficult.
Reason (R) : Because Zr and Hf lie in the same group of the Periodic Table.
Ans. (b)
27. Assertion (A) : Actinoids form relatively less stable complexes as compared to lanthanoids.
Reason (R) : Actinoids can utilise their 5f orbitals along with 6d orbitals in bonding but
lanthanoids do not use their 4f orbital for bonding.
Ans. (c)
node06\B0BC-BD\Kota\Board Material\Chemistry\Booklets\CBSE\Part-1

28. Assertion (A) : Cu cannot liberate hydrogen from acids.


Reason (R) : Because it has positive electrode potential.
Ans. (a)
29. Assertion (A) : The highest oxidation state of osmium is +8.
Reason (R) : Osmium is a 5d-block element.
Ans. (b)

E 239
Chemistry
ALLEN
EXERCISE-4 MCQ, A/R, CASE-BASED QUESTIONS
1. Transitional elements exhibit variable valencies because they release electrons from the
following orbits
(a) ns orbit (b) ns and np orbits
(c) (n – 1)d and ns orbits (d) (n – 1)d orbit
2. Highest (+7) oxidation state is shown by
(a) Co (b) Cr (c) V (d) Mn
3. The coinage metals are
(a) Iron, Cobalt, Nickel (b) Copper and Zinc
(c)Copper, Silver and Gold (d) Gold and Platinum
4. The catalytic activity of the transition metals and their compounds is ascribed to their
(a) Chemical reactivity
(b) Magnetic behaviour
(c) Unfilled d-orbitals
(d) Ability to adopt multiple oxidation states and their complexing ability
5. Transitional elements are named transition elements because their characters are
(a) In between s and p - block elements
(b) Like that of p and d - block elements
(c) They are members of I – A group
(d) They are like inactive elements
6. Elements which generally exhibit multiple oxidation states and whose ions are usually coloured
are
(a) Metalloids (b) Transition elements
(c) Non-metals (d) Gases
7. The general electronic configuration of transition elements is
(a) (n – 1)d1 – 5 (b) (n – 1)d1–10ns1
(c) (n – 1)d1–10 ns1–2 (d) ns2 (n – 1)d10
8. Zinc does not show variable valency like d-block elements because
(a) It is a soft metal
node06\B0BC-BD\Kota\Board Material\Chemistry\Booklets\CBSE\Part-1

(b) d-orbital is complete


(c) It is low melting
(d) Two electrons are present in the outermost orbit
9. Transition elements are coloured
(a) Due to small size (b) Due to metallic nature
(c) Due to unpaired d – electrons (d) All of these

240 E
CBSE
ALLEN
10. Lanthanum is grouped with f-block elements because
(a) It has partially filled f -orbitals
(b) It is just before Ce in the periodic table
(c) It has both partially filled f and d-orbitals
(d) The properties of Lanthanum are very similar to the elements of 4f block
11. The element having electronic configuration belongs to ns 2 (n  1)d110 (n  2)f 114
(a) s-block (b) p-block (c) d-block (d) f-block
12. Zinc, cadmium and mercury show the properties of
(a) Typical elements (b) Normal elements
(c) Transitional elements (d) Rare elements
13. Zinc and mercury do not show variable valency like d-block elements because
(a) They are soft
(b) Their d- shells are complete
(c) They have only two electrons in the outermost subshell
(d) Their d-shells are incomplete
14. Bullet-proof steel alloy is prepared by using
(a) Sc (b) Ni (c) Zr (d) Zn
15. Which of the following is diamagnetic transitional metal ion
(a) Ni+2 (b) Zn+2 (c) Co+2 (d) Cu+2
16. In first transition series, the melting point of Mn is low because
(a) Due to d10 configuration, metallic bonds are strong
(b) Due to d7 configuration, metallic bonds are weak
(c) Due to d5 configuration, metallic bonds are weak
(d) None of these
17. Lanthanoide for which +2 and +3 oxidation states are common is
(a) La (b) Nd (c) Ce (d) Eu
18. Lanthanoide contraction occurs because
(a) f-orbital electrons are easily lost
(b) f-orbital an incompletely filled
node06\B0BC-BD\Kota\Board Material\Chemistry\Booklets\CBSE\Part-1

(c) f-orbital electron an poor shielders of nuclear charge


(d) f-orbital do not come out on the surface of atom and are buried inside
19. The metal present in B12 is
(a) Magnesium (b) Iron (c) Cobalt (d) Manganese
20. Most common oxidation states of (cesium) are
(a) + 2, + 3 (b) + 2, + 4 (c) + 3, + 4 (d) + 3, + 5

E 241
Chemistry
ALLEN
21. Number of unpaired electrons in Mn2+ is
(a) 3 (b) 5 (c) 4 (d) 1
22. Lanthanoides and actinides resemble in
(a) Electronic configuration (b) Oxidation state
(c) Ionization energy (d) Formation of complexes
23. Transition metals show paramagnetism
(a) Due to characteristic configuration (b) High lattice energy
(c) Due to variable oxidation states (d) Due to unpaired electrons
24. Four successive members of the first row transition elements are listed below with their atomic
numbers. Which one of them is expected to have the highest third ionization enthalpy
(a) Vanadium (Z = 23) (b) Chromium (Z = 24)
(c) Iron (Z = 26) (d) Manganese (Z = 25)
25. How many unpaired electrons are there in Ni2+
(a) 2 (b) 4 (c) 5 (d) 0
26. Electronic configuration of Cu (Z = 29) is
(a) [Ar]3d 9 4s 2 (b) [Ar]3d10 4s1 (c) [Ar]3d 5 4s 2 (d) [Ar]3d 6 4s 2
27. The lanthanoide contraction is responsible for the fact that
(a) Zr and Y have about the same radius
(b) Zr and Nb have similar oxidation state
(c) Zr and Hf have about the same radius
(d) Zr and Zn have the same oxidation sate
28. Which of the following factors may be regarded as the main cause of lanthanoide contraction
(a) Poor shielding of one of 4f electron by another in the subshell
(b) Effective shielding of one of 4f electrons by another in the subshell
(c) Poorer shielding of 5d electrons by 4f electrons
(d) Greater shielding of 5d electron by 4f electrons

ANSWER KEY
Q. No. 1 2 3 4 5 6 7 8 9 10
node06\B0BC-BD\Kota\Board Material\Chemistry\Booklets\CBSE\Part-1

Ans. c d c d a b c b c d
Q. No. 11 12 13 14 15 16 17 18 19 20
Ans. d c b c b c a d c c
Q. No. 21 22 23 24 25 26 27 28
Ans. b b d d a b c a

242 E
CBSE
ALLEN
ASSERTION AND REASON
Note : In the following questions a statement of assertion followed by a statement of reason is
given. Choose the correct answer out of the following choices.
(a) Both (A) and (R) correct and (R) is correct explanation of (A).
(b) Both (A) and (R) correct and (R) is not correct explanation of (A).
(c) (A) is correct but (R) is wrong.
(d) (A) Assertion is wrong but (R) reason is correct.
1. Assertion (A) : Cr2+ is reducing, while Mn3+ is oxidising even both have d4-configuration.
Reason (R) : Configuration of Cu changes from d3 to d4.
2. Assertion (A) : KMnO4 oxidises oxalic acid to CO2 and itself changes to Mn2+ ion.
Reason (R) : KMnO4 acts as an oxidising agent.
3. Assertion (A) : Many trivalent lanthanoid ions are coloured both in solid state and in aqueous
solution.
Reason (R) : Colour of these ions is due to the presence of f-electrons.
4. Assertion (A) : The highest oxidation state of osmium is +8.
Reason (R) : Osmium is a 5d-series element.
5. Assertion (A) : Transition metals are good catalysts.
Reason (R) : V2O5 or Pt is used in the preparation of H2SO4 by contact process.
6. Assertion (A) : Zn is not a transition element.
Reason (R) : It has completely filled d-orbital, ground and common oxidation states.
7. Assertion (A) : Transition elements form coloured compounds.
Reason (R) : They have incomplete d-orbital.
8. Assertion (A) : Lanthanoids and Actinides show contraction.
Reason (R) : They have electrons filled in f-orbital.
9. Assertion (A) : Lanthanoids are called as f-block elements.
Reason (R) : They have valence electrons filled in f-orbital.
10. Assertion (A) : Lanthanium and actinium have similar ionic size.
Reason (R) : Lanthanoid show lanthanoid contraction.
11. Assertion (A) : Ceiv is stable.
node06\B0BC-BD\Kota\Board Material\Chemistry\Booklets\CBSE\Part-1

Reason (R) : It has attained noble gas configuration.


12. Assertion (A) : Tbiv has half filled f-orbital.
Reason (R) : It acts as an oxidant.
13. Assertion (A) : Actinides show structural variability.
Reason (R) : There is an irregularity in metallic radii.

E 243
Chemistry
ALLEN
14. Assertion (A) : Cu2+ iodide is not known.
Reason (R) : Cu2+ oxidises I– to iodine.
15. Assertion (A) : Transition metals have low melting points.
Reason (R) : The involvement of greater number of (n – 1)d and ns electrons in the interatomic
metallic bonding.
16. Assertion (A) : Separation of Zr and Hf is difficult.
Reason (R) : Because Zr and Hf lie in the same group of the periodic table.
17. Assertion (A) : Cu cannot liberate hydrogen from acids.
Reason (R) : Because it has positive electrode potential.
18. Assertion (A) : The highest oxidation state of osmium is +8.
Reason (R) : Osmium is a 5d-block element
19. Assertion (A) : Highest oxidation state is exhibited by transition metal lying in the middle of the
series.
Reason (R) : The highest oxidation state exhibited corresponds to number of (n – 1)d electrons.
20. Assertion (A) : Fe3+ is more stable than Fe2+
Reason (R) : Fe3+ has 3d5 configuration while Fe2+ has 3d6 configuration.
21. Assertion (A) : Vanadium had the ability to exhibit a wide range of oxidation states.
Reason (R) : The standard potential of vanadium are rather small, making a switch between
oxidation states relatively easy.
22. Assertion (A) : Transition metals like Fe, Cr and Mn form oxyions.
Reason (R) : Oxygen is highly electronegative and has a tendency to form multiple bonds.

ANSWER KEY
Q. No. 1 2 3 4 5 6 7 8 9 10
node06\B0BC-BD\Kota\Board Material\Chemistry\Booklets\CBSE\Part-1

Ans. c a a b b a b b a b
Q. No. 11 12 13 14 15 16 17 18 19 20
Ans. a b b a c c a a c a
Q. No. 21 22
Ans. a b

244 E
CBSE
ALLEN
CASE BASED QUESTIONS
Case-I
Transition metals show a great variety of oxidation states in its compounds (variable valency)
except the first and the last element. This is because of the fact that, the difference in the energy
of (n – 1) d-electrons and ns-electrons is low which implies that electrons from both energy
levels can take part in bonding.
In 3d-series, all elements show +2 oxidation state except Sc (Sc = + 3). Oxidation states first
increases from Sc to Mn due to increase in number of unpaired electrons and then decreases
because pairing takes place. Fe and Ni show zero oxidation state in metal carbonyls.
The oxidation state of a metal in a solvent depends on the nature of the solvent, e.g. Cu+ is
unstable in aqueous solution and undergo disproportionation reaction.
Low oxidation states are found if a complex compound has ligands capable of -acceptor
character and -bonding, e.g. Ni(CO)4 and Fe(CO)5, the oxidation state of nickel and iron is zero
in the presence of CO as ligand.
(a) The transition metal generally form coloured compound ?
Ans. Due to the presence of unpaired electrons,they undergo d-d transitions by absorbing light
from visible region and radiate the complementary colour.
OR
All scandium salts are white ? (At. no. of Sc = 21)
Ans. Sc3+ ions does not have any unpaired electron and has empty d-orbitals, therefore cannot
undergo d-d transitions. Hence its salts are white.
(b) Which of the following ion is coloured ?
Zn2+, Ti4+, Sc3+ and Co3+
Ans. Co3+
(c) The Cu+ salts are colourless while Cu2+ salts are coloured (At. no. of Cu = 29)
Ans. Cu+ ion does not have unpaired electron therefore it cannot undergo d-d transitions. That is
why Cu+ salts are colourless whereas Cu2+ salts are coloured due to the presence of one
unpaired electron, it can undergo d-d transition by absorbing light from the visible region
and radiate blue colour.
Case II
Within the 3d series, manganese exhibits oxidation states in aqueous solution from +2 to +7,
ranging from Mn2+(aq) to MnO 4 (aq) . Likewise, iron forms both Fe2+(aq) and Fe3+(aq) as well as
node06\B0BC-BD\Kota\Board Material\Chemistry\Booklets\CBSE\Part-1

the FeO24  ion. Cr and Mn form oxyions CrO24  , MnO 4 , owing to their willingness to form
multiple bonds.
The pattern with the early transition metals, d and f in the 3d series upto Mn, and for the 4d, 5d
metals upto Ru and Os is that the maximum oxidation state corresponds to the number of "outer
shell" electrons.

E 245
Chemistry
ALLEN
The highest oxidation states of 3d metals may depend upon the complex formation (e.g. the
stabilisation of Co by ammonia) or upon the pH (thus MnO 24  (aq)) is pronto to
disproportionation in acidic solution).
Within the 3d series, there is considerable variation in relative stability of oxidation states,
sometimes on moving from one metal to a neighbour; thus, for iron, Fe3+ is more stable than
Fe2+, especially in alkaline conditions, while the reverse is true for cobalt. The ability of
transition metals to exhibit a wide range of oxidation states is marked with metals such as
vanadium, where the standard potentials can be rather small, making a switch between states
relatively easy.
(a) Give the name and chemical formula of the ore from which potassium dichromate is
prepared ?
Ans. Potassium dichromate is prepared by chromite ore. The chemical formula of chromite ore
is FeCr2O4.
(b) Complete the following reaction :
Na2Cr2O7 + 2KCl 
Ans. Na2Cr2O7 + 2KCl  K 2 Cr2 O7  2NaCl
Potassium Sodium
dichromate chloride

(c) What are the oxidation states of Cr in chromate and dichromate ion ?
Ans. Oxidation states of Cr in both chromate and dichromate ion is +6.
(x )
In Cr2 O72
2x – 14 = –2
 x=+6
(x )
In CrO 24
x – 8 = –2
 x=+6
OR
Draw the structures of :
(i) Dichromate ions (ii) Chromate ions
Ans. The structures of dichromate and chromate ions are given below :
node06\B0BC-BD\Kota\Board Material\Chemistry\Booklets\CBSE\Part-1

(i) Dichromate ion (ii) Chromate ion


O O O O–
O Cr Cr O Cr
1 –
O O1 O O
O
Dichromate ion ( Chromate ion ( )

246 E
CBSE
ALLEN
PREVIOUS YEARS QUESTIONS
SECTION-A (ONE MARK QUESTIONS)
1. Out of the following transition elements, the maximum number of oxidation states are shown by
[1] (CBSE 2020)
(A) Sc (Z = 21) (B) Cr (Z = 24) (C) Mn (Z = 25) (D) Fe (Z = 26)
Ans. (C) Mn (Z = 25)
2. What is meant by ‘lanthanoid contraction’? [1] (CBSE 2019)
Ans. The overall decrease in atomic and ionic radil from Lanthanum to Lutetium is a unique feature in
the chemistry of the Lanthanoids. It has far reaching consequences in the chemistry of the third
transition series of the elements.
SECTION-B (TWO MARKS QUESTIONS)
3. Calculate the number of unpaired electrons in the following gaseous ions:
Mn3+ and Cr3+ (Atomic number of Cr = 24, Mn = 25)
Which one of them is most stable in aqueous solution and why ?
[2] (CBSE 2021 Compartment)
Ans. The number of unpaired electrons in Mn3+ = 4
The number of unpaired electrons in Cr3+ = 3
Cr3+ is most stable in aqueous solution due to  t 32g  half field stability.

4. Explain all the steps along with the chemical equation which are used in the preparation of
K2Cr2O7 from chromite ore. [2] (CBSE 2021 Compartment)
Ans. Potassium dichromate is prepared from chromite ore (FeCr2O4) in the following steps.
Step (1) : Preparation of sodium chromate
4FeCr2O4 + 16NaOH + 7O2 8Na2CrO4 + 2Fe2O3 + 8H2O
Step (2) : Conversion of sodium chromate into sodium dichromate
2Na2CrO4 + conc. H2SO4 Na2Cr2O7 + Na2SO4 + H2O
Step (3) : Conversion of sodium dichromate to potassium dichromate
Na2Cr2O7 + 2KCl  K2Cr2O7 + 2NaCl
Potassium dichromate being less soluble than sodium dichromate is obtained in the form of
orange coloured crystals and can be removed by filtration.
5. Complete the following chemical equations: [2] (CBSE 2020)
node06\B0BC-BD\Kota\Board Material\Chemistry\Booklets\CBSE\Part-1

(i) MnO 4 (aq)  S2 O32  (aq)  H 2 O (l) 

(ii) Cr2 O27 (aq)  Fe 2  (aq)  H  (aq) 

Ans. (i) 8 MnO 4 (aq)  3 S2 O32  (aq)  H 2 O(l)  8 MnO 2  6 SO 24  2 OH 

(ii) Cr2 O27 (aq)  6 Fe 2  (aq)  14 H  (aq)  2 Cr 3  6Fe 3  7H 2 O

E 247
Chemistry
ALLEN
6. How would you account for the following: [2] (CBSE 2020)
(i) Cr2+ is reducing in nature while with the same d-orbital configuration (d4) Mn3+ is an
oxidising agent.
(ii) In a transition series of metals, the metal which exhibits the greatest number of oxidation
states occurs in the middle of the series.
Ans. (i) Cr2+ has the configuration (d4) and easily changes to d3 has half filled orbitals and hence
stable. Cr2+ is reducing. On the other hand, Mn2+ is more stable due to half filled d5
configuration and Mn3+ easily changes to Mn2+ and hence is oxidising.
(ii) The transition metals reacts with a number of non-metals like oxygen, nitrogen, sulphur
and halogens to form binary compounds. The first series transition metal oxides are
generally formed from the reaction of metals with oxygen at high termperatures. These
oxides dissolve in acids and bases to form oxometallic salts. Potassium dichromate and
potassium permanganate are common examples. Potassium dichromate is prepared from
the chromite ore by fusion with alkali in presence of air and acidifying the extract.
Pyrolusite ore is used for the preparation of potassium permanganate. Both the dichromate
and the permanganate ions are strong oxidising agents.
7. Complete the following chemical reaction equation : [2] (CBSE 2019)
(i) MnO–4(aq) + C2O42–(aq) + H+(aq) 
(ii) Cr2O72–(aq) + Fe2+(aq) + H+(aq) 
Ans. (i) 2MnO–4 (aq) + 5C2O42–(aq) + 16H+(aq) 2Mn2+(aq) + 8H2O() + 10CO2(g)

(ii) Cr2O72–(aq) + 6Fe2+(aq) + 14H+(aq)  2Cr3+(aq) + 6Fe3+(aq) + 7H2O()


8. Write complete chemical equations for : [2] (CBSE 2019)
(i) Oxidation of Fe2+ by Cr2O72– in acid medium.
(ii) Oxidation of S2O32– by MnO4– in neutral aqueous medium.
Ans. (i) Acidified potassium dichromate dichromate oxidises iron (II) salt to iron (III).
Cr2O72– + 14H+ + 6Fe2+ 2Cr3+ + 6Fe3+ + 7H2O
(ii) In neutral aqueous medium, thiosulphate is oxidised almost quantitatively to sulphate.
8 MnO4– + 3S2O32– + H2O 8MnO2 + 6SO42– + 2OH–
9. When FeCr2O4 is fused with Na2CO3 in the presence of air it gives a yellow solution of
compound (A). Compound (A) on acidification gives compound (B). Compound (B) on reaction
with KCl forms an orange coloured compound (C). An acidified solution of compound (C)
oxidises Na2SO3 to (D). Identify (A), (B), (C) and (D). [2] (CBSE 2019)
node06\B0BC-BD\Kota\Board Material\Chemistry\Booklets\CBSE\Part-1

Ans. A = Na2CrO4 ; B = Na2Cr2O7 ; C = K2Cr2O7 ; D = Na2SO4


10. Complete and balance the following chemical equations : [2] (CBSE 2018)
(a) Fe 2   MnO 4  H  

(b) MnO 4  H 2 O  I  

Ans. 5Fe 2   MnO 4  8H   Mn 2   4H 2 O  5Fe 3
2MnO 4  H 2 O  I   2MnO2  2OH   IO3
248 E
CBSE
ALLEN
11. Why do transition elements show variable oxidation states? How is the variability in oxidation
states of d-block different from that of the p-block elements? [2] (CBSE 2018)
Ans. Due to comparable energies of ns & (n – 1)d orbitals / due to presence of unpaired electrons in
(n – 1)d orbitals.
In transition elements, oxidation states differ from each other by unity whereas in case of
p-block elements, the oxidation states differ by units of two / In transition elements, the higher
oxidation states are more stable for heavier elements in a group. In p–block elements, the lower
oxidation states are more stable for heavier members due to inert pair effect. (Any one difference)
12. Complete the following chemical reaction equations : [2] (CBSE 2018)
(i) Cr2O72– + I– + H+ 
(ii) MnO4– + NO2– + H+ 
Ans. (i) Cr2O72– + 6I– + 14H+  2Cr3+ + 3I2 + 7H2O
(ii) 2MnO4– + 5NO2– + 6H+  2Mn2+ + 5NO3– + 3H2O
13. State reasons for the following: [2] (CBSE 2018)
(i) Cu (I) ion is not stable in an aqueous solution.
(ii) Unlike Cr3+,Mn2+, Fe3+ and the subsequent other M2+ ions of the 3d series of elements, the
4d and the 5d series metals generally do not form stable cationic species.
Ans. (i) Cu+ in aqueous solution undergoes disproportionation i.e.,
2 Cu+ (aq)  Cu2+(aq) + Cu(s)
The E° value for this is favourable.
(ii) Cr2+ is stronger reducing agent than Fe2+
Reason : d4  d3 occurs in case of Cr2+ to Cr3+
But d6  d5 occurs in case of Fe2+ to Fe3+
In a medium (like water) d3 is more stable as compared to d5.
SECTION-C (THREE MARKS QUESTIONS)
14. Account for the following :
(i) Cr2+ is a strong reducing agent.
(ii) Ti3+ is coloured whereas Sc3+ is colourless in aqueous solution.
(iii) Zn, Cd and Hg are not called transition elements. [1×3=3] (CBSE_Term-II_2022)
node06\B0BC-BD\Kota\Board Material\Chemistry\Booklets\CBSE\Part-1

2+ 4
Ans. (i) Cr is strongly reducing in nature. It has a d configuration. While acting as a reducting
3+ 3 3
agent, it gets oxidized to Cr (electronic configuration, d ). This d configuration can be
written as t 23 g configuration, which is a more stable configuration.
3+ –
(ii) Sc is colourless is due to the absence of unpaired e as it attains 3d° configuration while
+3 1
Ti has 3d -configuration
(iii) Due to completely filled d-orbitals in their ground state as well as in oxidized state.
E 249
Chemistry
ALLEN
15. What is Lanthanoid contraction ? What is the cause of Lanthanoid contraction ? Write two
consequences of Lanthanoid contraction. [3] (CBSE_Term-II_2022)
Ans. A regular decrease (contraction) in the atomic and ionic radii of lanthanoides with increasing
atomic number is known as lanthanoid contraction.
Alloy containing lanthanoid metals is mischmetal.
Consequences of lanthanoid contraction
(i) There is similarity in the properties of second and third transition series.
(ii) Separation of lanthanoids is possible due to lanthanoide contraction.
(iii) It is due to lanthanoide contraction that there is variation in the basic strength of
lanthanoide hydroxides. (Basic strength decreases from La (OH)3 to Lu (OH)3.)
16. Give reasons for the following :
(a) Transition metals form complex compounds.
(b) E° values for (Zn2+ / Zn) and (Mn2+ / Mn) are more negative than expected.
(c) Actinoids show wide range of oxidation states. [3] (CBSE 2019)
Ans. (a) Due to small size, high ionic charge and availability of d-orbital
(b) Due to stable 3d10 configuration in Zn2+ and 3d5 configuration in Mn2+.
(c) Due to comparable energies of 5f, 6d and 7s orbitals/level 
17. Give reasons for the following :
(a) Transition metals have high enthalpies of atomization.
(b) Manganese has lower melting point even though it has a higher number of unpaired
electrons for bonding.
(c) Ce4+ is a strong oxidising agent. [3] (CBSE 2019)
Ans. (a) Because of strong interatomic interactions/strong metallic bonding between atoms.
(b) Due to stable 3d5 configuration, interatomic interaction is poor between unpaired electrons.
(c) Because Ce is more stable in +3 oxidation state.
18. How would you account for the following :- [3] (CBSE 2019)
(i) Many of the transition elements and their compounds can act good catalysts.
(ii) The metallic radii of the third (5d) series of transition elements virtually the same as those
of the corresponding members of the second series.
(iii) There is a greater range of oxidation states among the actinoids than among the
lanthanoids.
Ans. (i) Transition element are good catalysts because they posess variable oxidation state and can
node06\B0BC-BD\Kota\Board Material\Chemistry\Booklets\CBSE\Part-1

from intermediated compounds with most of the reaction. Furthermore it is also attributed
to the penultimate partially filled d-subshell possed by them.
e.g : Role of V2O5 in the manufacture of sulphuric acid by contact process.
SO2 + V2O5  V2O4 + SO3
2V2O4 + O2  2V2O5

250 E
CBSE
ALLEN
(ii) This is due to the intervention of the 4f orbitals which must be filled before the 5d series of
element begin. The filling of 4f before 5d orbitals results in regular decrease in atomic radii
called Lanthanoid contraction.
(iii) This is due to the fact that the 5f, 6d and 7s levels are of comparable energies.
19. How would you account for the following situations ? [3] (CBSE 2019)
(i) The transition metals generally form coloured compounds.
(ii) With 3d4 configuration, Cr2+ acts as reducing agent but Mn3+ acts as an oxidising agent.
(Atomic masses, Cr = 24, Mn = 25)
(iii) The actinoids exhibit a larger number of oxidation states than the corresponding
lanthanoids.
Ans. (i) The colour of compounds of transition elements depend upon the unpaired electrons
present in d-orbitals of transition element. If d-orbitals are completely vacant as in Sc3+,
Ti4+ or completely filled as in Cu+, Zn2+, the compounds will be colourless, but if any
unpaired electron is present in d-orbitals, the compound will be coloured due to d  d
transition. The unpaired electron is excited from one energy level to another energy level
within the same d-sub shell. For this purpose (d  d transition), the energy is absorbed
from visible region of radiation and reflected light will decide the colour of the compound.
(ii) It can be exaplained on the basis of E° values of Cr3+/Cr2+ (–0.4 V) and Mn3+/Mn2+
(+1.5 V). On the basis of these values, it is clear that Cr2+ has a great tendency to oxidise
into Cr3+, thus it acts as reducing agent.
Cr2+ (aq) Cr3+ (aq) + e–
While Mn3+ has a great tendency to reduce into Mn2+ thus it acts as an oxidising agent
Mn3+(aq) + e– Mn2+ (aq)
(iii) Actinoids show a large number of oxidation states because of small energy gap between 5f,
6d and 7s subshells.
20. Give reasons :
(a) E0 value for Mn3+/Mn2+ couple is much more positive than that for Fe3+/Fe2+.
(b) Iron has higher enthalpy of atomization than that of copper.
(c) Sc3+ is colourless in aqueous solution whereas Ti3+ is coloured. [3] (CBSE 2018)
node06\B0BC-BD\Kota\Board Material\Chemistry\Booklets\CBSE\Part-1

Ans. (a) E° value for Mn3+/Mn2+ couple is much more positive than that for Fe3+/Fe2+, due to the
Mn2+ have higher stability than M3+ due to half-filled d5 configuration.
(b) Iron has higher enthalpy of atomization than that of copper because Iron has higher number
of unpaired e– than Cu due to which extent of covalent bonding is more.
(c) Sc3+ is colourless is due to the absence of unpaired e– as it attains 3d° configuration while
Ti+3 has 3d1-configuration

E 251
Chemistry
ALLEN
21. (i) E0 value for the Mn3+/Mn2+ couple is positive (+1.5 V) whereas that of Cr3+/Cr2+ is negative
(– 0.4 V). Why?
(ii) Transition metals form coloured compounds. Why?
(iii) Complete the following equation :
2MnO 4  16H   5C 2 O 24   [3] (CBSE 2018)

Ans. (i) The large positive E0 value for Mn3+/Mn2+ shows that Mn2+ (3d5/half filled d orbital) is
much more stable than Mn3+ Whereas Cr3+ (t2g3 ) is more stable than Cr2+
(ii) Due to d–d transition/due to presence of unpaired electrons in d–orbitals which absorb light
in visible region
(iii) 2MnO 4  16H   5C 2 O24   2Mn 2   8H 2 O  10CO2

22. How would you account for the following [3] (CBSE 2018)
(i) Among lanthanoids, Ln(III) compounds are predominant. However, occasionally in
solutions or in solid compounds, +2 and +4 ions are also obtained.
(ii) The EM2 /M for copper is positive (0.34 V). Copper is the only metal in the first series of
transition elements showing this behaviour.
(iii) The metallic radii of the third (5d) series of transition metals are nearly the same as those of
the corresponding members of the second series.
Ans. (i) This irregularity arises mainly from the extrastability of empty, halffilled or filled f
subshell.
(ii) B'coz of high atomisation and low hydration enthalpy.
(iii) Poor shielding effect of 4f electrons known as Ln contraction.

SECTION-D (FOUR MARKS QUESTIONS)

23. The elements of 3d transition series are given as: [4] (CBSE 2019)
Sc Ti V Cr Mn Fe Co Ni Cu Zn
Answer the following:
(i) Write the element which is not regarded as a transition element. Give reason.
(ii) Which element has the highest m.p ?
(iii) Write the element which can show an oxidation state of +1.
node06\B0BC-BD\Kota\Board Material\Chemistry\Booklets\CBSE\Part-1

(iv) Which element is a strong oxidizing agent in +3 oxidation state and why ?
Ans. (i) Zn, because of not having partially filled d-orbital in its ground state or ionic state.
(ii) Cr
(iii) Cu
(iv) Mn, because Mn+2 has extra stability due to half-filled d-orbital

252 E
CBSE
ALLEN
24. The d-block of the periodic table contains the elements of the groups 3 – 12 and are
known as transition elements. In general, the electronic configuration of these elements is
(n – 1) d1–10 ns1–2. The d-orbitals of the penultimate energy level in their atoms receive electrons
giving rise to the three rows of the transition metals i.e., 3d, 4d and 5d series. However, Zn, Cd
and Hg are not regarded as transition elements. Transition elements exhibit certain characteristic
properties like variable oxidation states, complex formation, formation of coloured ions and
alloys, catalytic activity, etc. Transition metals are hard (except Zn, Cd and Hg) and have a high
melting point. [1+1+2=4] (CBSE 2019)
(i) Which element has the highest m.p. in 3d series ?
OR
Write the element which can show an oxidation state of +1.
(ii) Complete the following equation :
2MnO4–+ 16 H+ + 5C2O42– 
(iii) E0 value for the Mn3+ / Mn2+ couple is positive (+ 1.5 V) whereas that of Cr3+ / Cr2+ is
negative (– 0.4 V). Why?
Ans. (i) Cr
OR
Cu
(ii) 2MnO4– + 16H+ + 5C2O42–  2Mn2+ + 8H2O + 10CO2
(iii) The large positive E0 value for Mn3+ / Mn2+ shows that Mn2+ (3d5 / half filled d orbital) is
much more stable than Mn3+ Whereas Cr3+ (t2g3 ) is more stable than Cr2+.
SECTION-E (FIVE MARKS QUESTIONS)
25. (a) Account for the following : [1 × 3 = 3]
(i) Copper (I) compounds are white whereas Copper (II) compounds are coloured.
(ii) Chromates change their colour when kept in an acidic solution.
(iii) Zn, Cd, Hg are considered as d-block elements but not as transition elements.
(b) Calculate the spin-only moment of Co2+ (Z = 27) by writing the electronic configuration of
Co and Co2+ [2] (CBSE 2020)
OR
(a) Give three points of difference between lanthanoids and actinoids. [3]
(b) Give reason and select one atom/ion which will exhibit asked property : [1 × 2 = 2]
+3 3+
(i) Sc or Cr (Exhibit diamagnetic behaviour)
(ii) Cr or Cu (High melting and boiling point)
node06\B0BC-BD\Kota\Board Material\Chemistry\Booklets\CBSE\Part-1

Ans. (a) (i) Cu+1 (3d10) compounds are white because of absence of unpaired electrons while
Cu+2 (3d9) compounds are coloured due to unpaired e-/shows d-d transition.
(ii) chromate (CrO42–) changes to dichromate (Cr2O72-) ion in acidic medium.
(iii) due to completely filled d-orbitals in their ground state as well as in oxisized state.
(b) Co = [Ar]4s23d7, Co+2 = [Ar]3d7
= n(n  2) = 3(3  2)  15  3.92 B.M.

E 253
Chemistry
ALLEN
OR
(a) Lanthanoids Actinoids
(1) Most of them are not radioactive All are radioactive
(2) Don't show a wide range of oxidation state Show a wide range of oxidation states
(3) Most of their ions are colourless Most of their ions are coloured

(b) (i) Sc+3, because of absence of unpaired electron


(ii) Cr, because of presence of strong intermetallic bonding than Cu.
26. (a) Complete the following chemical equations: [5] (CBSE 2020)
 2
(i) MnO (aq)  S2 O
4 3 (aq)  H 2 O (l) 
(ii) Cr2 O 27 (aq)  Fe 2  (aq)  H  (aq) 
(b) Explain the following observations:
(i) La3+ (Z = 57) and Lu3+ (Z = 71) do not show any colour in solutions.
(ii) Among the divalent cations in the first series of transition elements, manganese
exhibits the maximum paramagnetism.
(iii) Cu+ ion is not known in aqueous solutions.
Ans. (a) (i) 8MnO 4 (aq)  3S2 O32  (aq)  H 2 O(l)  MnO 2 (s)  4SO 24 (aq)  2OH  (aq)
(ii) Cr2 O27 (aq)  6Fe 2  (aq)  14H  (aq)  2Cr 3 (aq)  6Fe3 (aq)  7H 2 O(l)
(b) (i) Because they have empty outer most shell.
(ii) Mn2+ has maximum number of unpaired electrons
(iii) Since Cu+ in aqueous solution undergoes disproportionation i.e.
2 Cu+ (aq)  Cu2+ (aq) + Cu(a)
Thus, the Eº value for this is favourable
27. (a) Complete the following chemical equations: [5] (CBSE 2020)
(i) Cr2 O27 (aq)  H 2S (g)  H  (aq) 
(ii) Cu 2  (aq)  I  (aq) 
(b) How would you account for the following:
(i) The oxidation power of oxoanions are in the order
VO2  Cr2 O27   MnO 4
(ii) The third ionization enthalpy of Mn (Z = 25) is exceptionally high.
(iii) Cr2+ is a stronger reducing agent than Fe2+.
Ans. (a) (i) Cr2O72– (aq) + 3H2S (g) + 8 H+(aq)  2 Cr3+ (aq) + 7 H2O (i) + 3S (s)
(ii) 2Cu2+ (aq) + 4 I- (aq)  Cu2 I2 (s) + I2(g)
node06\B0BC-BD\Kota\Board Material\Chemistry\Booklets\CBSE\Part-1

(b) (i) This is due to the increasing stability of the lower species to which they are produced.
(ii) Third ionization enthalpy of Mn is very high because the third electron has to be
removed from the stable half-filled 3d orbitals [ Mn (Z = 25) = 3d5 4s2]
(iii) Since Eº (Cr3+/Cr2+) is negative (–0.41V) whereas Eº (Fe3+/Fe2+) is positive (+0.77 V).
Thus Cr2+ is easily oxidized to Cr3+ but Fe2+ cannot be easily oxidized to Fe3+. Hence
Cr2+ is stronger reducing agent that Fe2+.
254 E
CBSE
ALLEN
28. (a) Account for the following : [5] (CBSE 2019)
(i) Transition metals show variable oxidation states.
(ii) Zn, Cd and Hg are soft metals.
(iii) Eo value for the Mn3+/Mn2+ couple is highly positive (+1.57 V) as compared to
Cr3+/Cr2+
(b) Write one similarity and one difference between the chemistry of lanthanoid and actinoid
elements.
OR
(a) Following are the transition metal ions of 3d series :
Ti4+, V2+, Mn3+, Cr3+
(Atomic numbers : Ti = 22, V =23, Mn = 25, Cr = 24)
Answer the following :
(i) Which ion is most stable in an aqueous solution and why ?
(ii) Which ion is a strong oxidising agent and why ?
(iii) Which ion is colourless and why ?
(b) Complete the following reactions :
(i) 2MnO4– + 16H+ + 5S2– 
heat
(ii) KMnO4  
Ans. (a) (i) Availability of partially filled d-orbitals/comparable energies of ns and (n–1)d
orbitals
(ii) Completely filled d-orbitals/absence of unpaired d electrons cause weak metallic
bonding
(iii) Because Mn2+ has d5 as a stable configuration whereas Cr3+ is more stable due to
stable t32g
(b) Similarity-both are stable in +3 oxidation state/both show contraction/irregular electronic
configuration (or any other suitable similarity) Difference-actinoids are radioactive and
lanthanoids are not/actionoids show wide range of oxidation states but lanthanoids don't
(or any other orrect difference)
OR
(a) 3+ 3
(i) Cr , half filled t 2g
(ii) Mn3+, due to stable d5 configuration in Mn2+
(iii) Ti4+, No unpaired electrons
(b) (i) 2MnO4– + 16H+ + 5S2– 2Mn2+ + 8H2O
(ii) 2KMnO4  K2MnO4 + MnO2 + O2
node06\B0BC-BD\Kota\Board Material\Chemistry\Booklets\CBSE\Part-1

29. (a) Why do transition elements show variable oxidation states? [5] (CBSE 2019)
(i) Name the clement showing maximum number of oxidation states among the first
series of transition metals from Sc (Z = 21) to Zn (Z = 30).
(ii) Name the element which shows only +3 oxidation state.
(b) What is lanthanold contraction? Name an important alloy which contains some of the
lanthanold metal.

E 255
Chemistry
ALLEN
Ans. (a) Transition elements have d orbitals which can accommodate a maximum of 10 electrons so
there is more scope for loss of any number of electron resulting into variable oxidation
states.
(i) Mn shows maximum number of oxidization states.
(ii) Sc shows only +3
(b) A regular decrease (contraction) in the atomic and ionic radii of lanthanoids with increasing
atomic number is known as lanthanoid contraction.
Alloy containing lanthanoid metals is mischmetal.
30. (a) Account for the following [5] (CBSE 2018)
(i) Mn shows the highest oxidation state of +7 with oxygen but with fluorine it shows
the highest oxidation state of +4.
(ii) Zirconium and Hafnium exhibit similar properties.
(iii) Transition metals act as catalysts.
(b) Complete the following equations:
(i) 2MnO + 4KOH + O   (ii) Cr O 2  + 14H+ + 6I–  
2 2 2 7

Ans. (a) (i) Ability of oxygen to form multiple bond .


(ii) Due to lanthanoid contraction.
(iii) Due to variable oxidation state/unpaired electrons
(b) (i) 2MnO2 + 4KOH + O2  2K2MnO4 + 2H2O
(ii) Cr2O72 – + 14H+ + 6I–  2Cr3++ 7H2O + 3I2
31. (a) Complete the following equations [5] (CBSE 2018)
(i) Cr2 O27  2OH  
 (ii) MnO 4  4H   3e  
(b) Account for the following
(i) Zn is not considered as a transition element.
(ii) Transition metals form a large number of complexes.
(iii) The Eº value for the Mn3+ /Mn2+ couple is much more positive than that for Cr3+ /Cr2+
couple.
OR
(i) With reference to structural variability and chemical reactivity, write the differences
between lanthanoids and actinoids.
(ii) Name a member of the lanthanoid series which is well known to exhibit + 4 oxidation state.
(iii) Complete the following equation.
node06\B0BC-BD\Kota\Board Material\Chemistry\Booklets\CBSE\Part-1

MnO 4  8H   5e  
 (iv) Out of Mn3+ and Cr3+ , which is more paramagnetic and why ?
(Atomic nos. : Mn = 25, Cr = 24)
Ans. (a) (i) Cr2O72– + 2OH–  2CrO42– + H2O
(ii) MnO4– + 4H+ + 3e–  MnO2 + 2H2O

256 E
CBSE
ALLEN
(b) (i) Zn, cannot be classified as transition elements because these have completely filled
d-subshell. In their ground state as well as in their common oxidation state.
(ii) Due to high nuclear charge presence of vacant d-orbital and variable oxidation state it
have great tendency to from complex.
(iii) Due to high sum of first two consquetive (IP1 + IP2).
OR
(i) In the lanthanoide series, the earlier members of the series are more reactive. They have
reactivity that is comparable to Ca. With an increase in the atomic number, the
lanthanoides start behaving similar to Al. Actinoids, on the other hand, are highly reactive
metals, especially when they are finely divided.
(ii) Ce+4
(iii) MnO4– + 8H+ + 5e–  Mn2+ + 4H2O
(iv) Mn+3 due to four unpaired electron in their 3d orbital.
node06\B0BC-BD\Kota\Board Material\Chemistry\Booklets\CBSE\Part-1

E 257
Chemistry
ALLEN
PRACTICE TEST
SECTION-A
1. Which one of the following statements is correct when SO2 is passed through acidified K2Cr2O7
solution ? [1]
(a) The solution turns blue (b) The solution is decolourized
(c) SO2 is reduced (d) Green Cr2(SO4)3 is formed
2. Reason of lanthanoid contraction is :- [1]
(a) Negligible screening effect of 'f' orbitals (b) Increasing nuclear charge
(c) Decreasing nuclear charge (d) Decreasing screening effect
3. Which of the following lanthanoid ions is diamagnetic? [1]
(Atoms, Ce = 58, Sm = 62, Yb = 70, Eu = 63)
(a) Yb2+ (b) Ce2+ (c) Sm2+ (d) Eu2+
4. Which of the following oxidation states is the most common among the lanthanoids :- [1]
(a) 2 (b) 5 (c) 3 (d) 4
5. Which of the following pairs has the same size ? [1]
(a) Zn2+, Hƒ4+ (b) Fe2+, Ni2+ (c) Zr4+, Ti4+
(d) Zr4+, Hƒ4+
6. Assertion (A) : All the metals except scandium form MO oxides which are ionic. [1]
Reason (R) : There is an irregularity in metallic radii.
7. Assertion (A) : TiC is an interstitial halide. [1]
Reason (R) : Ti only forms interstitial compounds.
SECTION-B
8. Although ‘F’ is more electronegative than ‘O’. the highest Mn fluoride In MnF4, whereas the
highest oxide is Mn2O7. Why ? [2]
 Out of Mn3+ and Cr3+ , which is more paramagnetic and why ?
 (Atomic nos. : Mn = 25, Cr = 24) [2]
10. (a) Transition metals form coloured compounds. Why? [2]
(b) Transition metals act as catalysts.
11. (a) Mn shows the highest oxidation state of +7 with oxygen but with fluorine it shows the
highest oxidation state of +4. [2]
(b) Copper (I) compounds are white whereas Copper (II) compounds are coloured.
SECTION-C
node06\B0BC-BD\Kota\Board Material\Chemistry\Booklets\CBSE\Part-1

12. The elements of 3d transition series are given as: [3]


Sc Ti V Cr Mn Fe Co Ni Cu Zn
Answer the following:
(i) Write the element which is not regarded as a transition element. Give reason.
(ii) Which element has the highest m.p ?
(iii) Write the element which can show an oxidation state of +1.
258 E
CBSE
ALLEN
13. Following are the transition metal ions of 3d series : [3]
Ti4+, V2+, Mn3+, Cr3+
(Atomic numbers : Ti = 22, V =23, Mn = 25, Cr = 24)
Answer the following :
(i) Which ion is most stable in an aqueous solution and why ?
(ii) Which ion is a strong oxidising agent and why ?
(iii) Which ion is colourless and why ?
SECTION-D
14. The rare earth elements of the modern periodic table are known as lanthanoids. They have
separate block in periodic table. The lanthanoid series consist of fourteen elements starting from
Cerium (atomic number - 58) to Lutetium (atomic number - 71). All lanthanoids generally
exhibit +3 oxidation state. In addition some lanthanoids show +2 and +4 oxidation state also. As
we move from left to right in lanthanoid series there is regular decrease in the size of an atom.
This is known as lanthanoid contraction. There are many industrial application as - formation of
misch metal, production of parts of Jet engine. [1+1+2=4]
(i) Write the name of any one lanthanoid element exhibiting +4 oxidation state.
(ii) Write name of two lanthanoid elements used in the formation of misch metal.
(iii) The basic nature of hydroxides of lanthanoid elements decreases moving from left to right.
Explain
OR
(i) What is the Lanthanoid contraction ?
(ii) The first ionisation energies of the 5d transition elements are higher than those of the 3d
and 4d transition elements in respective groups ?
SECTION-E
15. (a) Account for the following : [5]
(i) Transition metals show variable oxidation states.
(ii) Zn, Cd and Hg are soft metals.
(iii) Eo value for the Mn3+/Mn2+ couple is highly positive (+1.57 V) as compared to
node06\B0BC-BD\Kota\Board Material\Chemistry\Booklets\CBSE\Part-1

Cr3+/Cr2+
(b) Write one similarity and one difference between the chemistry of lanthanoid and actinoid
elements.

E 259
Chemistry
ALLEN
PRACTICE TEST SOLUTIONS
SECTION-A
1. (d)
2.
3. (a)
4. (c)
5. (d)
6. (c)
7. (c)
SECTION-B
8. This is because F can form only one bond and structure of MnF4 is which is which is tetrahedral
but
F

F Mn F
F

In Mn2O7 as 0 is present which can form 2 bonds or 1 and 1 bond so 0 can be present in
O O
Mn Mn
O O O
O
O

Such budge cannot


between two Mn atoms giving it a structure. be formed by F

9. Mn+3 due to four unpaired electron in their 3d orbital.


10. (a) Due to d - d transition / due to presence of unpaired electrons in d - orbitals which absorb
light in visible region.

(b) Due to variable oxidation state/unpaired electrons.


11. (a) Ability of oxygen to form multiple bond.
(b) Cu+1 (3d10) compounds are white because of absence of unpaired electrons while Cu+2 (3d9)
compounds are coloured due to unpaired e¯/shows d-d transition.
SECTION-C
node06\B0BC-BD\Kota\Board Material\Chemistry\Booklets\CBSE\Part-1

12. (i) Zn, because of not having partially filled d-orbital in its ground state or ionic state.
(ii) Cr
(iii) Cu
13. (i) Cr3+, half filled t32g
(ii) Mn3+, due to stable d5 configuration in Mn2+
(iii) Ti4+, No unpaired electrons
260 E
CBSE
ALLEN
SECTION-D
+4
14. (i) Cerium (Ce )
(ii) Ce(40.5%), La + neodymium (44%)
(iii) Due to the lanthanoid contraction, the covalent nature of Ln––OH bond decreases as the
basic character of hydroxides decreases from La(OH)3 to Lu(OH)3.
OR
(i) Lanthanoid contraction : In lanthanoid series when atomic number is increase then
atomic and ionic radii is decreases from one element to another element. It is known as
lanthanoid contraction.
(ii) It is due to lanthanoid contraction, which is due to poor shielding effect of 5d and 4f
electrons, effective nuclear charge increases. Hence ionisation energy of 5d transition
elements is more than that of 3d and 4d transition elements in respective groups.
SECTION-E
15. (a) (i) Availability of partially filled d-orbitals/comparable energies of ns and (n-1) d
orbitals
(ii) Completely filled d-orbitals/absence of unpaired d electrons cause weak metallic
bonding
(iii) Because Mn2+ has d5 as a stable configuration whereas Cr3+ is more stable due to
stable t32g
(b) Similarity-both are stable in +3 oxidation state/both show contraction/irregular electronic
configuration (or any other suitable similarity) Difference-actinoids are radioactive and
lanthanoids are not/actionoids show wide range of oxidation states but lanthanoids don't (or
any other orrect difference)
node06\B0BC-BD\Kota\Board Material\Chemistry\Booklets\CBSE\Part-1

E 261
Chemistry
ALLEN
UNIT-9 : COORDINATION COMPOUNDS
UNIT INDEX
• Theory 262-277
• Exercise-1_Intext Questions 278-282
• Exercise-2_NCERT Exercise Questions 283-299
• Exercise-3_Exemplar 300-305
• Exercise-4_MCQ, A/R, Case Based Questions 306-313
• Previous Years Questions 314-324
• Practice Test 325-328

THEORY
INTRODUCTION
(a) The concept of coordination compounds arises from the complex formation tendency of transition
elements.
(b) These compounds play a vital role in our lives. Haemoglobin of blood and chlorophyll of plants
are also coordination compounds of Fe, Mg respectively.
TYPE OF COMPOUNDS
Molecular or Addition Compounds
(a) When two or more simple salts are chemically combined together in fixed proportion by weight
the molecular or addition compounds are formed.
(b) Some common examples are as follows.
K2SO4 + Al2(SO4)3 + 24H2O K2SO4.Al2(SO4)3.24H2O
Alum
(c) Depending upon the behavior in an aqueous solution they are of two types
(i) Double salt (ii) Coordination or complex compounds.
(I) Double Salt
The addition compounds which loose their identity in aq. solution and get completely ionised in
water and give positive test of their constitute ions.
Ex. (i) K2SO4.Al2(SO4)3.24H2O (Potash Alum)
(ii) FeSO4(NH4)2SO4.6H2O (Mohr's salt)
(iii) KCl.MgCl2.6H2O (Carnallite)
KCl.MgCl2.6H2O 
H2O
 K+ + Mg2+ + 3Cl–
(II) Coordination or Complex Compounds
node06\B0BC-BD\Kota\Board Material\Chemistry\Booklets\CBSE\Part-1

(a) Those molecular or addition compounds which retain their identity in aq. solution or loose
the identity of constituents are called complex salt.
(b) Complexes retain their identity in aqueous solution. i.e., they do not dissociate into separate
ion.
K 4 [Fe(CN)6 ] 
H2 O
 4K   [Fe(CN)6 ]4
Potassium ferrocyanide Ferrocyanide ion

262 E
CBSE
ALLEN
(c) On the basis of type of ligands complex compounds are divided as follows :
(i) Homoleptic complexes :
Complex in which all the ligands are identical is called homoleptic complex.
Ex. [Co(NH3)6]+2
(ii) Heteroleptic complexes :
Complex in which all the ligands are not identical is known as heteroleptic complex.
Ex. [Fe(en)2Cl2]+1
TERMINOLOGY OF COMPLEX COMPOUNDS
(a) Complex Ion/Coordination sphere
• An aggregate of metal ion with anions, cation or neutral molecules is called as complex
ion.
• It is a non ionisable part of complex compound.
(b) Central Metal Ion
(i) The metal ion which forms complex ion in combination with anions, cation or neutral
molecules is called as central metal ion.
(ii) Central metal ion acts as an electron pair acceptor and forms coordinate bonds with donor
species.
(iii) Generally belongs to d block because d block elements have tendency to form complex due
to their high charge density and availability of vacant orbitals.
(c) Coordination Number
The total number of coordinate bonds formed by central metal ion with donor species is called as
coordination number of central metal ion.
(d) Ionisation Sphere
The part of complex compound which get ionised is known as ionisation sphere.
Central metal ion Coordination sphere

[Cu(NH3)4]SO4 Ionization sphere/Counter ion

Ligand Coordination number

(e) Oxidation State


It is a number which represents the electrical charge on the central metal atom of a complex ion.
Ex. the oxidation number of Fe, Co and Ni in [Fe(CN)6]4—, [Co(NH3)6]3+ and Ni(CO)4 is +2, +3
and zero respectively.
(f) Charge On the Complex Ion
node06\B0BC-BD\Kota\Board Material\Chemistry\Booklets\CBSE\Part-1

It is the algebraic sum of the total charge of the donor species and central metal ion.
(g) Ligands
(i) The anions, cation or neutral molecules which combine with central metal ion to form
complex ion are called as ligands.
(ii) They act as electron pair donor (Lewis bases) which donate e– pair to central metal ion and
form coordinate bonds.
(iii) no. of e– pair donated by a ligand to central metal ion is equal to its denticity.
E 263
Chemistry
ALLEN
(h) Naming of ligands
(i) o-suffix is used in the name of anionic ligands.
(ii) ium-suffix is used in the name of cationic ligands.
Classification of ligands :
On the basis of denticity :
1. Monodentate ligands : Which donate one e– pair to the central metal ion.
Neutral monodentate ligands
NH3 ammine
H 2O aqua
NO nitrosyl
PH3 phosphine
PPh3 triphenyl phosphine
O2 dioxygen
N2 dinitrogen
CH3–OH methyl alcohol
CH3–NH2 methyl amine
NH2CONH2 urea
NH2NH2 hydrazine
CH3–O–CH3 dimethyl ether
CO carbonyl
CS thiocarbonyl

Cationic monodentate ligands :


+
O2 oxygenium
NO+ nitrosylium or nitrosonium
NO2+ nitronium

hydrazinium
H2 N  N H3

Anionic monodentate ligands :



F Fluorido / fluoro
Cl– Chlorido / chloro
Br– bromido / bromo
I– iodido / iodo
O–2 oxido/oxo
S–2 sulphido/sulpho
OH– hydroxido / hydroxo
node06\B0BC-BD\Kota\Board Material\Chemistry\Booklets\CBSE\Part-1

O 2– superoxido / superoxo
O2–2 peroxido / peroxo
N–3 nitrido
N3– azido
(NH2)– amido
NH–2 imido
H– hydrido

264 E
CBSE
ALLEN
2. Bidentate ligands : Ligands which donates two e– pair to the central metal ion.
Ligands having denticity 2 or more than 2 form a stable ring with central metal ion that's why
they are known as chelating ligands.
no. of rings = Denticity – 1

O O
NH2 CH2 CH2 NH2
N N M+

O O
M+ M+
Ethylene diamine (en) dipyridyl(dipy) oxalato(ox) or C2O4–2

O H
H2N CH2 C O

O CH3 C CH C CH3 CH3 C N


O O M+
M + CH3 C N

glycinato(gly) M+ O
(unsymmetrical) acetyl acetonato (acac)– dimethyl glyoximato(dmg)–

3. Polydentate ligands

CH2 CH NH CH2 CH CH2–CH2–NH–CH2–CH2–NH–CH2–CH2

H2 N NH2 H2N NH2


+
M
+
M
Diethylenetriamine(dien) Triethylenetetraamine (trien)
Tridentate Tetradentate

OOC CH2 CH2 COO


N CH2 CH2 N
OOC CH2 CH2 COO
4–
Ethylene diaminetetraacetato (EDTA )
Hexadenatate

4. Ambidentate ligand : Ligands which have two different donor atoms but at one time one donor
atom can donate electron pair are called ambidentate ligands.
Ambidentate ligands have same molecular formula but have different donor atom.
(i) MCN cyanido (CN)
MNC isocyanido(NC)
(ii) M  O  C  N cyanato / cyanato-O
node06\B0BC-BD\Kota\Board Material\Chemistry\Booklets\CBSE\Part-1

MN CO isocyanato / cyanato-N


(iii) M  S  C  N Thiocyanato / Thiocyanato-S
MN CS Isothiocyanato/thiocyanato-N
(iv) M  O  N  O Nitrito / Nitrito-O
O
M N
O Nitro / Nitrito-N

E 265
Chemistry
ALLEN
Synergic bonding:-
(a)  donor and acceptor ligands
Ligands which donate their lone pair to central metal ion through  bond and simultaneously
accept e– pair from central metal ion through  bond.
The electronic configuration of CO molecule shows that it has lone pair of electrons on carbon
and oxygen atom each. Carbon atom can donate its electron pair of a transition metal atom (M),
forming OCM coordinate bond.
Since the metal atom in metal carbonyl is in zero oxidation state, the formation of M  CO 
bond accumulates a negative charge on the metal atom. The accumulation of negative charge on
the metal atom can be counter balanced by transferring some negative charge from the metal
atom to CO molecule (ligand). This transfer can be done by making a M  CO  bond by the
overlap between an appropriate filled d orbital on the metal atom and empty y or z molecular
orbital of CO molecule. This type of bonding between M and CO is called synergic bonding.
– M + + C  O  – M 
C O   bond

– +  bond / back bond



+ – –
 formed by full filled
M C O M CO 

d orbital of M and
+ + Vacant * 2p orbital of CO
+
+ – –

[Schematic of orbital overlaps in metal carbonyls]

Conclusion of synergic bonding :


(a) M–C bond strength increases; M–C bond length decreases; because double bond character
increases
(b) C–O bond strength decreases; C–O bond length increases; because bond order of C–O
decreases (CO accepts e– pair into its antibonding molecular orbital)
IUPAC NOMENCLATURE OF COORDINATION COMPOUNDS
Complex Compounds IUPAC Name

(i) K4[Fe(CN)6] Potassium hexacyanidoferrate (II)


(ii) K2[HgI4] Potassium tetraiodidomercurate (II)
(iii) [Co (NH3)6] Cl3 Hexamminecobalt (III) chloride
(iv) [Cr(H2O)4Cl2] Cl Tetraaquadichlorochromium (III) chloride
(v) [Pt(NH3)2Cl4] Diammine tetrachloroplatinum (IV)
(vi) [Co(NH3)3 Cl3] Triamminetrichlorocobalt (III)
node06\B0BC-BD\Kota\Board Material\Chemistry\Booklets\CBSE\Part-1

(vii) K3[Co(NO2)6 ] Potassium hexanitrocobaltate (III)


(viii) Na2[Fe(CN)5 NO] Sodium pentacyanonitrosoniumferrate (II)
(ix) [NiCl4]–2 Tetrachloronickelate (II) ion
(x) [Ru(NH3)5Cl]+2 Pentamminechlororuthenium (III) ion
(xi) [Fe(en)3]Cl3 Tris (ethylenediamine) iron (III) chloride
(xii) [Ni (Gly)2 ] Bis (glycinato) nickel (II)

266 E
CBSE
ALLEN
BONDING IN COORDINATION COMPOUNDS
(A) WERNER'S THEORY
This was postulated by Alfred Werner in 1893. The postulates of Werner's theory of complexes
are –
According to Werner central metal atom in a complex compound have 2 types of valencies :
Primary valencies Secondary valencies
(i) Equal to oxidation state of CMA (i) Equal to coordination number of CMA
(ii) Satisfied by anions (ii) satisfied by ligands
(iii) Non directional (iii) Directional
(iv) Ionisable (iv) Non-ionisable
(v) not helpful in geometry prediction of (v) helpful in geometry prediction of
complex ion complex ion

Experimental evidence of werner's theory


(a) Precipitation of primary valencies on the addition of suitable reagent. In a series of compounds
of cobalt (III) chloride with ammonia, it was found that some of the chloride ions could be
precipitated as AgCl on adding silver nitrate solution but some remained in solution.
1 mole CoCl36NH3 
AgNO
3 moles of AgCl
3

white ppt
Hence CoCl36NH3 is formulated as [Co(NH3)6]Cl3, which ionises as [Co(NH3)6]+3 + 3Cl–
(4 mole ions)
1 mole CoCl35NH3 
AgNO
2 moles of AgCl
3

hence CoCl35NH3 is formulated as [Co(NH3)5Cl]Cl2, which ionises as [Co(NH3)5Cl]+2 + 2Cl–


(3 mole ions)
1 mole CoCl34NH3 
AgNO
1 moles of AgCl
3

hence CoCl34NH3 is formulated as [Co(NH3)4Cl2]Cl, which ionises as [Co(NH3)4Cl2]+ + Cl–


(2 mole ions)
(B) VALENCE BOND THEORY
This theory was mainly developed by Linus Pauling. The main features of this theory are -
(a) Metal ion when it forms a complex compound undergoes formation of coordinate bond.
(b) During this bond formation, the metal ion acts as electron pair acceptor. For this the metal
ion provides vacant orbitals according to its coordination number.
node06\B0BC-BD\Kota\Board Material\Chemistry\Booklets\CBSE\Part-1

Ex. In the formation of [Fe(NH3 )6 ]3+, Fe+3 ion provides six vacant orbitals.
In [Cu(NH3 )4 ]2+, Cu+2 ion provides four vacant orbitals.
(c) These vacant orbitals undergo hybridisation before bond formation with ligands.
(d) The vacant hybrid orbitals of metal ion get overlapped with orbitals of ligands containing
lone pair of electrons.

E 267
Chemistry
ALLEN
(e) If in a complex strong ligand is present then it will cause pairing of unpaired e– in (n–1)d
orbitals of central metal ion and if (n–1)d orbitals are involved in hybridisation then inner
orbital complex is formed.
(f) If in the complex weak ligand is present then there will be no pairing of unpaired e– in
(n–1)d orbitals and if nd orbitals are involved in hybridisation, outer orbital complex is
formed.
(g) If unpaired e– is present in the complex then it will be paramagnetic otherwise it will be
diamagnetic.
Applications of valence bond Theory
(a) 6-coordinated complexes :
  [Co(NH3 )6 ]3+
e– configuration of 27Co = [Ar] 3d7 4s2 So, Co+3 = [Ar] 3d6 4s° 4p° 4d°
Co3+ =
3d 4s 4p 4d

Due to presence of strong ligand unpaired electrons get paired up so metal ion provides vacant
'3d' orbitals for hybridisation.

3d 4s 4p 4d
2 3
d sp hybridisation

 This is a diamagnetic complex.


 As inner 'd' orbitals are involved in hybridisation, hence it is an inner orbital complex.
 [CoF6]3–
e– configuration of Co27 = [Ar] 3d74s2  Co+3 = [Ar] 3d6 4s0 4p0 4d0
Co3+ =
 F– is a weak ligand, so unpaired e– do not get paired up and vacant 4d orbitals
involved in hybridisation.

3d 4s 4p 4d
3 2
sp d

 This is a paramagnetic complex.


 The outer 'd' orbitals are involved in hybridisation, hence it is an outer orbital complex.
(b) 4-coordinated complexes :
 [NiCl4]2–
node06\B0BC-BD\Kota\Board Material\Chemistry\Booklets\CBSE\Part-1

e– configuration of 28Ni = [Ar] 3d8 4s2


 Ni+2 = [Ar] 3d8 4s0 4p0
 There will be no pairing of unpaired e– in 3d orbitals.

3d 4s 4p
sp3 hybridisation

268 E
CBSE
ALLEN
 [NiCl4]2— has tetrahedral geometry.
 It is a paramagnetic complex.
 [Ni (CN)4]2–
8 2
28Ni = [Ar] 3d 4s Ni+2 = [Ar] 3d84s04s0
CN– is a strong ligand so unpaired e– in 3d orbitals get paired up

3d 4s 4p
2
dsp hybridisation
 [Ni(CN)4]2– has square planer geometry and diamganetic in nature.
 This complex is an inner orbital complex
 [Zn(NH3)4]2+
Zn = [Ar] 4s2 3d10
 Zn+2 = [Ar] 3d104s04p0
 Zn+2 has d10 configuration
So there is no possibility of pairing

3d 4s 4p
sp3

 Because the complex is formed by sp3 hybridisation, hence it has tetrahedral geometry.
 Since all electrons are paired, hence it is diamagnetic
 [Ni(CO)4]
Ni = [Ar] 3d8 4s2 4p°
 CO is a strong ligand so unpaired e– in (n–1)d orbitals get paired up.
Ni =
3d 4s 4p
3
sp hybridisation
3
 Complex has sp hybridisation
 It has tetrahedral geometry.
 It is diamagnetic complex as unpaired e– is absent.
 [Cu(NH3)4]2+
1
29Cu = [Ar] 4s 3d
10
 Cu+2 = [Ar] 3d9 4s0 4p0
  NH3 is a strong ligand so after rearrangement
 Complex has dsp2 hybridsation
node06\B0BC-BD\Kota\Board Material\Chemistry\Booklets\CBSE\Part-1

 It is inner orbital complex


 It has square planer geometry
 It is paramagenetic

3d 4s 4p
2
dsp

E 269
Chemistry
ALLEN
(C) Crystal field theory
Crystal Field Theory : This is a model of electronic structure of transition-metal complexes that
considers how the energies of the d-orbitals of a metal ion are affected by the electric field of the
ligand. According to this theory.
(a) Central metal ion is considered as a positive point charge, anionic ligand as a negative
point charge, neutral ligand as a point dipole and attraction between metal ion and ligand is
electrostatic in nature.
(b) In the electric field of ligands, the five d orbitals of the metal atom no longer have exactly
same energy while they are splitted into different energy levels. Splitting of five degenerate
d-orbitals of the metal ion into t2g and eg sets of orbitals having different energies is called
crystal field splitting.
(c) The extent of splitting of metal d-orbitals depends upon the nature and number of ligands
surrounding it and the charge on the central metal ion.
(d) The extent of splitting of metal d-orbitals determines the magnetic and spectroscopic
properties of the complexes.
SPLITTING IN OCTAHEDRAL COMPLEXES
In a octahedral complex, the co-ordination number is 6. The metal ion is at the centre and the
ligands occupy the six corners of the octahedron as shown in figure.
We know that two orbitals, d x  y and d z are oriented along the axis while the remaining three
2 2 2

orbitals, viz., dxy, dyz and dzx are oriented in between the axis.
Z

L L L
X
L

Y L

The two orbitals d x y and d z are designated as eg orbitals while the three orbitals dxy, dyz and dzx
2 2 2

are designated as t2g orbitals. As the six ligands approach the central ion along the axis hence eg
orbitals, is repelled more by the ligand than the t2g orbitals.
In other words, the energy of the eg orbitals increases much more than the energy of the t2g
orbitals.
Thus, in octahedral complexes, the five d-orbitals split up into two sets : one set consisting of two
orbitals ( d x -y and d z ) of higher energy (eg orbitals) and the other set consisting of three orbitals
2 2 2
node06\B0BC-BD\Kota\Board Material\Chemistry\Booklets\CBSE\Part-1

(dxy, dyz and dzx) of lower energy (t2g orbitals).


eg
0.6 
 = crystal field splitting
–0.4  energy of octahedral complex
State II t2g

State III

State I
270 E
CBSE
ALLEN
Splitting in tetrahedral complex
The co-ordination number for tetrahedral complexes is 4. The tetrahedral arrangement of four
ligands surrounding a metal ion may be visualized by placing ligands at the alternate corners of a
cube, as shown in figure.
Z
L
L
M+ X
L
Y L
Tetrahedral arrangement
of four lignads
It can be shown that in a tetrahedral structure, none of the d-orbitals points exactly towards the
ligands.
When ligand approaches it is more close dxy, dyz, dxz in comparision of d x y and d z because dxy, 2 2 2

dyz, dzx are between the axis and d z and d x  y are along the axis. So dxy, dyz, dzx feels more
2 2 2

repulsion as compare to d z and d x  y .


2 2 2

Thus, the d orbitals are also splitted into two groups but in a reverse order of octahedral complex.
The three orbitals, dxy, dyz and dxz, designated are now have higher energy than the two orbitals
d x  y and d z .
2 2 2

t2g
0.4  t 

= Spitting energy of tetrahedral complex


t
State II –0.6  t 

eg
State I State III

Calculation of crystal field stablizing energy (CFSE) 


For octahedral complex CFSE =  –0.4  n t   0.6  n   + Pairing energy (P.E.)
 2g eg 0

where n t = number of electron in t2g orbitals


2g

n eg = number of electron in eg orbitals


 0 = crystal field splitting energy
Factors affecting splitting energy ()
(i) Strength of ligand [C.F.S.E. is more in case of S.F.L. as compared to W.F.L.]
(ii) Oxidation state of central metal ion [C.F.S.E.  oxidation state]
(iii) Transition series (d-series) [C.F.S.E. , 5d > 4d > 3d]
(iv) Geometry [sp > 0 > t]
node06\B0BC-BD\Kota\Board Material\Chemistry\Booklets\CBSE\Part-1

(v) number of ligands


4 4
sp= 0 t= 0
3 9
Spectro-chemical series for ligands
Series which shows the relative strength of ligands
I–(weakest) < Br– < SCN– < Cl– < S–2 > F– < OH– < C2O42– < H2O < NCS–
< edta4– < NH3 < en < NO2– < CN– < CO(strongest)

E 271
Chemistry
ALLEN
APPLICATION OF CRYSTAL FIELD THEORY
Applications of C.F.T are
(i) To explain weather complex is high spin or low spin complex.
(ii) To explain colour property
(iii) To explain stability of complex
(II) COLOUR PROPERTY OF COMPLEX COMPOUNDS :
Reason for colour in complexes is due to d-d-transition
0 1
eg eg
energy

d-d transition

0
1
t2g t2g
ground state excited state

P V I
R B
Commentary colour wheel O
For example complementary colour of red is blue
BG
YG

Y G

green.
(III) STABILITY OF CO-ORDINATION COMPOUNDS
According to thermodynamic stability, the reaction between a metal ion and the ligands may be
considered as a Lewis acid base reaction. If the equilibrium constant is high then the
complex obtained is theromodynamically stable. The reaction can be written as follows :
M + nL  [MLn]
[ML n ]
The stability constant K, of the complex [MLn] is given by the relation, K = . The greater
[M][L]n
the value of K, more stable is the complex.
The stability of a complex ion also depends upon –
(i) Higher charge of the central metal ion.
(ii) Greater strength of the ligand.
(iii) Ring formation (chelation) in structure of complexes.
ISOMERISM IN COMPLEXES
Compounds which have the same molecular formula, but differ in their properties due to the
node06\B0BC-BD\Kota\Board Material\Chemistry\Booklets\CBSE\Part-1

difference in their structure are called as Isomers.


Classification of isomerism :
Type of isomerism

(A) Structural isomerism (B) Stereo isomerism

Ionization Hydrate Coordination Linkage Geometrical Optical

272 E
CBSE
ALLEN
(A) Structural Isomerism
It arises due to the difference in the type of chemical linkages and distribution of ligands within
and outside the coordination sphere.
(i) Ionisation Isomerism
Compounds having same molecular formula but give different type of ions in their aqueous
solution are known as ionisation isomer.
Due to exchange of ions between coordination sphere and ionisation sphere different ionisation
isomers are formed. So they are ionisation isomers.
Ex. (a) CoBrSO45NH3 can represent
(i) [Co(NH3)5Br] SO4 (red violet)
(ii) [Co(NH3 )5 SO4 ] Br (red).
These complexes give sulphate ion and bromide ion respectively
(b) [Pt(NH3)4Cl2] Br2 and [Pt(NH3 )4Br2]Cl2
(c) [Co(NH3)4(NO3)2 ]SO4 and [Co(NH3)4SO4](NO3)2
(ii) Hydrate Isomerism
This type of isomerism is due to presence of different number of water molecules inside the
complex ion.
Ex. (a) Cr(H2O)6Cl3 has three possible hydrate isomers.
(i) [Cr(H2O)6]Cl3 (violet)
(ii) [Cr(H2O)5Cl] Cl2 .H2O (green)
(iii) [Cr(H2O)4Cl2]Cl . 2H2O (dark green)
These complexes differ from one another with respect to the number of water molecules acting as
ligands.
(iii) Linkage Isomerism
(a) This type of isomerism arises due to presence of ambidentate ligands
(b) These ligands have two donor atoms but at a time only one atom is directly linked to the
central metal atom of the complex.
Ex. [Co(NH3)5NO2]Cl2 and [Co(NH3)5 ONO]Cl2
In NO2– ligand, the donor site is nitrogen while in ONO– ligand donor site is oxygen.
(iv) Coordination Isomerism
(a) This type of isomerism is exhibited when in complex compound both cation as well as anion is
complex.
(b) This type of isomerism is caused by the interchange of ligands between the two complex
ions of a complex compound.
node06\B0BC-BD\Kota\Board Material\Chemistry\Booklets\CBSE\Part-1

Ex. (i) [Zn(NH3 )4   ZnCl 4 ] 


 Coordination isomers
(ii) [Zn(NH3 )3 Cl   ZnCl 3 (NH3 )]

(B) Stereo Isomerism


(a) They have same molecular formula, same constitution but they differ only with respect to
the spatial orientation of ligands in space around the metal ion.
(b) The two types of stereo isomers which are possible are - Geometrical and optical.

E 273
Chemistry
ALLEN
(i) Geometrical Isomerism
(a) Due to change in relative positions of ligands around central metal ion, different forms are
obtained which are known as GI.
(b) When two identical ligands are coordinated to the metal ion from same side then it is
cis isomer. (Latin, cis means same).
(c) If the two identical ligands are coordinated to the metal ion from opposite side then it is
Trans isomer (in Latin,Trans means across).
(d) These geometrical isomers differ in physical as well as in chemical properties.
(e) Geometrical isomerism is most important in compounds with coordination numbers
4 and 6.
(f) 4-coordinated complexes with tetrahedral geometry do not exhibit cis - Trans isomerism,
because relative position of ligands remain same for each other.
(g) It is exhibited by 4-coordinated complexes with square planar geometry.
Note : There are only 5 possible cases which shows geometrical isomerism in square planar
complex. such as Ma2b2, Ma2bc, Mabcd, M(AB)2, M(AB)cd
Geometrical Isomers With Coordination Number = 4 (Square Planar Complexes)
(i) Complexes with general formula, Ma2b2 (where both a and b are monodentate) can have
Cis-and trans isomers.
a a a b
M M
b b b a
Cis-isomer Trans-isomer

Ex. [Pt (NH3)2Cl2]


H3N NH3
Cl NH3
Pt
Pt
Cl Cl
cis-platin H3N Cl
(used as a anti cancer) Trans-platin

(ii) Complexes with general formula Ma2bc can have Cis - and trans-isomers.
a a a c
M M
b c b a
node06\B0BC-BD\Kota\Board Material\Chemistry\Booklets\CBSE\Part-1

Cis trans

eg. [Pt(NH3)2ClBr]
Cl NH3 Br NH3
Pt Pt

Br NH3 H3N Cl
Cis Trans

274 E
CBSE
ALLEN
(iii) Complexes with general formula, Mabcd can have three geometrical isomers.
eg. [Pt(NH3) (Py) ClBr]
a b a d a c
M M M
d c c b b d
(i) (ii) (iii)

(iv) Complex with general formula [M(AB)2] have 2 geometrical isomers (Where AB is
unsymmetrical Bidentate ligand)
eg. [Pt(gly)2]
CH2 NH2 NH2 CH2 CH2 NH2 O CO
Pt Pt
CO O O CO CO O NH2 CH2
(cis) Trans

(v) Complex with general formula M(AB)cd can have two geometrical isomers (Where AB is
unsymmetrical Bidentate ligand)
eg. [Pt(gly)(NH3)Br]
A c A d
M M
B d B c
(i) (ii)

Geometrical isomers with Coordination number = 6 (Octahedral complex)


(i) [Ma4b2] has 2 geometrical isomers
eg. [Fe(NH3)4Cl2]
Cl Cl
2+ +2
NH3 Cl
H3N H3N
Fe Fe
NH3 NH3
H3N H3N
Cl NH3
Trans Cis

(ii) [Ma4bc] has 2 geometrical isomers


eg. [Fe(NH3)4ClCO3]
Cl Cl
+3
CO3 +3 NH3
H3N H3N
Fe Fe
NH3 NH3
H3N H3N
NH3 CO
Cis Trans

(iii) Facial and Meridional isomerism (Ma3b3 )


node06\B0BC-BD\Kota\Board Material\Chemistry\Booklets\CBSE\Part-1

eg. [Cr(NH3)3(H2O)3]+3
b b
a b a
b
a
b a a
a b
Facial (fac) Meridional[Mer]

E 275
Chemistry
ALLEN
(B) Optical Isomers
(a) Compounds which are mirror image of each other but non superimposable, are known as
optical isomers.
(b) An optically active complex is one which is asymmetric in nature i.e., not divisible into two
identical halves.
(c) The complex which rotates plane polarised light to left hand side is laevo rotatory i.e. ''
or '–' and if the complex rotates the plane polarised light to right hand side then it is dextro
rotatory 'd' or '+'.
(d) Thus complexes which have same physical and chemical properties but differ in their
action towards plane polarised light are called as optical isomers.
(e) The 'd' and '' isomers of a compound are called as Enantiomers.

Optical isomers with Coordination number = 6


(i) [M(AA)3]n+ [Co(en)3]3+
3+ 3+
en en
en en

Co Co

en en
d-form Mirror -form

(ii) [M(AA)2ab]n+ [Co(en)2NH3Cl]2+


2+ 2+
en en
Cl Cl

Co Co

H3N NH3
en en
Mirror Cis--isomer
Cis-d-isomer

APPLICATION OF COORDINATION CHEMISTRY


(a) In photography : undecomposed AgBr (at the time of image development) is removed from
photographic film by using hyposolution [Na2S2O3.5H2O]
node06\B0BC-BD\Kota\Board Material\Chemistry\Booklets\CBSE\Part-1

h 1
AgBr   Ag  Br
2 2
3
AgBr  2Na2 S2 O3  Na 3  Ag  S2 O3 2   NaBr
(undecomposed) so lub le complex

(b) Light blue colour of aqueous solution of CuSO4 turn into dark blue in presence of ammonia
 Cu  H2 O 4  SO4 
4NH3
Cu  NH3 4  SO4
light blue dark blue

276 E
CBSE
ALLEN
(c) cyano complex are used for electroplating of Ag or Au due to high stability.
e.g. [Ag(CN)2]–
(d) Wilkinson catalyst [RhCl(PPh3)3] is used for hydrogenation of alkene. (Homogeneous catalysis)
(e) [EDTA]4– is used for estimation of hardness of water.
(f) [Ca(EDTA)]2– is used to remove Pb poison.
[Ca(EDTA)]2– + Pb+2 [Pb(EDTA)]2– + Ca+2
(g) Biological importance
Chlorophyll (Mg) haemoglobin (Fe+2) Insulin (Zn)
Vitamin B12(Co) Corboxypeptide (Zn)
node06\B0BC-BD\Kota\Board Material\Chemistry\Booklets\CBSE\Part-1

E 277
Chemistry
ALLEN
EXERCISE-1 INTEXT QUESTIONS
Q.1 Write the formulas for the following coordination compounds :
(i) Tetraamminediaquacobalt(III) chloride
(ii) Potassium tetracyanonickelate (II)
(iii) Tris (ethane-1,2-diamine) chromium(III) chloride
(iv) Amminebromideochloridonitrito-N-platinate(II)
(v) Dichloridobis(ethane-1,2-diamine)platinum(IV) nitrate
(vi) Iron(III) hexacyanoferrate(II)
Ans. (i) Co(H 2O)2 (NH 3 ) 4  Cl3 (ii) K2[Ni(CN)4]

Cr(en)3  Cl3 (iv)  Pt(NH)3 BrCl(NO2 )



(iii)
(v)  PtCl2 (en)2  (NO3 )2 (vi) Fe 4  Fe(CN) 6 3
Q.2 Write the IUPAC name of the following coordination compounds :
(i) Co  NH3 6  Cl3 (ii) Co  NH3 5 Cl  Cl2

(iii) K3  Fe  CN 6  (iv) K 3  Fe  C2O 4 3 


(v) K 2  PdCl4  (vi)  Pt(NH 3 ) 2Cl(NH 2CH 3 )  Cl
Ans. (i) Hexamminecobalt(III) chloride
(ii) Pentaamminechloridocobalt(III) chloride
(iii) Potassium hexacyanoferrate(III)
(iv) Potassium trioxalatoferrate(III)
(v) Potassium tetrachloridopalladate(II)
(vi) Diamminechlorido(methylamine)platinum(II) chloride
Q.3 Indicate the types of isomerism exhibited by the following complexes and draw the structures for
these isomers :
(i) K[Cr(H2O)2(C2O4)2 (ii) [Co(en)3]Cl3
(iii) [Co(NH3)5(NO2)](NO3)2 (iv) [Pt(NH3)(H2O)Cl2]
Ans. (i) Both geometrical (cis-, trans-) isomers for K  Cr(H 2O) 2 (C 2O 4 ) 2  can exist. Also, optical
isomers for cis-isomer exist.
Geometrical isomers
OH2 OH2
XO OX XO OH2
Cr Cr
XO OX , XO OX
OH2 XO
node06\B0BC-BD\Kota\Board Material\Chemistry\Booklets\CBSE\Part-1

Trans Cis

Trans-isomer is optically inactive. On the other hand, cis-isomer is optically active.


OH2 OH
XO OH2 OH2 2 OX
Cr Cr
XO OX XO OX
XO OX

278 E
CBSE
ALLEN
(ii) Two-optical isomers for  CO(en)3  Cl3 exist.
CH2
H2N CH2 N
CH2 – H2N NH2 N N
Co Co
CH2 – H2N NH2 N N
H2N CH2 N
CH2
Two optical isomers are possible for this structure

N N
N N N N
Co Co
N N N N
N N

(iii) CO(NH3 )5 (NO 2 ) (NO3 )2


A pair of optical isomers
NH3 NH3
O2N NH3 H3N NO2
Co Co
H3N NH3 H3N NH3
NH3 NH3

It can also show linkage isomerism


CO(NH 3 )5 (NO 2 ) (NO3 )2 and CO(NH 3 )5 (ONO) (NO3 ) 2
It can also show ionization isomerism
Co(NH3 )5 (NO2 ) (NO3 )2 and  CO(NH 3 )5 (NO 3 )  (NO 3 ) (NO 2 )

(iv) Geometrical (cis-, trans-) isomers of  Pt(NH 3 ) (H 2O) Cl 2  can exist.

Cl NH3 H3N Cl
Pt Pt
Cl H2O Cl H2O
Cis Trans

Q.4 Give evidence that [Co(NH3)5Cl]SO4 and [Co(NH3)5SO4]Cl are ionization isomers.
Ans. When ionization isomers are dissolved in water, they ionize to give different ions. These ions
then react differently
[CO(NH3)5Cl]SO4 + Ba2+ BaSO4 
node06\B0BC-BD\Kota\Board Material\Chemistry\Booklets\CBSE\Part-1

White precipitate
[CO(NH3)5Cl]SO4 + Ag No reaction
+

[CO(NH3)5SO4]Cl + Ba2+ No reaction


[CO(NH3)5SO4]Cl + Ag+ AgCl 
White precipitate

E 279
Chemistry
ALLEN
Q.5 Explain on the basis of valence bond theory that [Ni(CN)4]2– ion with square planar structure is
diamagnetic and the [NiCl4]2– ion with tetrahedral geometry is paramagnetic.
Ans. Ni is in the +2 oxidation state i.e., in d8 configuration.
d8 configuration :

3d 4s 4p 4d
There are 4 CN– ions. Thus, it can either have a tetrahedral geometry or square planar geometry.
Since CN– ion is a strong field ligand, it causes the pairing of unpaired 3d electrons.

3d 4s 4p 4d

It now undergoes dsp2 hybridization. Since all electrons are paired, it is diamagnetic.
In case of [NiCl4]2–, Cl– ion is a weak field ligand. Therefore, it does not lead to the pairing of
unpaired 3d electrons. Therefore, it undergoes sp3 hybridization.

3d 4s 4p 4d

Since there are 2 unpaired electrons in this case, it is paramagnetic in nature.


Q.6 [NiCl4]2– is paramagnetic while [Ni(CO)4] is diamagnetic though both are tetrahedral. Why?
Ans. Though both [NiCl4]2– and [Ni(CO)4] are tetrahedral, their magnetic characters are different. This
is due to a difference in the nature of ligands. Cl– is a weak field ligand and it does not cause the
pairing of unpaired 3d electrons. Hence, [NiCl4]2– is paramagnetic.

Ni+2

3d 4s 4p 4d

In Ni(CO)4, Ni is in the zero oxidation state i.e., it has a configuration of 3d8 4s2.

3d 4s 4p 4d
But CO is a strong field ligand. Therefore, it causes the pairing of unpaired 3d electrons. Also, it
causes the 4s electrons to shift to the 3d orbital, thereby giving rise to sp3 hybridization. Since no
node06\B0BC-BD\Kota\Board Material\Chemistry\Booklets\CBSE\Part-1

unpaired electrons are present in this case, [Ni(CO)4] is diamagnetic.


Q.7 '[Fe(H2O)6]3+ is strongly paramagnetic whereas [Fe(CN)6]3– is weakly paramagnetic. Explain.

Ans. In both  Fe(H 2O) 6  and  Fe(CN)6  , Fe exists in the +3 oxidation state
3 3

i.e., in d5 configuration.

d5
280 E
CBSE
ALLEN
Since CN– is a strong field ligand, it causes the pairing of unpaired electrons. Therefore, there is
only one unpaired electron left in the d-orbital.

Therefore,  = n(n  2) = 1(1  2) = 3 = 1.732 BM


On the other hand, H2O is a weak field ligand. Therefore, it cannot cause the pairing of electrons.
This means that the number of unpaired electrons is 5.
Therefore,   n(n  2) = 5(5  2) = 35 = 6 BM

Thus, it is evident that  Fe(H 2O) 6  is strongly paramagnetic, while  Fe(CN)6  is weakly
3 3

paramagnetic.
Q.8 Explain [Co(NH3)6]3+ is an inner orbital complex whereas [Ni(NH3)6]2+ is an outer orbital
complex.
Ans. [Co(NH3)6]3+ [Ni(NH3)6]2+
Oxidation state of cobalt = +3 Oxidation state of Ni = +2 
Electronic configuration of cobalt = d6 Electronic configuration of Ni = d8 

3d 4s 4p 4d 3d 4s 4p 4d

NH3 being a strong field ligand causes the If NH3 causes the pairing, then only one 3d orbital is
2 3
pairing. Therefore, Cobalt can undergo d2sp3 empty. Thus, it cannot undergo d sp hybridization.
hybridisation Therefore, it undergoes sp3d2 hybridization.

3d 4s 4p 4d 3d 4s 4p 4d
d2sp3 sp3d2
Hence, it is an inner orbital complex. Hence, it forms an outer orbital complex.

Q.9 Predict the number of unpaired electrons in the square planar [Pt(CN)4]2– ion.

Ans.  Pt(CN)6 
2

In this complex, Pt is in the +2 state. It forms a square planar structure. This means that it
undergoes dsp2 hybridization. Now, the electronic configuration of Pt(+2) is 5d8.
node06\B0BC-BD\Kota\Board Material\Chemistry\Booklets\CBSE\Part-1

3d8
CN– being a strong field ligand causes the pairing of unpaired electrons. Hence, there are no
unpaired electrons in  Pt(CN) 4  .
2

E 281
Chemistry
ALLEN
Q.10 The hexaquo manganese(II) ion contains five unpaired electrons, while the hexacyanoion
contains only one unpaired electron. Explain using Crystal Field Theory.
Ans. [Mn(H2O)6]2+ [Mn(CN)6]4–
Mn is in the +2 oxidation state. Mn is in the +2 oxidation state. 
The electronic configuration is d5. The electronic configuration is d5. 
The crystal field is octahedral. Water is a The crystal field is octahedral. Cyanide is a
weak field ligand. Therefore, the arrangement strong field ligand. Therefore, the arrangement
of the electrons in  Mn(H 2O)6  of the electrons in  Mn(CN)6 
2 4
is t 32g eg2 . is t 52g eg0 .

Q.11 Calculate the overall complex dissociation equilibrium constant for the Cu(NH 3 ) 24 ion, given
that 4 for this complex is 2.1 × 1013.
Ans. 4 = 2.1 × 1013
The overall complex dissociation equilibrium constant is the reciprocal of the overall stability
constant 4.
1 1
 = 4.7 × 10–14
4 2.11013

node06\B0BC-BD\Kota\Board Material\Chemistry\Booklets\CBSE\Part-1

282 E
CBSE
ALLEN
EXERCISE-2 NCERT EXERCISE
1. Explain the bonding in coordination compounds in terms of Werner's postulates ?
Ans. Werner's postulates explain the bonding in coordination compounds as follows :
(i) A metal exhibits two types of valencies namely, primary and secondary valencies. Primary
valencies are satisfied by negative ions while secondary valencies are satisfied by both
negative and neutral ions.
(ii) A metal ion has a definite number of secondary valencies around the central atom. Also,
these valencies project in a specific direction in the space assigned to the definite geometry
of the coordination compound.
(iii) Primary valencies are usually ionizable, while secondary valencies are non-ionizable.
2. FeSO4 solution mixed with (NH4)2 SO4 solutions in 1 : 1 molar ratio gives the test of Fe2+ ion but
CuSO4 solution mixed with aqueous ammonia in 1 : 4 molar ratio does not give the test of Cu2+
ion. Explain why ?
Ans. (NH 4 )2 SO 4  FeSO 4  6H 2 O 
 FeSO 4 .(NH 4 )2 SO 4 .6H 2 O
Mohr's Salt

CuSO 4  4NH 3  5H 2 O 
 [Cu(NH 3 ) 4 ]SO 4 .5H 2 O
Tetraamminocopper( II )sulphate
Both the compounds i.e., FeSO4. (NH4)2 SO4. 6H2O and [Cu(NH3)4] SO4. 5H2O fall under the
category of addition compounds with only one major difference i.e., the former is an example of
a double salt, while the latter is a coordination compound.
3. Explain with two examples each of the following: coordination entity, ligand, coordination
number, coordination polyhedron, homoleptic and heteroleptic.
Ans. (i) Coordination entity : A coordination entity is an electrically charged radical or species
carrying a positive or negative charge. In a coordination entity, the central atom or ion is
surrounded by a suitable number of neutral molecules or negative ions (called ligands). For
example :
 Ni(NH3 )6  ,  Fe(NH3 )6 
2 3
cationic complex

 PtCl4  ,  Ag(CN)2   anionic complex


2 

 Ni  CO 4  , Co  NH3 4 Cl 2   neutral complex


(ii) Ligands : The neutral molecules or negatively charged ions that surround the metal atom
in a coordination entity or a coordinal complex are known as ligands. For example Cl–,
H2O, H2N CH2CH2NH2 etc.
(iii) Coordination number : The total number of ligands (either neutral molecules or negative
node06\B0BC-BD\Kota\Board Material\Chemistry\Booklets\CBSE\Part-1

ions) that get attached to the central metal atom in the coordination sphere is called the
coordination number of the central metal atom. It is also referred to as its ligancy.
For example :
(a) In the complex, K2[PtCl6], there as six chloride ions attached to Pt in the coordinate
sphere. Therefore, the coordination number of Pt is 6.
(b) Similarly, in the complex [Ni(NH3)4]Cl2, the coordination number of the central atom
(Ni) is 4.

E 283
Chemistry
ALLEN
(iv) Coordination polyhedron : Coordination polyhedrons about the central atom can be
defined as the spatial arrangement of the ligands that are directly attached to the central
metal ion in the coordination sphere. For example :
L
L L

M L M
(a) (b) L
L L
Square planar L Tetrahedral
(v) Homoleptic complexes : These are those complexes in which the metal ion is bound to
only one kind of a donor group.
For e.g.  Co(NH3 )6  ,  PtCl 4 
3 2
etc.
(vi) Heteroleptic complexes : Heteroleptic complexes are those complexes where the central
metal ion is bound to more than one type of a donor group.
For e.g.  Co(NH3 )4 Cl 2  ,  Co(NH3 )5 Cl 
 2

4. What is meant by unidentate, didentate and ambidentate ligands ? Give two examples for each.
Ans. A ligand may contain one or more unshared pairs of electrons which are called the donor sites of
ligands. Now, depending on the number of these donor sites, ligands can be classified as follows :
(a) Unidenatate ligands : Ligands with only one donor sites are called unidentate ligands. For

e.g., N H3 ,Cl — etc.
(b) Didentate ligands : Ligands that have two donor sites are called bidentate ligands. For e.g.,
(i) Ethane-1, 2-diamine (ii) Oxalate ion
H2N—CH2 COO—
2–
(C2O ) or
4
H2N—CH2 COO

(c) Ambidentate ligands : Ligands that can attach themseleves to the central metal atom
through two different atoms are called ambidentate ligands. For example :
O
M—N Nitro group
(i)
O
(The donor atom is N)
M—O—N = O
node06\B0BC-BD\Kota\Board Material\Chemistry\Booklets\CBSE\Part-1

Nitrito group

(The donor atom is oxygen)


M—SCN Thiocyanato
(ii)
(The donor atom is S)
M—NCS Isothiocyanato

(The donor atom is N)

284 E
CBSE
ALLEN
5. Specify the oxidation numbers of the metals in the following coordination entities :
(i) [Co(H2O)(CN)(en)2]2+ (ii) [CoBr2(en)2]+
(iii) [PtCl4]2– (iv) K3[Fe(CN)6]
(v) [Cr(NH3)3Cl3]
Ans. (i) [Co(H2O)(CN)(en)2]2+
Let the oxidation number of Co be x.
The charge on the complex is +2.
2+
Co (H2O) (CN) (en)2

x + 0 + (–1) + 2(0) = +2
x – 1 = +2
x = +3
(ii) [PtCl4]2–
Let the oxidation number of Pt be x.
The charge on the complex is –2.
2—

Pt (Cl)4

x + 4(–1) = –2
x = +2
+
Co (Br)2 (en)2
(iii)

x + 2(–1) + 2(0) = +1

x – 2= +1
x =+3
(iv) K3[Fe(CN)6]
3–

i.e., Fe (CN)6

x + 6(–1) = –3

x = +3
node06\B0BC-BD\Kota\Board Material\Chemistry\Booklets\CBSE\Part-1

Cr (NH3)3 Cl3
(v)

x + 3(0) + 3(-1) = 0

x–3=0
x = +3

E 285
Chemistry
ALLEN
6. Using IUPAC norms write the formulas for the following :
(i) Tetrahydroxozincate (II) (ii) Potassium tetrachloridopalladate (II)
(iii) Diamminedichloridoplatinum (II) (iv) Potassium tetracyanonickelate (II)
(v) Pentaamminenitrito-O-cobalt (III) (vi) Hexaamminecobalt (III) sulphate
(vii) Potassium tri(oxalato)chromate (III) (viii) Hexaammineplatinum (IV)
(ix) Tetrabromidocuprate (II) (x) Pentaamminenitrito-N-cobalt (III)
Ans. (i) [Zn(OH)4]2– (ii) K2[PdCl4]
(iii) [Pt(NH3)2Cl2] (iv) K2[Ni(CN)4]
2+
(v) [Co(ONO) (NH3)5] (vi) [Co(NH3)6]2 (SO4)3
(vii) K3[Cr(C2O4)3] (viii) [Pt(NH3)6]4+
(ix) [Cu(Br)4]2– (x) [Co[NO2)(NH3)5]2+
7. Using IUPAC norms write the systematic names of the following :
(i) [Co(NH3)6]Cl3 (ii) [Pt(NH3)2Cl(NH2CH3)]Cl
3+
(iii) [Ti(H2O)6] (iv) [Co(NH3)4Cl(NO2)]Cl
2+
(v) [Mn(H2O)6] (vi) [NiCl4]2–
(vii) [Ni(NH3)6]Cl2 (viii) [Co(en)3]3+
(ix) [Ni(CO)4]
Ans. (i) Hexaamminecobalt (III) chloride
(ii) Diamminechlorido (methylamine) platinum (II) chloride
(iii) Hexaaquatitanium (III) ion
(iv) Tetraamminechloridonitrito-N-Cobalt (III) chloride
(v) Hexaaquamanganese (II) ion
(vi) Tetrachloridonickelate (II) ion
(vii) Hexaamminenickel (II) chloride
(viii) Tris (ethane-1, 2-diammine) cobalt (III) ion
(ix) Tetracarbonylnickel (0)
8. List various types of isomerism possible for coordination compounds, giving an example of each.
Ans. Isomerism in coordination compounds

Stereoisomerism Structural isomersim

Geometrical Optical Solvate Linkage Coordination Ionization


isomerism isomerism isomerism isomerism isomerism isomerism
node06\B0BC-BD\Kota\Board Material\Chemistry\Booklets\CBSE\Part-1

(a) Geometrical isomerism : This type of isomerism is common in heteroleptic complexes. It


arises due to the different possible geometric arrangements of the ligands. For example :
Cl NH3 Cl NH3
Pt Pt
Cl NH3 NH3 Cl

Cis-isomer Trans-isomer

286 E
CBSE
ALLEN
(b) Optical isomerism : This type of isomerism arises in chiral molecules. Isomers are mirror
images of each other and are non-superimposable.
en en

en Co Co en

en en
3+
[Co(en)3] Mirror [Co(en)3]3+
(dextro) (Laevo)

(c) Linkage isomerism : This type of isomerism is found in complexes that contain
ambidentate ligands.
For example : [Co(NH3)5(NO2)]Cl2 and [Co(NH3)5(ONO)Cl2
Yellow form Red form
(d) Coordination isomerism : This type of isomerism arises when the ligands are
interchanged between cationic and anionic entities of different metal ions present in the
complex. For example [Co(NH3)6] [Cr(CN)6] and [Cr(NH3)6] [Co(CN)6]
(e) Ionization isomerism : This type of isomerism arises when a counter ion replaces a ligand
within the coordination sphere. For example
[Co(NH3)5 SO4] Br and [Co(NH3)5Br] SO4
(f) Solvate isomerism : Solvate isomers differ by whether or not the solvent molecule is
directly bonded to the metal ion or merely present as a free solvent molecule in the crystal
lattice.
[Cr(H2O)6]Cl3, [Cr(H2O)5Cl]Cl2. H2O, [Cr(H2O)4Cl2]Cl. 2H2O
Violet Blue-green Dark green
9. How many geometrical isomers are possible in the following coordination entities ?
(i) [Cr(C2O4)3]3+ (ii) [Co(NH3)3Cl3]
Ans. (i) For [Cr(C2O4)3]3–, no geometrical isomer is possible as it is a bidentate ligand.
O
OX O
XO

O
XO Cr OX
O

O
XO O
OX
node06\B0BC-BD\Kota\Board Material\Chemistry\Booklets\CBSE\Part-1

(ii) [Co(NH3)3Cl3], Two geometrical isomers are possible.


NH3 H3N NH3
Cl NH3 Cl
Co Co
Cl NH3 Cl Cl
Cl NH3
Facial Meridional

E 287
Chemistry
ALLEN
10. Draw the structure of optical isomers of :
(i) [Cr(C2O4)3]3– (ii) [PtCl2(en)2]2+ (iii) [Cr(NH3)2Cl2(en)]+
Ans. (i) [Cr(C2O4)3]3–
en en

en Co Co en

en en
3–
[Cr(OX)3] Mirror [Cr(OX)3]3–
(dextro) (Laevo)
(ii) [PtCl2(en)2]2+
Cl Cl
Cl
en Pt en
Pt

en en
Mirror
+
(iii) [Cr(NH3)2Cl2(en)]
en en
H3N NH3

Cr Cr

H3N Cl Cl NH3
Cl Cl
Mirror
11. Draw all the isomers (geometrical and optical) of :
(i) [CoCl2(en)2]+ (ii) [Co(NH3)Cl(en)2]2+ (iii) [Co(NH3)2Cl2(en)]+
Ans. (i) [CoCl2(en)2]+
Cl Cl

en Co Co en
en en

Cl Cl
Mirror
+
Trans [CoCl2(en)2] isomer-optically inactive
(Superimposable mirror images)
Cl Cl
Cl Cl
node06\B0BC-BD\Kota\Board Material\Chemistry\Booklets\CBSE\Part-1

en Co Co en

en en
Mirror
+
Cis [CoCl2(en)2] isomer-optically active
(non-superimposable mirror images)

In total, three isomers are possible.

288 E
CBSE
ALLEN
(ii) [Co(NH3)Cl(en)2]2+
Cl Cl
NH3
en en en
Co Co

NH3 en
Trans Cis

Cl Cl
NH3 H3N
en Co Co en

en en
Cis Mirror

Trans - isomers are optically inactive.


Cis - isomers are optically active.
(iii) [Co(NH3)2Cl2(en)]+
Cl Cl
Cl Cl
en Co Co en

NH3 NH3
NH3 NH3
Mirror
12. Write all the geometrical isomers of [Pt(NH3)(Br)(Cl)(py)] and how many of these will exhibit
optical isomers ?
Ans. [Pt(NH3)(Br)(Cl)(py)]
H3N Cl H3N Br H3N Cl
Pt Pt Pt
Py Br Py Cl Br Py

From the above isomers, none will exhibit optical isomers. Tetrahedral complexes rarely show
optical isomerization. They do so only in the presence of unsymmetrical chelating agents.
13. Aqueous copper sulphate solution (blue in colour) gives :
(i) A green precipitate with aqueous potassium fluoride, and
(ii) A bright green solution with aqueous potassium chloride
node06\B0BC-BD\Kota\Board Material\Chemistry\Booklets\CBSE\Part-1

Explain these experimental results.


Ans. Aqueous CuSO4 exists as [Cu(H2O)4]SO4. It is blue in colour due to the presence of
[Cu(H2O)4]2+ ions.
(i) When KF is added.
2 2
 Cu  H 2 O 4    4F  
  Cu  F 4   4H 2 O
(Green)

E 289
Chemistry
ALLEN
(ii) When KCl is added :

  CuCl 4 
2
 Cu  H 2 O 4    4Cl 
 2
 4H 2 O
(bright green)

In both these cases, the weak field ligand water is replaced by the F— and Cl— ions.
14. What is the coordination entity formed when excess of aqueous KCN is added to an aqueous
solution of copper sulphate? Why is it that no precipitate of copper sulphide is obtained when
H2S (g) is passed through this solution?
 K 2  Cu(CN)4 (aq)  K 2SO4(aq)
Ans. CuSO4(aq)  4KCN(aq) 

i.e.,  Cu(H 2 O) 4   4CN  


  Cu(CN)4   4H 2 O
2 2

Thus, the coordination entity formed in the process is K2[Cu(CN)4]. It is a very stable complex,
which does not ionize to give Cu2+ ions when added to water. Hence, Cu2+ ions are not
precipitated when H2S(g) is passed through the solution.
15. Discusss the nature of bonding in the following coordination entities on the basis of valence bond
theory :
(i) [Fe(CN)6]4– (ii) [FeF6]3– (iii) [Co(C2O4)3]3– (iv) [CoF6]3–
Ans. (i) [Fe(CN)6]4–
In the above coordination complex, iron exists in the +2 oxidation state.
Fe2+ : Electronic configuration is 3d6
Orbitals of Fe2+ ion :

3d 4s 4p

As CN— ion is a strong field ligand, it causes the pairing of the unpaired 3d electrons.

3d 4s 4p

Since there are six ligands around the central metal ion, the most feasible hybridization is
d2sp3.
d2sp3 hybridized orbitals of Fe2+ are :

3d 4s 4p
2 3
d sp

6 electron pairs from CN ions occupy the six hybrid d2sp3 orbitals.

node06\B0BC-BD\Kota\Board Material\Chemistry\Booklets\CBSE\Part-1

Then,

3d 4s 4p

6 pairs of electrons from 6CN ions

Hence, the geometry of the complex is octahedral and the complex is diamagnetic
(as there are no unpaired electrons).

290 E
CBSE
ALLEN
(ii) [FeF6]3–
In this complex, the oxidation state of Fe is +3.
Orbitals of Fe+3 ion :

3d 4s 4p 4d
There are 6 F ions. Thus, it will undergo d sp or sp d hybridization. As F– is a weak field
– 2 3 3 2

ligand, it does not cause the pairing of the electrons in the 3d orbitals. Hence, the most
feasible hybridization is sp3d2.
sp3d2 hybridized orbitals of Fe3+ it are :

3d 4s 4p 4d

sp3d2

3d 4s 4p 4d

sp3d2
6 electron pairs from F—ions

Hence, the geometry of the complex is found to be octahedral.


(iii) [Co(C2O4)3]3–
Cobalt exists in the +3 oxidation state in the given complex.
Orbitals of Co3+ ion :

3d 4s 4p 4d
Oxalate ion is a weak field ligand. Therefore, it cannot cause the pairing of the 3d orbital
electrons. As there are 6 ligands, hybridization has to be sp3d2.
sp3d2 hybridized orbitals of Co3+.

sp3d2

The 6 electron pairs from the 3 oxalate ions (oxalate anion is a bidentate ligand) occupy
these sp3d2 orbitals.

6 electron pairs from 3 oxalate ions

Hence, the geometry of the complex is found to be octahedral.


node06\B0BC-BD\Kota\Board Material\Chemistry\Booklets\CBSE\Part-1

(iv) [CoF6]3–
Cobalt exists in the +3 oxidation state.
Orbitals of Co3+ ion :

3d 4s 4p 4d

Again, fluoride ion is a weak field ligand. It cannot cause the pairing of the 3d electrons.
As a result, the Co3+ ion will undergo sp3d2 hybridization.
E 291
Chemistry
ALLEN
sp3d2 hybridized orbitals of Co3+ ion are :

3d 4s 4p 4d
3 2
sp d

3d 4s 4p 4d

sp3d2
6 electron pairs from F— ions

Hence, the geometry of the complex is octahedral and paramagnetic.


16. Draw figure to show the splitting of d obitals in an octahedral crystal field.
Ans. Energy dx2– y2,dz2
z eg

y +3/5
Barycentre

–2/5
x
Average energy t2g
of the d orbitals dxy dyz dxz
in a spherical field
Splitting of the d
Metal orbitals in a
d orbitals octahedral crystal field
2 2 2
dx –y –dz , dxydyzdxz

17. What is spectrochemical series ? Explain the difference between a weak field and a strong field
ligand.
Ans. A spectrochemical series is the arrangement of common ligands in the increasing order of their
crystal - field splitting energy (CFSE) values. The ligands present on the R.H.S of the series are
strong field ligands while that on the L.H.S are weak field ligands. Also, strong field ligands
cause higher splitting in the d orbitals that weak field ligands.
I–< Br– <S2– <SCN– <Cl– <N3– <OH– <C2O42– <~H2O < NCS– <edta4– < NH3 < en– <CN– < CO
18. What is crystal field splitting energy ? How does the magnitude of o decide the actual
configuration of d-orbitals in a coordination entity ?
Ans. The degenerate d-orbitals (in a spherical field environment) split into two levels i.e., eg and t2g in
the presence of ligands. The splitting of the degenerate levels due to the presence of ligands is
called the crystal – field splitting while the energy difference between the two levels (eg and t2g)
is called the crystal - field splitting energy. It is denoted by 0.
After the orbitals have split, the filling of the electrons takes place. After 1 electron (each) has
node06\B0BC-BD\Kota\Board Material\Chemistry\Booklets\CBSE\Part-1

been filled in the three t2g orbitals, the filling of the fourth electron takes place in two ways. It can
enter the eg orbitals (giving rise to t2g3 eg1 like electron configuration) or the pairing of the
electrons can take place in the t2g orbitals (giving rise to t2g4 eg0 like electronic configuration). If
the 0 value of a ligand is less than the pairing energy (P), then the electrons enter the eg orbital.
On the other hand, if the 0 value of a ligand is more than the pairing energy (P), then the
electrons enter the t2g orbital.

292 E
CBSE
ALLEN
19. [Cr(NH3)6]3+ is paramagnetic while [Ni(CN)4]2– is diamagnetic. Explain why ?
Ans. In [Cr(NH3)6]3+, Cr is in the +3 oxidation state i.e., d3 configuration. Since there are three
unparied electrons in 3d– orbitals
Cr3+
3d 4s 4p 4d
d2sp3

Therefore, it undergoes d2sp3 hybridization and the electrons in the 3d orbitals remain unpaired.
Hence, it is paramagnetic in nature.
In [Ni(CN)4]2– Ni exists in the +2 oxidation state i.e., d8 configuration.
Ni2+
3d 4s 4p

CN is a strong field ligand. It cause the pairing of the 3d orbital electrons. Then, Ni2+ undergoes

dsp2 hybridization.

3d 4s 4p
dsp2
As there are no unpaired electrons, it is diamagnetic.
20. A solution of [Ni(H2O)6]2+ is green but a solution of [Ni(CN)4]2– is colourless. Explain.

Ans. In [Ni(H2O)6]2+, H2 O is a weak field ligand. Therefore, there are unpaired elecrons in Ni2+. In
this complex, the 3d electrons from the lower energy level can be excited to the higher energy
level i.e., the possibility of d-d transition is present. Hence, [Ni(H2O)6]2+ is coloured.
In [Ni(CN)4]2– the electrons are all paired as CN– is a strong field ligand. Therefore, d-d
transition is not possible is [Ni(CN)4]2–. Hence, it is colourless.
21. [Fe(CN)6]4– and [Fe(H2O)6]2+ are of different colours in dilute solutions. Why ?
Ans. The colour of a particular coordination compound depends on the magnitude of the crystal-field
splitting energy, . This CFSE in turn depends on the nature of the ligand. In case of [Fe(CN)6]4–
and [Fe(H2O)6]2+, the colour differs because there is a difference in the CFSE. Now CN– is a
strong field ligand having a higher CFSE value of compound to the CFSE value of water. This
means that the absorption of energy for the intra d-d transition also differs. Hence the transmitted
colour also differs.
22. Discuss the nature of bonding in metal carbonyls.
Ans. The metal -carbon bonds in metals carbonyls have both  and  characters. A  bond is formed
when the carbonyl carbon donates a lone pair of electrons to the vacant orbital of the metal. A
bond is formed by the donation of a pair of electrons from the filled metal d orbital into the
vacent anti-bonding  orbital (also known as back bonding of the carbonyl group). The bond
strengthens the  bond and vica - versa. Thus, a synergic effect is created due to this metal -
node06\B0BC-BD\Kota\Board Material\Chemistry\Booklets\CBSE\Part-1

ligand bonding. This synergic effect strengthens the bond between CO and the metal.

 

M  C O


Synergic bonding in metal carbonyls

E 293
Chemistry
ALLEN
23. Give the oxidation state, d- orbital occupation and coordination number of the central metal ion
in the following complexes :
(i) K3[Co(C2O4)3] (ii) cis-[Cr(en)2Cl2]Cl
(iii) (NH4)2[CoF4] (iv) [Mn(H2O)6]SO4
Ans. (i) K3[Co(C2O4)3]
The central metal ion is Co3+.
Its coordination number is 6.
The oxidation state can be given as:
x – 6 = –3
x = +3
The d orbitals occupation for Co3+ is t 62g e0g .

(ii) cis-[Cr(en)2Cl2]Cl
The central metal ion is Cr3+.
The coordination number is 6.
The oxidation state can be given as :
x + 2(0) + 2(–1) = +1
x – 2 = +1
x = +3
The d orbitals occupation for Cr3+ is t 32g .

(iii) (NH4)2[CoF4]
The central metal ion is Co2+.
The coordination number is 4.
The oxidation state can be given as :
x – 4 = –2
x = +2
The d orbitals occupation for Co2+ is eg4 t2g3.
(iv) [Mn(H2O)6]SO4
The central metal ion is Mn2+.
node06\B0BC-BD\Kota\Board Material\Chemistry\Booklets\CBSE\Part-1

The coordination number is 6.


The oxidation state can be given as :
x + 0 = +2
x = +2
The d orbitals occupation for Mn2+ is t 32g e2g .

294 E
CBSE
ALLEN
24. Write down the IUPAC name for each of the following complexes and indicate the oxidation
state, electronic configuration and coordination number. Also give stereochemistry and magnetic
moment of the complex :
(i) K[Cr(H2O)2(C2O4)2]. 3H2O (ii) [Co(NH3)5Cl]Cl2
(iii) CrCl3(py)3 (iv) Cs[FeCl4]
(v) K4[Mn(CN)6]
Ans. (i) K[Cr (H2O)2 (C2O4)2]. 3H2O
IUPAC Name: Potassium diaquadioxalatochromate (III) trihydrate.
Oxidation state of chromium = +3
Electronic configuration : 3d3
Coordination number = 6
Shape : octahedral
Stereochemistry :
OH2 OH2
XO
O OX
O XO
O OH2
Cr Cr
O
XO O
OX O
XO OX
O
OX
OH2
Trans Cis

OH2 OH2 OH2 OH2


XO
O O
OX XO
O OX
O XO
O OH2 H2O OX
O
Cr Cr Cr Cr
O O
OX O O
OX OX OX
O
XO XO XO
O O O
XO
OH2 OH2 O
XO OX
O

Trans optically inactive Cis optically active

Magnetic moment,   n  n  2 

= 3 3  2  = 5 ~ 4BM

(ii) [Co(NH3)5Cl]Cl2
IUPAC name : Pentaamminechloridocobalt (III) chloride
node06\B0BC-BD\Kota\Board Material\Chemistry\Booklets\CBSE\Part-1

Oxidation state of Co = +3
Coordination number = 6
Shape : octahedral.
Electronic configuration : d6: t 62g .

E 295
Chemistry
ALLEN
Stereo chemistry :

OH2 OH2
O
XO OH2 H2O O
OX
Cr Cr
O
XO O
OX O
XO O
OX
XO
O OX
O

Cis optically active

Magnetic Moment = 0
(iii) CrCl3(py)3
IUPAC name : Trichloridotripyridinechromium (III)
Oxidation state of chromium = +3
Electronic configuration for: d3t2g
3

Coordination number = 6
Shape : octahedral.
Stereochemistry :
Cl Cl
Py Cl Py Cl
Cr Cr
Py Cl Py Py
Py Cl
Facial isomer Meriodional isomer

Both isomers are optically active. Therefore, a total of 4 isomers exist.

Magnetic moment,   n(n  2) = 3(3  2) = 15 ~ 4BM

(iv) Cs[FeCl4]
IUPAC name: Caesium tetrachloroferrate (III)
Oxidation state of Fe =+3
Electronic configuration d5: e 2g t 32 g
node06\B0BC-BD\Kota\Board Material\Chemistry\Booklets\CBSE\Part-1

Coordination number = 4
Shape : tetrahedral
Stereochemistry : optically inactive
Magnetic moment :

  n(n  2) = 5(5  2) = 35 ~ 6BM

296 E
CBSE
ALLEN
(v) K4[Mn(CN)6]
IUPAC Name : Potassium hexacyanomanganate (II)
Oxidation state of manganese = +2
Electronic configuration :d5: t 52g

Coordination number = 6
Shape : octahedral
Streochemistry : optically inactive

Magnetic moment,   n(n  2) = 1(1  2) = 3 = 1.732 BM


25. What is meant by stability of a coordination compound in solution? State the factors which
govern stability of complexes.
Ans. The stability of a complex in a solution refers to the degree of association between the two
species involved in a state of equilibrium. Stability can be expressed quantitatively in terms of
stability constant or formation constant.
 ML3
M + 3L 

[ML 3 ]
Stability constant,  =
[M][L]3

For this reaction, the greater the value of the stability constant, the greater is the proportion of
ML3 in the solution.
Stability can be of two types :
(a) Thermodynamic stability : The extent to which the complex will be formed or will be
transformed into another species at the point of equilibrium is determined by
thermodynamic stability.
(b) Kinetic stability : This helps is determining the speed with which the transformation will
occur to attain the state of equilibrium.
26. What is meant by the chelate effect ? Give an example.
Ans. When a ligand attaches to the metal ion in a manner that forms a ring, then the metal-ligand
association is found to be more stable. In other words, we can say that complex containing
chelate rings are more stable than complexes without rings. This is known as the chelate effect.
node06\B0BC-BD\Kota\Board Material\Chemistry\Booklets\CBSE\Part-1

For example : en

en Ni
  Ni(NH 3 ) 6 (aq ) log = 7.99
2 2
Ni (aq )  6NH 3(aq ) 
en

  Ni(en)3 (aq ) log = 18.1 (more stable)


2
Ni 2  (aq )  3en (aq ) 

E 297
Chemistry
ALLEN
27. Discuss briefly giving an example in each case the role of coordination compounds in :
(i) biological system
(ii) medicinal chemistry
(iii) analytical chemistry
(iv) extraction/metallurgy of metals
Ans. (i) Role of coordination compounds in biological system : We know that photosynthesis is
made possible by the presence of the chlorophyll pigment. This pigment is a coordination
compound of magnesium.
(ii) Role of coordination compounds in medicinal chemistry : Certain coordination
compounds of platinum (for example, cis-platin) are used for inhibiting the growth of
tumours.
(iii) Role of coordination compounds in analytical chemistry : During salt analysis, a
number of basic radicals are detected with the help of the colour changes they exhibit with
different reagents.
(iv) Role of coordination compounds in extraction or metallurgy of metals : From
[Au(CN)2]+ solution, gold is extracted by the addition of zinc metal.
28. How many ions are produced from the complex [Co(NH3)6]Cl2 in solution ?
(i) 6 (ii) 4 (iii) 3 (iv) 2
Ans. (iii) The given complex can be written as [Co(NH3)6]Cl2
Thus, [Co(NH3)6]2+ along with two Cl– ions are produced.
29. Amongst the following ions which one has the highest magnetic moment value ?
(i) [Cr(H2O6)]3+ (ii) [Fe(H2O)6]2+ (iii) [Zn(H2O)6]2+
Ans. (i) No. of unpaired electrons in [Cr(H2O6)]3+ = 3

Then,  = n(n+2) = 3(3  2) = 15 ~ 4BM

(ii) No. of unpaired electrons in [Fe(H2O)6]2+ = 4

Then,   4(4  2) = 24 ~ 5BM


(iii) No. of unpaired electrons in [Zn(H2O)6]2+ = 0
node06\B0BC-BD\Kota\Board Material\Chemistry\Booklets\CBSE\Part-1

Hence, [Fe(H2O)6]2+ has the highest magnetic moment value.


30. The oxidation number of cobalt in K[Co(CO)4] is
(i) +1 (ii) +3 (iii) –1 (iv) –3
Ans. We know that CO is a neutral ligand and K carries a charge of +1.
Therefore, the complex can be written as K+[Co(CO)4]–. Therefore, the oxidation number of Co
in the given complex is –1. Hence, option (iii) is correct.

298 E
CBSE
ALLEN
31. Amongst the following, the most stable complex is
(i) [Fe(H2O)6]3+ (ii) [Fe(NH3)6]3+ (iii) [Fe(C2O4)3]3– (iv) [FeCl6]3–
Ans. We know that the stability of a complex increases by chelation. Therefore, the most stable
complex is [Fe(C2O4)3]3-.
O=C O—

O=C O

Then,
O
C= O
O C=
O=C O O
Fe
O=C O O
O C

=O
C

=O
32. What will be the correct order for the wavelengths of absorption in the visible region for the
following:
[Ni(NO2)6]4–, [Ni(NH3)6]2+, [Ni(H2O)6]2+
Ans. The central metal ion in all the three complexes is the same. Therefore, absorption in the visible
region depends on the ligands. The order in which the CFSE value of the ligands increases in the
spectrochemical series is as follows :
H2O < NH3<NO2–
Thus, the amount of crystal-field splitting observed will be in the following order :
 0H O    0 NH   0 
2  3 
NO 2 
 

Hence, the wavelengths of absorption in the visible region will be in the order :

 Ni(H2O)6    Ni(NH3 )6    Ni(NO)6 


2 2 4
node06\B0BC-BD\Kota\Board Material\Chemistry\Booklets\CBSE\Part-1

E 299
Chemistry
ALLEN
EXERCISE-3 EXEMPLAR
1. The colour of the coordination compounds depends on the crystal field splitting. What will be the
correct order of absorption of wavelength of light in the visible region, for the complexes,
[Co(NH3)6]3+, [Co(CN)6]3–, [Co(H2O)6]3+
(a) [Co(CN)6]3– > [Co(NH3)6]3+ > [Co(H2O)6]3+
(b) [Co(NH3)6]3+ > [Co(H2O)6]3+ > [Co(CN)6]3–
(c) [Co(H2O)6]3+ > [Co(NH3)6]3+ > [Co(CN)6]3–
(d) [Co(CN)6]3– > [Co(NH3)6]3+ < [Co(H2O)6]3+
Ans. (c)
2. The correct IUPAC name of [Pt(NH3)2Cl2] is :
(a) Diamminedichloridoplatinum (II) (b) Diamminedichloridoplatinum (IV)
(c) Diamminedichloridoplatinum (0) (d) Dichloridodiammineplatinum (IV)
Ans. (a)
3. The stabilisation of coordination compounds due to chelation is called the chelate effect. Which
of the following is the most stable complex species ?
(a) [Fe(CO)5] (b) [Fe(CN)6]3– (c) [Fe(C2O4)3]3– (d) [Fe(H2O)6]3+
Ans. (c)
4. Indicate the complex ion which shows geometrical isomerism :
(a) [Cr(H2O)4 Cl2]+ (b) [Pt(NH3)3Cl]
(c) [Co(NH3)6]3+ (d) [Co(CN)5(NC)]3–
Ans. (a)
5. Due to the presence of ambidentate ligands coordination compounds show isomerism. Palladium
complexes of the type [Pd(C6H5)2(SCN)2] and [Pd(C6H5)2(NCS)2] are :
(a) linkage isomers (b) coordination isomers
(c) ionisation isomers (d) geometrical isomers
Ans. (a)
6. The compounds [Co(SO4) (NH3)5]Br and [Co(SO4) (NH3)5]Cl represent :
(a) linkage isomerism (b) ionisation isomerism
(c) coordination isomerism (d) no isomerism
node06\B0BC-BD\Kota\Board Material\Chemistry\Booklets\CBSE\Part-1

Ans. (b)
7. What kind of isomerism exists between [Cr(H2O)6]Cl3 (violet) and [Cr(H2O)5Cl]Cl2.H2O
(greyish-green) ?
(a) Linkage isomerism (b) Solvate isomerism
(c) Ionisation isomerism (d) Coordination isomerism
Ans. (b)

300 E
CBSE
ALLEN
8. IUPAC name of [Pt(NH3)2 Cl(NO2)] is :
(a) Platinum diaminechloronitrite
(b) Chloronitrito-N-ammineplatinum (II)
(c) Diamminechloridonitrito-N-platinum (II)
(d) Diamminechloronitrito-N-platinate (II)
Ans. (c)
(SHORT ANSWER TYPE QUESTIONS)
9. Arrange the following complexes in the increasing order of conductivity of their solution :
[Co(NH3)3Cl3], [Co(NH3)4Cl2]Cl, [Co(NH3)6]Cl3, [Cr(NH3)5Cl]Cl2
Ans. Ions or molecules present outside the coordination sphere are ionisable. A complex which gives
more ions on dissolution, is more conducting.
[Co(NH3)3Cl3] < [Co(NH3)4Cl2]Cl < [Cr(NH3)5Cl]Cl2 < [Co(NH3)6]Cl3
(1 ion) (2 ions) (3 ions) (4 ions)
Here, number of ions increases and conductivity increases.
10. A complex of the type [M(AA)2X2]n+ is known to be optically active. What does this indicate
about the structure of the complex ? Give one example of such complex.
Ans. An optically active complex of the type [M(AA)2X2]n+ indicates cis-octahedral structure.
e.g., cis-[Pt(en)2Cl2]2+ or cis-[Cr(en)2Cl2]+ because its mirror image isomers are
non-superimposable.
2+ 2+
en en
en en
Pt Pt
Cl Cl
Cl Cl

Non-superimposable isomers of [Pt(en)2Cl2]2+


Non-superimposable isomers of [Pt(en)2Cl2]2+.
11. Give the electronic configuration of the following complexes on the basis of crystal field splitting
theory. [CoF6]3–, [Fe(CN)6]4– and [Cu(NH3)6]2+
Ans. According to spectrochemical series, ligands can be arranged in a series in the order of increasing
field strength i.e., F– < NH3 < CN–.
Hence, CN– and NH3 being strong field ligand pair up the t2g electrons before filling eg set.
node06\B0BC-BD\Kota\Board Material\Chemistry\Booklets\CBSE\Part-1

[CoF6]3– ; Co3+ = (d6) t 42g eg2

eg

Degenerate t2g
d-orbital

E 301
Chemistry
ALLEN
 Fe(CN)64 , Fe 2  (d 6 )t 62q e0g 

eg

Degenerate t2g
d-orbital

Cu(NH3 )6 
2
,Cu 2  (d 9 ) t eg
6
e3g

eg

Degenerate t2g
d-orbital
12. Explain why [Fe(H2O)6]3+ has magnetic moment value of 5.92 BM whereas [Fe(CN)6]3– has a
value of only 1.74 BM ?
Ans. As we know,  m  n(n  2) BM

where, m = magnetic moment


n = number of unpaired electrons
It m = 1.74 i.e., n = 1
and m = 5.92 i.e., n = 5
[Fe(CN)6] involves d2sp3 hybridisation with one unpaired electron (as shown by its magnetic
3–

moment 1.74 BM) and [Fe(H2O)6]3+ involves sp3d2 hybridisation with the unpaired electrons
(because magnetic moment equal to 5.92 BM).
CN– is stronger ligand than H2O according to spectrochemical series, 0 > P for CN– hence,
fourth electron will pair itself. Whereas for water pairing will not happen for [Fe(CN)6]3– the
electronic configuration of Fe3+ is

× × × × × ×

d2sp3 hybridisation

One unpaired electron (n = 1)


For [Fe(H2O)6]3+ the electronic configuration of Fe3+ is
node06\B0BC-BD\Kota\Board Material\Chemistry\Booklets\CBSE\Part-1

× × × × × × × × ×

sp3d2 hybridisation

Five unpaired electron (n = 5)


Hence, [Fe(CN)6]3– and [Fe(H2O)6]3+ are inner orbital and outer orbital complex respectively.

302 E
CBSE
ALLEN
13. Arrange following complex ions is increasing order of crystal field splitting energy (0).
[Cr(Cl)6]3–, [Cr(CN)6]3–, [Cr(NH3)6]3+
Ans. CFSE is higher when the complex contains strong field ligand. Thus, crystal field splitting energy
increases in the order.
[Cr(Cl)6]3– < [Cr(NH3)6]3+ < [Cr(CN)6]3–
Because according to spectrochemical series the order of field strength is
Cl– < NH3 < CN–
14. Why do compounds having similar geometry have different magnetic moment ?
Ans. It is due to the presence of weak and strong field ligands in complexes. If CFSE is high, the
complex will show low value of magnetic moment and vice-versa. e.g., [CoF6]3– and
[Co(NH3]6]3+, the former is paramagnetic, and the latter is diamagnetic because F– is a weak field
ligand and NH3 is a strong field ligand while both have similar geometry.
]3– ]3+
F NH3
F F NH3 NH3
Co and Co
F F NH3 NH3
F NH3
15. CuSO4.5H2O is blue in colour while CuSO4 is colourless, Why ?
Ans. In CuSO4.5H2O water acts as ligand and causes crystal field splitting. Hence d-d transition is
possible thus CuSO4.5H2O is coloured. In the anhydrous CuSO4 due to the absence of water
(ligand), crystal field splitting is not possible and hence, it is colourless.
(MATCHING THE COLUMNS)
16. Match the complex ions given in Column I with the colours given in Column II and assign the
correct code.
Column-I Column-II
(Complex ion) (Colour)
(A) [Co(NH3)6]3+ (1) Violet
(B) [Ti(H2O)6]3+ (2) Green
2+
(C) [Ni(H2O)6] (3) Pale blue
(D) [Ni(H2O)4(en)]2+ (aq) (4) Yellowish orange
Ans. A-4, B-3, C-2, D-1
17. Match the coordination compounds given in Column I with the central metal atoms given in
Column II and assign the correct code.
Column-I Column-II
node06\B0BC-BD\Kota\Board Material\Chemistry\Booklets\CBSE\Part-1

(Coordination Compound) (Central metal atom)


(A) Chlorophyll (1) Rhodium
(B) Blood pigment (2) Cobalt
(C) Wilkinson catalyst (3) Magnesium
(D) Vitamin B12 (4) Iron
Ans. A-3, B-4, C-1, D-2
E 303
Chemistry
ALLEN
18. Match the complex ions given in Column I with the hybridisation and number of unpaired
electrons given in Column II and assign the correct code.

Column-I Column-II
(Complex ion) (Hybridisation,
number of unpaired
electrons)

(A) [Cr(H2O)6]3+ (1) dsp2, 1

(B) [Co(CN)4]2– (2) sp3d2, 5

(C) [Ni(NH3)6]2+ (3) d2sp3, 3

(D) [MnF6]4– (4) sp3d2, 2

Ans. A-3, B-1, C-4, D-2


19. Match the complex species given in Column I with the possible isomerism given in Column II
and assign the correct code.

Column-I Column-II
(Complex species) (Isomerism)

(A) [Co(NH3)4Cl2]+ (1) Optical

(B) cis-[Co(en)2Cl2]+ (2) Ionisation

(C) [Co(NH3)5(NO2)]Cl2 (3) Coordination

(D) [Co(NH3)6][Cr(CN)6] (4) Geometrical

Ans. A-4, B-1, C-2, D-3


20. Match the compounds given in Column-I with the oxidation state of cobalt present in it
(given in column II) and assign the correct code :

Column-I Column-II
(Compound) (Oxidation state of Co)

(A) [Co(NCS)(NH3)5](SO3) (1) +4


node06\B0BC-BD\Kota\Board Material\Chemistry\Booklets\CBSE\Part-1

(B) [Co(NH3)4Cl2]SO4 (2) 0

(C) Na4[Co(S2O3)3] (3) +2

(D) [Co2(CO)8] (4) +3

Ans. A-4, B-1, C-3, D-2

304 E
CBSE
ALLEN
(ASSERTION AND REASON TYPE)
Note: In the following questions a statement of assertion followed by a statement of reason is given.
Choose the correct answer out of the following choices.
(a) Both (A) and (R) correct and (R) is correct explanation of (A).
(b) Both (A) and (R) correct and (R) is not correct explanation of (A).
(c) (A) is correct but (R) is wrong.
(d) (A) Assertion is wrong but (R) reason is correct.
21. Assertion (A) : Toxic metal ions are removed by the chelating ligands.
Reason (R) : Chelate complexes tend to be more stable.
Ans. (a)
22. Assertion (A) : [Cr(H2O6)]Cl2 and [Fe(H2O)6]Cl2 are reducing in nature.
Reason (R) : Unpaired electrons are present in their d-orbitals.
Ans. (b)
23. Assertion (A) : Linkage isomerism arises in coordination compounds containing ambidentate
ligand.
Reason (R) : Ambidentate ligand has two different donor atoms.
Ans. (a)
24. Assertion (A) : Complexes of MX6 and MX5L type (X and L are unidentate) do not show
geometrical isomerism.
Reason (R) : Geometrical isomerism is not shown by complexes of coordination number 6.
Ans. (b)
25. Assertion (A) : [Fe(CN)6]3– ion shows magnetic moment corresponding to two unpaired
electrons.
Reason (R) : Because it has d2sp3 type hybridisation.
Ans. (d)
node06\B0BC-BD\Kota\Board Material\Chemistry\Booklets\CBSE\Part-1

E 305
Chemistry
ALLEN
EXERCISE-4 MCQ, A/R, CASE-BASED QUESTIONS
INTRODUCTION
1. In K4[Fe(CN)6]
(a) (CN) are linked with primary valency (b) (CN) are linked with secondary valency
(c) K are linked with secondary valency (d) K are linked with non-ionic valency
2. Which of the following acts as a bidentate ligand in complex formation
(a) Acetate (b) Oxalate (c) Thiocyanate (d) EDTA
3. The co-ordination number of cobalt in the complex [Co(en)2Br2]Cl2 is
(a) 2 (b) 6 (c) 5 (d) 4
4. Which of the following ligands forms a chelate
(a) Acetate (b) Oxalate (c) Cyanide (d) Ammonia
5. According to Werner's theory
(a) Primary valency can be ionized
(b) Secondary valency can be ionized
(c) Primary and secondary valencies both cannot be ionized
(d) Only primary valency cannot be ionized
6. Which of the following is not true for ligand-metal complex
(a) Larger the ligand, the more stable is the metal-ligand bond
(b) Highly charged ligand forms strong bond
(c) Larger the permanent dipole moment of ligand, the more stable is the bond
(d) Greater the ionization potential of central metal, the stronger is the bond
7. Which is the example of hexadentate ligand
(a) 2, 2—dipyridyl (b) Dimethyl glyoxime
(c) Aminodiacetate ion (d) Ethylene diammine tetra acetate ion [EDTA]
8. The coordination number of a metal in coordination compounds is
(a) Same as primary valency (b) Sum of primary and secondary valencies
(c) Same as secondary valency (d) None of these
9. Ligand in a complex salt are
(a) Anions linked by coordinate bonds to a central metal atom or ion
(b) Cations linked by coordinate bonds to a central metal atom or ion
(c) Molecules linked by coordinate bonds to a central metal atom or ion
node06\B0BC-BD\Kota\Board Material\Chemistry\Booklets\CBSE\Part-1

(d) Ions or molecules linked by coordinate bonds to a central metal atom or ion
10. The coordination number of a central metal atom in a complex is determined by
(a) The number of ligands around a metal ion bonded by sigma and pi-bonds both.
(b) The number around a metal ion bonded by pi-bonds
(c) The number of ligands around a metal ion bonded by sigma bonds
(d) The number of only anionic ligands bonded to the metal ion

306 E
CBSE
ALLEN
11. Potassium ferrocyanide is a
(a) Normal salt (b) Mixed salt (c) Double salt (d) Complex salt
12. That ion or molecule which forms a complex compound with transitional metal ion is called
(a) Recipient (b) Ligand (c) Coordinate ion (d) No special name
13. An example of a double salt is
(a) Bleaching powder (b) Hypo (c) K4[Fe(CN)6] (d) Potash alum
NOMENCLATURE OXIDATION
14. How many ions are produced in aqueous solution of [Co(H2O)6]Cl2
(a) 2 (b) 3 (c) 4 (d) 6
15. IUPAC name of [Pt(NH3)3(Br)(NO2)Cl]Cl is
(a) Triamminechlorobromonitroplatinum (IV) chloride
(b) Triamminebromonitrochloroplatinum (IV) chloride
(c) Triamminebromochloronitroplatinum (IV) chloride
(d) Triamminenitrochlorobromoplatinum (IV) chloride
16. The IUPAC name of [Ni(CO)4] is :
(a) Tetra carbonyl nickel (II) (b) Tetra carbonyl nickel (0)
(c) Tetra carbonyl nickelate (II) (d) Tetra carbonyl nickelate (0)

17. The oxidation number of Pt in [Pt(C2H4)Cl3] is
(a) + 1 (b) + 2 (c) + 3 (d) + 4
ISOMERISM AND MAGNETIC PROPERTIES
18. The number of unpaired electrons in the complex ion [CoF6]3– is (Atomic no. of Co = 27)
(a) Zero (b) 2 (c) 3 (d) 4
19. Which would exhibit co-ordination isomerism
(a) [Cr(NH3)6] [Co(CN)6] (b) [Co(en)2Cl2]
(c) [Cr(NH3)6]Cl3 (d) [Cr(en)2Cl2]+
20. [Co(NH3)5NO2]Cl2 and [Co(NH3)5(ONO)]Cl2 are related to each other as
(a) Geometrical isomers (b) Optical isomers
(c) Linkage isomers (d) Coordination isomers
21. [Co(NH3)5Br]SO4 and [Co(NH3)5SO4]Br are examples of which type of isomerism
(a) Linkage (b) Geometrical (c) Ionization (d) Optical
+3
22. [Ti(H2O)6] is paramagnetic in nature due to
(a) One unpaired (b) Two unpaired (c) Three unpaired (d) No unpaired
23. Coordination isomerism is caused by the interchange of ligands between the
node06\B0BC-BD\Kota\Board Material\Chemistry\Booklets\CBSE\Part-1

(a) Cis and Trans structure (b) Complex cation and complex anion
(c) Inner sphere and outer sphere (d) Low oxidation and higher oxidation states
2– 2–
24. Amongst Ni(CO)4, [Ni(CN)4] and [NiCl4] [IIT 1991]
(a) Ni(CO)4 and [NiCl4]2– are diamagnetic and [Ni(CN)4]2– is paramagnetic
(b) [NiCl4]2– and [Ni(CN)4]2– are diamagnetic and Ni(CO)4 is paramagnetic
(c) Ni(CO)4 and [Ni(CN)4]2– are diamagnetic and is paramagnetic
(d) Ni(CO)4 is diamagnetic and and [Ni(CN)4]2– are paramagnetic

E 307
Chemistry
ALLEN
25. [Co(NH3)4Cl2]+ exhibits
(a) Geometrical isomerism (b) Optical isomerism
(c) Bonding isomerism (d) Ionisation isomerism
26. Which of the following compounds exhibits linkage isomerism
(a) [Co(en)3]Cl3 (b) [Co(NH3)6[Cr(CN)6]
(c) [Co(en)2 NO2Cl] Br (d) [Co(NH3)5Cl]Br2
27. The possible number of optical isomers in [Co(en)2Cl2] are
(a) 2 (b) 3 (c) 4 (d) 6
28. The number of unpaired electrons in Ni(CO)4 is
(a) Zero (b) One (c) Three (d) Five
HYBRIDISATION AND GEOMETRY
29. One mole of the complex compound Co(NH3)5Cl3 gives 3 moles of ions on dissolution in water.
One mole of the same complex reacts with two moles of AgNO3 solution to yield two moles of
AgCl(s). The structure of the complex is
(a) [Co(NH3)5Cl]Cl2 (b) [Co(NH3)3Cl3].2NH3
(c) [Co(NH3)4Cl2]Cl.NH3 (d) [Co(NH3)4Cl]Cl2.NH3
30. In the formation of K4[Fe(CN)6] the hybridisation involved is
(a) sp2 (b) d2sp3 (c) d3sp2 (d) d4p
31. Which complex has square planar structure
(a) Ni(CO)4 (b) [NiCl4]2– (c) [Ni(H2O)6]2+ (d) [Cu(NH3)4]2+
32. [CoF6]–3 is formed by ..... hybridization
(a) d2sp3 (b) d3sp2 (c) d2sp3 (d) sp3d2

ANSWER KEY
Q. No. 1 2 3 4 5 6 7 8 9 10
Ans. b b b d a b d c d c
Q. No. 11 12 13 14 15 16 17 18 19 20
node06\B0BC-BD\Kota\Board Material\Chemistry\Booklets\CBSE\Part-1

Ans. d b d b c b b d a c
Q. No. 21 22 23 24 25 26 27 28 29 30
Ans. c a a c a c b a a b
Q. No. 31 32
Ans. d c

308 E
CBSE
ALLEN
ASSERTION AND REASON

Note : In the following questions a statement of assertion followed by a statement of reason is


given. Choose the correct answer out of the following choices.
(a) Both (A) and (R) correct and (R) is correct explanation of (A).
(b) Both (A) and (R) correct and (R) is not correct explanation of (A).
(c) (A) is correct but (R) is wrong.
(d) (A) Assertion is wrong but (R) reason is correct.
1. Assertion (A) : In the coordination compound [Co(H2NCH2CH2NH2)3]2, ethane-l,2-diamine is a
neutral ligand.
Reason (R) : Oxidation number of Co in the complex ion is +3.
2. Assertion (A) : Isomers differ in one or more physical or chemical properties.
Reason (R) : These have different arrangement of atoms.
3. Assertion (A) : Tetrahedral complexes do not show geometrical isomerism.
Reason (R) : The relative positions of the unidentate ligands attached to the central metal atom
are the same with respect to each other.
4. Assertion (A) : Oxidation number of Cr in [Cr(NH3)3(H2O)3]Cl3 is same as the charge of the
complex ion, +3.
Reason (R) : All the ligands are neutral molecules in this compound.
5. Assertion (A) : The stability of [Ni(en)3] Cl2 is lower than that of [Ni(NH3)6] Cl2.
Reason (R) : The geometry of Ni is trigonal bipyramidal in[Ni(en)3] Cl2.
6. Assertion (A) : The ligands of nitro and nitrito are called ambidentate ligands.
Reason (R) : These ligands give linkage isomers.
7. Assertion (A) : [Pt Cl2 (NH3)2] is square planner

Reason (R) : Cl- is a weak field ligand


8. Assertion (A) : [Cr(H2O)6]Cl2 and [Fe(H2O)6]Cl2 are reducing in nature.
node06\B0BC-BD\Kota\Board Material\Chemistry\Booklets\CBSE\Part-1

Reason (R) : Unpaired electrons are present in their d-orbitals.


9. Assertion (A) : [Ni(CO)4] is SP3 hybridized

Reason (R) : It is diamagnetic in nature.


10. Assertion (A) : Ionisation isomerism shown by compounds having same composition.
Reason (R) : Compounds having same ions in the solutions.

E 309
Chemistry
ALLEN
11. Assertion (A) : Coordination compounds show linkage isomerism.
Reason (R) : They have ambidentate ligands.
12. Assertion (A) : [Co(NH3)5SO4]Br and [Co(NH3)5Br]SO4 show ionisation isomerism.
Reason (R) : Because they have Co metal.
13. Assertion (A) : Heteroleptic complexes show geometrical isomerism.
Reason (R) : Complexes with coordination number 6 and 8 are heteroleptic.

node06\B0BC-BD\Kota\Board Material\Chemistry\Booklets\CBSE\Part-1

ANSWER KEY
Q. No. 1 2 3 4 5 6 7 8 9 10
Ans. b a a a c a b b b c
Q. No. 11 12 13
Ans. a c c

310 E
CBSE
ALLEN
CASE BASED QUESTIONS
Case-I
Crystal field theory was proposed by H. Bethe and van Vleck in 1930. L.Orgel in 1952, gave a
much more satisfactory explanation for the bonding and the properties of complexes. The crystal
field theory (CFT) is an electrostatic model which considers the metal-ligand bond to be ionic
arising purely from electrostatic interactions between the metal ion and the ligand. In an
octahedral coordination entity with six ligands surrounding the metal atom/ion, there will be
repulsion between the electrons in metal d-orbitals and the electrons (or negative charges) of the
ligands. Such a repulsion is more when the metal d-orbital is directed towards the ligand than
when it is away from the ligand.
The degeneracy of the d-orbitals has been removed due to ligand-metal electron repulsions in the
octahedral complex to yield three orbitals of lower energy, t2g set and two orbitals of higher
energy, eg set.
The splitting of d-orbitals in an octahedral crystal field can be seen in following diagram.
Energy
Z
eg
Y
3/5 0
X
Bary centre
0
Metal Average 2/5 0
d-orbital energy
of the d-
orbitals in dxy dxz dyz t2g
spherical Splitting of d-orbitals
crystal field in octahedral
Free metal ion
crystal field

(a) What is crystal field splitting energy ?


Ans. The difference of energy between two sets of d-orbitals after splitting is called crystal field
splitting energy or crystal field stabilisation energy (CFSE).
OR
What is the basis of formation of spectro-chemical series ?
Ans. Spectrochemical series : The arrangements of ligands in the order of their increasing field
strength, i.e., increasing crystal field splitting energy (CFSE) value is called
spectrochemical series.
Crystal field splitting energy is the basis of formation of the spectrochemical series.
(b) Predict the number of unpaired electrons and magnetic nature in the square planar
[Pt(CN)4]2– ion.
Ans. 78Pt is present in group 10 of the periodic table.
node06\B0BC-BD\Kota\Board Material\Chemistry\Booklets\CBSE\Part-1

Its outer configuration is 5d96s1.


5d 6s 6p
Pt2+(5d86s0) =

= ×× ×× ×× ××

CN CN– CN– CN–
dsp2-hybridisation
=

E 311
Chemistry
ALLEN
CN– is a strong field ligand. It causes pairing of 5d unpaired electrons which mnakes one d-
orbital empty for dsp2-hybridisation and form square planar complex. As, there are no
unpaired electrons in [Pt(CN)4]2– ion.
 = 0
(c) Answer the following :
(i) Which of the two K4[Fe(CN)6] or K3[Fe(CN)6] is more stable and why?
(ii) What are the various factors affecting crystal field splitting ?
Ans. (i) In K4[Fe(CN)6], oxidation state of Fe is +2.
In K3[Fe(CN)6], oxidation state of Fe is +3.
Therefore, electronic configuration of
Fe2+ = d6 = t 62g

Fe3+ = d5 = t 52g

C.F.S.E. of K4[Fe(CN)6] = –0.4 × 60 = –2.4 0


C.F.S.E. of K3[Fe(CN)6] = –0.4 × 50 = –2.0 0
More is the C.F.S.E. more is the stability of the complex. Therefore, K4[Fe(CN)6] is more
stable than K3[Fe(CN)6].
(ii) The crystal field splitting, 0 depends upon the field produced by the ligand and charge
on the metal ion.
Some ligands are able to produce strong fields in thatcase, the splitting will be large,
whereas others produces weak field and consequently results in small splitting of
d-orbitals.
Case II
Isomerism is a phenomenon in which compounds have the same molecular formula but different
physical and chemical properties on account of different structures. The two major types of
isomerism are structural and stereoisomerism. The structural isomerism is further divided into
four types:
Linkage, coordination, ionisation and solvate isomerism.
While, the stereoisomerism is divided into two types :
Geometrical and Optical isomerism.
node06\B0BC-BD\Kota\Board Material\Chemistry\Booklets\CBSE\Part-1

(a) What is meant by the term ionisation isomerism ?


Ans. Ionisation isomerism arises when compounds give different ions in the solution, althught
they have same composition.
(b) What type of coordination compounds show linkage isomerism ?
Ans. Linkage isomerism is shown by the coordination compounds containing ambidendate
ligands.

312 E
CBSE
ALLEN
(c) [Co(NH3)5(NO2)] (NO3)2 shows ionisation isomers as follows :
[Co(NH3)5(NO2)] (NO3)2 and [Co(NH3)5(NO3)] (NO2) (NO3).
Suggest an alternative isomerism exhibited by the given compound and also give the
number of ions produced from the complex.
Ans. [Co(NH3)5(NO2)] (NO3)2 shows linkage isomers as follows :
[Co(NH3)5(NO2)] (NO3)2 and [Co(NH3)5(NO3)] (NO2) (NO3)
The complex will dissociate in aqueous soluiton to produce three ions.
[Co(NH3)5(NO2)] (NO3)2  [Co(NH3)5(NO2]2+ + 2NO3

OR
Write all the isomers of :
(i) [Co(NH3)6] [Cr(C2O4)3]
(ii) [Co(NH3)5SO4] Br
Ans. (i) The given compound exhibits coordination isomerism. The two isomers are :
(a) [Co(NH3)6] [Cr(C2O4)3] (b) [Co(C2O4)3] [Cr(NH3)6]
(ii) The given compound exhibits ionisation isomerism . The two isomers are :
(a) [Co(NH3)5SO4] Br (b) [Co(NH3)5Br] SO4
node06\B0BC-BD\Kota\Board Material\Chemistry\Booklets\CBSE\Part-1

E 313
Chemistry
ALLEN
PREVIOUS YEARS QUESTIONS
SECTION-A (ONE MARK QUESTIONS)
1. What is the secondary valency of Cobalt in [Co(en2)Cl2]+ ? [1] (CBSE _2023)
(a) 6 (b) 4 (c) 2 (d) 8
Ans. (a)
2. Which of the following ligands is an ambidentate ligand ? [1] (CBSE _2023)
(a) CO (b) NO2 (c) NH3 (d) H2O
Ans. (b)
3. The formula of the complex Iron (III) hexacyanidoferrate (II) is : [1] (CBSE _2023)
(a) Fe2 [Fe(CN)6]3 (b) Fe4 [Fe(CN)6]3
(c) Fe [Fe(CN)6] (d) Fe 3 [Fe(CN) 6 ]2
Ans. (b)
4. The oxidation state of Fe in [Fe(CO)5] is [1] (CBSE _2023)
(a) 2 (b) 0 (c) +3 (d) +5
Ans. (b)
5. The formula of the complex dichloridobis (ethane –1, 2-diamine) platinum (IV) nitrate is
[1] (CBSE _2023)
(a) [Pt Cl2(en)2 (NO3)2] (b) [Pt Cl2(en)2] (NO3)2
(c) [Pt Cl2(en)2(NO3)] NO3 (d) [Pt (en)2(NO3)2] Cl2
Ans. (b)
6. Draw the geometrical isomers of [CoCl2(en)2]+. [1] (Compartment _CBSE 2021)
Cl Cl
Cl
en en Co
Ans. Co en

en Cl
Cis Trans
7. Assertion (A) : Linkage isomerism arises in coordination compounds becauses of ambidentate ligand.
Reason (R) : Ambidentate ligand like NO2 has two different donor atoms i.e., N and O.
[1] (CBSE 2020)
node06\B0BC-BD\Kota\Board Material\Chemistry\Booklets\CBSE\Part-1

Ans. (i) Both Assertion (A) and Reason (R) are correct statement and Reason (R) is the correct
explanation of the Assertion(A)
8. Give the formulae of the following compounds : [1] (CBSE 2020)
(i) Potassium tetra hydroxydozincate [II]
(ii) Hexaammine platinium [IV] Chloride
Ans. (i) K2[Zn(OH)4] (ii) [Pt(NH3)6]Cl4
314 E
CBSE
ALLEN
9. Write the coordination number and oxidation state of Platinum in the complex [Pt(en)2Cl2].
[1] (CBSE 2018)
Ans. Given complex [Pt (en)2 Cl2]
Coordination no. = denticity × no. of ligands = 2 × 2 + 1× 2 = 6
Charge on ligand + oxidation state = charge on complex of metal ion = –2 + x = 0
Oxidation tate of Pt = x
x + 2(0) + 2(–1) = 0
x  2

SECTION-B (TWO MARKS QUESTIONS)


10. (a) Write the IUPAC names of the following : [2] (CBSE 2023)
2+
(i) [Co(NH3)5(ONO)]
(ii) K2[NiCl4]
OR
(b) (i) What is chelate complex? Give on example.
(ii) What are heteroleptic complexes ? Give one example.
Ans. (a) (i) Pentaamminenitrito – O-cobalt (III)
(ii) Potassium tetrachloridonickelate (II)
OR
(b) (i) When a di- or polydentate ligand uses its two or more donor atoms to bind a single
metal ion, it is said to be a chelate ligand. Such complexes, called chelate complexes
eg. EDTA4–.
(ii) Complex in which a metal is bound to more then are kind of donor group
eg.[Co(NH3)5(ONO)]2+
11. Write IUPAC name of the complex [Co(en)2(NO2)Cl]+. What type of structural isomerism is
shown by this complex ? [2] (CBSE 2019)
OR
Using IUPAC norms, write the formulae for the following complexes :
(a) Hexaaquachromium (III) chloride (b) Sodium trioxalatoferrate (III)
Ans. Chloridobis(ethane-1,2-diamine)nitrito-N-cobalt(III)ion
Linkage isomerism 
node06\B0BC-BD\Kota\Board Material\Chemistry\Booklets\CBSE\Part-1

OR
(i) [Cr(H2O)6]Cl3 (ii) Na3[Fe(OX)3] 
12. (a) Although both [NiCl4]2– and [Ni(CO)4] have sp3 hybridisation yet [NiCl4]2– is paramagnetic
and [Ni(CO)4] is diamagnetic. Give reason. (Atomic no. of Ni = 28)
(b) Write the electronic configuration of d5 on the basis of crystal field theory when
(i) 0 < P and (ii) 0 > P [2] (CBSE 2019)

E 315
Chemistry
ALLEN
Ans. (a) In [NiCl4]2, Cl is a weak field ligand due to which there are two unpaired electrons in 3d
orbital whereas in [Ni(CN)4]2, CN is a strong field ligand due to which pairing leads to no
unpaired electron in 3d-orbital/Or structural representation.
(b) (i) t2g3eg2 (ii) t2g5eg0 
13. Defind the following terms with a suitable example of each :
(a) Chelate complex (b) Ambidentate ligand [2] (CBSE 2019)
OR
Using IUPAC norms, write the formulae for the following complexes : [2]
(a) Tetraamminediaquacobalt(III) chloride
(b) Dibromidobis(ethane-1, 2-diamine)platinum(IV) nitrate
Ans. (a) When a di- or polydentate ligand uses its two or more donor atoms to bind a single metal
ion, it is said to be a chelate ligand. The number of such ligating groups is called the
denticity of the ligand. Such complexes, called chelate complexes
(b) Ligand which can ligate through two different atoms is called ambidentate ligand.
OR
(a) [Co(NH3)4(H2O)2] Cl3
(b) [Pt Br2(en)2] [NO3)2]
14. Using valence bond theory, write the hybridisation and magnetic character of the complex
[Fe(CN)6]4–. (Atomic no. of Fe = 26) [2] (CBSE 2019)
Ans. d2 sp3; Diamagnetic nature
15. Defind the following terms with a suitable example of each : [2] (CBSE 2019)
(a) Polydentate ligand (b) Homoleptic complex
OR
Using IUPAC norms, write the formulae for the following complexes : [2]
(a) Potassium tri(oxalato)chromate(III)
(b) Hexaaquamanganese(II) sulphate
Ans. (a) Polydentate ligands range in the number of atoms used to bond to a central metal atom or ion.
e.g. EDTA
(b) Homoleptic complex are those which has all identical ligand. e.g. metal compound
OR
(a) K3 [Cr(OX)3]
node06\B0BC-BD\Kota\Board Material\Chemistry\Booklets\CBSE\Part-1

(b) [Mn(H2O)6]SO4
16. Using IUPAC norms write the formulae for the following :
(a) Tris (ethane-1,2-diamine)chromium(III)chloride
(b) Potassium tetrahydroxozincate(II) [2] (CBSE 2018)
Ans. (a) [Cr(en)3]Cl3
(b) K2[Zn(OH)4]

316 E
CBSE
ALLEN
17. (a) For the complex [Fe(CN)6]4–, write the hybridization, magnetic character and spin type of
thecomplex. (At. number : Fe = 26)
(b) Draw one of the geometrical isomers of the complex [Co(en)2Cl2]+ which is optically
active. [2] (CBSE 2018)
Ans. (a) d2sp3,
Diamagnetic, low spin
Cl +
Cl
(b) en Co

en
18. When a coordination compound NiCl2.6H2O is mixed with AgNO3, 2 moles of AgCl are
precipitated per mole of the compound. Write [2] (CBSE 2018)
(i) Structural formula of the complex
(ii) IUPAC name of the complex
Ans. (i) [Ni(H2O)6] Cl2
(ii) Hexaaquanickel(II) chloride
SECTION-C (THREE MARKS QUESTIONS)
19. (a) Draw the geometrical isomers of [Co(en)2Cl2]2+. Which geometrical isomer of
[Co(en)2Cl2]2+ is not optically active and why ?
(b) Write the hybridisation and magnetic behaviour of [CoF6]3–
[Given: Atomic number of Co = 27] [3] (CBSE 2023)
+
Ans. (a) [CoCl2(en)2]
Cl Cl

en Co en Co en
en

Cl Cl
Mirror
+
Trans [CoCl2(en)2] isomer-optically inactive
(Superimposable mirror images)

Cl Cl
Cl Cl
node06\B0BC-BD\Kota\Board Material\Chemistry\Booklets\CBSE\Part-1

en Co Co en

en en
Mirror
+
Cis [CoCl2(en)2] isomer-optically active
(non-superimposable mirror images)

E 317
Chemistry
ALLEN
(b) [CoF6]3–
Cobalt exists in the +3 oxidation state.
3+
Orbitals of Co ion
3d 4s 4p 4d
3 2
sp d hybridised
3+
orbitals of Co
3 2
3d sp d hybrid 4d
3–
[CoF 6]
(outer orbital or
high spin complex)
3d six pairs of electrons 4d

from six F ions

20. (a) Using valence bond theory, predict the hybridization and magnetic character of following :
[CoF6]3– [Atomic number of Co = 27]
(b) Write IUPAC name of the following complex : [CoBr2(en)2]+
(c) How many ions are produced from the complex [Co(NH3)6]Cl2 in solution ?
[1× 3 = 3](CBSE-Term-II_ 2022)
Ans. (a) [CoF6]3– Hybridization = sp3d2
Magnet character = Paramagnetic
(b) dibromidobis – ethane-1,2-diamine.cobalt(I)
(c) [Co(NH3)6]Cl2  [Co(NH3)6]3+ + 2Cl–
(aq. sol.)  3 ions produced
21. (i) Write the IUPAC name of the complex [Cr(NH3)4 Cl2] Cl.
(ii) What type of isomerism is exhibited by the complex [Co(en)3]3+ (en = ethane-1,2-diamine)
(iii) Why is [NiCl4]2– paramagnetic but [Ni(CO)4] is diamagnetic ?
(At. nos. : Cr = 24, Co = 27, Ni = 28) [3] (CBSE 2020)
Ans. (i) Tetraamminedichloridochromium(III) chloride
3+ 3+
en
en

(ii) en Co Co en

en en
dextro mirror laevo
Optical Isomer
node06\B0BC-BD\Kota\Board Material\Chemistry\Booklets\CBSE\Part-1

(iii) In Ni(CO)4, Ni is in zero oxidation state whereas in NiCl42–, it is in +2 oxidation state. In


the presence of CO ligand, the unpaired d electrons of Ni pair up but Cl– being a weak
ligand is unable to pair up the unpaired electrons.
22. What is meant by crystal field splitting energy? On the basis of crystal field theory, write the
electronic configuration of d4 in terms of t2g and eg in an octahedral field when
(i) 0 > P (ii) 0 < P [3] (CBSE 2020)

318 E
CBSE
ALLEN
Ans. The difference of energy two sets of d-orbitals (which are no longer degenerate under different
kinds of electric fields).
Is called crystal field stabilization energy or (CFSE) represented by symbol 0.
d4 when 0 > P t42geg°.
d4 when 0 < P t32geg1.
23. For the complex [NiCl4]2–, write
(i) the IUPAC name
(ii) the hybridization type.
(iii) the shape of the complex
(Atomic no. of Ni = 28) [3] (CBSE 2020)
Ans. [NiCl4]2–
(i) Tetrachlorido nickelate (II) ion
(ii) sp3
(iii) tetrahedral
24. Name the following coordination entities and draw the structures of their stereoisomers :
(i) [Co(en)2Cl2]+ (en = ethylenediamine)
(ii) [Cr(C2O4)3]3–
(iii) [Co(NH3)3Cl3]
(Atomic numbers Cr = 24, Co = 27) [3] (CBSE 2019)
Ans. (i) Dichlorido bis ethylenediamine cobalt (III) ion
Cl Cl
Cl
en Co en Co
en

en Cl
Cis Trans
(ii) Trioxalato chromate (III) ion
ox ox

ox Co Co ox

ox ox
node06\B0BC-BD\Kota\Board Material\Chemistry\Booklets\CBSE\Part-1

3– 3–
[Cr(OX)3] Mirror [Cr(OX)3]
(dextro) (Laevo)
(iii) Triamine trichlorido cobalt (III)
NH3 NH3
Cl NH3 NH3 Cl
Co Co
Cl NH3 Cl Cl
Cl NH3
Fac mer

E 319
Chemistry
ALLEN
25. Explain the following terms giving a suitable example in each case :
(i) Ambidentate ligand
(ii) Denticity of a ligand
(iii) Crystal field splitting in an octahedral field [3] (CBSE 2019)
Ans. (i) Ambidentate ligand : Ligand which can link through two different atoms is called
ambident ligand. Example - SCN-, CNO–, CN-, NO 2 can link through ‘N’ as well as O
while SCN– can link through ‘S’ as well ‘N’ atom.
(ii) Denticity of a ligand : Total number of lone pair donated by a ligand when it is bonded
with metal is called denticity or number of doner sites on a ligand is called denticity.
- -
CN has 1, COO
has 2,
COO-
H2N – CH2 – CH2 – NH – CH2 – CH2 – NH2 has 3.
(iii) Crystal field spliting in an octahedral field : In a octahedral complex, the co-ordination
number is 6. The metal ion is at the centre and the ligands occupy the six corners of the
octahedron as shown in figure.
We know that two orbitals, d x2  y2 and d z 2 are oriented along the axis while the remaining
three orbitals, viz., dxy, dyz and dzx are oriented in between the axis.
The two orbtials d x2  y2 and d z 2 are designated as eg orbitals while the three orbitals dxy, dyz
and dzx are designated as t2g orbitals. As the six ligands approach the central ion along the
axis, eg orbitals, is repelled more by the ligand than the t2g orbitals.
Z

In other words, the energy of the d z 2 and d x2  y2 orbitals increases much more than the
energy of the dxy, dyz and dzx orbitals.
Thus, in octahedral complexes, the five d-orbitals split up into two sets : one set
consisting of two orbitals ( d x2 – y2 and ) of higher energy (eg orbitals) and the other set
consisting of three orbitals (dxy, dyz and dzx ) of lower energy (t2g orbitals).
eg
node06\B0BC-BD\Kota\Board Material\Chemistry\Booklets\CBSE\Part-1

.6 

–.4 
t2g

The state I represents degeneracy of al the five d-orbitals in the isolated central ion. The
state II represents hypothetical degeneracy of all the orbitals at a higher energy level if the
negative charge of all the ligands is assumed to be uniformly affecting the electrons in the
d-orbitals of the metal ion. The state III represents crystal field splitting discussed above

320 E
CBSE
ALLEN
26. Explain the following cases giving appropriate reasons:
(i) Nickel does not form low spin octahedral complexes.
(ii) The -complexes are known for the transition metals only.
(iii) Co2+ is easily oxidised to Co+3 in the presence of a strong ligand. [3] (CBSE 2019)
Ans. (i) Since nickel in zero oxidation state contain no unpaired electron.
(ii) The transition metal/ions have electrons in d- orbitals which can be shared to  -acid
ligands through synergic bonding.
For example : CH2 = CH2 and C6 H6 etc.

(iii) Co3+ can from very stable low spin octahedral complex.
27. Compare the following complexes with respect to structural shapes of units, magnetic behaviour
and hybrid orbitals involved in units :
[Co(NH3)6]3+, [Cr(NH3)6]3+, Ni(CO)4
(At. Nos. : Co = 27, Cr= 24, Ni = 28) [3] (CBSE 2019)

Ans. S. No. Complexes Magnetic behaviour Hybridisation Shape


3+ 2 3
1 [Co(NH3)6] Diamagnetic d sp Octahedral
2 [Cr(NH3)6]3+ Paramagnetic d2sp3 Octahedral
3 [Ni(CO)4] Diamagnetic sp3 Tetrahedral

28. Giving a suitable example for each, explain the following :


(i) Crystal field splitting
(ii) Linkage isomerism
(iii) Ambidentate ligand [3] (CBSE 2019)
Ans. (i) Crystal field splitting : In a free transition metal ion, all the five orbitals are degenerate,
but when it is involved in a complex formation, the degeneracy split. This is called crystal
field splitting.
(ii) Linkage isomerism : Linkage isomerism arises in a coordination compound containing
ambidentate ligand. A simple example is provided by complexes containing the thiocyanate
ligand, NCS–.
(iii) Ambidentate ligand : Unidentate ligands containing more than one coordinating atoms are
called ambidentate ligands. For example,
node06\B0BC-BD\Kota\Board Material\Chemistry\Booklets\CBSE\Part-1

NO–2, CN–, SCN– are all ambidentate ligands.


29. (a) What is the basis of formation of the spectro-chemical series ?
(b) Draw the structures of geometrical isomer of the following coordination complexes :
[Co(NH3)3Cl3] and [CoCl2(en)2]+
(en = ethylenediamine and atomic number of Co is 27) [3] (CBSE 2019)

E 321
Chemistry
ALLEN
Ans. (a) Spectro-chemical series : The arrangement of ligands are in their increasing
field strength i.e. increasing crystal field splitting energy (CFSE) value, is called spectro-
chemical series. Crystal field splitting energy is the basis of formation of the spectro
chemical series.
It is an experimentally determined series based on the absorption of light by complexes
with different ligands.
I– <Br–< SCN– < Cl– < S2–< F–< OH–< C2O42–< H2O < NCS–< edta4–< NH3 < en < CN–<CO
(b) (i) The cis and trans isomers of [Co(NH3)3Cl3] may be represented as
Cl NH3 NH3 NH3
NH3 Cl

Co Co

Cl NH3 Cl Cl
Cl NH3
cis-isomer trans-isomer
+
(ii) [CoCl2 (en)2] ion exists in two geometrical isomers as shown below.
Cl Cl
Cl
en en
Co Co
en

Cl en
trans-isomer cis-isomer
30. (a) Write the formula of the following coordination compound :
Iron(III) hexacyanoferrate(II)
(b) What type of isomerism is exhibited by the complex [Co(NH3)5Cl]SO4 ?
(c) Write the hybridisation and number of unpaired electrons in the complex [CoF6]3–.
(Atomic No. of Co = 27). [3] (CBSE 2018)
Ans. (a) Fe4[Fe(CN)6]3
(b) [CO(NH3)5Cl]SO4 (Ionisation isomers)
(c) [COF6]–3
(i) Hybridisation  Sp3d2
(ii) Unpaired e–  4 unpaired electrons.
31. (a) What type of isomerism is shown by the complex [Co(NH3)5(SCN)]2+ ?
(b) Why is [NiCl4]2– paramagnetic while [Ni(CN)4]2– is diamagnetic ?
node06\B0BC-BD\Kota\Board Material\Chemistry\Booklets\CBSE\Part-1

(Atomic number of Ni = 28)


(c) Why are low spin tetrahedral complexes rarely observed [3](CBSE 2018)
Ans. (a) Linkage isomerism
(b) In [NiCl4]2–, due to the presence of Cl–, a weak field ligand no pairing occurs whereas in
[Ni(CN)4]2–, CN–is a strong field ligand and pairing takes place.
(c) Because of very low CFSE which is not able to pair up the electrons.

322 E
CBSE
ALLEN
SECTION-D (FOUR MARKS QUESTIONS)
32. In an octahedral coordination entity with six ligands surrounding the metal atom/ion, there will
be repulsion between the electrons in metal d orbitals and the electrons (or negative charges) of
the ligands. Such a repulsion is more when the metal d orbital is directed towards the ligand than
when it is away from the ligand. Thus, the dx2-y2 and dz2 orbitals.
Which point towards the axes along the direction of the ligand will experience more repulsion
and will be raised in energy; and the dxy, dyz and dxz orbitals which are directed between the
axes will be lowered in energy relative to the average energy in the spherical crystal field. Thus,
the degeneracy of the d orbitals has been removed due to ligand electron-metal electron
repulsions in the octahedral complex to yield three orbitals of lower energy, t2g set and two
orbitals of higher energy, eg set. ?
Answer the following questions :
(a) Write the relation between t and O.
4
Ans. t = O
9
(b) Why is CO a stronger ligand than Cl– ?
Ans. It is because CO can form (sigma) as well as -bond.
(c) What is spectrochemical series? Explain the difference between a weak field and a strong
field ligand.
Ans. A spectrochemical series is the arrangement of common ligands in the increasing order of
their crystal - field splitting energy (CFSE) values. The ligands present on the R.H.S of the
series are strong field ligands while that on the L.H.S are weak field ligands. Also, strong
field ligands cause higher splitting in the d orbitals that weak field ligands.
I– < Br– < S2– < SCN– <Cl– <N3– < OH– < C2O42– < ~H2O < NCS– < edta4– < NH3 < en– < CN– < CO
OR
What is meant by crystal field splitting energy? On the basis of crystal field theory, write
the electronic configuration of d4 in terms of t2g and eg in an octahedral field when

(i) 0 > P (ii) 0 < P


node06\B0BC-BD\Kota\Board Material\Chemistry\Booklets\CBSE\Part-1

Ans. The difference of energy two sets of d-orbitals (which are no longer degenerate under
different kinds of electric fields).
Is called crystal field stabilization energy or (CFSE) represented by symbol 0.
d4 when 0 > P t42g eg°.
d4 when 0 < P t32g eg1.

E 323
Chemistry
ALLEN
SECTION-E (FIVE MARKS QUESTIONS)

33. (i) What type of isomerism is shown by [Co(NH3)5ONO]Cl2?


(ii) On the basis of crystal field theory, write the electronic configuration for d4 ion if 0 < P.
(iii) Write the hybridization and shape of [Fe(CN)6]3–.
(Atomic number of Fe = 26)
(iv) Write down the IUPAC name of the following complex :
[Pt(NH3)(H2O)Cl2]
(v) Write the formula for the following complex :
tris(ethan-1, 2-diamine)chromium(III) chloride [5] (CBSE 2018)
Ans. (i) Linkage isomerism
(ii) t2g3 eg1 / Diagrammatic representation

(iii) d2sp3, Octahedral


(iv) Ammineaquadichloridoplatinum(II)
(v) [Cr(en)3]Cl3

node06\B0BC-BD\Kota\Board Material\Chemistry\Booklets\CBSE\Part-1

324 E
CBSE
ALLEN
PRACTICE TEST
SECTION-A
1. The name of complex ion, [Fe(CN)6]3– is :- [1]
(a) Tricyanoferrate (III) ion (b) Hexacyanidoferrate (III) ion
(c) Hexacyanoiron (III) ion (d) Hexacyanitoferrate (III) ion
2. The hybridization involved in complex [Ni(CN)4]2– is (At.No. Ni = 28) [1]
(a) d2sp2 (b) d2sp3 (c) dsp2 (d) sp3
3. The sum of coordination number and oxidation number of the metal M in the complex
[M(en)2(C2O4)]Cl (where en is ethylenediamine) is :- [1]
(a) 7 (b) 8 (c) 9 (d) 6
4. Which one of the following electrolytes has the same value of van't Hoff's factor (i) as that of the
Al2(SO4)3 (if all are 100% ionised) ? [1]
(a) K3[Fe(CN)6] (b) Al(SO4)3
(c) K4[Fe(CN)6] (d) K2SO4
5. Crystal field stabilization energy for high spin d4 octahedral complex is :- [1]
(a) –0.6 0 (b) –1.8 0 (c) –1.6 0 + P (d) –1.2 0
Note : In the following questions a statement of assertion followed by a statement of reason is
given. Choose the correct answer out of the following choices.
(a) Both (A) and (R) correct and (R) is correct explanation of (A).
(b) Both (A) and (R) correct and (R) is not correct explanation of (A).
(c) (A) is correct but (R) is wrong.
(d) (A) Assertion is wrong but (R) reason is correct.
6. Assertion (A) : [Ni(CN)4]2– is square planar and diamagnetic.
Reason (R) : It has no unpaired electrons due to presence of strong field ligand. [1]
+
7. Assertion (A) : The total number of isomers shown by [Co(en)2Cl2l complex ion is three.
Reason (R) : [Co(en)2Cl2]+ complex ion has an octahedral geometry. [1]
SECTION-B
8. Why is [NiCl4]2– paramagnetic while [Ni(CN)4]2– is diamagnetic ?
(Atomic number of Ni = 28) [2]
9. Write the corrdination number and oxidation state of Platinum in the complex [Pt(en)2Cl2]. [2]
node06\B0BC-BD\Kota\Board Material\Chemistry\Booklets\CBSE\Part-1

10. Write the hybridisation and number of unpaired electrons in the complex [CoF6]3–.
(Atomic No. of Co = 27). [2]
11. When a coordination compound NiCl2.6H2O is mixed with AgNO3, 2 moles of AgCl are
precipitated per mole of the compound. Write [2]
(i) Structural formula of the complex
(ii) IUPAC name of the complex

E 325
Chemistry
ALLEN
SECTION-C
12. Using IUPAC norms write the formulae for the following : [3]
(a) Tris (ethane-1,2-diamine)chromium(III)chloride
(b) Potassium tetrahydroxozincate(II)
(c) Hexaammine platinium [IV] Chloride
13. For the complex [Fe(CN)6]4–, write the hybridization, magnetic character and spin type of the
complex. (At. number : Fe = 26) [3]
SECTION-D
14. The Lewis approach to chemical bonding failed to shed light on the formation of chemical bonds.
Also, valence shell electron pair repulsion theory of (VSEPR theory) has limited applications
(and also failed in predicting the geometry corresponding to complex molecules). In order to
address these issues, the valence bond theory was put forth by the German physicists Walter
Heinrich Heitler and Fritz Wolfgang London. The Schrodinger wave equation was also used to
explain the formation of a covalent bond between two hydrogen atoms. This theory focuses on
the concepts of electronic configuration, atomic orbitals (and their overlapping) and the
hydridization of these atomic orbitals. Chemical bonds are formed from the overlapping of
atomic orbital wherein the electrons are localized in the corresponding bond region. the valence
bond theory also goes on to explain the electronic structure of the molecules formed by this
overlapping of atomic orbitals. It also emphasizes that the nucleus of one atom in a molecule is
attracted to the electrons of the other atoms. [1+1+2=4]
Answer the following questions :
(a) What type of hybrid orbital is associated with Ni atom in [Ni(CN)4]2– ?
OR
What is the oxidation coordination number of central metal ion in [Fe(C2O4)3]3– ?
(b) Using valence bond theory predict the shape of [Ni(CO)4].[Ni = 28]
(c) For the complex, [Fe(en)2Cl2]Cl (en = ethylenediamine), identify and write :
(i) oxidation number of iron
(ii) number of hybrid orbitals and the shape of the complex.
SECTION-E
15. (a) What is meant by crystal field splitting energy? On the basis of crystal field theory, write
the electronic configuration of d4 in terms of t2g and eg in an octahedral field when [5]
(i) 0 > P
node06\B0BC-BD\Kota\Board Material\Chemistry\Booklets\CBSE\Part-1

(ii) 0 < P
(b) (i) Write the IUPAC name of the complex [Cr(NH3)4 Cl2] Cl.
(ii) Write IUPAC name of the complex [Co(en)2(NO2)Cl]+.
(iii) Why is [NiCl4]2– paramagnetic but [Ni(CO)4] is diamagnetic ?
(At. nos. : Cr = 24, Co = 27, Ni = 28)
326 E
CBSE
ALLEN
PRACTICE TEST SOLUTIONS
SECTION-A
1. (b)
2. (c)
3. (c)
4. (b)
5. (a)
6. (a)
7. (b)
SECTION-B
8. In [NiCl4]2–, due to the presence of Cl¯, a weak field ligand no pairing occurs whereas in
[Ni(CN)4]2, CN¯ is a strong field ligand and pairing takes place.
9. Given complex [Pt (en)2 Cl2]
Coordination no. = denticity × no. of ligands
=2×2+2×1=6
Charge on ligand + oxidation state = charge on complex of metal ion
–2+x=0
x = +2
10. [COF6]–3
(i) Hybridisation  Sp3d2
(ii) Unpaired e¯  4 unpaired electrons.
11. (i) [Ni(H2O)6] Cl2
(ii) Hexaaquanickel(II) chloride
SECTION-C
12. (i) [Cr(en)3]Cl3
(ii) K2[Zn(OH)4]
(iii) [Pt(NH3)6]Cl4
13. d2sp3,
node06\B0BC-BD\Kota\Board Material\Chemistry\Booklets\CBSE\Part-1

Diamagnetic,
low spin
SECTION-D
2
14. (a) dsp
OR
+3

E 327
Chemistry
ALLEN
(b) Oxidation state of Ni in Ni(CO)4 is zero.
3d 4s 4s
Ni (28) E.C. =

Hybridisation =

sp3 hybridisation

Ni(CO)4 =
Four pairs of electrons
from four CO molecules
Shape - Tetrahedral
Hybridisation - sp3
(c) (i) Let the oxidation number of iron (Fe) be x in [Fe(en)2Cl2]Cl.
x + 0 + 2 (–1) = +1 ; x = +3
3d5 4s0
(ii) Fe3+ = [Ar] 3d5 4s0 =

Fe3+ (in presence of ligand)


[Fe(en)2Cl2]–

      ×××× ×× ××××××

d2sp3-hybridisation
d2sp3 - hybridisation (six hybrid orbitals)
Shape - Octahedral
SECTION-E
15. (a) The difference of energy two sets of d-orbitals (which are no longer degenerate under
different kinds of electric fields).
Is called crystal field stabilization energy or (CFSE) represented by symbol 0.
d4 when 0 > P t42g eg°.
d4 when 0 < P t32g eg1.
node06\B0BC-BD\Kota\Board Material\Chemistry\Booklets\CBSE\Part-1

(b) (i) Tetraamminedichloridochromium(III) chloride.


(ii) Chloridobis(ethane-1,2-diamine)nitrito-N-cobalt(III)ion.
(iii) In Ni(CO)4, Ni is in zero oxidation state whereas in NiCl42–, it is in +2 oxidation
state. In the presence of CO ligand, the unpaired d electrons of Ni pair up but Cl¯
being a weak ligand is unable to pair up the unpaired electrons.

328 E

You might also like